]> code.communitydata.science - stats_class_2020.git/commitdiff
pset3 solutions
authoraaronshaw <aaron.d.shaw@gmail.com>
Mon, 12 Oct 2020 17:41:49 +0000 (12:41 -0500)
committeraaronshaw <aaron.d.shaw@gmail.com>
Mon, 12 Oct 2020 17:41:49 +0000 (12:41 -0500)
psets/pset3-worked_solution.html [new file with mode: 0644]
psets/pset3-worked_solution.pdf [new file with mode: 0644]
psets/pset3-worked_solution.rmd [new file with mode: 0644]

diff --git a/psets/pset3-worked_solution.html b/psets/pset3-worked_solution.html
new file mode 100644 (file)
index 0000000..0970f3e
--- /dev/null
@@ -0,0 +1,2269 @@
+<!DOCTYPE html>
+
+<html>
+
+<head>
+
+<meta charset="utf-8" />
+<meta name="generator" content="pandoc" />
+<meta http-equiv="X-UA-Compatible" content="IE=EDGE" />
+
+
+<meta name="author" content="Aaron Shaw" />
+
+<meta name="date" content="2020-10-12" />
+
+<title>Problem set 3: Worked solutions</title>
+
+<script>/*! jQuery v1.11.3 | (c) 2005, 2015 jQuery Foundation, Inc. | jquery.org/license */
+!function(a,b){"object"==typeof module&&"object"==typeof module.exports?module.exports=a.document?b(a,!0):function(a){if(!a.document)throw new Error("jQuery requires a window with a document");return b(a)}:b(a)}("undefined"!=typeof window?window:this,function(a,b){var c=[],d=c.slice,e=c.concat,f=c.push,g=c.indexOf,h={},i=h.toString,j=h.hasOwnProperty,k={},l="1.11.3",m=function(a,b){return new m.fn.init(a,b)},n=/^[\s\uFEFF\xA0]+|[\s\uFEFF\xA0]+$/g,o=/^-ms-/,p=/-([\da-z])/gi,q=function(a,b){return b.toUpperCase()};m.fn=m.prototype={jquery:l,constructor:m,selector:"",length:0,toArray:function(){return d.call(this)},get:function(a){return null!=a?0>a?this[a+this.length]:this[a]:d.call(this)},pushStack:function(a){var b=m.merge(this.constructor(),a);return b.prevObject=this,b.context=this.context,b},each:function(a,b){return m.each(this,a,b)},map:function(a){return this.pushStack(m.map(this,function(b,c){return a.call(b,c,b)}))},slice:function(){return this.pushStack(d.apply(this,arguments))},first:function(){return this.eq(0)},last:function(){return this.eq(-1)},eq:function(a){var b=this.length,c=+a+(0>a?b:0);return this.pushStack(c>=0&&b>c?[this[c]]:[])},end:function(){return this.prevObject||this.constructor(null)},push:f,sort:c.sort,splice:c.splice},m.extend=m.fn.extend=function(){var a,b,c,d,e,f,g=arguments[0]||{},h=1,i=arguments.length,j=!1;for("boolean"==typeof g&&(j=g,g=arguments[h]||{},h++),"object"==typeof g||m.isFunction(g)||(g={}),h===i&&(g=this,h--);i>h;h++)if(null!=(e=arguments[h]))for(d in e)a=g[d],c=e[d],g!==c&&(j&&c&&(m.isPlainObject(c)||(b=m.isArray(c)))?(b?(b=!1,f=a&&m.isArray(a)?a:[]):f=a&&m.isPlainObject(a)?a:{},g[d]=m.extend(j,f,c)):void 0!==c&&(g[d]=c));return g},m.extend({expando:"jQuery"+(l+Math.random()).replace(/\D/g,""),isReady:!0,error:function(a){throw new Error(a)},noop:function(){},isFunction:function(a){return"function"===m.type(a)},isArray:Array.isArray||function(a){return"array"===m.type(a)},isWindow:function(a){return null!=a&&a==a.window},isNumeric:function(a){return!m.isArray(a)&&a-parseFloat(a)+1>=0},isEmptyObject:function(a){var b;for(b in a)return!1;return!0},isPlainObject:function(a){var b;if(!a||"object"!==m.type(a)||a.nodeType||m.isWindow(a))return!1;try{if(a.constructor&&!j.call(a,"constructor")&&!j.call(a.constructor.prototype,"isPrototypeOf"))return!1}catch(c){return!1}if(k.ownLast)for(b in a)return j.call(a,b);for(b in a);return void 0===b||j.call(a,b)},type:function(a){return null==a?a+"":"object"==typeof a||"function"==typeof a?h[i.call(a)]||"object":typeof a},globalEval:function(b){b&&m.trim(b)&&(a.execScript||function(b){a.eval.call(a,b)})(b)},camelCase:function(a){return a.replace(o,"ms-").replace(p,q)},nodeName:function(a,b){return a.nodeName&&a.nodeName.toLowerCase()===b.toLowerCase()},each:function(a,b,c){var d,e=0,f=a.length,g=r(a);if(c){if(g){for(;f>e;e++)if(d=b.apply(a[e],c),d===!1)break}else for(e in a)if(d=b.apply(a[e],c),d===!1)break}else if(g){for(;f>e;e++)if(d=b.call(a[e],e,a[e]),d===!1)break}else for(e in a)if(d=b.call(a[e],e,a[e]),d===!1)break;return a},trim:function(a){return null==a?"":(a+"").replace(n,"")},makeArray:function(a,b){var c=b||[];return null!=a&&(r(Object(a))?m.merge(c,"string"==typeof a?[a]:a):f.call(c,a)),c},inArray:function(a,b,c){var d;if(b){if(g)return g.call(b,a,c);for(d=b.length,c=c?0>c?Math.max(0,d+c):c:0;d>c;c++)if(c in b&&b[c]===a)return c}return-1},merge:function(a,b){var c=+b.length,d=0,e=a.length;while(c>d)a[e++]=b[d++];if(c!==c)while(void 0!==b[d])a[e++]=b[d++];return a.length=e,a},grep:function(a,b,c){for(var d,e=[],f=0,g=a.length,h=!c;g>f;f++)d=!b(a[f],f),d!==h&&e.push(a[f]);return e},map:function(a,b,c){var d,f=0,g=a.length,h=r(a),i=[];if(h)for(;g>f;f++)d=b(a[f],f,c),null!=d&&i.push(d);else for(f in a)d=b(a[f],f,c),null!=d&&i.push(d);return e.apply([],i)},guid:1,proxy:function(a,b){var c,e,f;return"string"==typeof b&&(f=a[b],b=a,a=f),m.isFunction(a)?(c=d.call(arguments,2),e=function(){return a.apply(b||this,c.concat(d.call(arguments)))},e.guid=a.guid=a.guid||m.guid++,e):void 0},now:function(){return+new Date},support:k}),m.each("Boolean Number String Function Array Date RegExp Object Error".split(" "),function(a,b){h["[object "+b+"]"]=b.toLowerCase()});function r(a){var b="length"in a&&a.length,c=m.type(a);return"function"===c||m.isWindow(a)?!1:1===a.nodeType&&b?!0:"array"===c||0===b||"number"==typeof b&&b>0&&b-1 in a}var s=function(a){var b,c,d,e,f,g,h,i,j,k,l,m,n,o,p,q,r,s,t,u="sizzle"+1*new Date,v=a.document,w=0,x=0,y=ha(),z=ha(),A=ha(),B=function(a,b){return a===b&&(l=!0),0},C=1<<31,D={}.hasOwnProperty,E=[],F=E.pop,G=E.push,H=E.push,I=E.slice,J=function(a,b){for(var c=0,d=a.length;d>c;c++)if(a[c]===b)return c;return-1},K="checked|selected|async|autofocus|autoplay|controls|defer|disabled|hidden|ismap|loop|multiple|open|readonly|required|scoped",L="[\\x20\\t\\r\\n\\f]",M="(?:\\\\.|[\\w-]|[^\\x00-\\xa0])+",N=M.replace("w","w#"),O="\\["+L+"*("+M+")(?:"+L+"*([*^$|!~]?=)"+L+"*(?:'((?:\\\\.|[^\\\\'])*)'|\"((?:\\\\.|[^\\\\\"])*)\"|("+N+"))|)"+L+"*\\]",P=":("+M+")(?:\\((('((?:\\\\.|[^\\\\'])*)'|\"((?:\\\\.|[^\\\\\"])*)\")|((?:\\\\.|[^\\\\()[\\]]|"+O+")*)|.*)\\)|)",Q=new RegExp(L+"+","g"),R=new RegExp("^"+L+"+|((?:^|[^\\\\])(?:\\\\.)*)"+L+"+$","g"),S=new RegExp("^"+L+"*,"+L+"*"),T=new RegExp("^"+L+"*([>+~]|"+L+")"+L+"*"),U=new RegExp("="+L+"*([^\\]'\"]*?)"+L+"*\\]","g"),V=new RegExp(P),W=new RegExp("^"+N+"$"),X={ID:new RegExp("^#("+M+")"),CLASS:new RegExp("^\\.("+M+")"),TAG:new RegExp("^("+M.replace("w","w*")+")"),ATTR:new RegExp("^"+O),PSEUDO:new RegExp("^"+P),CHILD:new RegExp("^:(only|first|last|nth|nth-last)-(child|of-type)(?:\\("+L+"*(even|odd|(([+-]|)(\\d*)n|)"+L+"*(?:([+-]|)"+L+"*(\\d+)|))"+L+"*\\)|)","i"),bool:new RegExp("^(?:"+K+")$","i"),needsContext:new RegExp("^"+L+"*[>+~]|:(even|odd|eq|gt|lt|nth|first|last)(?:\\("+L+"*((?:-\\d)?\\d*)"+L+"*\\)|)(?=[^-]|$)","i")},Y=/^(?:input|select|textarea|button)$/i,Z=/^h\d$/i,$=/^[^{]+\{\s*\[native \w/,_=/^(?:#([\w-]+)|(\w+)|\.([\w-]+))$/,aa=/[+~]/,ba=/'|\\/g,ca=new RegExp("\\\\([\\da-f]{1,6}"+L+"?|("+L+")|.)","ig"),da=function(a,b,c){var d="0x"+b-65536;return d!==d||c?b:0>d?String.fromCharCode(d+65536):String.fromCharCode(d>>10|55296,1023&d|56320)},ea=function(){m()};try{H.apply(E=I.call(v.childNodes),v.childNodes),E[v.childNodes.length].nodeType}catch(fa){H={apply:E.length?function(a,b){G.apply(a,I.call(b))}:function(a,b){var c=a.length,d=0;while(a[c++]=b[d++]);a.length=c-1}}}function ga(a,b,d,e){var f,h,j,k,l,o,r,s,w,x;if((b?b.ownerDocument||b:v)!==n&&m(b),b=b||n,d=d||[],k=b.nodeType,"string"!=typeof a||!a||1!==k&&9!==k&&11!==k)return d;if(!e&&p){if(11!==k&&(f=_.exec(a)))if(j=f[1]){if(9===k){if(h=b.getElementById(j),!h||!h.parentNode)return d;if(h.id===j)return d.push(h),d}else if(b.ownerDocument&&(h=b.ownerDocument.getElementById(j))&&t(b,h)&&h.id===j)return d.push(h),d}else{if(f[2])return H.apply(d,b.getElementsByTagName(a)),d;if((j=f[3])&&c.getElementsByClassName)return H.apply(d,b.getElementsByClassName(j)),d}if(c.qsa&&(!q||!q.test(a))){if(s=r=u,w=b,x=1!==k&&a,1===k&&"object"!==b.nodeName.toLowerCase()){o=g(a),(r=b.getAttribute("id"))?s=r.replace(ba,"\\$&"):b.setAttribute("id",s),s="[id='"+s+"'] ",l=o.length;while(l--)o[l]=s+ra(o[l]);w=aa.test(a)&&pa(b.parentNode)||b,x=o.join(",")}if(x)try{return H.apply(d,w.querySelectorAll(x)),d}catch(y){}finally{r||b.removeAttribute("id")}}}return i(a.replace(R,"$1"),b,d,e)}function ha(){var a=[];function b(c,e){return a.push(c+" ")>d.cacheLength&&delete b[a.shift()],b[c+" "]=e}return b}function ia(a){return a[u]=!0,a}function ja(a){var b=n.createElement("div");try{return!!a(b)}catch(c){return!1}finally{b.parentNode&&b.parentNode.removeChild(b),b=null}}function ka(a,b){var c=a.split("|"),e=a.length;while(e--)d.attrHandle[c[e]]=b}function la(a,b){var c=b&&a,d=c&&1===a.nodeType&&1===b.nodeType&&(~b.sourceIndex||C)-(~a.sourceIndex||C);if(d)return d;if(c)while(c=c.nextSibling)if(c===b)return-1;return a?1:-1}function ma(a){return function(b){var c=b.nodeName.toLowerCase();return"input"===c&&b.type===a}}function na(a){return function(b){var c=b.nodeName.toLowerCase();return("input"===c||"button"===c)&&b.type===a}}function oa(a){return ia(function(b){return b=+b,ia(function(c,d){var e,f=a([],c.length,b),g=f.length;while(g--)c[e=f[g]]&&(c[e]=!(d[e]=c[e]))})})}function pa(a){return a&&"undefined"!=typeof a.getElementsByTagName&&a}c=ga.support={},f=ga.isXML=function(a){var b=a&&(a.ownerDocument||a).documentElement;return b?"HTML"!==b.nodeName:!1},m=ga.setDocument=function(a){var b,e,g=a?a.ownerDocument||a:v;return g!==n&&9===g.nodeType&&g.documentElement?(n=g,o=g.documentElement,e=g.defaultView,e&&e!==e.top&&(e.addEventListener?e.addEventListener("unload",ea,!1):e.attachEvent&&e.attachEvent("onunload",ea)),p=!f(g),c.attributes=ja(function(a){return a.className="i",!a.getAttribute("className")}),c.getElementsByTagName=ja(function(a){return a.appendChild(g.createComment("")),!a.getElementsByTagName("*").length}),c.getElementsByClassName=$.test(g.getElementsByClassName),c.getById=ja(function(a){return o.appendChild(a).id=u,!g.getElementsByName||!g.getElementsByName(u).length}),c.getById?(d.find.ID=function(a,b){if("undefined"!=typeof b.getElementById&&p){var c=b.getElementById(a);return c&&c.parentNode?[c]:[]}},d.filter.ID=function(a){var b=a.replace(ca,da);return function(a){return a.getAttribute("id")===b}}):(delete d.find.ID,d.filter.ID=function(a){var b=a.replace(ca,da);return function(a){var c="undefined"!=typeof a.getAttributeNode&&a.getAttributeNode("id");return c&&c.value===b}}),d.find.TAG=c.getElementsByTagName?function(a,b){return"undefined"!=typeof b.getElementsByTagName?b.getElementsByTagName(a):c.qsa?b.querySelectorAll(a):void 0}:function(a,b){var c,d=[],e=0,f=b.getElementsByTagName(a);if("*"===a){while(c=f[e++])1===c.nodeType&&d.push(c);return d}return f},d.find.CLASS=c.getElementsByClassName&&function(a,b){return p?b.getElementsByClassName(a):void 0},r=[],q=[],(c.qsa=$.test(g.querySelectorAll))&&(ja(function(a){o.appendChild(a).innerHTML="<a id='"+u+"'></a><select id='"+u+"-\f]' msallowcapture=''><option selected=''></option></select>",a.querySelectorAll("[msallowcapture^='']").length&&q.push("[*^$]="+L+"*(?:''|\"\")"),a.querySelectorAll("[selected]").length||q.push("\\["+L+"*(?:value|"+K+")"),a.querySelectorAll("[id~="+u+"-]").length||q.push("~="),a.querySelectorAll(":checked").length||q.push(":checked"),a.querySelectorAll("a#"+u+"+*").length||q.push(".#.+[+~]")}),ja(function(a){var b=g.createElement("input");b.setAttribute("type","hidden"),a.appendChild(b).setAttribute("name","D"),a.querySelectorAll("[name=d]").length&&q.push("name"+L+"*[*^$|!~]?="),a.querySelectorAll(":enabled").length||q.push(":enabled",":disabled"),a.querySelectorAll("*,:x"),q.push(",.*:")})),(c.matchesSelector=$.test(s=o.matches||o.webkitMatchesSelector||o.mozMatchesSelector||o.oMatchesSelector||o.msMatchesSelector))&&ja(function(a){c.disconnectedMatch=s.call(a,"div"),s.call(a,"[s!='']:x"),r.push("!=",P)}),q=q.length&&new RegExp(q.join("|")),r=r.length&&new RegExp(r.join("|")),b=$.test(o.compareDocumentPosition),t=b||$.test(o.contains)?function(a,b){var c=9===a.nodeType?a.documentElement:a,d=b&&b.parentNode;return a===d||!(!d||1!==d.nodeType||!(c.contains?c.contains(d):a.compareDocumentPosition&&16&a.compareDocumentPosition(d)))}:function(a,b){if(b)while(b=b.parentNode)if(b===a)return!0;return!1},B=b?function(a,b){if(a===b)return l=!0,0;var d=!a.compareDocumentPosition-!b.compareDocumentPosition;return d?d:(d=(a.ownerDocument||a)===(b.ownerDocument||b)?a.compareDocumentPosition(b):1,1&d||!c.sortDetached&&b.compareDocumentPosition(a)===d?a===g||a.ownerDocument===v&&t(v,a)?-1:b===g||b.ownerDocument===v&&t(v,b)?1:k?J(k,a)-J(k,b):0:4&d?-1:1)}:function(a,b){if(a===b)return l=!0,0;var c,d=0,e=a.parentNode,f=b.parentNode,h=[a],i=[b];if(!e||!f)return a===g?-1:b===g?1:e?-1:f?1:k?J(k,a)-J(k,b):0;if(e===f)return la(a,b);c=a;while(c=c.parentNode)h.unshift(c);c=b;while(c=c.parentNode)i.unshift(c);while(h[d]===i[d])d++;return d?la(h[d],i[d]):h[d]===v?-1:i[d]===v?1:0},g):n},ga.matches=function(a,b){return ga(a,null,null,b)},ga.matchesSelector=function(a,b){if((a.ownerDocument||a)!==n&&m(a),b=b.replace(U,"='$1']"),!(!c.matchesSelector||!p||r&&r.test(b)||q&&q.test(b)))try{var d=s.call(a,b);if(d||c.disconnectedMatch||a.document&&11!==a.document.nodeType)return d}catch(e){}return ga(b,n,null,[a]).length>0},ga.contains=function(a,b){return(a.ownerDocument||a)!==n&&m(a),t(a,b)},ga.attr=function(a,b){(a.ownerDocument||a)!==n&&m(a);var e=d.attrHandle[b.toLowerCase()],f=e&&D.call(d.attrHandle,b.toLowerCase())?e(a,b,!p):void 0;return void 0!==f?f:c.attributes||!p?a.getAttribute(b):(f=a.getAttributeNode(b))&&f.specified?f.value:null},ga.error=function(a){throw new Error("Syntax error, unrecognized expression: "+a)},ga.uniqueSort=function(a){var b,d=[],e=0,f=0;if(l=!c.detectDuplicates,k=!c.sortStable&&a.slice(0),a.sort(B),l){while(b=a[f++])b===a[f]&&(e=d.push(f));while(e--)a.splice(d[e],1)}return k=null,a},e=ga.getText=function(a){var b,c="",d=0,f=a.nodeType;if(f){if(1===f||9===f||11===f){if("string"==typeof a.textContent)return a.textContent;for(a=a.firstChild;a;a=a.nextSibling)c+=e(a)}else if(3===f||4===f)return a.nodeValue}else while(b=a[d++])c+=e(b);return c},d=ga.selectors={cacheLength:50,createPseudo:ia,match:X,attrHandle:{},find:{},relative:{">":{dir:"parentNode",first:!0}," ":{dir:"parentNode"},"+":{dir:"previousSibling",first:!0},"~":{dir:"previousSibling"}},preFilter:{ATTR:function(a){return a[1]=a[1].replace(ca,da),a[3]=(a[3]||a[4]||a[5]||"").replace(ca,da),"~="===a[2]&&(a[3]=" "+a[3]+" "),a.slice(0,4)},CHILD:function(a){return a[1]=a[1].toLowerCase(),"nth"===a[1].slice(0,3)?(a[3]||ga.error(a[0]),a[4]=+(a[4]?a[5]+(a[6]||1):2*("even"===a[3]||"odd"===a[3])),a[5]=+(a[7]+a[8]||"odd"===a[3])):a[3]&&ga.error(a[0]),a},PSEUDO:function(a){var b,c=!a[6]&&a[2];return X.CHILD.test(a[0])?null:(a[3]?a[2]=a[4]||a[5]||"":c&&V.test(c)&&(b=g(c,!0))&&(b=c.indexOf(")",c.length-b)-c.length)&&(a[0]=a[0].slice(0,b),a[2]=c.slice(0,b)),a.slice(0,3))}},filter:{TAG:function(a){var b=a.replace(ca,da).toLowerCase();return"*"===a?function(){return!0}:function(a){return a.nodeName&&a.nodeName.toLowerCase()===b}},CLASS:function(a){var b=y[a+" "];return b||(b=new RegExp("(^|"+L+")"+a+"("+L+"|$)"))&&y(a,function(a){return b.test("string"==typeof a.className&&a.className||"undefined"!=typeof a.getAttribute&&a.getAttribute("class")||"")})},ATTR:function(a,b,c){return function(d){var e=ga.attr(d,a);return null==e?"!="===b:b?(e+="","="===b?e===c:"!="===b?e!==c:"^="===b?c&&0===e.indexOf(c):"*="===b?c&&e.indexOf(c)>-1:"$="===b?c&&e.slice(-c.length)===c:"~="===b?(" "+e.replace(Q," ")+" ").indexOf(c)>-1:"|="===b?e===c||e.slice(0,c.length+1)===c+"-":!1):!0}},CHILD:function(a,b,c,d,e){var f="nth"!==a.slice(0,3),g="last"!==a.slice(-4),h="of-type"===b;return 1===d&&0===e?function(a){return!!a.parentNode}:function(b,c,i){var j,k,l,m,n,o,p=f!==g?"nextSibling":"previousSibling",q=b.parentNode,r=h&&b.nodeName.toLowerCase(),s=!i&&!h;if(q){if(f){while(p){l=b;while(l=l[p])if(h?l.nodeName.toLowerCase()===r:1===l.nodeType)return!1;o=p="only"===a&&!o&&"nextSibling"}return!0}if(o=[g?q.firstChild:q.lastChild],g&&s){k=q[u]||(q[u]={}),j=k[a]||[],n=j[0]===w&&j[1],m=j[0]===w&&j[2],l=n&&q.childNodes[n];while(l=++n&&l&&l[p]||(m=n=0)||o.pop())if(1===l.nodeType&&++m&&l===b){k[a]=[w,n,m];break}}else if(s&&(j=(b[u]||(b[u]={}))[a])&&j[0]===w)m=j[1];else while(l=++n&&l&&l[p]||(m=n=0)||o.pop())if((h?l.nodeName.toLowerCase()===r:1===l.nodeType)&&++m&&(s&&((l[u]||(l[u]={}))[a]=[w,m]),l===b))break;return m-=e,m===d||m%d===0&&m/d>=0}}},PSEUDO:function(a,b){var c,e=d.pseudos[a]||d.setFilters[a.toLowerCase()]||ga.error("unsupported pseudo: "+a);return e[u]?e(b):e.length>1?(c=[a,a,"",b],d.setFilters.hasOwnProperty(a.toLowerCase())?ia(function(a,c){var d,f=e(a,b),g=f.length;while(g--)d=J(a,f[g]),a[d]=!(c[d]=f[g])}):function(a){return e(a,0,c)}):e}},pseudos:{not:ia(function(a){var b=[],c=[],d=h(a.replace(R,"$1"));return d[u]?ia(function(a,b,c,e){var f,g=d(a,null,e,[]),h=a.length;while(h--)(f=g[h])&&(a[h]=!(b[h]=f))}):function(a,e,f){return b[0]=a,d(b,null,f,c),b[0]=null,!c.pop()}}),has:ia(function(a){return function(b){return ga(a,b).length>0}}),contains:ia(function(a){return a=a.replace(ca,da),function(b){return(b.textContent||b.innerText||e(b)).indexOf(a)>-1}}),lang:ia(function(a){return W.test(a||"")||ga.error("unsupported lang: "+a),a=a.replace(ca,da).toLowerCase(),function(b){var c;do if(c=p?b.lang:b.getAttribute("xml:lang")||b.getAttribute("lang"))return c=c.toLowerCase(),c===a||0===c.indexOf(a+"-");while((b=b.parentNode)&&1===b.nodeType);return!1}}),target:function(b){var c=a.location&&a.location.hash;return c&&c.slice(1)===b.id},root:function(a){return a===o},focus:function(a){return a===n.activeElement&&(!n.hasFocus||n.hasFocus())&&!!(a.type||a.href||~a.tabIndex)},enabled:function(a){return a.disabled===!1},disabled:function(a){return a.disabled===!0},checked:function(a){var b=a.nodeName.toLowerCase();return"input"===b&&!!a.checked||"option"===b&&!!a.selected},selected:function(a){return a.parentNode&&a.parentNode.selectedIndex,a.selected===!0},empty:function(a){for(a=a.firstChild;a;a=a.nextSibling)if(a.nodeType<6)return!1;return!0},parent:function(a){return!d.pseudos.empty(a)},header:function(a){return Z.test(a.nodeName)},input:function(a){return Y.test(a.nodeName)},button:function(a){var b=a.nodeName.toLowerCase();return"input"===b&&"button"===a.type||"button"===b},text:function(a){var b;return"input"===a.nodeName.toLowerCase()&&"text"===a.type&&(null==(b=a.getAttribute("type"))||"text"===b.toLowerCase())},first:oa(function(){return[0]}),last:oa(function(a,b){return[b-1]}),eq:oa(function(a,b,c){return[0>c?c+b:c]}),even:oa(function(a,b){for(var c=0;b>c;c+=2)a.push(c);return a}),odd:oa(function(a,b){for(var c=1;b>c;c+=2)a.push(c);return a}),lt:oa(function(a,b,c){for(var d=0>c?c+b:c;--d>=0;)a.push(d);return a}),gt:oa(function(a,b,c){for(var d=0>c?c+b:c;++d<b;)a.push(d);return a})}},d.pseudos.nth=d.pseudos.eq;for(b in{radio:!0,checkbox:!0,file:!0,password:!0,image:!0})d.pseudos[b]=ma(b);for(b in{submit:!0,reset:!0})d.pseudos[b]=na(b);function qa(){}qa.prototype=d.filters=d.pseudos,d.setFilters=new qa,g=ga.tokenize=function(a,b){var c,e,f,g,h,i,j,k=z[a+" "];if(k)return b?0:k.slice(0);h=a,i=[],j=d.preFilter;while(h){(!c||(e=S.exec(h)))&&(e&&(h=h.slice(e[0].length)||h),i.push(f=[])),c=!1,(e=T.exec(h))&&(c=e.shift(),f.push({value:c,type:e[0].replace(R," ")}),h=h.slice(c.length));for(g in d.filter)!(e=X[g].exec(h))||j[g]&&!(e=j[g](e))||(c=e.shift(),f.push({value:c,type:g,matches:e}),h=h.slice(c.length));if(!c)break}return b?h.length:h?ga.error(a):z(a,i).slice(0)};function ra(a){for(var b=0,c=a.length,d="";c>b;b++)d+=a[b].value;return d}function sa(a,b,c){var d=b.dir,e=c&&"parentNode"===d,f=x++;return b.first?function(b,c,f){while(b=b[d])if(1===b.nodeType||e)return a(b,c,f)}:function(b,c,g){var h,i,j=[w,f];if(g){while(b=b[d])if((1===b.nodeType||e)&&a(b,c,g))return!0}else while(b=b[d])if(1===b.nodeType||e){if(i=b[u]||(b[u]={}),(h=i[d])&&h[0]===w&&h[1]===f)return j[2]=h[2];if(i[d]=j,j[2]=a(b,c,g))return!0}}}function ta(a){return a.length>1?function(b,c,d){var e=a.length;while(e--)if(!a[e](b,c,d))return!1;return!0}:a[0]}function ua(a,b,c){for(var d=0,e=b.length;e>d;d++)ga(a,b[d],c);return c}function va(a,b,c,d,e){for(var f,g=[],h=0,i=a.length,j=null!=b;i>h;h++)(f=a[h])&&(!c||c(f,d,e))&&(g.push(f),j&&b.push(h));return g}function wa(a,b,c,d,e,f){return d&&!d[u]&&(d=wa(d)),e&&!e[u]&&(e=wa(e,f)),ia(function(f,g,h,i){var j,k,l,m=[],n=[],o=g.length,p=f||ua(b||"*",h.nodeType?[h]:h,[]),q=!a||!f&&b?p:va(p,m,a,h,i),r=c?e||(f?a:o||d)?[]:g:q;if(c&&c(q,r,h,i),d){j=va(r,n),d(j,[],h,i),k=j.length;while(k--)(l=j[k])&&(r[n[k]]=!(q[n[k]]=l))}if(f){if(e||a){if(e){j=[],k=r.length;while(k--)(l=r[k])&&j.push(q[k]=l);e(null,r=[],j,i)}k=r.length;while(k--)(l=r[k])&&(j=e?J(f,l):m[k])>-1&&(f[j]=!(g[j]=l))}}else r=va(r===g?r.splice(o,r.length):r),e?e(null,g,r,i):H.apply(g,r)})}function xa(a){for(var b,c,e,f=a.length,g=d.relative[a[0].type],h=g||d.relative[" "],i=g?1:0,k=sa(function(a){return a===b},h,!0),l=sa(function(a){return J(b,a)>-1},h,!0),m=[function(a,c,d){var e=!g&&(d||c!==j)||((b=c).nodeType?k(a,c,d):l(a,c,d));return b=null,e}];f>i;i++)if(c=d.relative[a[i].type])m=[sa(ta(m),c)];else{if(c=d.filter[a[i].type].apply(null,a[i].matches),c[u]){for(e=++i;f>e;e++)if(d.relative[a[e].type])break;return wa(i>1&&ta(m),i>1&&ra(a.slice(0,i-1).concat({value:" "===a[i-2].type?"*":""})).replace(R,"$1"),c,e>i&&xa(a.slice(i,e)),f>e&&xa(a=a.slice(e)),f>e&&ra(a))}m.push(c)}return ta(m)}function ya(a,b){var c=b.length>0,e=a.length>0,f=function(f,g,h,i,k){var l,m,o,p=0,q="0",r=f&&[],s=[],t=j,u=f||e&&d.find.TAG("*",k),v=w+=null==t?1:Math.random()||.1,x=u.length;for(k&&(j=g!==n&&g);q!==x&&null!=(l=u[q]);q++){if(e&&l){m=0;while(o=a[m++])if(o(l,g,h)){i.push(l);break}k&&(w=v)}c&&((l=!o&&l)&&p--,f&&r.push(l))}if(p+=q,c&&q!==p){m=0;while(o=b[m++])o(r,s,g,h);if(f){if(p>0)while(q--)r[q]||s[q]||(s[q]=F.call(i));s=va(s)}H.apply(i,s),k&&!f&&s.length>0&&p+b.length>1&&ga.uniqueSort(i)}return k&&(w=v,j=t),r};return c?ia(f):f}return h=ga.compile=function(a,b){var c,d=[],e=[],f=A[a+" "];if(!f){b||(b=g(a)),c=b.length;while(c--)f=xa(b[c]),f[u]?d.push(f):e.push(f);f=A(a,ya(e,d)),f.selector=a}return f},i=ga.select=function(a,b,e,f){var i,j,k,l,m,n="function"==typeof a&&a,o=!f&&g(a=n.selector||a);if(e=e||[],1===o.length){if(j=o[0]=o[0].slice(0),j.length>2&&"ID"===(k=j[0]).type&&c.getById&&9===b.nodeType&&p&&d.relative[j[1].type]){if(b=(d.find.ID(k.matches[0].replace(ca,da),b)||[])[0],!b)return e;n&&(b=b.parentNode),a=a.slice(j.shift().value.length)}i=X.needsContext.test(a)?0:j.length;while(i--){if(k=j[i],d.relative[l=k.type])break;if((m=d.find[l])&&(f=m(k.matches[0].replace(ca,da),aa.test(j[0].type)&&pa(b.parentNode)||b))){if(j.splice(i,1),a=f.length&&ra(j),!a)return H.apply(e,f),e;break}}}return(n||h(a,o))(f,b,!p,e,aa.test(a)&&pa(b.parentNode)||b),e},c.sortStable=u.split("").sort(B).join("")===u,c.detectDuplicates=!!l,m(),c.sortDetached=ja(function(a){return 1&a.compareDocumentPosition(n.createElement("div"))}),ja(function(a){return a.innerHTML="<a href='#'></a>","#"===a.firstChild.getAttribute("href")})||ka("type|href|height|width",function(a,b,c){return c?void 0:a.getAttribute(b,"type"===b.toLowerCase()?1:2)}),c.attributes&&ja(function(a){return a.innerHTML="<input/>",a.firstChild.setAttribute("value",""),""===a.firstChild.getAttribute("value")})||ka("value",function(a,b,c){return c||"input"!==a.nodeName.toLowerCase()?void 0:a.defaultValue}),ja(function(a){return null==a.getAttribute("disabled")})||ka(K,function(a,b,c){var d;return c?void 0:a[b]===!0?b.toLowerCase():(d=a.getAttributeNode(b))&&d.specified?d.value:null}),ga}(a);m.find=s,m.expr=s.selectors,m.expr[":"]=m.expr.pseudos,m.unique=s.uniqueSort,m.text=s.getText,m.isXMLDoc=s.isXML,m.contains=s.contains;var t=m.expr.match.needsContext,u=/^<(\w+)\s*\/?>(?:<\/\1>|)$/,v=/^.[^:#\[\.,]*$/;function w(a,b,c){if(m.isFunction(b))return m.grep(a,function(a,d){return!!b.call(a,d,a)!==c});if(b.nodeType)return m.grep(a,function(a){return a===b!==c});if("string"==typeof b){if(v.test(b))return m.filter(b,a,c);b=m.filter(b,a)}return m.grep(a,function(a){return m.inArray(a,b)>=0!==c})}m.filter=function(a,b,c){var d=b[0];return c&&(a=":not("+a+")"),1===b.length&&1===d.nodeType?m.find.matchesSelector(d,a)?[d]:[]:m.find.matches(a,m.grep(b,function(a){return 1===a.nodeType}))},m.fn.extend({find:function(a){var b,c=[],d=this,e=d.length;if("string"!=typeof a)return this.pushStack(m(a).filter(function(){for(b=0;e>b;b++)if(m.contains(d[b],this))return!0}));for(b=0;e>b;b++)m.find(a,d[b],c);return c=this.pushStack(e>1?m.unique(c):c),c.selector=this.selector?this.selector+" "+a:a,c},filter:function(a){return this.pushStack(w(this,a||[],!1))},not:function(a){return this.pushStack(w(this,a||[],!0))},is:function(a){return!!w(this,"string"==typeof a&&t.test(a)?m(a):a||[],!1).length}});var x,y=a.document,z=/^(?:\s*(<[\w\W]+>)[^>]*|#([\w-]*))$/,A=m.fn.init=function(a,b){var c,d;if(!a)return this;if("string"==typeof a){if(c="<"===a.charAt(0)&&">"===a.charAt(a.length-1)&&a.length>=3?[null,a,null]:z.exec(a),!c||!c[1]&&b)return!b||b.jquery?(b||x).find(a):this.constructor(b).find(a);if(c[1]){if(b=b instanceof m?b[0]:b,m.merge(this,m.parseHTML(c[1],b&&b.nodeType?b.ownerDocument||b:y,!0)),u.test(c[1])&&m.isPlainObject(b))for(c in b)m.isFunction(this[c])?this[c](b[c]):this.attr(c,b[c]);return this}if(d=y.getElementById(c[2]),d&&d.parentNode){if(d.id!==c[2])return x.find(a);this.length=1,this[0]=d}return this.context=y,this.selector=a,this}return a.nodeType?(this.context=this[0]=a,this.length=1,this):m.isFunction(a)?"undefined"!=typeof x.ready?x.ready(a):a(m):(void 0!==a.selector&&(this.selector=a.selector,this.context=a.context),m.makeArray(a,this))};A.prototype=m.fn,x=m(y);var B=/^(?:parents|prev(?:Until|All))/,C={children:!0,contents:!0,next:!0,prev:!0};m.extend({dir:function(a,b,c){var d=[],e=a[b];while(e&&9!==e.nodeType&&(void 0===c||1!==e.nodeType||!m(e).is(c)))1===e.nodeType&&d.push(e),e=e[b];return d},sibling:function(a,b){for(var c=[];a;a=a.nextSibling)1===a.nodeType&&a!==b&&c.push(a);return c}}),m.fn.extend({has:function(a){var b,c=m(a,this),d=c.length;return this.filter(function(){for(b=0;d>b;b++)if(m.contains(this,c[b]))return!0})},closest:function(a,b){for(var c,d=0,e=this.length,f=[],g=t.test(a)||"string"!=typeof a?m(a,b||this.context):0;e>d;d++)for(c=this[d];c&&c!==b;c=c.parentNode)if(c.nodeType<11&&(g?g.index(c)>-1:1===c.nodeType&&m.find.matchesSelector(c,a))){f.push(c);break}return this.pushStack(f.length>1?m.unique(f):f)},index:function(a){return a?"string"==typeof a?m.inArray(this[0],m(a)):m.inArray(a.jquery?a[0]:a,this):this[0]&&this[0].parentNode?this.first().prevAll().length:-1},add:function(a,b){return this.pushStack(m.unique(m.merge(this.get(),m(a,b))))},addBack:function(a){return this.add(null==a?this.prevObject:this.prevObject.filter(a))}});function D(a,b){do a=a[b];while(a&&1!==a.nodeType);return a}m.each({parent:function(a){var b=a.parentNode;return b&&11!==b.nodeType?b:null},parents:function(a){return m.dir(a,"parentNode")},parentsUntil:function(a,b,c){return m.dir(a,"parentNode",c)},next:function(a){return D(a,"nextSibling")},prev:function(a){return D(a,"previousSibling")},nextAll:function(a){return m.dir(a,"nextSibling")},prevAll:function(a){return m.dir(a,"previousSibling")},nextUntil:function(a,b,c){return m.dir(a,"nextSibling",c)},prevUntil:function(a,b,c){return m.dir(a,"previousSibling",c)},siblings:function(a){return m.sibling((a.parentNode||{}).firstChild,a)},children:function(a){return m.sibling(a.firstChild)},contents:function(a){return m.nodeName(a,"iframe")?a.contentDocument||a.contentWindow.document:m.merge([],a.childNodes)}},function(a,b){m.fn[a]=function(c,d){var e=m.map(this,b,c);return"Until"!==a.slice(-5)&&(d=c),d&&"string"==typeof d&&(e=m.filter(d,e)),this.length>1&&(C[a]||(e=m.unique(e)),B.test(a)&&(e=e.reverse())),this.pushStack(e)}});var E=/\S+/g,F={};function G(a){var b=F[a]={};return m.each(a.match(E)||[],function(a,c){b[c]=!0}),b}m.Callbacks=function(a){a="string"==typeof a?F[a]||G(a):m.extend({},a);var b,c,d,e,f,g,h=[],i=!a.once&&[],j=function(l){for(c=a.memory&&l,d=!0,f=g||0,g=0,e=h.length,b=!0;h&&e>f;f++)if(h[f].apply(l[0],l[1])===!1&&a.stopOnFalse){c=!1;break}b=!1,h&&(i?i.length&&j(i.shift()):c?h=[]:k.disable())},k={add:function(){if(h){var d=h.length;!function f(b){m.each(b,function(b,c){var d=m.type(c);"function"===d?a.unique&&k.has(c)||h.push(c):c&&c.length&&"string"!==d&&f(c)})}(arguments),b?e=h.length:c&&(g=d,j(c))}return this},remove:function(){return h&&m.each(arguments,function(a,c){var d;while((d=m.inArray(c,h,d))>-1)h.splice(d,1),b&&(e>=d&&e--,f>=d&&f--)}),this},has:function(a){return a?m.inArray(a,h)>-1:!(!h||!h.length)},empty:function(){return h=[],e=0,this},disable:function(){return h=i=c=void 0,this},disabled:function(){return!h},lock:function(){return i=void 0,c||k.disable(),this},locked:function(){return!i},fireWith:function(a,c){return!h||d&&!i||(c=c||[],c=[a,c.slice?c.slice():c],b?i.push(c):j(c)),this},fire:function(){return k.fireWith(this,arguments),this},fired:function(){return!!d}};return k},m.extend({Deferred:function(a){var b=[["resolve","done",m.Callbacks("once memory"),"resolved"],["reject","fail",m.Callbacks("once memory"),"rejected"],["notify","progress",m.Callbacks("memory")]],c="pending",d={state:function(){return c},always:function(){return e.done(arguments).fail(arguments),this},then:function(){var a=arguments;return m.Deferred(function(c){m.each(b,function(b,f){var g=m.isFunction(a[b])&&a[b];e[f[1]](function(){var a=g&&g.apply(this,arguments);a&&m.isFunction(a.promise)?a.promise().done(c.resolve).fail(c.reject).progress(c.notify):c[f[0]+"With"](this===d?c.promise():this,g?[a]:arguments)})}),a=null}).promise()},promise:function(a){return null!=a?m.extend(a,d):d}},e={};return d.pipe=d.then,m.each(b,function(a,f){var g=f[2],h=f[3];d[f[1]]=g.add,h&&g.add(function(){c=h},b[1^a][2].disable,b[2][2].lock),e[f[0]]=function(){return e[f[0]+"With"](this===e?d:this,arguments),this},e[f[0]+"With"]=g.fireWith}),d.promise(e),a&&a.call(e,e),e},when:function(a){var b=0,c=d.call(arguments),e=c.length,f=1!==e||a&&m.isFunction(a.promise)?e:0,g=1===f?a:m.Deferred(),h=function(a,b,c){return function(e){b[a]=this,c[a]=arguments.length>1?d.call(arguments):e,c===i?g.notifyWith(b,c):--f||g.resolveWith(b,c)}},i,j,k;if(e>1)for(i=new Array(e),j=new Array(e),k=new Array(e);e>b;b++)c[b]&&m.isFunction(c[b].promise)?c[b].promise().done(h(b,k,c)).fail(g.reject).progress(h(b,j,i)):--f;return f||g.resolveWith(k,c),g.promise()}});var H;m.fn.ready=function(a){return m.ready.promise().done(a),this},m.extend({isReady:!1,readyWait:1,holdReady:function(a){a?m.readyWait++:m.ready(!0)},ready:function(a){if(a===!0?!--m.readyWait:!m.isReady){if(!y.body)return setTimeout(m.ready);m.isReady=!0,a!==!0&&--m.readyWait>0||(H.resolveWith(y,[m]),m.fn.triggerHandler&&(m(y).triggerHandler("ready"),m(y).off("ready")))}}});function I(){y.addEventListener?(y.removeEventListener("DOMContentLoaded",J,!1),a.removeEventListener("load",J,!1)):(y.detachEvent("onreadystatechange",J),a.detachEvent("onload",J))}function J(){(y.addEventListener||"load"===event.type||"complete"===y.readyState)&&(I(),m.ready())}m.ready.promise=function(b){if(!H)if(H=m.Deferred(),"complete"===y.readyState)setTimeout(m.ready);else if(y.addEventListener)y.addEventListener("DOMContentLoaded",J,!1),a.addEventListener("load",J,!1);else{y.attachEvent("onreadystatechange",J),a.attachEvent("onload",J);var c=!1;try{c=null==a.frameElement&&y.documentElement}catch(d){}c&&c.doScroll&&!function e(){if(!m.isReady){try{c.doScroll("left")}catch(a){return setTimeout(e,50)}I(),m.ready()}}()}return H.promise(b)};var K="undefined",L;for(L in m(k))break;k.ownLast="0"!==L,k.inlineBlockNeedsLayout=!1,m(function(){var a,b,c,d;c=y.getElementsByTagName("body")[0],c&&c.style&&(b=y.createElement("div"),d=y.createElement("div"),d.style.cssText="position:absolute;border:0;width:0;height:0;top:0;left:-9999px",c.appendChild(d).appendChild(b),typeof b.style.zoom!==K&&(b.style.cssText="display:inline;margin:0;border:0;padding:1px;width:1px;zoom:1",k.inlineBlockNeedsLayout=a=3===b.offsetWidth,a&&(c.style.zoom=1)),c.removeChild(d))}),function(){var a=y.createElement("div");if(null==k.deleteExpando){k.deleteExpando=!0;try{delete a.test}catch(b){k.deleteExpando=!1}}a=null}(),m.acceptData=function(a){var b=m.noData[(a.nodeName+" ").toLowerCase()],c=+a.nodeType||1;return 1!==c&&9!==c?!1:!b||b!==!0&&a.getAttribute("classid")===b};var M=/^(?:\{[\w\W]*\}|\[[\w\W]*\])$/,N=/([A-Z])/g;function O(a,b,c){if(void 0===c&&1===a.nodeType){var d="data-"+b.replace(N,"-$1").toLowerCase();if(c=a.getAttribute(d),"string"==typeof c){try{c="true"===c?!0:"false"===c?!1:"null"===c?null:+c+""===c?+c:M.test(c)?m.parseJSON(c):c}catch(e){}m.data(a,b,c)}else c=void 0}return c}function P(a){var b;for(b in a)if(("data"!==b||!m.isEmptyObject(a[b]))&&"toJSON"!==b)return!1;
+
+return!0}function Q(a,b,d,e){if(m.acceptData(a)){var f,g,h=m.expando,i=a.nodeType,j=i?m.cache:a,k=i?a[h]:a[h]&&h;if(k&&j[k]&&(e||j[k].data)||void 0!==d||"string"!=typeof b)return k||(k=i?a[h]=c.pop()||m.guid++:h),j[k]||(j[k]=i?{}:{toJSON:m.noop}),("object"==typeof b||"function"==typeof b)&&(e?j[k]=m.extend(j[k],b):j[k].data=m.extend(j[k].data,b)),g=j[k],e||(g.data||(g.data={}),g=g.data),void 0!==d&&(g[m.camelCase(b)]=d),"string"==typeof b?(f=g[b],null==f&&(f=g[m.camelCase(b)])):f=g,f}}function R(a,b,c){if(m.acceptData(a)){var d,e,f=a.nodeType,g=f?m.cache:a,h=f?a[m.expando]:m.expando;if(g[h]){if(b&&(d=c?g[h]:g[h].data)){m.isArray(b)?b=b.concat(m.map(b,m.camelCase)):b in d?b=[b]:(b=m.camelCase(b),b=b in d?[b]:b.split(" ")),e=b.length;while(e--)delete d[b[e]];if(c?!P(d):!m.isEmptyObject(d))return}(c||(delete g[h].data,P(g[h])))&&(f?m.cleanData([a],!0):k.deleteExpando||g!=g.window?delete g[h]:g[h]=null)}}}m.extend({cache:{},noData:{"applet ":!0,"embed ":!0,"object ":"clsid:D27CDB6E-AE6D-11cf-96B8-444553540000"},hasData:function(a){return a=a.nodeType?m.cache[a[m.expando]]:a[m.expando],!!a&&!P(a)},data:function(a,b,c){return Q(a,b,c)},removeData:function(a,b){return R(a,b)},_data:function(a,b,c){return Q(a,b,c,!0)},_removeData:function(a,b){return R(a,b,!0)}}),m.fn.extend({data:function(a,b){var c,d,e,f=this[0],g=f&&f.attributes;if(void 0===a){if(this.length&&(e=m.data(f),1===f.nodeType&&!m._data(f,"parsedAttrs"))){c=g.length;while(c--)g[c]&&(d=g[c].name,0===d.indexOf("data-")&&(d=m.camelCase(d.slice(5)),O(f,d,e[d])));m._data(f,"parsedAttrs",!0)}return e}return"object"==typeof a?this.each(function(){m.data(this,a)}):arguments.length>1?this.each(function(){m.data(this,a,b)}):f?O(f,a,m.data(f,a)):void 0},removeData:function(a){return this.each(function(){m.removeData(this,a)})}}),m.extend({queue:function(a,b,c){var d;return a?(b=(b||"fx")+"queue",d=m._data(a,b),c&&(!d||m.isArray(c)?d=m._data(a,b,m.makeArray(c)):d.push(c)),d||[]):void 0},dequeue:function(a,b){b=b||"fx";var c=m.queue(a,b),d=c.length,e=c.shift(),f=m._queueHooks(a,b),g=function(){m.dequeue(a,b)};"inprogress"===e&&(e=c.shift(),d--),e&&("fx"===b&&c.unshift("inprogress"),delete f.stop,e.call(a,g,f)),!d&&f&&f.empty.fire()},_queueHooks:function(a,b){var c=b+"queueHooks";return m._data(a,c)||m._data(a,c,{empty:m.Callbacks("once memory").add(function(){m._removeData(a,b+"queue"),m._removeData(a,c)})})}}),m.fn.extend({queue:function(a,b){var c=2;return"string"!=typeof a&&(b=a,a="fx",c--),arguments.length<c?m.queue(this[0],a):void 0===b?this:this.each(function(){var c=m.queue(this,a,b);m._queueHooks(this,a),"fx"===a&&"inprogress"!==c[0]&&m.dequeue(this,a)})},dequeue:function(a){return this.each(function(){m.dequeue(this,a)})},clearQueue:function(a){return this.queue(a||"fx",[])},promise:function(a,b){var c,d=1,e=m.Deferred(),f=this,g=this.length,h=function(){--d||e.resolveWith(f,[f])};"string"!=typeof a&&(b=a,a=void 0),a=a||"fx";while(g--)c=m._data(f[g],a+"queueHooks"),c&&c.empty&&(d++,c.empty.add(h));return h(),e.promise(b)}});var S=/[+-]?(?:\d*\.|)\d+(?:[eE][+-]?\d+|)/.source,T=["Top","Right","Bottom","Left"],U=function(a,b){return a=b||a,"none"===m.css(a,"display")||!m.contains(a.ownerDocument,a)},V=m.access=function(a,b,c,d,e,f,g){var h=0,i=a.length,j=null==c;if("object"===m.type(c)){e=!0;for(h in c)m.access(a,b,h,c[h],!0,f,g)}else if(void 0!==d&&(e=!0,m.isFunction(d)||(g=!0),j&&(g?(b.call(a,d),b=null):(j=b,b=function(a,b,c){return j.call(m(a),c)})),b))for(;i>h;h++)b(a[h],c,g?d:d.call(a[h],h,b(a[h],c)));return e?a:j?b.call(a):i?b(a[0],c):f},W=/^(?:checkbox|radio)$/i;!function(){var a=y.createElement("input"),b=y.createElement("div"),c=y.createDocumentFragment();if(b.innerHTML="  <link/><table></table><a href='/a'>a</a><input type='checkbox'/>",k.leadingWhitespace=3===b.firstChild.nodeType,k.tbody=!b.getElementsByTagName("tbody").length,k.htmlSerialize=!!b.getElementsByTagName("link").length,k.html5Clone="<:nav></:nav>"!==y.createElement("nav").cloneNode(!0).outerHTML,a.type="checkbox",a.checked=!0,c.appendChild(a),k.appendChecked=a.checked,b.innerHTML="<textarea>x</textarea>",k.noCloneChecked=!!b.cloneNode(!0).lastChild.defaultValue,c.appendChild(b),b.innerHTML="<input type='radio' checked='checked' name='t'/>",k.checkClone=b.cloneNode(!0).cloneNode(!0).lastChild.checked,k.noCloneEvent=!0,b.attachEvent&&(b.attachEvent("onclick",function(){k.noCloneEvent=!1}),b.cloneNode(!0).click()),null==k.deleteExpando){k.deleteExpando=!0;try{delete b.test}catch(d){k.deleteExpando=!1}}}(),function(){var b,c,d=y.createElement("div");for(b in{submit:!0,change:!0,focusin:!0})c="on"+b,(k[b+"Bubbles"]=c in a)||(d.setAttribute(c,"t"),k[b+"Bubbles"]=d.attributes[c].expando===!1);d=null}();var X=/^(?:input|select|textarea)$/i,Y=/^key/,Z=/^(?:mouse|pointer|contextmenu)|click/,$=/^(?:focusinfocus|focusoutblur)$/,_=/^([^.]*)(?:\.(.+)|)$/;function aa(){return!0}function ba(){return!1}function ca(){try{return y.activeElement}catch(a){}}m.event={global:{},add:function(a,b,c,d,e){var f,g,h,i,j,k,l,n,o,p,q,r=m._data(a);if(r){c.handler&&(i=c,c=i.handler,e=i.selector),c.guid||(c.guid=m.guid++),(g=r.events)||(g=r.events={}),(k=r.handle)||(k=r.handle=function(a){return typeof m===K||a&&m.event.triggered===a.type?void 0:m.event.dispatch.apply(k.elem,arguments)},k.elem=a),b=(b||"").match(E)||[""],h=b.length;while(h--)f=_.exec(b[h])||[],o=q=f[1],p=(f[2]||"").split(".").sort(),o&&(j=m.event.special[o]||{},o=(e?j.delegateType:j.bindType)||o,j=m.event.special[o]||{},l=m.extend({type:o,origType:q,data:d,handler:c,guid:c.guid,selector:e,needsContext:e&&m.expr.match.needsContext.test(e),namespace:p.join(".")},i),(n=g[o])||(n=g[o]=[],n.delegateCount=0,j.setup&&j.setup.call(a,d,p,k)!==!1||(a.addEventListener?a.addEventListener(o,k,!1):a.attachEvent&&a.attachEvent("on"+o,k))),j.add&&(j.add.call(a,l),l.handler.guid||(l.handler.guid=c.guid)),e?n.splice(n.delegateCount++,0,l):n.push(l),m.event.global[o]=!0);a=null}},remove:function(a,b,c,d,e){var f,g,h,i,j,k,l,n,o,p,q,r=m.hasData(a)&&m._data(a);if(r&&(k=r.events)){b=(b||"").match(E)||[""],j=b.length;while(j--)if(h=_.exec(b[j])||[],o=q=h[1],p=(h[2]||"").split(".").sort(),o){l=m.event.special[o]||{},o=(d?l.delegateType:l.bindType)||o,n=k[o]||[],h=h[2]&&new RegExp("(^|\\.)"+p.join("\\.(?:.*\\.|)")+"(\\.|$)"),i=f=n.length;while(f--)g=n[f],!e&&q!==g.origType||c&&c.guid!==g.guid||h&&!h.test(g.namespace)||d&&d!==g.selector&&("**"!==d||!g.selector)||(n.splice(f,1),g.selector&&n.delegateCount--,l.remove&&l.remove.call(a,g));i&&!n.length&&(l.teardown&&l.teardown.call(a,p,r.handle)!==!1||m.removeEvent(a,o,r.handle),delete k[o])}else for(o in k)m.event.remove(a,o+b[j],c,d,!0);m.isEmptyObject(k)&&(delete r.handle,m._removeData(a,"events"))}},trigger:function(b,c,d,e){var f,g,h,i,k,l,n,o=[d||y],p=j.call(b,"type")?b.type:b,q=j.call(b,"namespace")?b.namespace.split("."):[];if(h=l=d=d||y,3!==d.nodeType&&8!==d.nodeType&&!$.test(p+m.event.triggered)&&(p.indexOf(".")>=0&&(q=p.split("."),p=q.shift(),q.sort()),g=p.indexOf(":")<0&&"on"+p,b=b[m.expando]?b:new m.Event(p,"object"==typeof b&&b),b.isTrigger=e?2:3,b.namespace=q.join("."),b.namespace_re=b.namespace?new RegExp("(^|\\.)"+q.join("\\.(?:.*\\.|)")+"(\\.|$)"):null,b.result=void 0,b.target||(b.target=d),c=null==c?[b]:m.makeArray(c,[b]),k=m.event.special[p]||{},e||!k.trigger||k.trigger.apply(d,c)!==!1)){if(!e&&!k.noBubble&&!m.isWindow(d)){for(i=k.delegateType||p,$.test(i+p)||(h=h.parentNode);h;h=h.parentNode)o.push(h),l=h;l===(d.ownerDocument||y)&&o.push(l.defaultView||l.parentWindow||a)}n=0;while((h=o[n++])&&!b.isPropagationStopped())b.type=n>1?i:k.bindType||p,f=(m._data(h,"events")||{})[b.type]&&m._data(h,"handle"),f&&f.apply(h,c),f=g&&h[g],f&&f.apply&&m.acceptData(h)&&(b.result=f.apply(h,c),b.result===!1&&b.preventDefault());if(b.type=p,!e&&!b.isDefaultPrevented()&&(!k._default||k._default.apply(o.pop(),c)===!1)&&m.acceptData(d)&&g&&d[p]&&!m.isWindow(d)){l=d[g],l&&(d[g]=null),m.event.triggered=p;try{d[p]()}catch(r){}m.event.triggered=void 0,l&&(d[g]=l)}return b.result}},dispatch:function(a){a=m.event.fix(a);var b,c,e,f,g,h=[],i=d.call(arguments),j=(m._data(this,"events")||{})[a.type]||[],k=m.event.special[a.type]||{};if(i[0]=a,a.delegateTarget=this,!k.preDispatch||k.preDispatch.call(this,a)!==!1){h=m.event.handlers.call(this,a,j),b=0;while((f=h[b++])&&!a.isPropagationStopped()){a.currentTarget=f.elem,g=0;while((e=f.handlers[g++])&&!a.isImmediatePropagationStopped())(!a.namespace_re||a.namespace_re.test(e.namespace))&&(a.handleObj=e,a.data=e.data,c=((m.event.special[e.origType]||{}).handle||e.handler).apply(f.elem,i),void 0!==c&&(a.result=c)===!1&&(a.preventDefault(),a.stopPropagation()))}return k.postDispatch&&k.postDispatch.call(this,a),a.result}},handlers:function(a,b){var c,d,e,f,g=[],h=b.delegateCount,i=a.target;if(h&&i.nodeType&&(!a.button||"click"!==a.type))for(;i!=this;i=i.parentNode||this)if(1===i.nodeType&&(i.disabled!==!0||"click"!==a.type)){for(e=[],f=0;h>f;f++)d=b[f],c=d.selector+" ",void 0===e[c]&&(e[c]=d.needsContext?m(c,this).index(i)>=0:m.find(c,this,null,[i]).length),e[c]&&e.push(d);e.length&&g.push({elem:i,handlers:e})}return h<b.length&&g.push({elem:this,handlers:b.slice(h)}),g},fix:function(a){if(a[m.expando])return a;var b,c,d,e=a.type,f=a,g=this.fixHooks[e];g||(this.fixHooks[e]=g=Z.test(e)?this.mouseHooks:Y.test(e)?this.keyHooks:{}),d=g.props?this.props.concat(g.props):this.props,a=new m.Event(f),b=d.length;while(b--)c=d[b],a[c]=f[c];return a.target||(a.target=f.srcElement||y),3===a.target.nodeType&&(a.target=a.target.parentNode),a.metaKey=!!a.metaKey,g.filter?g.filter(a,f):a},props:"altKey bubbles cancelable ctrlKey currentTarget eventPhase metaKey relatedTarget shiftKey target timeStamp view which".split(" "),fixHooks:{},keyHooks:{props:"char charCode key keyCode".split(" "),filter:function(a,b){return null==a.which&&(a.which=null!=b.charCode?b.charCode:b.keyCode),a}},mouseHooks:{props:"button buttons clientX clientY fromElement offsetX offsetY pageX pageY screenX screenY toElement".split(" "),filter:function(a,b){var c,d,e,f=b.button,g=b.fromElement;return null==a.pageX&&null!=b.clientX&&(d=a.target.ownerDocument||y,e=d.documentElement,c=d.body,a.pageX=b.clientX+(e&&e.scrollLeft||c&&c.scrollLeft||0)-(e&&e.clientLeft||c&&c.clientLeft||0),a.pageY=b.clientY+(e&&e.scrollTop||c&&c.scrollTop||0)-(e&&e.clientTop||c&&c.clientTop||0)),!a.relatedTarget&&g&&(a.relatedTarget=g===a.target?b.toElement:g),a.which||void 0===f||(a.which=1&f?1:2&f?3:4&f?2:0),a}},special:{load:{noBubble:!0},focus:{trigger:function(){if(this!==ca()&&this.focus)try{return this.focus(),!1}catch(a){}},delegateType:"focusin"},blur:{trigger:function(){return this===ca()&&this.blur?(this.blur(),!1):void 0},delegateType:"focusout"},click:{trigger:function(){return m.nodeName(this,"input")&&"checkbox"===this.type&&this.click?(this.click(),!1):void 0},_default:function(a){return m.nodeName(a.target,"a")}},beforeunload:{postDispatch:function(a){void 0!==a.result&&a.originalEvent&&(a.originalEvent.returnValue=a.result)}}},simulate:function(a,b,c,d){var e=m.extend(new m.Event,c,{type:a,isSimulated:!0,originalEvent:{}});d?m.event.trigger(e,null,b):m.event.dispatch.call(b,e),e.isDefaultPrevented()&&c.preventDefault()}},m.removeEvent=y.removeEventListener?function(a,b,c){a.removeEventListener&&a.removeEventListener(b,c,!1)}:function(a,b,c){var d="on"+b;a.detachEvent&&(typeof a[d]===K&&(a[d]=null),a.detachEvent(d,c))},m.Event=function(a,b){return this instanceof m.Event?(a&&a.type?(this.originalEvent=a,this.type=a.type,this.isDefaultPrevented=a.defaultPrevented||void 0===a.defaultPrevented&&a.returnValue===!1?aa:ba):this.type=a,b&&m.extend(this,b),this.timeStamp=a&&a.timeStamp||m.now(),void(this[m.expando]=!0)):new m.Event(a,b)},m.Event.prototype={isDefaultPrevented:ba,isPropagationStopped:ba,isImmediatePropagationStopped:ba,preventDefault:function(){var a=this.originalEvent;this.isDefaultPrevented=aa,a&&(a.preventDefault?a.preventDefault():a.returnValue=!1)},stopPropagation:function(){var a=this.originalEvent;this.isPropagationStopped=aa,a&&(a.stopPropagation&&a.stopPropagation(),a.cancelBubble=!0)},stopImmediatePropagation:function(){var a=this.originalEvent;this.isImmediatePropagationStopped=aa,a&&a.stopImmediatePropagation&&a.stopImmediatePropagation(),this.stopPropagation()}},m.each({mouseenter:"mouseover",mouseleave:"mouseout",pointerenter:"pointerover",pointerleave:"pointerout"},function(a,b){m.event.special[a]={delegateType:b,bindType:b,handle:function(a){var c,d=this,e=a.relatedTarget,f=a.handleObj;return(!e||e!==d&&!m.contains(d,e))&&(a.type=f.origType,c=f.handler.apply(this,arguments),a.type=b),c}}}),k.submitBubbles||(m.event.special.submit={setup:function(){return m.nodeName(this,"form")?!1:void m.event.add(this,"click._submit keypress._submit",function(a){var b=a.target,c=m.nodeName(b,"input")||m.nodeName(b,"button")?b.form:void 0;c&&!m._data(c,"submitBubbles")&&(m.event.add(c,"submit._submit",function(a){a._submit_bubble=!0}),m._data(c,"submitBubbles",!0))})},postDispatch:function(a){a._submit_bubble&&(delete a._submit_bubble,this.parentNode&&!a.isTrigger&&m.event.simulate("submit",this.parentNode,a,!0))},teardown:function(){return m.nodeName(this,"form")?!1:void m.event.remove(this,"._submit")}}),k.changeBubbles||(m.event.special.change={setup:function(){return X.test(this.nodeName)?(("checkbox"===this.type||"radio"===this.type)&&(m.event.add(this,"propertychange._change",function(a){"checked"===a.originalEvent.propertyName&&(this._just_changed=!0)}),m.event.add(this,"click._change",function(a){this._just_changed&&!a.isTrigger&&(this._just_changed=!1),m.event.simulate("change",this,a,!0)})),!1):void m.event.add(this,"beforeactivate._change",function(a){var b=a.target;X.test(b.nodeName)&&!m._data(b,"changeBubbles")&&(m.event.add(b,"change._change",function(a){!this.parentNode||a.isSimulated||a.isTrigger||m.event.simulate("change",this.parentNode,a,!0)}),m._data(b,"changeBubbles",!0))})},handle:function(a){var b=a.target;return this!==b||a.isSimulated||a.isTrigger||"radio"!==b.type&&"checkbox"!==b.type?a.handleObj.handler.apply(this,arguments):void 0},teardown:function(){return m.event.remove(this,"._change"),!X.test(this.nodeName)}}),k.focusinBubbles||m.each({focus:"focusin",blur:"focusout"},function(a,b){var c=function(a){m.event.simulate(b,a.target,m.event.fix(a),!0)};m.event.special[b]={setup:function(){var d=this.ownerDocument||this,e=m._data(d,b);e||d.addEventListener(a,c,!0),m._data(d,b,(e||0)+1)},teardown:function(){var d=this.ownerDocument||this,e=m._data(d,b)-1;e?m._data(d,b,e):(d.removeEventListener(a,c,!0),m._removeData(d,b))}}}),m.fn.extend({on:function(a,b,c,d,e){var f,g;if("object"==typeof a){"string"!=typeof b&&(c=c||b,b=void 0);for(f in a)this.on(f,b,c,a[f],e);return this}if(null==c&&null==d?(d=b,c=b=void 0):null==d&&("string"==typeof b?(d=c,c=void 0):(d=c,c=b,b=void 0)),d===!1)d=ba;else if(!d)return this;return 1===e&&(g=d,d=function(a){return m().off(a),g.apply(this,arguments)},d.guid=g.guid||(g.guid=m.guid++)),this.each(function(){m.event.add(this,a,d,c,b)})},one:function(a,b,c,d){return this.on(a,b,c,d,1)},off:function(a,b,c){var d,e;if(a&&a.preventDefault&&a.handleObj)return d=a.handleObj,m(a.delegateTarget).off(d.namespace?d.origType+"."+d.namespace:d.origType,d.selector,d.handler),this;if("object"==typeof a){for(e in a)this.off(e,b,a[e]);return this}return(b===!1||"function"==typeof b)&&(c=b,b=void 0),c===!1&&(c=ba),this.each(function(){m.event.remove(this,a,c,b)})},trigger:function(a,b){return this.each(function(){m.event.trigger(a,b,this)})},triggerHandler:function(a,b){var c=this[0];return c?m.event.trigger(a,b,c,!0):void 0}});function da(a){var b=ea.split("|"),c=a.createDocumentFragment();if(c.createElement)while(b.length)c.createElement(b.pop());return c}var ea="abbr|article|aside|audio|bdi|canvas|data|datalist|details|figcaption|figure|footer|header|hgroup|mark|meter|nav|output|progress|section|summary|time|video",fa=/ jQuery\d+="(?:null|\d+)"/g,ga=new RegExp("<(?:"+ea+")[\\s/>]","i"),ha=/^\s+/,ia=/<(?!area|br|col|embed|hr|img|input|link|meta|param)(([\w:]+)[^>]*)\/>/gi,ja=/<([\w:]+)/,ka=/<tbody/i,la=/<|&#?\w+;/,ma=/<(?:script|style|link)/i,na=/checked\s*(?:[^=]|=\s*.checked.)/i,oa=/^$|\/(?:java|ecma)script/i,pa=/^true\/(.*)/,qa=/^\s*<!(?:\[CDATA\[|--)|(?:\]\]|--)>\s*$/g,ra={option:[1,"<select multiple='multiple'>","</select>"],legend:[1,"<fieldset>","</fieldset>"],area:[1,"<map>","</map>"],param:[1,"<object>","</object>"],thead:[1,"<table>","</table>"],tr:[2,"<table><tbody>","</tbody></table>"],col:[2,"<table><tbody></tbody><colgroup>","</colgroup></table>"],td:[3,"<table><tbody><tr>","</tr></tbody></table>"],_default:k.htmlSerialize?[0,"",""]:[1,"X<div>","</div>"]},sa=da(y),ta=sa.appendChild(y.createElement("div"));ra.optgroup=ra.option,ra.tbody=ra.tfoot=ra.colgroup=ra.caption=ra.thead,ra.th=ra.td;function ua(a,b){var c,d,e=0,f=typeof a.getElementsByTagName!==K?a.getElementsByTagName(b||"*"):typeof a.querySelectorAll!==K?a.querySelectorAll(b||"*"):void 0;if(!f)for(f=[],c=a.childNodes||a;null!=(d=c[e]);e++)!b||m.nodeName(d,b)?f.push(d):m.merge(f,ua(d,b));return void 0===b||b&&m.nodeName(a,b)?m.merge([a],f):f}function va(a){W.test(a.type)&&(a.defaultChecked=a.checked)}function wa(a,b){return m.nodeName(a,"table")&&m.nodeName(11!==b.nodeType?b:b.firstChild,"tr")?a.getElementsByTagName("tbody")[0]||a.appendChild(a.ownerDocument.createElement("tbody")):a}function xa(a){return a.type=(null!==m.find.attr(a,"type"))+"/"+a.type,a}function ya(a){var b=pa.exec(a.type);return b?a.type=b[1]:a.removeAttribute("type"),a}function za(a,b){for(var c,d=0;null!=(c=a[d]);d++)m._data(c,"globalEval",!b||m._data(b[d],"globalEval"))}function Aa(a,b){if(1===b.nodeType&&m.hasData(a)){var c,d,e,f=m._data(a),g=m._data(b,f),h=f.events;if(h){delete g.handle,g.events={};for(c in h)for(d=0,e=h[c].length;e>d;d++)m.event.add(b,c,h[c][d])}g.data&&(g.data=m.extend({},g.data))}}function Ba(a,b){var c,d,e;if(1===b.nodeType){if(c=b.nodeName.toLowerCase(),!k.noCloneEvent&&b[m.expando]){e=m._data(b);for(d in e.events)m.removeEvent(b,d,e.handle);b.removeAttribute(m.expando)}"script"===c&&b.text!==a.text?(xa(b).text=a.text,ya(b)):"object"===c?(b.parentNode&&(b.outerHTML=a.outerHTML),k.html5Clone&&a.innerHTML&&!m.trim(b.innerHTML)&&(b.innerHTML=a.innerHTML)):"input"===c&&W.test(a.type)?(b.defaultChecked=b.checked=a.checked,b.value!==a.value&&(b.value=a.value)):"option"===c?b.defaultSelected=b.selected=a.defaultSelected:("input"===c||"textarea"===c)&&(b.defaultValue=a.defaultValue)}}m.extend({clone:function(a,b,c){var d,e,f,g,h,i=m.contains(a.ownerDocument,a);if(k.html5Clone||m.isXMLDoc(a)||!ga.test("<"+a.nodeName+">")?f=a.cloneNode(!0):(ta.innerHTML=a.outerHTML,ta.removeChild(f=ta.firstChild)),!(k.noCloneEvent&&k.noCloneChecked||1!==a.nodeType&&11!==a.nodeType||m.isXMLDoc(a)))for(d=ua(f),h=ua(a),g=0;null!=(e=h[g]);++g)d[g]&&Ba(e,d[g]);if(b)if(c)for(h=h||ua(a),d=d||ua(f),g=0;null!=(e=h[g]);g++)Aa(e,d[g]);else Aa(a,f);return d=ua(f,"script"),d.length>0&&za(d,!i&&ua(a,"script")),d=h=e=null,f},buildFragment:function(a,b,c,d){for(var e,f,g,h,i,j,l,n=a.length,o=da(b),p=[],q=0;n>q;q++)if(f=a[q],f||0===f)if("object"===m.type(f))m.merge(p,f.nodeType?[f]:f);else if(la.test(f)){h=h||o.appendChild(b.createElement("div")),i=(ja.exec(f)||["",""])[1].toLowerCase(),l=ra[i]||ra._default,h.innerHTML=l[1]+f.replace(ia,"<$1></$2>")+l[2],e=l[0];while(e--)h=h.lastChild;if(!k.leadingWhitespace&&ha.test(f)&&p.push(b.createTextNode(ha.exec(f)[0])),!k.tbody){f="table"!==i||ka.test(f)?"<table>"!==l[1]||ka.test(f)?0:h:h.firstChild,e=f&&f.childNodes.length;while(e--)m.nodeName(j=f.childNodes[e],"tbody")&&!j.childNodes.length&&f.removeChild(j)}m.merge(p,h.childNodes),h.textContent="";while(h.firstChild)h.removeChild(h.firstChild);h=o.lastChild}else p.push(b.createTextNode(f));h&&o.removeChild(h),k.appendChecked||m.grep(ua(p,"input"),va),q=0;while(f=p[q++])if((!d||-1===m.inArray(f,d))&&(g=m.contains(f.ownerDocument,f),h=ua(o.appendChild(f),"script"),g&&za(h),c)){e=0;while(f=h[e++])oa.test(f.type||"")&&c.push(f)}return h=null,o},cleanData:function(a,b){for(var d,e,f,g,h=0,i=m.expando,j=m.cache,l=k.deleteExpando,n=m.event.special;null!=(d=a[h]);h++)if((b||m.acceptData(d))&&(f=d[i],g=f&&j[f])){if(g.events)for(e in g.events)n[e]?m.event.remove(d,e):m.removeEvent(d,e,g.handle);j[f]&&(delete j[f],l?delete d[i]:typeof d.removeAttribute!==K?d.removeAttribute(i):d[i]=null,c.push(f))}}}),m.fn.extend({text:function(a){return V(this,function(a){return void 0===a?m.text(this):this.empty().append((this[0]&&this[0].ownerDocument||y).createTextNode(a))},null,a,arguments.length)},append:function(){return this.domManip(arguments,function(a){if(1===this.nodeType||11===this.nodeType||9===this.nodeType){var b=wa(this,a);b.appendChild(a)}})},prepend:function(){return this.domManip(arguments,function(a){if(1===this.nodeType||11===this.nodeType||9===this.nodeType){var b=wa(this,a);b.insertBefore(a,b.firstChild)}})},before:function(){return this.domManip(arguments,function(a){this.parentNode&&this.parentNode.insertBefore(a,this)})},after:function(){return this.domManip(arguments,function(a){this.parentNode&&this.parentNode.insertBefore(a,this.nextSibling)})},remove:function(a,b){for(var c,d=a?m.filter(a,this):this,e=0;null!=(c=d[e]);e++)b||1!==c.nodeType||m.cleanData(ua(c)),c.parentNode&&(b&&m.contains(c.ownerDocument,c)&&za(ua(c,"script")),c.parentNode.removeChild(c));return this},empty:function(){for(var a,b=0;null!=(a=this[b]);b++){1===a.nodeType&&m.cleanData(ua(a,!1));while(a.firstChild)a.removeChild(a.firstChild);a.options&&m.nodeName(a,"select")&&(a.options.length=0)}return this},clone:function(a,b){return a=null==a?!1:a,b=null==b?a:b,this.map(function(){return m.clone(this,a,b)})},html:function(a){return V(this,function(a){var b=this[0]||{},c=0,d=this.length;if(void 0===a)return 1===b.nodeType?b.innerHTML.replace(fa,""):void 0;if(!("string"!=typeof a||ma.test(a)||!k.htmlSerialize&&ga.test(a)||!k.leadingWhitespace&&ha.test(a)||ra[(ja.exec(a)||["",""])[1].toLowerCase()])){a=a.replace(ia,"<$1></$2>");try{for(;d>c;c++)b=this[c]||{},1===b.nodeType&&(m.cleanData(ua(b,!1)),b.innerHTML=a);b=0}catch(e){}}b&&this.empty().append(a)},null,a,arguments.length)},replaceWith:function(){var a=arguments[0];return this.domManip(arguments,function(b){a=this.parentNode,m.cleanData(ua(this)),a&&a.replaceChild(b,this)}),a&&(a.length||a.nodeType)?this:this.remove()},detach:function(a){return this.remove(a,!0)},domManip:function(a,b){a=e.apply([],a);var c,d,f,g,h,i,j=0,l=this.length,n=this,o=l-1,p=a[0],q=m.isFunction(p);if(q||l>1&&"string"==typeof p&&!k.checkClone&&na.test(p))return this.each(function(c){var d=n.eq(c);q&&(a[0]=p.call(this,c,d.html())),d.domManip(a,b)});if(l&&(i=m.buildFragment(a,this[0].ownerDocument,!1,this),c=i.firstChild,1===i.childNodes.length&&(i=c),c)){for(g=m.map(ua(i,"script"),xa),f=g.length;l>j;j++)d=i,j!==o&&(d=m.clone(d,!0,!0),f&&m.merge(g,ua(d,"script"))),b.call(this[j],d,j);if(f)for(h=g[g.length-1].ownerDocument,m.map(g,ya),j=0;f>j;j++)d=g[j],oa.test(d.type||"")&&!m._data(d,"globalEval")&&m.contains(h,d)&&(d.src?m._evalUrl&&m._evalUrl(d.src):m.globalEval((d.text||d.textContent||d.innerHTML||"").replace(qa,"")));i=c=null}return this}}),m.each({appendTo:"append",prependTo:"prepend",insertBefore:"before",insertAfter:"after",replaceAll:"replaceWith"},function(a,b){m.fn[a]=function(a){for(var c,d=0,e=[],g=m(a),h=g.length-1;h>=d;d++)c=d===h?this:this.clone(!0),m(g[d])[b](c),f.apply(e,c.get());return this.pushStack(e)}});var Ca,Da={};function Ea(b,c){var d,e=m(c.createElement(b)).appendTo(c.body),f=a.getDefaultComputedStyle&&(d=a.getDefaultComputedStyle(e[0]))?d.display:m.css(e[0],"display");return e.detach(),f}function Fa(a){var b=y,c=Da[a];return c||(c=Ea(a,b),"none"!==c&&c||(Ca=(Ca||m("<iframe frameborder='0' width='0' height='0'/>")).appendTo(b.documentElement),b=(Ca[0].contentWindow||Ca[0].contentDocument).document,b.write(),b.close(),c=Ea(a,b),Ca.detach()),Da[a]=c),c}!function(){var a;k.shrinkWrapBlocks=function(){if(null!=a)return a;a=!1;var b,c,d;return c=y.getElementsByTagName("body")[0],c&&c.style?(b=y.createElement("div"),d=y.createElement("div"),d.style.cssText="position:absolute;border:0;width:0;height:0;top:0;left:-9999px",c.appendChild(d).appendChild(b),typeof b.style.zoom!==K&&(b.style.cssText="-webkit-box-sizing:content-box;-moz-box-sizing:content-box;box-sizing:content-box;display:block;margin:0;border:0;padding:1px;width:1px;zoom:1",b.appendChild(y.createElement("div")).style.width="5px",a=3!==b.offsetWidth),c.removeChild(d),a):void 0}}();var Ga=/^margin/,Ha=new RegExp("^("+S+")(?!px)[a-z%]+$","i"),Ia,Ja,Ka=/^(top|right|bottom|left)$/;a.getComputedStyle?(Ia=function(b){return b.ownerDocument.defaultView.opener?b.ownerDocument.defaultView.getComputedStyle(b,null):a.getComputedStyle(b,null)},Ja=function(a,b,c){var d,e,f,g,h=a.style;return c=c||Ia(a),g=c?c.getPropertyValue(b)||c[b]:void 0,c&&(""!==g||m.contains(a.ownerDocument,a)||(g=m.style(a,b)),Ha.test(g)&&Ga.test(b)&&(d=h.width,e=h.minWidth,f=h.maxWidth,h.minWidth=h.maxWidth=h.width=g,g=c.width,h.width=d,h.minWidth=e,h.maxWidth=f)),void 0===g?g:g+""}):y.documentElement.currentStyle&&(Ia=function(a){return a.currentStyle},Ja=function(a,b,c){var d,e,f,g,h=a.style;return c=c||Ia(a),g=c?c[b]:void 0,null==g&&h&&h[b]&&(g=h[b]),Ha.test(g)&&!Ka.test(b)&&(d=h.left,e=a.runtimeStyle,f=e&&e.left,f&&(e.left=a.currentStyle.left),h.left="fontSize"===b?"1em":g,g=h.pixelLeft+"px",h.left=d,f&&(e.left=f)),void 0===g?g:g+""||"auto"});function La(a,b){return{get:function(){var c=a();if(null!=c)return c?void delete this.get:(this.get=b).apply(this,arguments)}}}!function(){var b,c,d,e,f,g,h;if(b=y.createElement("div"),b.innerHTML="  <link/><table></table><a href='/a'>a</a><input type='checkbox'/>",d=b.getElementsByTagName("a")[0],c=d&&d.style){c.cssText="float:left;opacity:.5",k.opacity="0.5"===c.opacity,k.cssFloat=!!c.cssFloat,b.style.backgroundClip="content-box",b.cloneNode(!0).style.backgroundClip="",k.clearCloneStyle="content-box"===b.style.backgroundClip,k.boxSizing=""===c.boxSizing||""===c.MozBoxSizing||""===c.WebkitBoxSizing,m.extend(k,{reliableHiddenOffsets:function(){return null==g&&i(),g},boxSizingReliable:function(){return null==f&&i(),f},pixelPosition:function(){return null==e&&i(),e},reliableMarginRight:function(){return null==h&&i(),h}});function i(){var b,c,d,i;c=y.getElementsByTagName("body")[0],c&&c.style&&(b=y.createElement("div"),d=y.createElement("div"),d.style.cssText="position:absolute;border:0;width:0;height:0;top:0;left:-9999px",c.appendChild(d).appendChild(b),b.style.cssText="-webkit-box-sizing:border-box;-moz-box-sizing:border-box;box-sizing:border-box;display:block;margin-top:1%;top:1%;border:1px;padding:1px;width:4px;position:absolute",e=f=!1,h=!0,a.getComputedStyle&&(e="1%"!==(a.getComputedStyle(b,null)||{}).top,f="4px"===(a.getComputedStyle(b,null)||{width:"4px"}).width,i=b.appendChild(y.createElement("div")),i.style.cssText=b.style.cssText="-webkit-box-sizing:content-box;-moz-box-sizing:content-box;box-sizing:content-box;display:block;margin:0;border:0;padding:0",i.style.marginRight=i.style.width="0",b.style.width="1px",h=!parseFloat((a.getComputedStyle(i,null)||{}).marginRight),b.removeChild(i)),b.innerHTML="<table><tr><td></td><td>t</td></tr></table>",i=b.getElementsByTagName("td"),i[0].style.cssText="margin:0;border:0;padding:0;display:none",g=0===i[0].offsetHeight,g&&(i[0].style.display="",i[1].style.display="none",g=0===i[0].offsetHeight),c.removeChild(d))}}}(),m.swap=function(a,b,c,d){var e,f,g={};for(f in b)g[f]=a.style[f],a.style[f]=b[f];e=c.apply(a,d||[]);for(f in b)a.style[f]=g[f];return e};var Ma=/alpha\([^)]*\)/i,Na=/opacity\s*=\s*([^)]*)/,Oa=/^(none|table(?!-c[ea]).+)/,Pa=new RegExp("^("+S+")(.*)$","i"),Qa=new RegExp("^([+-])=("+S+")","i"),Ra={position:"absolute",visibility:"hidden",display:"block"},Sa={letterSpacing:"0",fontWeight:"400"},Ta=["Webkit","O","Moz","ms"];function Ua(a,b){if(b in a)return b;var c=b.charAt(0).toUpperCase()+b.slice(1),d=b,e=Ta.length;while(e--)if(b=Ta[e]+c,b in a)return b;return d}function Va(a,b){for(var c,d,e,f=[],g=0,h=a.length;h>g;g++)d=a[g],d.style&&(f[g]=m._data(d,"olddisplay"),c=d.style.display,b?(f[g]||"none"!==c||(d.style.display=""),""===d.style.display&&U(d)&&(f[g]=m._data(d,"olddisplay",Fa(d.nodeName)))):(e=U(d),(c&&"none"!==c||!e)&&m._data(d,"olddisplay",e?c:m.css(d,"display"))));for(g=0;h>g;g++)d=a[g],d.style&&(b&&"none"!==d.style.display&&""!==d.style.display||(d.style.display=b?f[g]||"":"none"));return a}function Wa(a,b,c){var d=Pa.exec(b);return d?Math.max(0,d[1]-(c||0))+(d[2]||"px"):b}function Xa(a,b,c,d,e){for(var f=c===(d?"border":"content")?4:"width"===b?1:0,g=0;4>f;f+=2)"margin"===c&&(g+=m.css(a,c+T[f],!0,e)),d?("content"===c&&(g-=m.css(a,"padding"+T[f],!0,e)),"margin"!==c&&(g-=m.css(a,"border"+T[f]+"Width",!0,e))):(g+=m.css(a,"padding"+T[f],!0,e),"padding"!==c&&(g+=m.css(a,"border"+T[f]+"Width",!0,e)));return g}function Ya(a,b,c){var d=!0,e="width"===b?a.offsetWidth:a.offsetHeight,f=Ia(a),g=k.boxSizing&&"border-box"===m.css(a,"boxSizing",!1,f);if(0>=e||null==e){if(e=Ja(a,b,f),(0>e||null==e)&&(e=a.style[b]),Ha.test(e))return e;d=g&&(k.boxSizingReliable()||e===a.style[b]),e=parseFloat(e)||0}return e+Xa(a,b,c||(g?"border":"content"),d,f)+"px"}m.extend({cssHooks:{opacity:{get:function(a,b){if(b){var c=Ja(a,"opacity");return""===c?"1":c}}}},cssNumber:{columnCount:!0,fillOpacity:!0,flexGrow:!0,flexShrink:!0,fontWeight:!0,lineHeight:!0,opacity:!0,order:!0,orphans:!0,widows:!0,zIndex:!0,zoom:!0},cssProps:{"float":k.cssFloat?"cssFloat":"styleFloat"},style:function(a,b,c,d){if(a&&3!==a.nodeType&&8!==a.nodeType&&a.style){var e,f,g,h=m.camelCase(b),i=a.style;if(b=m.cssProps[h]||(m.cssProps[h]=Ua(i,h)),g=m.cssHooks[b]||m.cssHooks[h],void 0===c)return g&&"get"in g&&void 0!==(e=g.get(a,!1,d))?e:i[b];if(f=typeof c,"string"===f&&(e=Qa.exec(c))&&(c=(e[1]+1)*e[2]+parseFloat(m.css(a,b)),f="number"),null!=c&&c===c&&("number"!==f||m.cssNumber[h]||(c+="px"),k.clearCloneStyle||""!==c||0!==b.indexOf("background")||(i[b]="inherit"),!(g&&"set"in g&&void 0===(c=g.set(a,c,d)))))try{i[b]=c}catch(j){}}},css:function(a,b,c,d){var e,f,g,h=m.camelCase(b);return b=m.cssProps[h]||(m.cssProps[h]=Ua(a.style,h)),g=m.cssHooks[b]||m.cssHooks[h],g&&"get"in g&&(f=g.get(a,!0,c)),void 0===f&&(f=Ja(a,b,d)),"normal"===f&&b in Sa&&(f=Sa[b]),""===c||c?(e=parseFloat(f),c===!0||m.isNumeric(e)?e||0:f):f}}),m.each(["height","width"],function(a,b){m.cssHooks[b]={get:function(a,c,d){return c?Oa.test(m.css(a,"display"))&&0===a.offsetWidth?m.swap(a,Ra,function(){return Ya(a,b,d)}):Ya(a,b,d):void 0},set:function(a,c,d){var e=d&&Ia(a);return Wa(a,c,d?Xa(a,b,d,k.boxSizing&&"border-box"===m.css(a,"boxSizing",!1,e),e):0)}}}),k.opacity||(m.cssHooks.opacity={get:function(a,b){return Na.test((b&&a.currentStyle?a.currentStyle.filter:a.style.filter)||"")?.01*parseFloat(RegExp.$1)+"":b?"1":""},set:function(a,b){var c=a.style,d=a.currentStyle,e=m.isNumeric(b)?"alpha(opacity="+100*b+")":"",f=d&&d.filter||c.filter||"";c.zoom=1,(b>=1||""===b)&&""===m.trim(f.replace(Ma,""))&&c.removeAttribute&&(c.removeAttribute("filter"),""===b||d&&!d.filter)||(c.filter=Ma.test(f)?f.replace(Ma,e):f+" "+e)}}),m.cssHooks.marginRight=La(k.reliableMarginRight,function(a,b){return b?m.swap(a,{display:"inline-block"},Ja,[a,"marginRight"]):void 0}),m.each({margin:"",padding:"",border:"Width"},function(a,b){m.cssHooks[a+b]={expand:function(c){for(var d=0,e={},f="string"==typeof c?c.split(" "):[c];4>d;d++)e[a+T[d]+b]=f[d]||f[d-2]||f[0];return e}},Ga.test(a)||(m.cssHooks[a+b].set=Wa)}),m.fn.extend({css:function(a,b){return V(this,function(a,b,c){var d,e,f={},g=0;if(m.isArray(b)){for(d=Ia(a),e=b.length;e>g;g++)f[b[g]]=m.css(a,b[g],!1,d);return f}return void 0!==c?m.style(a,b,c):m.css(a,b)},a,b,arguments.length>1)},show:function(){return Va(this,!0)},hide:function(){return Va(this)},toggle:function(a){return"boolean"==typeof a?a?this.show():this.hide():this.each(function(){U(this)?m(this).show():m(this).hide()})}});function Za(a,b,c,d,e){
+return new Za.prototype.init(a,b,c,d,e)}m.Tween=Za,Za.prototype={constructor:Za,init:function(a,b,c,d,e,f){this.elem=a,this.prop=c,this.easing=e||"swing",this.options=b,this.start=this.now=this.cur(),this.end=d,this.unit=f||(m.cssNumber[c]?"":"px")},cur:function(){var a=Za.propHooks[this.prop];return a&&a.get?a.get(this):Za.propHooks._default.get(this)},run:function(a){var b,c=Za.propHooks[this.prop];return this.options.duration?this.pos=b=m.easing[this.easing](a,this.options.duration*a,0,1,this.options.duration):this.pos=b=a,this.now=(this.end-this.start)*b+this.start,this.options.step&&this.options.step.call(this.elem,this.now,this),c&&c.set?c.set(this):Za.propHooks._default.set(this),this}},Za.prototype.init.prototype=Za.prototype,Za.propHooks={_default:{get:function(a){var b;return null==a.elem[a.prop]||a.elem.style&&null!=a.elem.style[a.prop]?(b=m.css(a.elem,a.prop,""),b&&"auto"!==b?b:0):a.elem[a.prop]},set:function(a){m.fx.step[a.prop]?m.fx.step[a.prop](a):a.elem.style&&(null!=a.elem.style[m.cssProps[a.prop]]||m.cssHooks[a.prop])?m.style(a.elem,a.prop,a.now+a.unit):a.elem[a.prop]=a.now}}},Za.propHooks.scrollTop=Za.propHooks.scrollLeft={set:function(a){a.elem.nodeType&&a.elem.parentNode&&(a.elem[a.prop]=a.now)}},m.easing={linear:function(a){return a},swing:function(a){return.5-Math.cos(a*Math.PI)/2}},m.fx=Za.prototype.init,m.fx.step={};var $a,_a,ab=/^(?:toggle|show|hide)$/,bb=new RegExp("^(?:([+-])=|)("+S+")([a-z%]*)$","i"),cb=/queueHooks$/,db=[ib],eb={"*":[function(a,b){var c=this.createTween(a,b),d=c.cur(),e=bb.exec(b),f=e&&e[3]||(m.cssNumber[a]?"":"px"),g=(m.cssNumber[a]||"px"!==f&&+d)&&bb.exec(m.css(c.elem,a)),h=1,i=20;if(g&&g[3]!==f){f=f||g[3],e=e||[],g=+d||1;do h=h||".5",g/=h,m.style(c.elem,a,g+f);while(h!==(h=c.cur()/d)&&1!==h&&--i)}return e&&(g=c.start=+g||+d||0,c.unit=f,c.end=e[1]?g+(e[1]+1)*e[2]:+e[2]),c}]};function fb(){return setTimeout(function(){$a=void 0}),$a=m.now()}function gb(a,b){var c,d={height:a},e=0;for(b=b?1:0;4>e;e+=2-b)c=T[e],d["margin"+c]=d["padding"+c]=a;return b&&(d.opacity=d.width=a),d}function hb(a,b,c){for(var d,e=(eb[b]||[]).concat(eb["*"]),f=0,g=e.length;g>f;f++)if(d=e[f].call(c,b,a))return d}function ib(a,b,c){var d,e,f,g,h,i,j,l,n=this,o={},p=a.style,q=a.nodeType&&U(a),r=m._data(a,"fxshow");c.queue||(h=m._queueHooks(a,"fx"),null==h.unqueued&&(h.unqueued=0,i=h.empty.fire,h.empty.fire=function(){h.unqueued||i()}),h.unqueued++,n.always(function(){n.always(function(){h.unqueued--,m.queue(a,"fx").length||h.empty.fire()})})),1===a.nodeType&&("height"in b||"width"in b)&&(c.overflow=[p.overflow,p.overflowX,p.overflowY],j=m.css(a,"display"),l="none"===j?m._data(a,"olddisplay")||Fa(a.nodeName):j,"inline"===l&&"none"===m.css(a,"float")&&(k.inlineBlockNeedsLayout&&"inline"!==Fa(a.nodeName)?p.zoom=1:p.display="inline-block")),c.overflow&&(p.overflow="hidden",k.shrinkWrapBlocks()||n.always(function(){p.overflow=c.overflow[0],p.overflowX=c.overflow[1],p.overflowY=c.overflow[2]}));for(d in b)if(e=b[d],ab.exec(e)){if(delete b[d],f=f||"toggle"===e,e===(q?"hide":"show")){if("show"!==e||!r||void 0===r[d])continue;q=!0}o[d]=r&&r[d]||m.style(a,d)}else j=void 0;if(m.isEmptyObject(o))"inline"===("none"===j?Fa(a.nodeName):j)&&(p.display=j);else{r?"hidden"in r&&(q=r.hidden):r=m._data(a,"fxshow",{}),f&&(r.hidden=!q),q?m(a).show():n.done(function(){m(a).hide()}),n.done(function(){var b;m._removeData(a,"fxshow");for(b in o)m.style(a,b,o[b])});for(d in o)g=hb(q?r[d]:0,d,n),d in r||(r[d]=g.start,q&&(g.end=g.start,g.start="width"===d||"height"===d?1:0))}}function jb(a,b){var c,d,e,f,g;for(c in a)if(d=m.camelCase(c),e=b[d],f=a[c],m.isArray(f)&&(e=f[1],f=a[c]=f[0]),c!==d&&(a[d]=f,delete a[c]),g=m.cssHooks[d],g&&"expand"in g){f=g.expand(f),delete a[d];for(c in f)c in a||(a[c]=f[c],b[c]=e)}else b[d]=e}function kb(a,b,c){var d,e,f=0,g=db.length,h=m.Deferred().always(function(){delete i.elem}),i=function(){if(e)return!1;for(var b=$a||fb(),c=Math.max(0,j.startTime+j.duration-b),d=c/j.duration||0,f=1-d,g=0,i=j.tweens.length;i>g;g++)j.tweens[g].run(f);return h.notifyWith(a,[j,f,c]),1>f&&i?c:(h.resolveWith(a,[j]),!1)},j=h.promise({elem:a,props:m.extend({},b),opts:m.extend(!0,{specialEasing:{}},c),originalProperties:b,originalOptions:c,startTime:$a||fb(),duration:c.duration,tweens:[],createTween:function(b,c){var d=m.Tween(a,j.opts,b,c,j.opts.specialEasing[b]||j.opts.easing);return j.tweens.push(d),d},stop:function(b){var c=0,d=b?j.tweens.length:0;if(e)return this;for(e=!0;d>c;c++)j.tweens[c].run(1);return b?h.resolveWith(a,[j,b]):h.rejectWith(a,[j,b]),this}}),k=j.props;for(jb(k,j.opts.specialEasing);g>f;f++)if(d=db[f].call(j,a,k,j.opts))return d;return m.map(k,hb,j),m.isFunction(j.opts.start)&&j.opts.start.call(a,j),m.fx.timer(m.extend(i,{elem:a,anim:j,queue:j.opts.queue})),j.progress(j.opts.progress).done(j.opts.done,j.opts.complete).fail(j.opts.fail).always(j.opts.always)}m.Animation=m.extend(kb,{tweener:function(a,b){m.isFunction(a)?(b=a,a=["*"]):a=a.split(" ");for(var c,d=0,e=a.length;e>d;d++)c=a[d],eb[c]=eb[c]||[],eb[c].unshift(b)},prefilter:function(a,b){b?db.unshift(a):db.push(a)}}),m.speed=function(a,b,c){var d=a&&"object"==typeof a?m.extend({},a):{complete:c||!c&&b||m.isFunction(a)&&a,duration:a,easing:c&&b||b&&!m.isFunction(b)&&b};return d.duration=m.fx.off?0:"number"==typeof d.duration?d.duration:d.duration in m.fx.speeds?m.fx.speeds[d.duration]:m.fx.speeds._default,(null==d.queue||d.queue===!0)&&(d.queue="fx"),d.old=d.complete,d.complete=function(){m.isFunction(d.old)&&d.old.call(this),d.queue&&m.dequeue(this,d.queue)},d},m.fn.extend({fadeTo:function(a,b,c,d){return this.filter(U).css("opacity",0).show().end().animate({opacity:b},a,c,d)},animate:function(a,b,c,d){var e=m.isEmptyObject(a),f=m.speed(b,c,d),g=function(){var b=kb(this,m.extend({},a),f);(e||m._data(this,"finish"))&&b.stop(!0)};return g.finish=g,e||f.queue===!1?this.each(g):this.queue(f.queue,g)},stop:function(a,b,c){var d=function(a){var b=a.stop;delete a.stop,b(c)};return"string"!=typeof a&&(c=b,b=a,a=void 0),b&&a!==!1&&this.queue(a||"fx",[]),this.each(function(){var b=!0,e=null!=a&&a+"queueHooks",f=m.timers,g=m._data(this);if(e)g[e]&&g[e].stop&&d(g[e]);else for(e in g)g[e]&&g[e].stop&&cb.test(e)&&d(g[e]);for(e=f.length;e--;)f[e].elem!==this||null!=a&&f[e].queue!==a||(f[e].anim.stop(c),b=!1,f.splice(e,1));(b||!c)&&m.dequeue(this,a)})},finish:function(a){return a!==!1&&(a=a||"fx"),this.each(function(){var b,c=m._data(this),d=c[a+"queue"],e=c[a+"queueHooks"],f=m.timers,g=d?d.length:0;for(c.finish=!0,m.queue(this,a,[]),e&&e.stop&&e.stop.call(this,!0),b=f.length;b--;)f[b].elem===this&&f[b].queue===a&&(f[b].anim.stop(!0),f.splice(b,1));for(b=0;g>b;b++)d[b]&&d[b].finish&&d[b].finish.call(this);delete c.finish})}}),m.each(["toggle","show","hide"],function(a,b){var c=m.fn[b];m.fn[b]=function(a,d,e){return null==a||"boolean"==typeof a?c.apply(this,arguments):this.animate(gb(b,!0),a,d,e)}}),m.each({slideDown:gb("show"),slideUp:gb("hide"),slideToggle:gb("toggle"),fadeIn:{opacity:"show"},fadeOut:{opacity:"hide"},fadeToggle:{opacity:"toggle"}},function(a,b){m.fn[a]=function(a,c,d){return this.animate(b,a,c,d)}}),m.timers=[],m.fx.tick=function(){var a,b=m.timers,c=0;for($a=m.now();c<b.length;c++)a=b[c],a()||b[c]!==a||b.splice(c--,1);b.length||m.fx.stop(),$a=void 0},m.fx.timer=function(a){m.timers.push(a),a()?m.fx.start():m.timers.pop()},m.fx.interval=13,m.fx.start=function(){_a||(_a=setInterval(m.fx.tick,m.fx.interval))},m.fx.stop=function(){clearInterval(_a),_a=null},m.fx.speeds={slow:600,fast:200,_default:400},m.fn.delay=function(a,b){return a=m.fx?m.fx.speeds[a]||a:a,b=b||"fx",this.queue(b,function(b,c){var d=setTimeout(b,a);c.stop=function(){clearTimeout(d)}})},function(){var a,b,c,d,e;b=y.createElement("div"),b.setAttribute("className","t"),b.innerHTML="  <link/><table></table><a href='/a'>a</a><input type='checkbox'/>",d=b.getElementsByTagName("a")[0],c=y.createElement("select"),e=c.appendChild(y.createElement("option")),a=b.getElementsByTagName("input")[0],d.style.cssText="top:1px",k.getSetAttribute="t"!==b.className,k.style=/top/.test(d.getAttribute("style")),k.hrefNormalized="/a"===d.getAttribute("href"),k.checkOn=!!a.value,k.optSelected=e.selected,k.enctype=!!y.createElement("form").enctype,c.disabled=!0,k.optDisabled=!e.disabled,a=y.createElement("input"),a.setAttribute("value",""),k.input=""===a.getAttribute("value"),a.value="t",a.setAttribute("type","radio"),k.radioValue="t"===a.value}();var lb=/\r/g;m.fn.extend({val:function(a){var b,c,d,e=this[0];{if(arguments.length)return d=m.isFunction(a),this.each(function(c){var e;1===this.nodeType&&(e=d?a.call(this,c,m(this).val()):a,null==e?e="":"number"==typeof e?e+="":m.isArray(e)&&(e=m.map(e,function(a){return null==a?"":a+""})),b=m.valHooks[this.type]||m.valHooks[this.nodeName.toLowerCase()],b&&"set"in b&&void 0!==b.set(this,e,"value")||(this.value=e))});if(e)return b=m.valHooks[e.type]||m.valHooks[e.nodeName.toLowerCase()],b&&"get"in b&&void 0!==(c=b.get(e,"value"))?c:(c=e.value,"string"==typeof c?c.replace(lb,""):null==c?"":c)}}}),m.extend({valHooks:{option:{get:function(a){var b=m.find.attr(a,"value");return null!=b?b:m.trim(m.text(a))}},select:{get:function(a){for(var b,c,d=a.options,e=a.selectedIndex,f="select-one"===a.type||0>e,g=f?null:[],h=f?e+1:d.length,i=0>e?h:f?e:0;h>i;i++)if(c=d[i],!(!c.selected&&i!==e||(k.optDisabled?c.disabled:null!==c.getAttribute("disabled"))||c.parentNode.disabled&&m.nodeName(c.parentNode,"optgroup"))){if(b=m(c).val(),f)return b;g.push(b)}return g},set:function(a,b){var c,d,e=a.options,f=m.makeArray(b),g=e.length;while(g--)if(d=e[g],m.inArray(m.valHooks.option.get(d),f)>=0)try{d.selected=c=!0}catch(h){d.scrollHeight}else d.selected=!1;return c||(a.selectedIndex=-1),e}}}}),m.each(["radio","checkbox"],function(){m.valHooks[this]={set:function(a,b){return m.isArray(b)?a.checked=m.inArray(m(a).val(),b)>=0:void 0}},k.checkOn||(m.valHooks[this].get=function(a){return null===a.getAttribute("value")?"on":a.value})});var mb,nb,ob=m.expr.attrHandle,pb=/^(?:checked|selected)$/i,qb=k.getSetAttribute,rb=k.input;m.fn.extend({attr:function(a,b){return V(this,m.attr,a,b,arguments.length>1)},removeAttr:function(a){return this.each(function(){m.removeAttr(this,a)})}}),m.extend({attr:function(a,b,c){var d,e,f=a.nodeType;if(a&&3!==f&&8!==f&&2!==f)return typeof a.getAttribute===K?m.prop(a,b,c):(1===f&&m.isXMLDoc(a)||(b=b.toLowerCase(),d=m.attrHooks[b]||(m.expr.match.bool.test(b)?nb:mb)),void 0===c?d&&"get"in d&&null!==(e=d.get(a,b))?e:(e=m.find.attr(a,b),null==e?void 0:e):null!==c?d&&"set"in d&&void 0!==(e=d.set(a,c,b))?e:(a.setAttribute(b,c+""),c):void m.removeAttr(a,b))},removeAttr:function(a,b){var c,d,e=0,f=b&&b.match(E);if(f&&1===a.nodeType)while(c=f[e++])d=m.propFix[c]||c,m.expr.match.bool.test(c)?rb&&qb||!pb.test(c)?a[d]=!1:a[m.camelCase("default-"+c)]=a[d]=!1:m.attr(a,c,""),a.removeAttribute(qb?c:d)},attrHooks:{type:{set:function(a,b){if(!k.radioValue&&"radio"===b&&m.nodeName(a,"input")){var c=a.value;return a.setAttribute("type",b),c&&(a.value=c),b}}}}}),nb={set:function(a,b,c){return b===!1?m.removeAttr(a,c):rb&&qb||!pb.test(c)?a.setAttribute(!qb&&m.propFix[c]||c,c):a[m.camelCase("default-"+c)]=a[c]=!0,c}},m.each(m.expr.match.bool.source.match(/\w+/g),function(a,b){var c=ob[b]||m.find.attr;ob[b]=rb&&qb||!pb.test(b)?function(a,b,d){var e,f;return d||(f=ob[b],ob[b]=e,e=null!=c(a,b,d)?b.toLowerCase():null,ob[b]=f),e}:function(a,b,c){return c?void 0:a[m.camelCase("default-"+b)]?b.toLowerCase():null}}),rb&&qb||(m.attrHooks.value={set:function(a,b,c){return m.nodeName(a,"input")?void(a.defaultValue=b):mb&&mb.set(a,b,c)}}),qb||(mb={set:function(a,b,c){var d=a.getAttributeNode(c);return d||a.setAttributeNode(d=a.ownerDocument.createAttribute(c)),d.value=b+="","value"===c||b===a.getAttribute(c)?b:void 0}},ob.id=ob.name=ob.coords=function(a,b,c){var d;return c?void 0:(d=a.getAttributeNode(b))&&""!==d.value?d.value:null},m.valHooks.button={get:function(a,b){var c=a.getAttributeNode(b);return c&&c.specified?c.value:void 0},set:mb.set},m.attrHooks.contenteditable={set:function(a,b,c){mb.set(a,""===b?!1:b,c)}},m.each(["width","height"],function(a,b){m.attrHooks[b]={set:function(a,c){return""===c?(a.setAttribute(b,"auto"),c):void 0}}})),k.style||(m.attrHooks.style={get:function(a){return a.style.cssText||void 0},set:function(a,b){return a.style.cssText=b+""}});var sb=/^(?:input|select|textarea|button|object)$/i,tb=/^(?:a|area)$/i;m.fn.extend({prop:function(a,b){return V(this,m.prop,a,b,arguments.length>1)},removeProp:function(a){return a=m.propFix[a]||a,this.each(function(){try{this[a]=void 0,delete this[a]}catch(b){}})}}),m.extend({propFix:{"for":"htmlFor","class":"className"},prop:function(a,b,c){var d,e,f,g=a.nodeType;if(a&&3!==g&&8!==g&&2!==g)return f=1!==g||!m.isXMLDoc(a),f&&(b=m.propFix[b]||b,e=m.propHooks[b]),void 0!==c?e&&"set"in e&&void 0!==(d=e.set(a,c,b))?d:a[b]=c:e&&"get"in e&&null!==(d=e.get(a,b))?d:a[b]},propHooks:{tabIndex:{get:function(a){var b=m.find.attr(a,"tabindex");return b?parseInt(b,10):sb.test(a.nodeName)||tb.test(a.nodeName)&&a.href?0:-1}}}}),k.hrefNormalized||m.each(["href","src"],function(a,b){m.propHooks[b]={get:function(a){return a.getAttribute(b,4)}}}),k.optSelected||(m.propHooks.selected={get:function(a){var b=a.parentNode;return b&&(b.selectedIndex,b.parentNode&&b.parentNode.selectedIndex),null}}),m.each(["tabIndex","readOnly","maxLength","cellSpacing","cellPadding","rowSpan","colSpan","useMap","frameBorder","contentEditable"],function(){m.propFix[this.toLowerCase()]=this}),k.enctype||(m.propFix.enctype="encoding");var ub=/[\t\r\n\f]/g;m.fn.extend({addClass:function(a){var b,c,d,e,f,g,h=0,i=this.length,j="string"==typeof a&&a;if(m.isFunction(a))return this.each(function(b){m(this).addClass(a.call(this,b,this.className))});if(j)for(b=(a||"").match(E)||[];i>h;h++)if(c=this[h],d=1===c.nodeType&&(c.className?(" "+c.className+" ").replace(ub," "):" ")){f=0;while(e=b[f++])d.indexOf(" "+e+" ")<0&&(d+=e+" ");g=m.trim(d),c.className!==g&&(c.className=g)}return this},removeClass:function(a){var b,c,d,e,f,g,h=0,i=this.length,j=0===arguments.length||"string"==typeof a&&a;if(m.isFunction(a))return this.each(function(b){m(this).removeClass(a.call(this,b,this.className))});if(j)for(b=(a||"").match(E)||[];i>h;h++)if(c=this[h],d=1===c.nodeType&&(c.className?(" "+c.className+" ").replace(ub," "):"")){f=0;while(e=b[f++])while(d.indexOf(" "+e+" ")>=0)d=d.replace(" "+e+" "," ");g=a?m.trim(d):"",c.className!==g&&(c.className=g)}return this},toggleClass:function(a,b){var c=typeof a;return"boolean"==typeof b&&"string"===c?b?this.addClass(a):this.removeClass(a):this.each(m.isFunction(a)?function(c){m(this).toggleClass(a.call(this,c,this.className,b),b)}:function(){if("string"===c){var b,d=0,e=m(this),f=a.match(E)||[];while(b=f[d++])e.hasClass(b)?e.removeClass(b):e.addClass(b)}else(c===K||"boolean"===c)&&(this.className&&m._data(this,"__className__",this.className),this.className=this.className||a===!1?"":m._data(this,"__className__")||"")})},hasClass:function(a){for(var b=" "+a+" ",c=0,d=this.length;d>c;c++)if(1===this[c].nodeType&&(" "+this[c].className+" ").replace(ub," ").indexOf(b)>=0)return!0;return!1}}),m.each("blur focus focusin focusout load resize scroll unload click dblclick mousedown mouseup mousemove mouseover mouseout mouseenter mouseleave change select submit keydown keypress keyup error contextmenu".split(" "),function(a,b){m.fn[b]=function(a,c){return arguments.length>0?this.on(b,null,a,c):this.trigger(b)}}),m.fn.extend({hover:function(a,b){return this.mouseenter(a).mouseleave(b||a)},bind:function(a,b,c){return this.on(a,null,b,c)},unbind:function(a,b){return this.off(a,null,b)},delegate:function(a,b,c,d){return this.on(b,a,c,d)},undelegate:function(a,b,c){return 1===arguments.length?this.off(a,"**"):this.off(b,a||"**",c)}});var vb=m.now(),wb=/\?/,xb=/(,)|(\[|{)|(}|])|"(?:[^"\\\r\n]|\\["\\\/bfnrt]|\\u[\da-fA-F]{4})*"\s*:?|true|false|null|-?(?!0\d)\d+(?:\.\d+|)(?:[eE][+-]?\d+|)/g;m.parseJSON=function(b){if(a.JSON&&a.JSON.parse)return a.JSON.parse(b+"");var c,d=null,e=m.trim(b+"");return e&&!m.trim(e.replace(xb,function(a,b,e,f){return c&&b&&(d=0),0===d?a:(c=e||b,d+=!f-!e,"")}))?Function("return "+e)():m.error("Invalid JSON: "+b)},m.parseXML=function(b){var c,d;if(!b||"string"!=typeof b)return null;try{a.DOMParser?(d=new DOMParser,c=d.parseFromString(b,"text/xml")):(c=new ActiveXObject("Microsoft.XMLDOM"),c.async="false",c.loadXML(b))}catch(e){c=void 0}return c&&c.documentElement&&!c.getElementsByTagName("parsererror").length||m.error("Invalid XML: "+b),c};var yb,zb,Ab=/#.*$/,Bb=/([?&])_=[^&]*/,Cb=/^(.*?):[ \t]*([^\r\n]*)\r?$/gm,Db=/^(?:about|app|app-storage|.+-extension|file|res|widget):$/,Eb=/^(?:GET|HEAD)$/,Fb=/^\/\//,Gb=/^([\w.+-]+:)(?:\/\/(?:[^\/?#]*@|)([^\/?#:]*)(?::(\d+)|)|)/,Hb={},Ib={},Jb="*/".concat("*");try{zb=location.href}catch(Kb){zb=y.createElement("a"),zb.href="",zb=zb.href}yb=Gb.exec(zb.toLowerCase())||[];function Lb(a){return function(b,c){"string"!=typeof b&&(c=b,b="*");var d,e=0,f=b.toLowerCase().match(E)||[];if(m.isFunction(c))while(d=f[e++])"+"===d.charAt(0)?(d=d.slice(1)||"*",(a[d]=a[d]||[]).unshift(c)):(a[d]=a[d]||[]).push(c)}}function Mb(a,b,c,d){var e={},f=a===Ib;function g(h){var i;return e[h]=!0,m.each(a[h]||[],function(a,h){var j=h(b,c,d);return"string"!=typeof j||f||e[j]?f?!(i=j):void 0:(b.dataTypes.unshift(j),g(j),!1)}),i}return g(b.dataTypes[0])||!e["*"]&&g("*")}function Nb(a,b){var c,d,e=m.ajaxSettings.flatOptions||{};for(d in b)void 0!==b[d]&&((e[d]?a:c||(c={}))[d]=b[d]);return c&&m.extend(!0,a,c),a}function Ob(a,b,c){var d,e,f,g,h=a.contents,i=a.dataTypes;while("*"===i[0])i.shift(),void 0===e&&(e=a.mimeType||b.getResponseHeader("Content-Type"));if(e)for(g in h)if(h[g]&&h[g].test(e)){i.unshift(g);break}if(i[0]in c)f=i[0];else{for(g in c){if(!i[0]||a.converters[g+" "+i[0]]){f=g;break}d||(d=g)}f=f||d}return f?(f!==i[0]&&i.unshift(f),c[f]):void 0}function Pb(a,b,c,d){var e,f,g,h,i,j={},k=a.dataTypes.slice();if(k[1])for(g in a.converters)j[g.toLowerCase()]=a.converters[g];f=k.shift();while(f)if(a.responseFields[f]&&(c[a.responseFields[f]]=b),!i&&d&&a.dataFilter&&(b=a.dataFilter(b,a.dataType)),i=f,f=k.shift())if("*"===f)f=i;else if("*"!==i&&i!==f){if(g=j[i+" "+f]||j["* "+f],!g)for(e in j)if(h=e.split(" "),h[1]===f&&(g=j[i+" "+h[0]]||j["* "+h[0]])){g===!0?g=j[e]:j[e]!==!0&&(f=h[0],k.unshift(h[1]));break}if(g!==!0)if(g&&a["throws"])b=g(b);else try{b=g(b)}catch(l){return{state:"parsererror",error:g?l:"No conversion from "+i+" to "+f}}}return{state:"success",data:b}}m.extend({active:0,lastModified:{},etag:{},ajaxSettings:{url:zb,type:"GET",isLocal:Db.test(yb[1]),global:!0,processData:!0,async:!0,contentType:"application/x-www-form-urlencoded; charset=UTF-8",accepts:{"*":Jb,text:"text/plain",html:"text/html",xml:"application/xml, text/xml",json:"application/json, text/javascript"},contents:{xml:/xml/,html:/html/,json:/json/},responseFields:{xml:"responseXML",text:"responseText",json:"responseJSON"},converters:{"* text":String,"text html":!0,"text json":m.parseJSON,"text xml":m.parseXML},flatOptions:{url:!0,context:!0}},ajaxSetup:function(a,b){return b?Nb(Nb(a,m.ajaxSettings),b):Nb(m.ajaxSettings,a)},ajaxPrefilter:Lb(Hb),ajaxTransport:Lb(Ib),ajax:function(a,b){"object"==typeof a&&(b=a,a=void 0),b=b||{};var c,d,e,f,g,h,i,j,k=m.ajaxSetup({},b),l=k.context||k,n=k.context&&(l.nodeType||l.jquery)?m(l):m.event,o=m.Deferred(),p=m.Callbacks("once memory"),q=k.statusCode||{},r={},s={},t=0,u="canceled",v={readyState:0,getResponseHeader:function(a){var b;if(2===t){if(!j){j={};while(b=Cb.exec(f))j[b[1].toLowerCase()]=b[2]}b=j[a.toLowerCase()]}return null==b?null:b},getAllResponseHeaders:function(){return 2===t?f:null},setRequestHeader:function(a,b){var c=a.toLowerCase();return t||(a=s[c]=s[c]||a,r[a]=b),this},overrideMimeType:function(a){return t||(k.mimeType=a),this},statusCode:function(a){var b;if(a)if(2>t)for(b in a)q[b]=[q[b],a[b]];else v.always(a[v.status]);return this},abort:function(a){var b=a||u;return i&&i.abort(b),x(0,b),this}};if(o.promise(v).complete=p.add,v.success=v.done,v.error=v.fail,k.url=((a||k.url||zb)+"").replace(Ab,"").replace(Fb,yb[1]+"//"),k.type=b.method||b.type||k.method||k.type,k.dataTypes=m.trim(k.dataType||"*").toLowerCase().match(E)||[""],null==k.crossDomain&&(c=Gb.exec(k.url.toLowerCase()),k.crossDomain=!(!c||c[1]===yb[1]&&c[2]===yb[2]&&(c[3]||("http:"===c[1]?"80":"443"))===(yb[3]||("http:"===yb[1]?"80":"443")))),k.data&&k.processData&&"string"!=typeof k.data&&(k.data=m.param(k.data,k.traditional)),Mb(Hb,k,b,v),2===t)return v;h=m.event&&k.global,h&&0===m.active++&&m.event.trigger("ajaxStart"),k.type=k.type.toUpperCase(),k.hasContent=!Eb.test(k.type),e=k.url,k.hasContent||(k.data&&(e=k.url+=(wb.test(e)?"&":"?")+k.data,delete k.data),k.cache===!1&&(k.url=Bb.test(e)?e.replace(Bb,"$1_="+vb++):e+(wb.test(e)?"&":"?")+"_="+vb++)),k.ifModified&&(m.lastModified[e]&&v.setRequestHeader("If-Modified-Since",m.lastModified[e]),m.etag[e]&&v.setRequestHeader("If-None-Match",m.etag[e])),(k.data&&k.hasContent&&k.contentType!==!1||b.contentType)&&v.setRequestHeader("Content-Type",k.contentType),v.setRequestHeader("Accept",k.dataTypes[0]&&k.accepts[k.dataTypes[0]]?k.accepts[k.dataTypes[0]]+("*"!==k.dataTypes[0]?", "+Jb+"; q=0.01":""):k.accepts["*"]);for(d in k.headers)v.setRequestHeader(d,k.headers[d]);if(k.beforeSend&&(k.beforeSend.call(l,v,k)===!1||2===t))return v.abort();u="abort";for(d in{success:1,error:1,complete:1})v[d](k[d]);if(i=Mb(Ib,k,b,v)){v.readyState=1,h&&n.trigger("ajaxSend",[v,k]),k.async&&k.timeout>0&&(g=setTimeout(function(){v.abort("timeout")},k.timeout));try{t=1,i.send(r,x)}catch(w){if(!(2>t))throw w;x(-1,w)}}else x(-1,"No Transport");function x(a,b,c,d){var j,r,s,u,w,x=b;2!==t&&(t=2,g&&clearTimeout(g),i=void 0,f=d||"",v.readyState=a>0?4:0,j=a>=200&&300>a||304===a,c&&(u=Ob(k,v,c)),u=Pb(k,u,v,j),j?(k.ifModified&&(w=v.getResponseHeader("Last-Modified"),w&&(m.lastModified[e]=w),w=v.getResponseHeader("etag"),w&&(m.etag[e]=w)),204===a||"HEAD"===k.type?x="nocontent":304===a?x="notmodified":(x=u.state,r=u.data,s=u.error,j=!s)):(s=x,(a||!x)&&(x="error",0>a&&(a=0))),v.status=a,v.statusText=(b||x)+"",j?o.resolveWith(l,[r,x,v]):o.rejectWith(l,[v,x,s]),v.statusCode(q),q=void 0,h&&n.trigger(j?"ajaxSuccess":"ajaxError",[v,k,j?r:s]),p.fireWith(l,[v,x]),h&&(n.trigger("ajaxComplete",[v,k]),--m.active||m.event.trigger("ajaxStop")))}return v},getJSON:function(a,b,c){return m.get(a,b,c,"json")},getScript:function(a,b){return m.get(a,void 0,b,"script")}}),m.each(["get","post"],function(a,b){m[b]=function(a,c,d,e){return m.isFunction(c)&&(e=e||d,d=c,c=void 0),m.ajax({url:a,type:b,dataType:e,data:c,success:d})}}),m._evalUrl=function(a){return m.ajax({url:a,type:"GET",dataType:"script",async:!1,global:!1,"throws":!0})},m.fn.extend({wrapAll:function(a){if(m.isFunction(a))return this.each(function(b){m(this).wrapAll(a.call(this,b))});if(this[0]){var b=m(a,this[0].ownerDocument).eq(0).clone(!0);this[0].parentNode&&b.insertBefore(this[0]),b.map(function(){var a=this;while(a.firstChild&&1===a.firstChild.nodeType)a=a.firstChild;return a}).append(this)}return this},wrapInner:function(a){return this.each(m.isFunction(a)?function(b){m(this).wrapInner(a.call(this,b))}:function(){var b=m(this),c=b.contents();c.length?c.wrapAll(a):b.append(a)})},wrap:function(a){var b=m.isFunction(a);return this.each(function(c){m(this).wrapAll(b?a.call(this,c):a)})},unwrap:function(){return this.parent().each(function(){m.nodeName(this,"body")||m(this).replaceWith(this.childNodes)}).end()}}),m.expr.filters.hidden=function(a){return a.offsetWidth<=0&&a.offsetHeight<=0||!k.reliableHiddenOffsets()&&"none"===(a.style&&a.style.display||m.css(a,"display"))},m.expr.filters.visible=function(a){return!m.expr.filters.hidden(a)};var Qb=/%20/g,Rb=/\[\]$/,Sb=/\r?\n/g,Tb=/^(?:submit|button|image|reset|file)$/i,Ub=/^(?:input|select|textarea|keygen)/i;function Vb(a,b,c,d){var e;if(m.isArray(b))m.each(b,function(b,e){c||Rb.test(a)?d(a,e):Vb(a+"["+("object"==typeof e?b:"")+"]",e,c,d)});else if(c||"object"!==m.type(b))d(a,b);else for(e in b)Vb(a+"["+e+"]",b[e],c,d)}m.param=function(a,b){var c,d=[],e=function(a,b){b=m.isFunction(b)?b():null==b?"":b,d[d.length]=encodeURIComponent(a)+"="+encodeURIComponent(b)};if(void 0===b&&(b=m.ajaxSettings&&m.ajaxSettings.traditional),m.isArray(a)||a.jquery&&!m.isPlainObject(a))m.each(a,function(){e(this.name,this.value)});else for(c in a)Vb(c,a[c],b,e);return d.join("&").replace(Qb,"+")},m.fn.extend({serialize:function(){return m.param(this.serializeArray())},serializeArray:function(){return this.map(function(){var a=m.prop(this,"elements");return a?m.makeArray(a):this}).filter(function(){var a=this.type;return this.name&&!m(this).is(":disabled")&&Ub.test(this.nodeName)&&!Tb.test(a)&&(this.checked||!W.test(a))}).map(function(a,b){var c=m(this).val();return null==c?null:m.isArray(c)?m.map(c,function(a){return{name:b.name,value:a.replace(Sb,"\r\n")}}):{name:b.name,value:c.replace(Sb,"\r\n")}}).get()}}),m.ajaxSettings.xhr=void 0!==a.ActiveXObject?function(){return!this.isLocal&&/^(get|post|head|put|delete|options)$/i.test(this.type)&&Zb()||$b()}:Zb;var Wb=0,Xb={},Yb=m.ajaxSettings.xhr();a.attachEvent&&a.attachEvent("onunload",function(){for(var a in Xb)Xb[a](void 0,!0)}),k.cors=!!Yb&&"withCredentials"in Yb,Yb=k.ajax=!!Yb,Yb&&m.ajaxTransport(function(a){if(!a.crossDomain||k.cors){var b;return{send:function(c,d){var e,f=a.xhr(),g=++Wb;if(f.open(a.type,a.url,a.async,a.username,a.password),a.xhrFields)for(e in a.xhrFields)f[e]=a.xhrFields[e];a.mimeType&&f.overrideMimeType&&f.overrideMimeType(a.mimeType),a.crossDomain||c["X-Requested-With"]||(c["X-Requested-With"]="XMLHttpRequest");for(e in c)void 0!==c[e]&&f.setRequestHeader(e,c[e]+"");f.send(a.hasContent&&a.data||null),b=function(c,e){var h,i,j;if(b&&(e||4===f.readyState))if(delete Xb[g],b=void 0,f.onreadystatechange=m.noop,e)4!==f.readyState&&f.abort();else{j={},h=f.status,"string"==typeof f.responseText&&(j.text=f.responseText);try{i=f.statusText}catch(k){i=""}h||!a.isLocal||a.crossDomain?1223===h&&(h=204):h=j.text?200:404}j&&d(h,i,j,f.getAllResponseHeaders())},a.async?4===f.readyState?setTimeout(b):f.onreadystatechange=Xb[g]=b:b()},abort:function(){b&&b(void 0,!0)}}}});function Zb(){try{return new a.XMLHttpRequest}catch(b){}}function $b(){try{return new a.ActiveXObject("Microsoft.XMLHTTP")}catch(b){}}m.ajaxSetup({accepts:{script:"text/javascript, application/javascript, application/ecmascript, application/x-ecmascript"},contents:{script:/(?:java|ecma)script/},converters:{"text script":function(a){return m.globalEval(a),a}}}),m.ajaxPrefilter("script",function(a){void 0===a.cache&&(a.cache=!1),a.crossDomain&&(a.type="GET",a.global=!1)}),m.ajaxTransport("script",function(a){if(a.crossDomain){var b,c=y.head||m("head")[0]||y.documentElement;return{send:function(d,e){b=y.createElement("script"),b.async=!0,a.scriptCharset&&(b.charset=a.scriptCharset),b.src=a.url,b.onload=b.onreadystatechange=function(a,c){(c||!b.readyState||/loaded|complete/.test(b.readyState))&&(b.onload=b.onreadystatechange=null,b.parentNode&&b.parentNode.removeChild(b),b=null,c||e(200,"success"))},c.insertBefore(b,c.firstChild)},abort:function(){b&&b.onload(void 0,!0)}}}});var _b=[],ac=/(=)\?(?=&|$)|\?\?/;m.ajaxSetup({jsonp:"callback",jsonpCallback:function(){var a=_b.pop()||m.expando+"_"+vb++;return this[a]=!0,a}}),m.ajaxPrefilter("json jsonp",function(b,c,d){var e,f,g,h=b.jsonp!==!1&&(ac.test(b.url)?"url":"string"==typeof b.data&&!(b.contentType||"").indexOf("application/x-www-form-urlencoded")&&ac.test(b.data)&&"data");return h||"jsonp"===b.dataTypes[0]?(e=b.jsonpCallback=m.isFunction(b.jsonpCallback)?b.jsonpCallback():b.jsonpCallback,h?b[h]=b[h].replace(ac,"$1"+e):b.jsonp!==!1&&(b.url+=(wb.test(b.url)?"&":"?")+b.jsonp+"="+e),b.converters["script json"]=function(){return g||m.error(e+" was not called"),g[0]},b.dataTypes[0]="json",f=a[e],a[e]=function(){g=arguments},d.always(function(){a[e]=f,b[e]&&(b.jsonpCallback=c.jsonpCallback,_b.push(e)),g&&m.isFunction(f)&&f(g[0]),g=f=void 0}),"script"):void 0}),m.parseHTML=function(a,b,c){if(!a||"string"!=typeof a)return null;"boolean"==typeof b&&(c=b,b=!1),b=b||y;var d=u.exec(a),e=!c&&[];return d?[b.createElement(d[1])]:(d=m.buildFragment([a],b,e),e&&e.length&&m(e).remove(),m.merge([],d.childNodes))};var bc=m.fn.load;m.fn.load=function(a,b,c){if("string"!=typeof a&&bc)return bc.apply(this,arguments);var d,e,f,g=this,h=a.indexOf(" ");return h>=0&&(d=m.trim(a.slice(h,a.length)),a=a.slice(0,h)),m.isFunction(b)?(c=b,b=void 0):b&&"object"==typeof b&&(f="POST"),g.length>0&&m.ajax({url:a,type:f,dataType:"html",data:b}).done(function(a){e=arguments,g.html(d?m("<div>").append(m.parseHTML(a)).find(d):a)}).complete(c&&function(a,b){g.each(c,e||[a.responseText,b,a])}),this},m.each(["ajaxStart","ajaxStop","ajaxComplete","ajaxError","ajaxSuccess","ajaxSend"],function(a,b){m.fn[b]=function(a){return this.on(b,a)}}),m.expr.filters.animated=function(a){return m.grep(m.timers,function(b){return a===b.elem}).length};var cc=a.document.documentElement;function dc(a){return m.isWindow(a)?a:9===a.nodeType?a.defaultView||a.parentWindow:!1}m.offset={setOffset:function(a,b,c){var d,e,f,g,h,i,j,k=m.css(a,"position"),l=m(a),n={};"static"===k&&(a.style.position="relative"),h=l.offset(),f=m.css(a,"top"),i=m.css(a,"left"),j=("absolute"===k||"fixed"===k)&&m.inArray("auto",[f,i])>-1,j?(d=l.position(),g=d.top,e=d.left):(g=parseFloat(f)||0,e=parseFloat(i)||0),m.isFunction(b)&&(b=b.call(a,c,h)),null!=b.top&&(n.top=b.top-h.top+g),null!=b.left&&(n.left=b.left-h.left+e),"using"in b?b.using.call(a,n):l.css(n)}},m.fn.extend({offset:function(a){if(arguments.length)return void 0===a?this:this.each(function(b){m.offset.setOffset(this,a,b)});var b,c,d={top:0,left:0},e=this[0],f=e&&e.ownerDocument;if(f)return b=f.documentElement,m.contains(b,e)?(typeof e.getBoundingClientRect!==K&&(d=e.getBoundingClientRect()),c=dc(f),{top:d.top+(c.pageYOffset||b.scrollTop)-(b.clientTop||0),left:d.left+(c.pageXOffset||b.scrollLeft)-(b.clientLeft||0)}):d},position:function(){if(this[0]){var a,b,c={top:0,left:0},d=this[0];return"fixed"===m.css(d,"position")?b=d.getBoundingClientRect():(a=this.offsetParent(),b=this.offset(),m.nodeName(a[0],"html")||(c=a.offset()),c.top+=m.css(a[0],"borderTopWidth",!0),c.left+=m.css(a[0],"borderLeftWidth",!0)),{top:b.top-c.top-m.css(d,"marginTop",!0),left:b.left-c.left-m.css(d,"marginLeft",!0)}}},offsetParent:function(){return this.map(function(){var a=this.offsetParent||cc;while(a&&!m.nodeName(a,"html")&&"static"===m.css(a,"position"))a=a.offsetParent;return a||cc})}}),m.each({scrollLeft:"pageXOffset",scrollTop:"pageYOffset"},function(a,b){var c=/Y/.test(b);m.fn[a]=function(d){return V(this,function(a,d,e){var f=dc(a);return void 0===e?f?b in f?f[b]:f.document.documentElement[d]:a[d]:void(f?f.scrollTo(c?m(f).scrollLeft():e,c?e:m(f).scrollTop()):a[d]=e)},a,d,arguments.length,null)}}),m.each(["top","left"],function(a,b){m.cssHooks[b]=La(k.pixelPosition,function(a,c){return c?(c=Ja(a,b),Ha.test(c)?m(a).position()[b]+"px":c):void 0})}),m.each({Height:"height",Width:"width"},function(a,b){m.each({padding:"inner"+a,content:b,"":"outer"+a},function(c,d){m.fn[d]=function(d,e){var f=arguments.length&&(c||"boolean"!=typeof d),g=c||(d===!0||e===!0?"margin":"border");return V(this,function(b,c,d){var e;return m.isWindow(b)?b.document.documentElement["client"+a]:9===b.nodeType?(e=b.documentElement,Math.max(b.body["scroll"+a],e["scroll"+a],b.body["offset"+a],e["offset"+a],e["client"+a])):void 0===d?m.css(b,c,g):m.style(b,c,d,g)},b,f?d:void 0,f,null)}})}),m.fn.size=function(){return this.length},m.fn.andSelf=m.fn.addBack,"function"==typeof define&&define.amd&&define("jquery",[],function(){return m});var ec=a.jQuery,fc=a.$;return m.noConflict=function(b){return a.$===m&&(a.$=fc),b&&a.jQuery===m&&(a.jQuery=ec),m},typeof b===K&&(a.jQuery=a.$=m),m});
+</script>
+<meta name="viewport" content="width=device-width, initial-scale=1" />
+<style type="text/css">@font-face {
+font-family: 'Raleway';
+font-style: normal;
+font-weight: 400;
+src: url(data:application/x-font-truetype;base64,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) format('truetype');
+}
+@font-face {
+font-family: 'Raleway';
+font-style: normal;
+font-weight: 700;
+src: url(data:application/x-font-truetype;base64,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) format('truetype');
+}
+html{font-family:sans-serif;-ms-text-size-adjust:100%;-webkit-text-size-adjust:100%}body{margin:0}article,aside,details,figcaption,figure,footer,header,hgroup,main,menu,nav,section,summary{display:block}audio,canvas,progress,video{display:inline-block;vertical-align:baseline}audio:not([controls]){display:none;height:0}[hidden],template{display:none}a{background-color:transparent}a:active,a:hover{outline:0}abbr[title]{border-bottom:1px dotted}b,strong{font-weight:bold}dfn{font-style:italic}h1{font-size:2em;margin:0.67em 0}mark{background:#ff0;color:#000}small{font-size:80%}sub,sup{font-size:75%;line-height:0;position:relative;vertical-align:baseline}sup{top:-0.5em}sub{bottom:-0.25em}img{border:0}svg:not(:root){overflow:hidden}figure{margin:1em 40px}hr{-webkit-box-sizing:content-box;-moz-box-sizing:content-box;box-sizing:content-box;height:0}pre{overflow:auto}code,kbd,pre,samp{font-family:monospace, monospace;font-size:1em}button,input,optgroup,select,textarea{color:inherit;font:inherit;margin:0}button{overflow:visible}button,select{text-transform:none}button,html input[type="button"],input[type="reset"],input[type="submit"]{-webkit-appearance:button;cursor:pointer}button[disabled],html input[disabled]{cursor:default}button::-moz-focus-inner,input::-moz-focus-inner{border:0;padding:0}input{line-height:normal}input[type="checkbox"],input[type="radio"]{-webkit-box-sizing:border-box;-moz-box-sizing:border-box;box-sizing:border-box;padding:0}input[type="number"]::-webkit-inner-spin-button,input[type="number"]::-webkit-outer-spin-button{height:auto}input[type="search"]{-webkit-appearance:textfield;-webkit-box-sizing:content-box;-moz-box-sizing:content-box;box-sizing:content-box}input[type="search"]::-webkit-search-cancel-button,input[type="search"]::-webkit-search-decoration{-webkit-appearance:none}fieldset{border:1px solid #c0c0c0;margin:0 2px;padding:0.35em 0.625em 0.75em}legend{border:0;padding:0}textarea{overflow:auto}optgroup{font-weight:bold}table{border-collapse:collapse;border-spacing:0}td,th{padding:0}@media print{*,*:before,*:after{background:transparent !important;color:#000 !important;-webkit-box-shadow:none !important;box-shadow:none !important;text-shadow:none !important}a,a:visited{text-decoration:underline}a[href]:after{content:" (" attr(href) ")"}abbr[title]:after{content:" (" attr(title) ")"}a[href^="#"]:after,a[href^="javascript:"]:after{content:""}pre,blockquote{border:1px solid #999;page-break-inside:avoid}thead{display:table-header-group}tr,img{page-break-inside:avoid}img{max-width:100% !important}p,h2,h3{orphans:3;widows:3}h2,h3{page-break-after:avoid}.navbar{display:none}.btn>.caret,.dropup>.btn>.caret{border-top-color:#000 !important}.label{border:1px solid #000}.table{border-collapse:collapse !important}.table td,.table th{background-color:#fff !important}.table-bordered th,.table-bordered td{border:1px solid #ddd !important}}@font-face{font-family:'Glyphicons Halflings';src:url(data:application/vnd.ms-fontobject;base64,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);src:url(data:application/vnd.ms-fontobject;base64,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) format('embedded-opentype'),url(data:application/font-woff;base64,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) format('woff'),url(data:application/x-font-truetype;base64,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) format('truetype'),url(data:image/svg+xml;base64,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) format('svg')}.glyphicon{position:relative;top:1px;display:inline-block;font-family:'Glyphicons Halflings';font-style:normal;font-weight:normal;line-height:1;-webkit-font-smoothing:antialiased;-moz-osx-font-smoothing:grayscale}.glyphicon-asterisk:before{content:"\002a"}.glyphicon-plus:before{content:"\002b"}.glyphicon-euro:before,.glyphicon-eur:before{content:"\20ac"}.glyphicon-minus:before{content:"\2212"}.glyphicon-cloud:before{content:"\2601"}.glyphicon-envelope:before{content:"\2709"}.glyphicon-pencil:before{content:"\270f"}.glyphicon-glass:before{content:"\e001"}.glyphicon-music:before{content:"\e002"}.glyphicon-search:before{content:"\e003"}.glyphicon-heart:before{content:"\e005"}.glyphicon-star:before{content:"\e006"}.glyphicon-star-empty:before{content:"\e007"}.glyphicon-user:before{content:"\e008"}.glyphicon-film:before{content:"\e009"}.glyphicon-th-large:before{content:"\e010"}.glyphicon-th:before{content:"\e011"}.glyphicon-th-list:before{content:"\e012"}.glyphicon-ok:before{content:"\e013"}.glyphicon-remove:before{content:"\e014"}.glyphicon-zoom-in:before{content:"\e015"}.glyphicon-zoom-out:before{content:"\e016"}.glyphicon-off:before{content:"\e017"}.glyphicon-signal:before{content:"\e018"}.glyphicon-cog:before{content:"\e019"}.glyphicon-trash:before{content:"\e020"}.glyphicon-home:before{content:"\e021"}.glyphicon-file:before{content:"\e022"}.glyphicon-time:before{content:"\e023"}.glyphicon-road:before{content:"\e024"}.glyphicon-download-alt:before{content:"\e025"}.glyphicon-download:before{content:"\e026"}.glyphicon-upload:before{content:"\e027"}.glyphicon-inbox:before{content:"\e028"}.glyphicon-play-circle:before{content:"\e029"}.glyphicon-repeat:before{content:"\e030"}.glyphicon-refresh:before{content:"\e031"}.glyphicon-list-alt:before{content:"\e032"}.glyphicon-lock:before{content:"\e033"}.glyphicon-flag:before{content:"\e034"}.glyphicon-headphones:before{content:"\e035"}.glyphicon-volume-off:before{content:"\e036"}.glyphicon-volume-down:before{content:"\e037"}.glyphicon-volume-up:before{content:"\e038"}.glyphicon-qrcode:before{content:"\e039"}.glyphicon-barcode:before{content:"\e040"}.glyphicon-tag:before{content:"\e041"}.glyphicon-tags:before{content:"\e042"}.glyphicon-book:before{content:"\e043"}.glyphicon-bookmark:before{content:"\e044"}.glyphicon-print:before{content:"\e045"}.glyphicon-camera:before{content:"\e046"}.glyphicon-font:before{content:"\e047"}.glyphicon-bold:before{content:"\e048"}.glyphicon-italic:before{content:"\e049"}.glyphicon-text-height:before{content:"\e050"}.glyphicon-text-width:before{content:"\e051"}.glyphicon-align-left:before{content:"\e052"}.glyphicon-align-center:before{content:"\e053"}.glyphicon-align-right:before{content:"\e054"}.glyphicon-align-justify:before{content:"\e055"}.glyphicon-list:before{content:"\e056"}.glyphicon-indent-left:before{content:"\e057"}.glyphicon-indent-right:before{content:"\e058"}.glyphicon-facetime-video:before{content:"\e059"}.glyphicon-picture:before{content:"\e060"}.glyphicon-map-marker:before{content:"\e062"}.glyphicon-adjust:before{content:"\e063"}.glyphicon-tint:before{content:"\e064"}.glyphicon-edit:before{content:"\e065"}.glyphicon-share:before{content:"\e066"}.glyphicon-check:before{content:"\e067"}.glyphicon-move:before{content:"\e068"}.glyphicon-step-backward:before{content:"\e069"}.glyphicon-fast-backward:before{content:"\e070"}.glyphicon-backward:before{content:"\e071"}.glyphicon-play:before{content:"\e072"}.glyphicon-pause:before{content:"\e073"}.glyphicon-stop:before{content:"\e074"}.glyphicon-forward:before{content:"\e075"}.glyphicon-fast-forward:before{content:"\e076"}.glyphicon-step-forward:before{content:"\e077"}.glyphicon-eject:before{content:"\e078"}.glyphicon-chevron-left:before{content:"\e079"}.glyphicon-chevron-right:before{content:"\e080"}.glyphicon-plus-sign:before{content:"\e081"}.glyphicon-minus-sign:before{content:"\e082"}.glyphicon-remove-sign:before{content:"\e083"}.glyphicon-ok-sign:before{content:"\e084"}.glyphicon-question-sign:before{content:"\e085"}.glyphicon-info-sign:before{content:"\e086"}.glyphicon-screenshot:before{content:"\e087"}.glyphicon-remove-circle:before{content:"\e088"}.glyphicon-ok-circle:before{content:"\e089"}.glyphicon-ban-circle:before{content:"\e090"}.glyphicon-arrow-left:before{content:"\e091"}.glyphicon-arrow-right:before{content:"\e092"}.glyphicon-arrow-up:before{content:"\e093"}.glyphicon-arrow-down:before{content:"\e094"}.glyphicon-share-alt:before{content:"\e095"}.glyphicon-resize-full:before{content:"\e096"}.glyphicon-resize-small:before{content:"\e097"}.glyphicon-exclamation-sign:before{content:"\e101"}.glyphicon-gift:before{content:"\e102"}.glyphicon-leaf:before{content:"\e103"}.glyphicon-fire:before{content:"\e104"}.glyphicon-eye-open:before{content:"\e105"}.glyphicon-eye-close:before{content:"\e106"}.glyphicon-warning-sign:before{content:"\e107"}.glyphicon-plane:before{content:"\e108"}.glyphicon-calendar:before{content:"\e109"}.glyphicon-random:before{content:"\e110"}.glyphicon-comment:before{content:"\e111"}.glyphicon-magnet:before{content:"\e112"}.glyphicon-chevron-up:before{content:"\e113"}.glyphicon-chevron-down:before{content:"\e114"}.glyphicon-retweet:before{content:"\e115"}.glyphicon-shopping-cart:before{content:"\e116"}.glyphicon-folder-close:before{content:"\e117"}.glyphicon-folder-open:before{content:"\e118"}.glyphicon-resize-vertical:before{content:"\e119"}.glyphicon-resize-horizontal:before{content:"\e120"}.glyphicon-hdd:before{content:"\e121"}.glyphicon-bullhorn:before{content:"\e122"}.glyphicon-bell:before{content:"\e123"}.glyphicon-certificate:before{content:"\e124"}.glyphicon-thumbs-up:before{content:"\e125"}.glyphicon-thumbs-down:before{content:"\e126"}.glyphicon-hand-right:before{content:"\e127"}.glyphicon-hand-left:before{content:"\e128"}.glyphicon-hand-up:before{content:"\e129"}.glyphicon-hand-down:before{content:"\e130"}.glyphicon-circle-arrow-right:before{content:"\e131"}.glyphicon-circle-arrow-left:before{content:"\e132"}.glyphicon-circle-arrow-up:before{content:"\e133"}.glyphicon-circle-arrow-down:before{content:"\e134"}.glyphicon-globe:before{content:"\e135"}.glyphicon-wrench:before{content:"\e136"}.glyphicon-tasks:before{content:"\e137"}.glyphicon-filter:before{content:"\e138"}.glyphicon-briefcase:before{content:"\e139"}.glyphicon-fullscreen:before{content:"\e140"}.glyphicon-dashboard:before{content:"\e141"}.glyphicon-paperclip:before{content:"\e142"}.glyphicon-heart-empty:before{content:"\e143"}.glyphicon-link:before{content:"\e144"}.glyphicon-phone:before{content:"\e145"}.glyphicon-pushpin:before{content:"\e146"}.glyphicon-usd:before{content:"\e148"}.glyphicon-gbp:before{content:"\e149"}.glyphicon-sort:before{content:"\e150"}.glyphicon-sort-by-alphabet:before{content:"\e151"}.glyphicon-sort-by-alphabet-alt:before{content:"\e152"}.glyphicon-sort-by-order:before{content:"\e153"}.glyphicon-sort-by-order-alt:before{content:"\e154"}.glyphicon-sort-by-attributes:before{content:"\e155"}.glyphicon-sort-by-attributes-alt:before{content:"\e156"}.glyphicon-unchecked:before{content:"\e157"}.glyphicon-expand:before{content:"\e158"}.glyphicon-collapse-down:before{content:"\e159"}.glyphicon-collapse-up:before{content:"\e160"}.glyphicon-log-in:before{content:"\e161"}.glyphicon-flash:before{content:"\e162"}.glyphicon-log-out:before{content:"\e163"}.glyphicon-new-window:before{content:"\e164"}.glyphicon-record:before{content:"\e165"}.glyphicon-save:before{content:"\e166"}.glyphicon-open:before{content:"\e167"}.glyphicon-saved:before{content:"\e168"}.glyphicon-import:before{content:"\e169"}.glyphicon-export:before{content:"\e170"}.glyphicon-send:before{content:"\e171"}.glyphicon-floppy-disk:before{content:"\e172"}.glyphicon-floppy-saved:before{content:"\e173"}.glyphicon-floppy-remove:before{content:"\e174"}.glyphicon-floppy-save:before{content:"\e175"}.glyphicon-floppy-open:before{content:"\e176"}.glyphicon-credit-card:before{content:"\e177"}.glyphicon-transfer:before{content:"\e178"}.glyphicon-cutlery:before{content:"\e179"}.glyphicon-header:before{content:"\e180"}.glyphicon-compressed:before{content:"\e181"}.glyphicon-earphone:before{content:"\e182"}.glyphicon-phone-alt:before{content:"\e183"}.glyphicon-tower:before{content:"\e184"}.glyphicon-stats:before{content:"\e185"}.glyphicon-sd-video:before{content:"\e186"}.glyphicon-hd-video:before{content:"\e187"}.glyphicon-subtitles:before{content:"\e188"}.glyphicon-sound-stereo:before{content:"\e189"}.glyphicon-sound-dolby:before{content:"\e190"}.glyphicon-sound-5-1:before{content:"\e191"}.glyphicon-sound-6-1:before{content:"\e192"}.glyphicon-sound-7-1:before{content:"\e193"}.glyphicon-copyright-mark:before{content:"\e194"}.glyphicon-registration-mark:before{content:"\e195"}.glyphicon-cloud-download:before{content:"\e197"}.glyphicon-cloud-upload:before{content:"\e198"}.glyphicon-tree-conifer:before{content:"\e199"}.glyphicon-tree-deciduous:before{content:"\e200"}.glyphicon-cd:before{content:"\e201"}.glyphicon-save-file:before{content:"\e202"}.glyphicon-open-file:before{content:"\e203"}.glyphicon-level-up:before{content:"\e204"}.glyphicon-copy:before{content:"\e205"}.glyphicon-paste:before{content:"\e206"}.glyphicon-alert:before{content:"\e209"}.glyphicon-equalizer:before{content:"\e210"}.glyphicon-king:before{content:"\e211"}.glyphicon-queen:before{content:"\e212"}.glyphicon-pawn:before{content:"\e213"}.glyphicon-bishop:before{content:"\e214"}.glyphicon-knight:before{content:"\e215"}.glyphicon-baby-formula:before{content:"\e216"}.glyphicon-tent:before{content:"\26fa"}.glyphicon-blackboard:before{content:"\e218"}.glyphicon-bed:before{content:"\e219"}.glyphicon-apple:before{content:"\f8ff"}.glyphicon-erase:before{content:"\e221"}.glyphicon-hourglass:before{content:"\231b"}.glyphicon-lamp:before{content:"\e223"}.glyphicon-duplicate:before{content:"\e224"}.glyphicon-piggy-bank:before{content:"\e225"}.glyphicon-scissors:before{content:"\e226"}.glyphicon-bitcoin:before{content:"\e227"}.glyphicon-btc:before{content:"\e227"}.glyphicon-xbt:before{content:"\e227"}.glyphicon-yen:before{content:"\00a5"}.glyphicon-jpy:before{content:"\00a5"}.glyphicon-ruble:before{content:"\20bd"}.glyphicon-rub:before{content:"\20bd"}.glyphicon-scale:before{content:"\e230"}.glyphicon-ice-lolly:before{content:"\e231"}.glyphicon-ice-lolly-tasted:before{content:"\e232"}.glyphicon-education:before{content:"\e233"}.glyphicon-option-horizontal:before{content:"\e234"}.glyphicon-option-vertical:before{content:"\e235"}.glyphicon-menu-hamburger:before{content:"\e236"}.glyphicon-modal-window:before{content:"\e237"}.glyphicon-oil:before{content:"\e238"}.glyphicon-grain:before{content:"\e239"}.glyphicon-sunglasses:before{content:"\e240"}.glyphicon-text-size:before{content:"\e241"}.glyphicon-text-color:before{content:"\e242"}.glyphicon-text-background:before{content:"\e243"}.glyphicon-object-align-top:before{content:"\e244"}.glyphicon-object-align-bottom:before{content:"\e245"}.glyphicon-object-align-horizontal:before{content:"\e246"}.glyphicon-object-align-left:before{content:"\e247"}.glyphicon-object-align-vertical:before{content:"\e248"}.glyphicon-object-align-right:before{content:"\e249"}.glyphicon-triangle-right:before{content:"\e250"}.glyphicon-triangle-left:before{content:"\e251"}.glyphicon-triangle-bottom:before{content:"\e252"}.glyphicon-triangle-top:before{content:"\e253"}.glyphicon-console:before{content:"\e254"}.glyphicon-superscript:before{content:"\e255"}.glyphicon-subscript:before{content:"\e256"}.glyphicon-menu-left:before{content:"\e257"}.glyphicon-menu-right:before{content:"\e258"}.glyphicon-menu-down:before{content:"\e259"}.glyphicon-menu-up:before{content:"\e260"}*{-webkit-box-sizing:border-box;-moz-box-sizing:border-box;box-sizing:border-box}*:before,*:after{-webkit-box-sizing:border-box;-moz-box-sizing:border-box;box-sizing:border-box}html{font-size:10px;-webkit-tap-highlight-color:rgba(0,0,0,0)}body{font-family:Georgia,"Times New Roman",Times,serif;font-size:16px;line-height:1.42857143;color:#333333;background-color:#ffffff}input,button,select,textarea{font-family:inherit;font-size:inherit;line-height:inherit}a{color:#4582ec;text-decoration:none}a:hover,a:focus{color:#134fb8;text-decoration:underline}a:focus{outline:thin dotted;outline:5px auto -webkit-focus-ring-color;outline-offset:-2px}figure{margin:0}img{vertical-align:middle}.img-responsive,.thumbnail>img,.thumbnail a>img,.carousel-inner>.item>img,.carousel-inner>.item>a>img{display:block;max-width:100%;height:auto}.img-rounded{border-radius:6px}.img-thumbnail{padding:4px;line-height:1.42857143;background-color:#ffffff;border:1px solid #dddddd;border-radius:4px;-webkit-transition:all .2s ease-in-out;-o-transition:all .2s ease-in-out;transition:all .2s ease-in-out;display:inline-block;max-width:100%;height:auto}.img-circle{border-radius:50%}hr{margin-top:22px;margin-bottom:22px;border:0;border-top:1px solid #eeeeee}.sr-only{position:absolute;width:1px;height:1px;margin:-1px;padding:0;overflow:hidden;clip:rect(0, 0, 0, 0);border:0}.sr-only-focusable:active,.sr-only-focusable:focus{position:static;width:auto;height:auto;margin:0;overflow:visible;clip:auto}[role="button"]{cursor:pointer}h1,h2,h3,h4,h5,h6,.h1,.h2,.h3,.h4,.h5,.h6{font-family:"Raleway","Helvetica Neue",Helvetica,Arial,sans-serif;font-weight:bold;line-height:1.1;color:inherit}h1 small,h2 small,h3 small,h4 small,h5 small,h6 small,.h1 small,.h2 small,.h3 small,.h4 small,.h5 small,.h6 small,h1 .small,h2 .small,h3 .small,h4 .small,h5 .small,h6 .small,.h1 .small,.h2 .small,.h3 .small,.h4 .small,.h5 .small,.h6 .small{font-weight:normal;line-height:1;color:#b3b3b3}h1,.h1,h2,.h2,h3,.h3{margin-top:22px;margin-bottom:11px}h1 small,.h1 small,h2 small,.h2 small,h3 small,.h3 small,h1 .small,.h1 .small,h2 .small,.h2 .small,h3 .small,.h3 .small{font-size:65%}h4,.h4,h5,.h5,h6,.h6{margin-top:11px;margin-bottom:11px}h4 small,.h4 small,h5 small,.h5 small,h6 small,.h6 small,h4 .small,.h4 .small,h5 .small,.h5 .small,h6 .small,.h6 .small{font-size:75%}h1,.h1{font-size:41px}h2,.h2{font-size:34px}h3,.h3{font-size:28px}h4,.h4{font-size:20px}h5,.h5{font-size:16px}h6,.h6{font-size:14px}p{margin:0 0 11px}.lead{margin-bottom:22px;font-size:18px;font-weight:300;line-height:1.4}@media (min-width:768px){.lead{font-size:24px}}small,.small{font-size:87%}mark,.mark{background-color:#fcf8e3;padding:.2em}.text-left{text-align:left}.text-right{text-align:right}.text-center{text-align:center}.text-justify{text-align:justify}.text-nowrap{white-space:nowrap}.text-lowercase{text-transform:lowercase}.text-uppercase{text-transform:uppercase}.text-capitalize{text-transform:capitalize}.text-muted{color:#b3b3b3}.text-primary{color:#4582ec}a.text-primary:hover,a.text-primary:focus{color:#1863e6}.text-success{color:#3fad46}a.text-success:hover,a.text-success:focus{color:#318837}.text-info{color:#5bc0de}a.text-info:hover,a.text-info:focus{color:#31b0d5}.text-warning{color:#f0ad4e}a.text-warning:hover,a.text-warning:focus{color:#ec971f}.text-danger{color:#d9534f}a.text-danger:hover,a.text-danger:focus{color:#c9302c}.bg-primary{color:#fff;background-color:#4582ec}a.bg-primary:hover,a.bg-primary:focus{background-color:#1863e6}.bg-success{background-color:#dff0d8}a.bg-success:hover,a.bg-success:focus{background-color:#c1e2b3}.bg-info{background-color:#d9edf7}a.bg-info:hover,a.bg-info:focus{background-color:#afd9ee}.bg-warning{background-color:#fcf8e3}a.bg-warning:hover,a.bg-warning:focus{background-color:#f7ecb5}.bg-danger{background-color:#f2dede}a.bg-danger:hover,a.bg-danger:focus{background-color:#e4b9b9}.page-header{padding-bottom:10px;margin:44px 0 22px;border-bottom:1px solid #dddddd}ul,ol{margin-top:0;margin-bottom:11px}ul ul,ol ul,ul ol,ol ol{margin-bottom:0}.list-unstyled{padding-left:0;list-style:none}.list-inline{padding-left:0;list-style:none;margin-left:-5px}.list-inline>li{display:inline-block;padding-left:5px;padding-right:5px}dl{margin-top:0;margin-bottom:22px}dt,dd{line-height:1.42857143}dt{font-weight:bold}dd{margin-left:0}@media (min-width:768px){.dl-horizontal dt{float:left;width:160px;clear:left;text-align:right;overflow:hidden;text-overflow:ellipsis;white-space:nowrap}.dl-horizontal dd{margin-left:180px}}abbr[title],abbr[data-original-title]{cursor:help;border-bottom:1px dotted #b3b3b3}.initialism{font-size:90%;text-transform:uppercase}blockquote{padding:11px 22px;margin:0 0 22px;font-size:20px;border-left:5px solid #4582ec}blockquote p:last-child,blockquote ul:last-child,blockquote ol:last-child{margin-bottom:0}blockquote footer,blockquote small,blockquote .small{display:block;font-size:80%;line-height:1.42857143;color:#333333}blockquote footer:before,blockquote small:before,blockquote .small:before{content:'\2014 \00A0'}.blockquote-reverse,blockquote.pull-right{padding-right:15px;padding-left:0;border-right:5px solid #4582ec;border-left:0;text-align:right}.blockquote-reverse footer:before,blockquote.pull-right footer:before,.blockquote-reverse small:before,blockquote.pull-right small:before,.blockquote-reverse .small:before,blockquote.pull-right .small:before{content:''}.blockquote-reverse footer:after,blockquote.pull-right footer:after,.blockquote-reverse small:after,blockquote.pull-right small:after,.blockquote-reverse .small:after,blockquote.pull-right .small:after{content:'\00A0 \2014'}address{margin-bottom:22px;font-style:normal;line-height:1.42857143}code,kbd,pre,samp{font-family:monospace}code{padding:2px 4px;font-size:90%;color:#c7254e;background-color:#f9f2f4;border-radius:4px}kbd{padding:2px 4px;font-size:90%;color:#ffffff;background-color:#333333;border-radius:3px;-webkit-box-shadow:inset 0 -1px 0 rgba(0,0,0,0.25);box-shadow:inset 0 -1px 0 rgba(0,0,0,0.25)}kbd kbd{padding:0;font-size:100%;font-weight:bold;-webkit-box-shadow:none;box-shadow:none}pre{display:block;padding:10.5px;margin:0 0 11px;font-size:15px;line-height:1.42857143;word-break:break-all;word-wrap:break-word;color:#333333;background-color:#f5f5f5;border:1px solid #cccccc;border-radius:4px}pre code{padding:0;font-size:inherit;color:inherit;white-space:pre-wrap;background-color:transparent;border-radius:0}.pre-scrollable{max-height:340px;overflow-y:scroll}.container{margin-right:auto;margin-left:auto;padding-left:15px;padding-right:15px}@media (min-width:768px){.container{width:750px}}@media (min-width:992px){.container{width:970px}}@media (min-width:1200px){.container{width:1170px}}.container-fluid{margin-right:auto;margin-left:auto;padding-left:15px;padding-right:15px}.row{margin-left:-15px;margin-right:-15px}.col-xs-1,.col-sm-1,.col-md-1,.col-lg-1,.col-xs-2,.col-sm-2,.col-md-2,.col-lg-2,.col-xs-3,.col-sm-3,.col-md-3,.col-lg-3,.col-xs-4,.col-sm-4,.col-md-4,.col-lg-4,.col-xs-5,.col-sm-5,.col-md-5,.col-lg-5,.col-xs-6,.col-sm-6,.col-md-6,.col-lg-6,.col-xs-7,.col-sm-7,.col-md-7,.col-lg-7,.col-xs-8,.col-sm-8,.col-md-8,.col-lg-8,.col-xs-9,.col-sm-9,.col-md-9,.col-lg-9,.col-xs-10,.col-sm-10,.col-md-10,.col-lg-10,.col-xs-11,.col-sm-11,.col-md-11,.col-lg-11,.col-xs-12,.col-sm-12,.col-md-12,.col-lg-12{position:relative;min-height:1px;padding-left:15px;padding-right:15px}.col-xs-1,.col-xs-2,.col-xs-3,.col-xs-4,.col-xs-5,.col-xs-6,.col-xs-7,.col-xs-8,.col-xs-9,.col-xs-10,.col-xs-11,.col-xs-12{float:left}.col-xs-12{width:100%}.col-xs-11{width:91.66666667%}.col-xs-10{width:83.33333333%}.col-xs-9{width:75%}.col-xs-8{width:66.66666667%}.col-xs-7{width:58.33333333%}.col-xs-6{width:50%}.col-xs-5{width:41.66666667%}.col-xs-4{width:33.33333333%}.col-xs-3{width:25%}.col-xs-2{width:16.66666667%}.col-xs-1{width:8.33333333%}.col-xs-pull-12{right:100%}.col-xs-pull-11{right:91.66666667%}.col-xs-pull-10{right:83.33333333%}.col-xs-pull-9{right:75%}.col-xs-pull-8{right:66.66666667%}.col-xs-pull-7{right:58.33333333%}.col-xs-pull-6{right:50%}.col-xs-pull-5{right:41.66666667%}.col-xs-pull-4{right:33.33333333%}.col-xs-pull-3{right:25%}.col-xs-pull-2{right:16.66666667%}.col-xs-pull-1{right:8.33333333%}.col-xs-pull-0{right:auto}.col-xs-push-12{left:100%}.col-xs-push-11{left:91.66666667%}.col-xs-push-10{left:83.33333333%}.col-xs-push-9{left:75%}.col-xs-push-8{left:66.66666667%}.col-xs-push-7{left:58.33333333%}.col-xs-push-6{left:50%}.col-xs-push-5{left:41.66666667%}.col-xs-push-4{left:33.33333333%}.col-xs-push-3{left:25%}.col-xs-push-2{left:16.66666667%}.col-xs-push-1{left:8.33333333%}.col-xs-push-0{left:auto}.col-xs-offset-12{margin-left:100%}.col-xs-offset-11{margin-left:91.66666667%}.col-xs-offset-10{margin-left:83.33333333%}.col-xs-offset-9{margin-left:75%}.col-xs-offset-8{margin-left:66.66666667%}.col-xs-offset-7{margin-left:58.33333333%}.col-xs-offset-6{margin-left:50%}.col-xs-offset-5{margin-left:41.66666667%}.col-xs-offset-4{margin-left:33.33333333%}.col-xs-offset-3{margin-left:25%}.col-xs-offset-2{margin-left:16.66666667%}.col-xs-offset-1{margin-left:8.33333333%}.col-xs-offset-0{margin-left:0%}@media (min-width:768px){.col-sm-1,.col-sm-2,.col-sm-3,.col-sm-4,.col-sm-5,.col-sm-6,.col-sm-7,.col-sm-8,.col-sm-9,.col-sm-10,.col-sm-11,.col-sm-12{float:left}.col-sm-12{width:100%}.col-sm-11{width:91.66666667%}.col-sm-10{width:83.33333333%}.col-sm-9{width:75%}.col-sm-8{width:66.66666667%}.col-sm-7{width:58.33333333%}.col-sm-6{width:50%}.col-sm-5{width:41.66666667%}.col-sm-4{width:33.33333333%}.col-sm-3{width:25%}.col-sm-2{width:16.66666667%}.col-sm-1{width:8.33333333%}.col-sm-pull-12{right:100%}.col-sm-pull-11{right:91.66666667%}.col-sm-pull-10{right:83.33333333%}.col-sm-pull-9{right:75%}.col-sm-pull-8{right:66.66666667%}.col-sm-pull-7{right:58.33333333%}.col-sm-pull-6{right:50%}.col-sm-pull-5{right:41.66666667%}.col-sm-pull-4{right:33.33333333%}.col-sm-pull-3{right:25%}.col-sm-pull-2{right:16.66666667%}.col-sm-pull-1{right:8.33333333%}.col-sm-pull-0{right:auto}.col-sm-push-12{left:100%}.col-sm-push-11{left:91.66666667%}.col-sm-push-10{left:83.33333333%}.col-sm-push-9{left:75%}.col-sm-push-8{left:66.66666667%}.col-sm-push-7{left:58.33333333%}.col-sm-push-6{left:50%}.col-sm-push-5{left:41.66666667%}.col-sm-push-4{left:33.33333333%}.col-sm-push-3{left:25%}.col-sm-push-2{left:16.66666667%}.col-sm-push-1{left:8.33333333%}.col-sm-push-0{left:auto}.col-sm-offset-12{margin-left:100%}.col-sm-offset-11{margin-left:91.66666667%}.col-sm-offset-10{margin-left:83.33333333%}.col-sm-offset-9{margin-left:75%}.col-sm-offset-8{margin-left:66.66666667%}.col-sm-offset-7{margin-left:58.33333333%}.col-sm-offset-6{margin-left:50%}.col-sm-offset-5{margin-left:41.66666667%}.col-sm-offset-4{margin-left:33.33333333%}.col-sm-offset-3{margin-left:25%}.col-sm-offset-2{margin-left:16.66666667%}.col-sm-offset-1{margin-left:8.33333333%}.col-sm-offset-0{margin-left:0%}}@media (min-width:992px){.col-md-1,.col-md-2,.col-md-3,.col-md-4,.col-md-5,.col-md-6,.col-md-7,.col-md-8,.col-md-9,.col-md-10,.col-md-11,.col-md-12{float:left}.col-md-12{width:100%}.col-md-11{width:91.66666667%}.col-md-10{width:83.33333333%}.col-md-9{width:75%}.col-md-8{width:66.66666667%}.col-md-7{width:58.33333333%}.col-md-6{width:50%}.col-md-5{width:41.66666667%}.col-md-4{width:33.33333333%}.col-md-3{width:25%}.col-md-2{width:16.66666667%}.col-md-1{width:8.33333333%}.col-md-pull-12{right:100%}.col-md-pull-11{right:91.66666667%}.col-md-pull-10{right:83.33333333%}.col-md-pull-9{right:75%}.col-md-pull-8{right:66.66666667%}.col-md-pull-7{right:58.33333333%}.col-md-pull-6{right:50%}.col-md-pull-5{right:41.66666667%}.col-md-pull-4{right:33.33333333%}.col-md-pull-3{right:25%}.col-md-pull-2{right:16.66666667%}.col-md-pull-1{right:8.33333333%}.col-md-pull-0{right:auto}.col-md-push-12{left:100%}.col-md-push-11{left:91.66666667%}.col-md-push-10{left:83.33333333%}.col-md-push-9{left:75%}.col-md-push-8{left:66.66666667%}.col-md-push-7{left:58.33333333%}.col-md-push-6{left:50%}.col-md-push-5{left:41.66666667%}.col-md-push-4{left:33.33333333%}.col-md-push-3{left:25%}.col-md-push-2{left:16.66666667%}.col-md-push-1{left:8.33333333%}.col-md-push-0{left:auto}.col-md-offset-12{margin-left:100%}.col-md-offset-11{margin-left:91.66666667%}.col-md-offset-10{margin-left:83.33333333%}.col-md-offset-9{margin-left:75%}.col-md-offset-8{margin-left:66.66666667%}.col-md-offset-7{margin-left:58.33333333%}.col-md-offset-6{margin-left:50%}.col-md-offset-5{margin-left:41.66666667%}.col-md-offset-4{margin-left:33.33333333%}.col-md-offset-3{margin-left:25%}.col-md-offset-2{margin-left:16.66666667%}.col-md-offset-1{margin-left:8.33333333%}.col-md-offset-0{margin-left:0%}}@media (min-width:1200px){.col-lg-1,.col-lg-2,.col-lg-3,.col-lg-4,.col-lg-5,.col-lg-6,.col-lg-7,.col-lg-8,.col-lg-9,.col-lg-10,.col-lg-11,.col-lg-12{float:left}.col-lg-12{width:100%}.col-lg-11{width:91.66666667%}.col-lg-10{width:83.33333333%}.col-lg-9{width:75%}.col-lg-8{width:66.66666667%}.col-lg-7{width:58.33333333%}.col-lg-6{width:50%}.col-lg-5{width:41.66666667%}.col-lg-4{width:33.33333333%}.col-lg-3{width:25%}.col-lg-2{width:16.66666667%}.col-lg-1{width:8.33333333%}.col-lg-pull-12{right:100%}.col-lg-pull-11{right:91.66666667%}.col-lg-pull-10{right:83.33333333%}.col-lg-pull-9{right:75%}.col-lg-pull-8{right:66.66666667%}.col-lg-pull-7{right:58.33333333%}.col-lg-pull-6{right:50%}.col-lg-pull-5{right:41.66666667%}.col-lg-pull-4{right:33.33333333%}.col-lg-pull-3{right:25%}.col-lg-pull-2{right:16.66666667%}.col-lg-pull-1{right:8.33333333%}.col-lg-pull-0{right:auto}.col-lg-push-12{left:100%}.col-lg-push-11{left:91.66666667%}.col-lg-push-10{left:83.33333333%}.col-lg-push-9{left:75%}.col-lg-push-8{left:66.66666667%}.col-lg-push-7{left:58.33333333%}.col-lg-push-6{left:50%}.col-lg-push-5{left:41.66666667%}.col-lg-push-4{left:33.33333333%}.col-lg-push-3{left:25%}.col-lg-push-2{left:16.66666667%}.col-lg-push-1{left:8.33333333%}.col-lg-push-0{left:auto}.col-lg-offset-12{margin-left:100%}.col-lg-offset-11{margin-left:91.66666667%}.col-lg-offset-10{margin-left:83.33333333%}.col-lg-offset-9{margin-left:75%}.col-lg-offset-8{margin-left:66.66666667%}.col-lg-offset-7{margin-left:58.33333333%}.col-lg-offset-6{margin-left:50%}.col-lg-offset-5{margin-left:41.66666667%}.col-lg-offset-4{margin-left:33.33333333%}.col-lg-offset-3{margin-left:25%}.col-lg-offset-2{margin-left:16.66666667%}.col-lg-offset-1{margin-left:8.33333333%}.col-lg-offset-0{margin-left:0%}}table{background-color:transparent}caption{padding-top:8px;padding-bottom:8px;color:#b3b3b3;text-align:left}th{}.table{width:100%;max-width:100%;margin-bottom:22px}.table>thead>tr>th,.table>tbody>tr>th,.table>tfoot>tr>th,.table>thead>tr>td,.table>tbody>tr>td,.table>tfoot>tr>td{padding:8px;line-height:1.42857143;vertical-align:top;border-top:1px solid #dddddd}.table>thead>tr>th{vertical-align:bottom;border-bottom:2px solid #dddddd}.table>caption+thead>tr:first-child>th,.table>colgroup+thead>tr:first-child>th,.table>thead:first-child>tr:first-child>th,.table>caption+thead>tr:first-child>td,.table>colgroup+thead>tr:first-child>td,.table>thead:first-child>tr:first-child>td{border-top:0}.table>tbody+tbody{border-top:2px solid #dddddd}.table .table{background-color:#ffffff}.table-condensed>thead>tr>th,.table-condensed>tbody>tr>th,.table-condensed>tfoot>tr>th,.table-condensed>thead>tr>td,.table-condensed>tbody>tr>td,.table-condensed>tfoot>tr>td{padding:5px}.table-bordered{border:1px solid #dddddd}.table-bordered>thead>tr>th,.table-bordered>tbody>tr>th,.table-bordered>tfoot>tr>th,.table-bordered>thead>tr>td,.table-bordered>tbody>tr>td,.table-bordered>tfoot>tr>td{border:1px solid #dddddd}.table-bordered>thead>tr>th,.table-bordered>thead>tr>td{border-bottom-width:2px}.table-striped>tbody>tr:nth-of-type(odd){background-color:#f9f9f9}.table-hover>tbody>tr:hover{background-color:#f5f5f5}table col[class*="col-"]{position:static;float:none;display:table-column}table td[class*="col-"],table th[class*="col-"]{position:static;float:none;display:table-cell}.table>thead>tr>td.active,.table>tbody>tr>td.active,.table>tfoot>tr>td.active,.table>thead>tr>th.active,.table>tbody>tr>th.active,.table>tfoot>tr>th.active,.table>thead>tr.active>td,.table>tbody>tr.active>td,.table>tfoot>tr.active>td,.table>thead>tr.active>th,.table>tbody>tr.active>th,.table>tfoot>tr.active>th{background-color:#f5f5f5}.table-hover>tbody>tr>td.active:hover,.table-hover>tbody>tr>th.active:hover,.table-hover>tbody>tr.active:hover>td,.table-hover>tbody>tr:hover>.active,.table-hover>tbody>tr.active:hover>th{background-color:#e8e8e8}.table>thead>tr>td.success,.table>tbody>tr>td.success,.table>tfoot>tr>td.success,.table>thead>tr>th.success,.table>tbody>tr>th.success,.table>tfoot>tr>th.success,.table>thead>tr.success>td,.table>tbody>tr.success>td,.table>tfoot>tr.success>td,.table>thead>tr.success>th,.table>tbody>tr.success>th,.table>tfoot>tr.success>th{background-color:#dff0d8}.table-hover>tbody>tr>td.success:hover,.table-hover>tbody>tr>th.success:hover,.table-hover>tbody>tr.success:hover>td,.table-hover>tbody>tr:hover>.success,.table-hover>tbody>tr.success:hover>th{background-color:#d0e9c6}.table>thead>tr>td.info,.table>tbody>tr>td.info,.table>tfoot>tr>td.info,.table>thead>tr>th.info,.table>tbody>tr>th.info,.table>tfoot>tr>th.info,.table>thead>tr.info>td,.table>tbody>tr.info>td,.table>tfoot>tr.info>td,.table>thead>tr.info>th,.table>tbody>tr.info>th,.table>tfoot>tr.info>th{background-color:#d9edf7}.table-hover>tbody>tr>td.info:hover,.table-hover>tbody>tr>th.info:hover,.table-hover>tbody>tr.info:hover>td,.table-hover>tbody>tr:hover>.info,.table-hover>tbody>tr.info:hover>th{background-color:#c4e3f3}.table>thead>tr>td.warning,.table>tbody>tr>td.warning,.table>tfoot>tr>td.warning,.table>thead>tr>th.warning,.table>tbody>tr>th.warning,.table>tfoot>tr>th.warning,.table>thead>tr.warning>td,.table>tbody>tr.warning>td,.table>tfoot>tr.warning>td,.table>thead>tr.warning>th,.table>tbody>tr.warning>th,.table>tfoot>tr.warning>th{background-color:#fcf8e3}.table-hover>tbody>tr>td.warning:hover,.table-hover>tbody>tr>th.warning:hover,.table-hover>tbody>tr.warning:hover>td,.table-hover>tbody>tr:hover>.warning,.table-hover>tbody>tr.warning:hover>th{background-color:#faf2cc}.table>thead>tr>td.danger,.table>tbody>tr>td.danger,.table>tfoot>tr>td.danger,.table>thead>tr>th.danger,.table>tbody>tr>th.danger,.table>tfoot>tr>th.danger,.table>thead>tr.danger>td,.table>tbody>tr.danger>td,.table>tfoot>tr.danger>td,.table>thead>tr.danger>th,.table>tbody>tr.danger>th,.table>tfoot>tr.danger>th{background-color:#f2dede}.table-hover>tbody>tr>td.danger:hover,.table-hover>tbody>tr>th.danger:hover,.table-hover>tbody>tr.danger:hover>td,.table-hover>tbody>tr:hover>.danger,.table-hover>tbody>tr.danger:hover>th{background-color:#ebcccc}.table-responsive{overflow-x:auto;min-height:0.01%}@media screen and (max-width:767px){.table-responsive{width:100%;margin-bottom:16.5px;overflow-y:hidden;-ms-overflow-style:-ms-autohiding-scrollbar;border:1px solid #dddddd}.table-responsive>.table{margin-bottom:0}.table-responsive>.table>thead>tr>th,.table-responsive>.table>tbody>tr>th,.table-responsive>.table>tfoot>tr>th,.table-responsive>.table>thead>tr>td,.table-responsive>.table>tbody>tr>td,.table-responsive>.table>tfoot>tr>td{white-space:nowrap}.table-responsive>.table-bordered{border:0}.table-responsive>.table-bordered>thead>tr>th:first-child,.table-responsive>.table-bordered>tbody>tr>th:first-child,.table-responsive>.table-bordered>tfoot>tr>th:first-child,.table-responsive>.table-bordered>thead>tr>td:first-child,.table-responsive>.table-bordered>tbody>tr>td:first-child,.table-responsive>.table-bordered>tfoot>tr>td:first-child{border-left:0}.table-responsive>.table-bordered>thead>tr>th:last-child,.table-responsive>.table-bordered>tbody>tr>th:last-child,.table-responsive>.table-bordered>tfoot>tr>th:last-child,.table-responsive>.table-bordered>thead>tr>td:last-child,.table-responsive>.table-bordered>tbody>tr>td:last-child,.table-responsive>.table-bordered>tfoot>tr>td:last-child{border-right:0}.table-responsive>.table-bordered>tbody>tr:last-child>th,.table-responsive>.table-bordered>tfoot>tr:last-child>th,.table-responsive>.table-bordered>tbody>tr:last-child>td,.table-responsive>.table-bordered>tfoot>tr:last-child>td{border-bottom:0}}fieldset{padding:0;margin:0;border:0;min-width:0}legend{display:block;width:100%;padding:0;margin-bottom:22px;font-size:24px;line-height:inherit;color:#333333;border:0;border-bottom:1px solid #e5e5e5}label{display:inline-block;max-width:100%;margin-bottom:5px;font-weight:bold}input[type="search"]{-webkit-box-sizing:border-box;-moz-box-sizing:border-box;box-sizing:border-box}input[type="radio"],input[type="checkbox"]{margin:4px 0 0;margin-top:1px \9;line-height:normal}input[type="file"]{display:block}input[type="range"]{display:block;width:100%}select[multiple],select[size]{height:auto}input[type="file"]:focus,input[type="radio"]:focus,input[type="checkbox"]:focus{outline:thin dotted;outline:5px auto -webkit-focus-ring-color;outline-offset:-2px}output{display:block;padding-top:9px;font-size:16px;line-height:1.42857143;color:#333333}.form-control{display:block;width:100%;height:40px;padding:8px 12px;font-size:16px;line-height:1.42857143;color:#333333;background-color:#ffffff;background-image:none;border:1px solid #dddddd;border-radius:4px;-webkit-box-shadow:inset 0 1px 1px rgba(0,0,0,0.075);box-shadow:inset 0 1px 1px rgba(0,0,0,0.075);-webkit-transition:border-color ease-in-out .15s,-webkit-box-shadow ease-in-out .15s;-o-transition:border-color ease-in-out .15s,box-shadow ease-in-out .15s;transition:border-color ease-in-out .15s,box-shadow ease-in-out .15s}.form-control:focus{border-color:#66afe9;outline:0;-webkit-box-shadow:inset 0 1px 1px rgba(0,0,0,0.075),0 0 8px rgba(102,175,233,0.6);box-shadow:inset 0 1px 1px rgba(0,0,0,0.075),0 0 8px rgba(102,175,233,0.6)}.form-control::-moz-placeholder{color:#b3b3b3;opacity:1}.form-control:-ms-input-placeholder{color:#b3b3b3}.form-control::-webkit-input-placeholder{color:#b3b3b3}.form-control::-ms-expand{border:0;background-color:transparent}.form-control[disabled],.form-control[readonly],fieldset[disabled] .form-control{background-color:#eeeeee;opacity:1}.form-control[disabled],fieldset[disabled] .form-control{cursor:not-allowed}textarea.form-control{height:auto}input[type="search"]{-webkit-appearance:none}@media screen and (-webkit-min-device-pixel-ratio:0){input[type="date"].form-control,input[type="time"].form-control,input[type="datetime-local"].form-control,input[type="month"].form-control{line-height:40px}input[type="date"].input-sm,input[type="time"].input-sm,input[type="datetime-local"].input-sm,input[type="month"].input-sm,.input-group-sm input[type="date"],.input-group-sm input[type="time"],.input-group-sm input[type="datetime-local"],.input-group-sm input[type="month"]{line-height:33px}input[type="date"].input-lg,input[type="time"].input-lg,input[type="datetime-local"].input-lg,input[type="month"].input-lg,.input-group-lg input[type="date"],.input-group-lg input[type="time"],.input-group-lg input[type="datetime-local"],.input-group-lg input[type="month"]{line-height:57px}}.form-group{margin-bottom:15px}.radio,.checkbox{position:relative;display:block;margin-top:10px;margin-bottom:10px}.radio label,.checkbox label{min-height:22px;padding-left:20px;margin-bottom:0;font-weight:normal;cursor:pointer}.radio input[type="radio"],.radio-inline input[type="radio"],.checkbox input[type="checkbox"],.checkbox-inline input[type="checkbox"]{position:absolute;margin-left:-20px;margin-top:4px \9}.radio+.radio,.checkbox+.checkbox{margin-top:-5px}.radio-inline,.checkbox-inline{position:relative;display:inline-block;padding-left:20px;margin-bottom:0;vertical-align:middle;font-weight:normal;cursor:pointer}.radio-inline+.radio-inline,.checkbox-inline+.checkbox-inline{margin-top:0;margin-left:10px}input[type="radio"][disabled],input[type="checkbox"][disabled],input[type="radio"].disabled,input[type="checkbox"].disabled,fieldset[disabled] input[type="radio"],fieldset[disabled] input[type="checkbox"]{cursor:not-allowed}.radio-inline.disabled,.checkbox-inline.disabled,fieldset[disabled] .radio-inline,fieldset[disabled] .checkbox-inline{cursor:not-allowed}.radio.disabled label,.checkbox.disabled label,fieldset[disabled] .radio label,fieldset[disabled] .checkbox label{cursor:not-allowed}.form-control-static{padding-top:9px;padding-bottom:9px;margin-bottom:0;min-height:38px}.form-control-static.input-lg,.form-control-static.input-sm{padding-left:0;padding-right:0}.input-sm{height:33px;padding:5px 10px;font-size:14px;line-height:1.5;border-radius:3px}select.input-sm{height:33px;line-height:33px}textarea.input-sm,select[multiple].input-sm{height:auto}.form-group-sm .form-control{height:33px;padding:5px 10px;font-size:14px;line-height:1.5;border-radius:3px}.form-group-sm select.form-control{height:33px;line-height:33px}.form-group-sm textarea.form-control,.form-group-sm select[multiple].form-control{height:auto}.form-group-sm .form-control-static{height:33px;min-height:36px;padding:6px 10px;font-size:14px;line-height:1.5}.input-lg{height:57px;padding:14px 16px;font-size:20px;line-height:1.3333333;border-radius:6px}select.input-lg{height:57px;line-height:57px}textarea.input-lg,select[multiple].input-lg{height:auto}.form-group-lg .form-control{height:57px;padding:14px 16px;font-size:20px;line-height:1.3333333;border-radius:6px}.form-group-lg select.form-control{height:57px;line-height:57px}.form-group-lg textarea.form-control,.form-group-lg select[multiple].form-control{height:auto}.form-group-lg .form-control-static{height:57px;min-height:42px;padding:15px 16px;font-size:20px;line-height:1.3333333}.has-feedback{position:relative}.has-feedback .form-control{padding-right:50px}.form-control-feedback{position:absolute;top:0;right:0;z-index:2;display:block;width:40px;height:40px;line-height:40px;text-align:center;pointer-events:none}.input-lg+.form-control-feedback,.input-group-lg+.form-control-feedback,.form-group-lg .form-control+.form-control-feedback{width:57px;height:57px;line-height:57px}.input-sm+.form-control-feedback,.input-group-sm+.form-control-feedback,.form-group-sm .form-control+.form-control-feedback{width:33px;height:33px;line-height:33px}.has-success .help-block,.has-success .control-label,.has-success .radio,.has-success .checkbox,.has-success .radio-inline,.has-success .checkbox-inline,.has-success.radio label,.has-success.checkbox label,.has-success.radio-inline label,.has-success.checkbox-inline label{color:#3fad46}.has-success .form-control{border-color:#3fad46;-webkit-box-shadow:inset 0 1px 1px rgba(0,0,0,0.075);box-shadow:inset 0 1px 1px rgba(0,0,0,0.075)}.has-success .form-control:focus{border-color:#318837;-webkit-box-shadow:inset 0 1px 1px rgba(0,0,0,0.075),0 0 6px #81d186;box-shadow:inset 0 1px 1px rgba(0,0,0,0.075),0 0 6px #81d186}.has-success .input-group-addon{color:#3fad46;border-color:#3fad46;background-color:#dff0d8}.has-success .form-control-feedback{color:#3fad46}.has-warning .help-block,.has-warning .control-label,.has-warning .radio,.has-warning .checkbox,.has-warning .radio-inline,.has-warning .checkbox-inline,.has-warning.radio label,.has-warning.checkbox label,.has-warning.radio-inline label,.has-warning.checkbox-inline label{color:#f0ad4e}.has-warning .form-control{border-color:#f0ad4e;-webkit-box-shadow:inset 0 1px 1px rgba(0,0,0,0.075);box-shadow:inset 0 1px 1px rgba(0,0,0,0.075)}.has-warning .form-control:focus{border-color:#ec971f;-webkit-box-shadow:inset 0 1px 1px rgba(0,0,0,0.075),0 0 6px #f8d9ac;box-shadow:inset 0 1px 1px rgba(0,0,0,0.075),0 0 6px #f8d9ac}.has-warning .input-group-addon{color:#f0ad4e;border-color:#f0ad4e;background-color:#fcf8e3}.has-warning .form-control-feedback{color:#f0ad4e}.has-error .help-block,.has-error .control-label,.has-error .radio,.has-error .checkbox,.has-error .radio-inline,.has-error .checkbox-inline,.has-error.radio label,.has-error.checkbox label,.has-error.radio-inline label,.has-error.checkbox-inline label{color:#d9534f}.has-error .form-control{border-color:#d9534f;-webkit-box-shadow:inset 0 1px 1px rgba(0,0,0,0.075);box-shadow:inset 0 1px 1px rgba(0,0,0,0.075)}.has-error .form-control:focus{border-color:#c9302c;-webkit-box-shadow:inset 0 1px 1px rgba(0,0,0,0.075),0 0 6px #eba5a3;box-shadow:inset 0 1px 1px rgba(0,0,0,0.075),0 0 6px #eba5a3}.has-error .input-group-addon{color:#d9534f;border-color:#d9534f;background-color:#f2dede}.has-error .form-control-feedback{color:#d9534f}.has-feedback label~.form-control-feedback{top:27px}.has-feedback label.sr-only~.form-control-feedback{top:0}.help-block{display:block;margin-top:5px;margin-bottom:10px;color:#737373}@media (min-width:768px){.form-inline .form-group{display:inline-block;margin-bottom:0;vertical-align:middle}.form-inline .form-control{display:inline-block;width:auto;vertical-align:middle}.form-inline .form-control-static{display:inline-block}.form-inline .input-group{display:inline-table;vertical-align:middle}.form-inline .input-group .input-group-addon,.form-inline .input-group .input-group-btn,.form-inline .input-group .form-control{width:auto}.form-inline .input-group>.form-control{width:100%}.form-inline .control-label{margin-bottom:0;vertical-align:middle}.form-inline .radio,.form-inline .checkbox{display:inline-block;margin-top:0;margin-bottom:0;vertical-align:middle}.form-inline .radio label,.form-inline .checkbox label{padding-left:0}.form-inline .radio input[type="radio"],.form-inline .checkbox input[type="checkbox"]{position:relative;margin-left:0}.form-inline .has-feedback .form-control-feedback{top:0}}.form-horizontal .radio,.form-horizontal .checkbox,.form-horizontal .radio-inline,.form-horizontal .checkbox-inline{margin-top:0;margin-bottom:0;padding-top:9px}.form-horizontal .radio,.form-horizontal .checkbox{min-height:31px}.form-horizontal .form-group{margin-left:-15px;margin-right:-15px}@media (min-width:768px){.form-horizontal .control-label{text-align:right;margin-bottom:0;padding-top:9px}}.form-horizontal .has-feedback .form-control-feedback{right:15px}@media (min-width:768px){.form-horizontal .form-group-lg .control-label{padding-top:15px;font-size:20px}}@media (min-width:768px){.form-horizontal .form-group-sm .control-label{padding-top:6px;font-size:14px}}.btn{display:inline-block;margin-bottom:0;font-weight:normal;text-align:center;vertical-align:middle;-ms-touch-action:manipulation;touch-action:manipulation;cursor:pointer;background-image:none;border:1px solid transparent;white-space:nowrap;padding:8px 12px;font-size:16px;line-height:1.42857143;border-radius:4px;-webkit-user-select:none;-moz-user-select:none;-ms-user-select:none;user-select:none}.btn:focus,.btn:active:focus,.btn.active:focus,.btn.focus,.btn:active.focus,.btn.active.focus{outline:thin dotted;outline:5px auto -webkit-focus-ring-color;outline-offset:-2px}.btn:hover,.btn:focus,.btn.focus{color:#333333;text-decoration:none}.btn:active,.btn.active{outline:0;background-image:none;-webkit-box-shadow:inset 0 3px 5px rgba(0,0,0,0.125);box-shadow:inset 0 3px 5px rgba(0,0,0,0.125)}.btn.disabled,.btn[disabled],fieldset[disabled] .btn{cursor:not-allowed;opacity:0.65;filter:alpha(opacity=65);-webkit-box-shadow:none;box-shadow:none}a.btn.disabled,fieldset[disabled] a.btn{pointer-events:none}.btn-default{color:#333333;background-color:#ffffff;border-color:#dddddd}.btn-default:focus,.btn-default.focus{color:#333333;background-color:#e6e6e6;border-color:#9d9d9d}.btn-default:hover{color:#333333;background-color:#e6e6e6;border-color:#bebebe}.btn-default:active,.btn-default.active,.open>.dropdown-toggle.btn-default{color:#333333;background-color:#e6e6e6;border-color:#bebebe}.btn-default:active:hover,.btn-default.active:hover,.open>.dropdown-toggle.btn-default:hover,.btn-default:active:focus,.btn-default.active:focus,.open>.dropdown-toggle.btn-default:focus,.btn-default:active.focus,.btn-default.active.focus,.open>.dropdown-toggle.btn-default.focus{color:#333333;background-color:#d4d4d4;border-color:#9d9d9d}.btn-default:active,.btn-default.active,.open>.dropdown-toggle.btn-default{background-image:none}.btn-default.disabled:hover,.btn-default[disabled]:hover,fieldset[disabled] .btn-default:hover,.btn-default.disabled:focus,.btn-default[disabled]:focus,fieldset[disabled] .btn-default:focus,.btn-default.disabled.focus,.btn-default[disabled].focus,fieldset[disabled] .btn-default.focus{background-color:#ffffff;border-color:#dddddd}.btn-default .badge{color:#ffffff;background-color:#333333}.btn-primary{color:#ffffff;background-color:#4582ec;border-color:#4582ec}.btn-primary:focus,.btn-primary.focus{color:#ffffff;background-color:#1863e6;border-color:#1045a1}.btn-primary:hover{color:#ffffff;background-color:#1863e6;border-color:#175fdd}.btn-primary:active,.btn-primary.active,.open>.dropdown-toggle.btn-primary{color:#ffffff;background-color:#1863e6;border-color:#175fdd}.btn-primary:active:hover,.btn-primary.active:hover,.open>.dropdown-toggle.btn-primary:hover,.btn-primary:active:focus,.btn-primary.active:focus,.open>.dropdown-toggle.btn-primary:focus,.btn-primary:active.focus,.btn-primary.active.focus,.open>.dropdown-toggle.btn-primary.focus{color:#ffffff;background-color:#1455c6;border-color:#1045a1}.btn-primary:active,.btn-primary.active,.open>.dropdown-toggle.btn-primary{background-image:none}.btn-primary.disabled:hover,.btn-primary[disabled]:hover,fieldset[disabled] .btn-primary:hover,.btn-primary.disabled:focus,.btn-primary[disabled]:focus,fieldset[disabled] .btn-primary:focus,.btn-primary.disabled.focus,.btn-primary[disabled].focus,fieldset[disabled] .btn-primary.focus{background-color:#4582ec;border-color:#4582ec}.btn-primary .badge{color:#4582ec;background-color:#ffffff}.btn-success{color:#ffffff;background-color:#3fad46;border-color:#3fad46}.btn-success:focus,.btn-success.focus{color:#ffffff;background-color:#318837;border-color:#1d5020}.btn-success:hover{color:#ffffff;background-color:#318837;border-color:#2f8034}.btn-success:active,.btn-success.active,.open>.dropdown-toggle.btn-success{color:#ffffff;background-color:#318837;border-color:#2f8034}.btn-success:active:hover,.btn-success.active:hover,.open>.dropdown-toggle.btn-success:hover,.btn-success:active:focus,.btn-success.active:focus,.open>.dropdown-toggle.btn-success:focus,.btn-success:active.focus,.btn-success.active.focus,.open>.dropdown-toggle.btn-success.focus{color:#ffffff;background-color:#286d2c;border-color:#1d5020}.btn-success:active,.btn-success.active,.open>.dropdown-toggle.btn-success{background-image:none}.btn-success.disabled:hover,.btn-success[disabled]:hover,fieldset[disabled] .btn-success:hover,.btn-success.disabled:focus,.btn-success[disabled]:focus,fieldset[disabled] .btn-success:focus,.btn-success.disabled.focus,.btn-success[disabled].focus,fieldset[disabled] .btn-success.focus{background-color:#3fad46;border-color:#3fad46}.btn-success .badge{color:#3fad46;background-color:#ffffff}.btn-info{color:#ffffff;background-color:#5bc0de;border-color:#5bc0de}.btn-info:focus,.btn-info.focus{color:#ffffff;background-color:#31b0d5;border-color:#1f7e9a}.btn-info:hover{color:#ffffff;background-color:#31b0d5;border-color:#2aabd2}.btn-info:active,.btn-info.active,.open>.dropdown-toggle.btn-info{color:#ffffff;background-color:#31b0d5;border-color:#2aabd2}.btn-info:active:hover,.btn-info.active:hover,.open>.dropdown-toggle.btn-info:hover,.btn-info:active:focus,.btn-info.active:focus,.open>.dropdown-toggle.btn-info:focus,.btn-info:active.focus,.btn-info.active.focus,.open>.dropdown-toggle.btn-info.focus{color:#ffffff;background-color:#269abc;border-color:#1f7e9a}.btn-info:active,.btn-info.active,.open>.dropdown-toggle.btn-info{background-image:none}.btn-info.disabled:hover,.btn-info[disabled]:hover,fieldset[disabled] .btn-info:hover,.btn-info.disabled:focus,.btn-info[disabled]:focus,fieldset[disabled] .btn-info:focus,.btn-info.disabled.focus,.btn-info[disabled].focus,fieldset[disabled] .btn-info.focus{background-color:#5bc0de;border-color:#5bc0de}.btn-info .badge{color:#5bc0de;background-color:#ffffff}.btn-warning{color:#ffffff;background-color:#f0ad4e;border-color:#f0ad4e}.btn-warning:focus,.btn-warning.focus{color:#ffffff;background-color:#ec971f;border-color:#b06d0f}.btn-warning:hover{color:#ffffff;background-color:#ec971f;border-color:#eb9316}.btn-warning:active,.btn-warning.active,.open>.dropdown-toggle.btn-warning{color:#ffffff;background-color:#ec971f;border-color:#eb9316}.btn-warning:active:hover,.btn-warning.active:hover,.open>.dropdown-toggle.btn-warning:hover,.btn-warning:active:focus,.btn-warning.active:focus,.open>.dropdown-toggle.btn-warning:focus,.btn-warning:active.focus,.btn-warning.active.focus,.open>.dropdown-toggle.btn-warning.focus{color:#ffffff;background-color:#d58512;border-color:#b06d0f}.btn-warning:active,.btn-warning.active,.open>.dropdown-toggle.btn-warning{background-image:none}.btn-warning.disabled:hover,.btn-warning[disabled]:hover,fieldset[disabled] .btn-warning:hover,.btn-warning.disabled:focus,.btn-warning[disabled]:focus,fieldset[disabled] .btn-warning:focus,.btn-warning.disabled.focus,.btn-warning[disabled].focus,fieldset[disabled] .btn-warning.focus{background-color:#f0ad4e;border-color:#f0ad4e}.btn-warning .badge{color:#f0ad4e;background-color:#ffffff}.btn-danger{color:#ffffff;background-color:#d9534f;border-color:#d9534f}.btn-danger:focus,.btn-danger.focus{color:#ffffff;background-color:#c9302c;border-color:#8b211e}.btn-danger:hover{color:#ffffff;background-color:#c9302c;border-color:#c12e2a}.btn-danger:active,.btn-danger.active,.open>.dropdown-toggle.btn-danger{color:#ffffff;background-color:#c9302c;border-color:#c12e2a}.btn-danger:active:hover,.btn-danger.active:hover,.open>.dropdown-toggle.btn-danger:hover,.btn-danger:active:focus,.btn-danger.active:focus,.open>.dropdown-toggle.btn-danger:focus,.btn-danger:active.focus,.btn-danger.active.focus,.open>.dropdown-toggle.btn-danger.focus{color:#ffffff;background-color:#ac2925;border-color:#8b211e}.btn-danger:active,.btn-danger.active,.open>.dropdown-toggle.btn-danger{background-image:none}.btn-danger.disabled:hover,.btn-danger[disabled]:hover,fieldset[disabled] .btn-danger:hover,.btn-danger.disabled:focus,.btn-danger[disabled]:focus,fieldset[disabled] .btn-danger:focus,.btn-danger.disabled.focus,.btn-danger[disabled].focus,fieldset[disabled] .btn-danger.focus{background-color:#d9534f;border-color:#d9534f}.btn-danger .badge{color:#d9534f;background-color:#ffffff}.btn-link{color:#4582ec;font-weight:normal;border-radius:0}.btn-link,.btn-link:active,.btn-link.active,.btn-link[disabled],fieldset[disabled] .btn-link{background-color:transparent;-webkit-box-shadow:none;box-shadow:none}.btn-link,.btn-link:hover,.btn-link:focus,.btn-link:active{border-color:transparent}.btn-link:hover,.btn-link:focus{color:#134fb8;text-decoration:underline;background-color:transparent}.btn-link[disabled]:hover,fieldset[disabled] .btn-link:hover,.btn-link[disabled]:focus,fieldset[disabled] .btn-link:focus{color:#b3b3b3;text-decoration:none}.btn-lg,.btn-group-lg>.btn{padding:14px 16px;font-size:20px;line-height:1.3333333;border-radius:6px}.btn-sm,.btn-group-sm>.btn{padding:5px 10px;font-size:14px;line-height:1.5;border-radius:3px}.btn-xs,.btn-group-xs>.btn{padding:1px 5px;font-size:14px;line-height:1.5;border-radius:3px}.btn-block{display:block;width:100%}.btn-block+.btn-block{margin-top:5px}input[type="submit"].btn-block,input[type="reset"].btn-block,input[type="button"].btn-block{width:100%}.fade{opacity:0;-webkit-transition:opacity 0.15s linear;-o-transition:opacity 0.15s linear;transition:opacity 0.15s linear}.fade.in{opacity:1}.collapse{display:none}.collapse.in{display:block}tr.collapse.in{display:table-row}tbody.collapse.in{display:table-row-group}.collapsing{position:relative;height:0;overflow:hidden;-webkit-transition-property:height, visibility;-o-transition-property:height, visibility;transition-property:height, visibility;-webkit-transition-duration:0.35s;-o-transition-duration:0.35s;transition-duration:0.35s;-webkit-transition-timing-function:ease;-o-transition-timing-function:ease;transition-timing-function:ease}.caret{display:inline-block;width:0;height:0;margin-left:2px;vertical-align:middle;border-top:4px dashed;border-top:4px solid \9;border-right:4px solid transparent;border-left:4px solid transparent}.dropup,.dropdown{position:relative}.dropdown-toggle:focus{outline:0}.dropdown-menu{position:absolute;top:100%;left:0;z-index:1000;display:none;float:left;min-width:160px;padding:5px 0;margin:2px 0 0;list-style:none;font-size:16px;text-align:left;background-color:#ffffff;border:1px solid #cccccc;border:1px solid rgba(0,0,0,0.15);border-radius:4px;-webkit-box-shadow:0 6px 12px rgba(0,0,0,0.175);box-shadow:0 6px 12px rgba(0,0,0,0.175);-webkit-background-clip:padding-box;background-clip:padding-box}.dropdown-menu.pull-right{right:0;left:auto}.dropdown-menu .divider{height:1px;margin:10px 0;overflow:hidden;background-color:#e5e5e5}.dropdown-menu>li>a{display:block;padding:3px 20px;clear:both;font-weight:normal;line-height:1.42857143;color:#333333;white-space:nowrap}.dropdown-menu>li>a:hover,.dropdown-menu>li>a:focus{text-decoration:none;color:#ffffff;background-color:#4582ec}.dropdown-menu>.active>a,.dropdown-menu>.active>a:hover,.dropdown-menu>.active>a:focus{color:#ffffff;text-decoration:none;outline:0;background-color:#4582ec}.dropdown-menu>.disabled>a,.dropdown-menu>.disabled>a:hover,.dropdown-menu>.disabled>a:focus{color:#b3b3b3}.dropdown-menu>.disabled>a:hover,.dropdown-menu>.disabled>a:focus{text-decoration:none;background-color:transparent;background-image:none;filter:progid:DXImageTransform.Microsoft.gradient(enabled=false);cursor:not-allowed}.open>.dropdown-menu{display:block}.open>a{outline:0}.dropdown-menu-right{left:auto;right:0}.dropdown-menu-left{left:0;right:auto}.dropdown-header{display:block;padding:3px 20px;font-size:14px;line-height:1.42857143;color:#b3b3b3;white-space:nowrap}.dropdown-backdrop{position:fixed;left:0;right:0;bottom:0;top:0;z-index:990}.pull-right>.dropdown-menu{right:0;left:auto}.dropup .caret,.navbar-fixed-bottom .dropdown .caret{border-top:0;border-bottom:4px dashed;border-bottom:4px solid \9;content:""}.dropup .dropdown-menu,.navbar-fixed-bottom .dropdown .dropdown-menu{top:auto;bottom:100%;margin-bottom:2px}@media (min-width:768px){.navbar-right .dropdown-menu{left:auto;right:0}.navbar-right .dropdown-menu-left{left:0;right:auto}}.btn-group,.btn-group-vertical{position:relative;display:inline-block;vertical-align:middle}.btn-group>.btn,.btn-group-vertical>.btn{position:relative;float:left}.btn-group>.btn:hover,.btn-group-vertical>.btn:hover,.btn-group>.btn:focus,.btn-group-vertical>.btn:focus,.btn-group>.btn:active,.btn-group-vertical>.btn:active,.btn-group>.btn.active,.btn-group-vertical>.btn.active{z-index:2}.btn-group .btn+.btn,.btn-group .btn+.btn-group,.btn-group .btn-group+.btn,.btn-group .btn-group+.btn-group{margin-left:-1px}.btn-toolbar{margin-left:-5px}.btn-toolbar .btn,.btn-toolbar .btn-group,.btn-toolbar .input-group{float:left}.btn-toolbar>.btn,.btn-toolbar>.btn-group,.btn-toolbar>.input-group{margin-left:5px}.btn-group>.btn:not(:first-child):not(:last-child):not(.dropdown-toggle){border-radius:0}.btn-group>.btn:first-child{margin-left:0}.btn-group>.btn:first-child:not(:last-child):not(.dropdown-toggle){border-bottom-right-radius:0;border-top-right-radius:0}.btn-group>.btn:last-child:not(:first-child),.btn-group>.dropdown-toggle:not(:first-child){border-bottom-left-radius:0;border-top-left-radius:0}.btn-group>.btn-group{float:left}.btn-group>.btn-group:not(:first-child):not(:last-child)>.btn{border-radius:0}.btn-group>.btn-group:first-child:not(:last-child)>.btn:last-child,.btn-group>.btn-group:first-child:not(:last-child)>.dropdown-toggle{border-bottom-right-radius:0;border-top-right-radius:0}.btn-group>.btn-group:last-child:not(:first-child)>.btn:first-child{border-bottom-left-radius:0;border-top-left-radius:0}.btn-group .dropdown-toggle:active,.btn-group.open .dropdown-toggle{outline:0}.btn-group>.btn+.dropdown-toggle{padding-left:8px;padding-right:8px}.btn-group>.btn-lg+.dropdown-toggle{padding-left:12px;padding-right:12px}.btn-group.open .dropdown-toggle{-webkit-box-shadow:inset 0 3px 5px rgba(0,0,0,0.125);box-shadow:inset 0 3px 5px rgba(0,0,0,0.125)}.btn-group.open .dropdown-toggle.btn-link{-webkit-box-shadow:none;box-shadow:none}.btn .caret{margin-left:0}.btn-lg .caret{border-width:5px 5px 0;border-bottom-width:0}.dropup .btn-lg .caret{border-width:0 5px 5px}.btn-group-vertical>.btn,.btn-group-vertical>.btn-group,.btn-group-vertical>.btn-group>.btn{display:block;float:none;width:100%;max-width:100%}.btn-group-vertical>.btn-group>.btn{float:none}.btn-group-vertical>.btn+.btn,.btn-group-vertical>.btn+.btn-group,.btn-group-vertical>.btn-group+.btn,.btn-group-vertical>.btn-group+.btn-group{margin-top:-1px;margin-left:0}.btn-group-vertical>.btn:not(:first-child):not(:last-child){border-radius:0}.btn-group-vertical>.btn:first-child:not(:last-child){border-top-right-radius:4px;border-top-left-radius:4px;border-bottom-right-radius:0;border-bottom-left-radius:0}.btn-group-vertical>.btn:last-child:not(:first-child){border-top-right-radius:0;border-top-left-radius:0;border-bottom-right-radius:4px;border-bottom-left-radius:4px}.btn-group-vertical>.btn-group:not(:first-child):not(:last-child)>.btn{border-radius:0}.btn-group-vertical>.btn-group:first-child:not(:last-child)>.btn:last-child,.btn-group-vertical>.btn-group:first-child:not(:last-child)>.dropdown-toggle{border-bottom-right-radius:0;border-bottom-left-radius:0}.btn-group-vertical>.btn-group:last-child:not(:first-child)>.btn:first-child{border-top-right-radius:0;border-top-left-radius:0}.btn-group-justified{display:table;width:100%;table-layout:fixed;border-collapse:separate}.btn-group-justified>.btn,.btn-group-justified>.btn-group{float:none;display:table-cell;width:1%}.btn-group-justified>.btn-group .btn{width:100%}.btn-group-justified>.btn-group .dropdown-menu{left:auto}[data-toggle="buttons"]>.btn input[type="radio"],[data-toggle="buttons"]>.btn-group>.btn input[type="radio"],[data-toggle="buttons"]>.btn input[type="checkbox"],[data-toggle="buttons"]>.btn-group>.btn input[type="checkbox"]{position:absolute;clip:rect(0, 0, 0, 0);pointer-events:none}.input-group{position:relative;display:table;border-collapse:separate}.input-group[class*="col-"]{float:none;padding-left:0;padding-right:0}.input-group .form-control{position:relative;z-index:2;float:left;width:100%;margin-bottom:0}.input-group .form-control:focus{z-index:3}.input-group-lg>.form-control,.input-group-lg>.input-group-addon,.input-group-lg>.input-group-btn>.btn{height:57px;padding:14px 16px;font-size:20px;line-height:1.3333333;border-radius:6px}select.input-group-lg>.form-control,select.input-group-lg>.input-group-addon,select.input-group-lg>.input-group-btn>.btn{height:57px;line-height:57px}textarea.input-group-lg>.form-control,textarea.input-group-lg>.input-group-addon,textarea.input-group-lg>.input-group-btn>.btn,select[multiple].input-group-lg>.form-control,select[multiple].input-group-lg>.input-group-addon,select[multiple].input-group-lg>.input-group-btn>.btn{height:auto}.input-group-sm>.form-control,.input-group-sm>.input-group-addon,.input-group-sm>.input-group-btn>.btn{height:33px;padding:5px 10px;font-size:14px;line-height:1.5;border-radius:3px}select.input-group-sm>.form-control,select.input-group-sm>.input-group-addon,select.input-group-sm>.input-group-btn>.btn{height:33px;line-height:33px}textarea.input-group-sm>.form-control,textarea.input-group-sm>.input-group-addon,textarea.input-group-sm>.input-group-btn>.btn,select[multiple].input-group-sm>.form-control,select[multiple].input-group-sm>.input-group-addon,select[multiple].input-group-sm>.input-group-btn>.btn{height:auto}.input-group-addon,.input-group-btn,.input-group .form-control{display:table-cell}.input-group-addon:not(:first-child):not(:last-child),.input-group-btn:not(:first-child):not(:last-child),.input-group .form-control:not(:first-child):not(:last-child){border-radius:0}.input-group-addon,.input-group-btn{width:1%;white-space:nowrap;vertical-align:middle}.input-group-addon{padding:8px 12px;font-size:16px;font-weight:normal;line-height:1;color:#333333;text-align:center;background-color:#eeeeee;border:1px solid #dddddd;border-radius:4px}.input-group-addon.input-sm{padding:5px 10px;font-size:14px;border-radius:3px}.input-group-addon.input-lg{padding:14px 16px;font-size:20px;border-radius:6px}.input-group-addon input[type="radio"],.input-group-addon input[type="checkbox"]{margin-top:0}.input-group .form-control:first-child,.input-group-addon:first-child,.input-group-btn:first-child>.btn,.input-group-btn:first-child>.btn-group>.btn,.input-group-btn:first-child>.dropdown-toggle,.input-group-btn:last-child>.btn:not(:last-child):not(.dropdown-toggle),.input-group-btn:last-child>.btn-group:not(:last-child)>.btn{border-bottom-right-radius:0;border-top-right-radius:0}.input-group-addon:first-child{border-right:0}.input-group .form-control:last-child,.input-group-addon:last-child,.input-group-btn:last-child>.btn,.input-group-btn:last-child>.btn-group>.btn,.input-group-btn:last-child>.dropdown-toggle,.input-group-btn:first-child>.btn:not(:first-child),.input-group-btn:first-child>.btn-group:not(:first-child)>.btn{border-bottom-left-radius:0;border-top-left-radius:0}.input-group-addon:last-child{border-left:0}.input-group-btn{position:relative;font-size:0;white-space:nowrap}.input-group-btn>.btn{position:relative}.input-group-btn>.btn+.btn{margin-left:-1px}.input-group-btn>.btn:hover,.input-group-btn>.btn:focus,.input-group-btn>.btn:active{z-index:2}.input-group-btn:first-child>.btn,.input-group-btn:first-child>.btn-group{margin-right:-1px}.input-group-btn:last-child>.btn,.input-group-btn:last-child>.btn-group{z-index:2;margin-left:-1px}.nav{margin-bottom:0;padding-left:0;list-style:none}.nav>li{position:relative;display:block}.nav>li>a{position:relative;display:block;padding:10px 15px}.nav>li>a:hover,.nav>li>a:focus{text-decoration:none;background-color:#eeeeee}.nav>li.disabled>a{color:#b3b3b3}.nav>li.disabled>a:hover,.nav>li.disabled>a:focus{color:#b3b3b3;text-decoration:none;background-color:transparent;cursor:not-allowed}.nav .open>a,.nav .open>a:hover,.nav .open>a:focus{background-color:#eeeeee;border-color:#4582ec}.nav .nav-divider{height:1px;margin:10px 0;overflow:hidden;background-color:#e5e5e5}.nav>li>a>img{max-width:none}.nav-tabs{border-bottom:1px solid #dddddd}.nav-tabs>li{float:left;margin-bottom:-1px}.nav-tabs>li>a{margin-right:2px;line-height:1.42857143;border:1px solid transparent;border-radius:4px 4px 0 0}.nav-tabs>li>a:hover{border-color:#eeeeee #eeeeee #dddddd}.nav-tabs>li.active>a,.nav-tabs>li.active>a:hover,.nav-tabs>li.active>a:focus{color:#555555;background-color:#ffffff;border:1px solid #dddddd;border-bottom-color:transparent;cursor:default}.nav-tabs.nav-justified{width:100%;border-bottom:0}.nav-tabs.nav-justified>li{float:none}.nav-tabs.nav-justified>li>a{text-align:center;margin-bottom:5px}.nav-tabs.nav-justified>.dropdown .dropdown-menu{top:auto;left:auto}@media (min-width:768px){.nav-tabs.nav-justified>li{display:table-cell;width:1%}.nav-tabs.nav-justified>li>a{margin-bottom:0}}.nav-tabs.nav-justified>li>a{margin-right:0;border-radius:4px}.nav-tabs.nav-justified>.active>a,.nav-tabs.nav-justified>.active>a:hover,.nav-tabs.nav-justified>.active>a:focus{border:1px solid #dddddd}@media (min-width:768px){.nav-tabs.nav-justified>li>a{border-bottom:1px solid #dddddd;border-radius:4px 4px 0 0}.nav-tabs.nav-justified>.active>a,.nav-tabs.nav-justified>.active>a:hover,.nav-tabs.nav-justified>.active>a:focus{border-bottom-color:#ffffff}}.nav-pills>li{float:left}.nav-pills>li>a{border-radius:4px}.nav-pills>li+li{margin-left:2px}.nav-pills>li.active>a,.nav-pills>li.active>a:hover,.nav-pills>li.active>a:focus{color:#ffffff;background-color:#4582ec}.nav-stacked>li{float:none}.nav-stacked>li+li{margin-top:2px;margin-left:0}.nav-justified{width:100%}.nav-justified>li{float:none}.nav-justified>li>a{text-align:center;margin-bottom:5px}.nav-justified>.dropdown .dropdown-menu{top:auto;left:auto}@media (min-width:768px){.nav-justified>li{display:table-cell;width:1%}.nav-justified>li>a{margin-bottom:0}}.nav-tabs-justified{border-bottom:0}.nav-tabs-justified>li>a{margin-right:0;border-radius:4px}.nav-tabs-justified>.active>a,.nav-tabs-justified>.active>a:hover,.nav-tabs-justified>.active>a:focus{border:1px solid #dddddd}@media (min-width:768px){.nav-tabs-justified>li>a{border-bottom:1px solid #dddddd;border-radius:4px 4px 0 0}.nav-tabs-justified>.active>a,.nav-tabs-justified>.active>a:hover,.nav-tabs-justified>.active>a:focus{border-bottom-color:#ffffff}}.tab-content>.tab-pane{display:none}.tab-content>.active{display:block}.nav-tabs .dropdown-menu{margin-top:-1px;border-top-right-radius:0;border-top-left-radius:0}.navbar{position:relative;min-height:65px;margin-bottom:22px;border:1px solid transparent}@media (min-width:768px){.navbar{border-radius:4px}}@media (min-width:768px){.navbar-header{float:left}}.navbar-collapse{overflow-x:visible;padding-right:15px;padding-left:15px;border-top:1px solid transparent;-webkit-box-shadow:inset 0 1px 0 rgba(255,255,255,0.1);box-shadow:inset 0 1px 0 rgba(255,255,255,0.1);-webkit-overflow-scrolling:touch}.navbar-collapse.in{overflow-y:auto}@media (min-width:768px){.navbar-collapse{width:auto;border-top:0;-webkit-box-shadow:none;box-shadow:none}.navbar-collapse.collapse{display:block !important;height:auto !important;padding-bottom:0;overflow:visible !important}.navbar-collapse.in{overflow-y:visible}.navbar-fixed-top .navbar-collapse,.navbar-static-top .navbar-collapse,.navbar-fixed-bottom .navbar-collapse{padding-left:0;padding-right:0}}.navbar-fixed-top .navbar-collapse,.navbar-fixed-bottom .navbar-collapse{max-height:340px}@media (max-device-width:480px) and (orientation:landscape){.navbar-fixed-top .navbar-collapse,.navbar-fixed-bottom .navbar-collapse{max-height:200px}}.container>.navbar-header,.container-fluid>.navbar-header,.container>.navbar-collapse,.container-fluid>.navbar-collapse{margin-right:-15px;margin-left:-15px}@media (min-width:768px){.container>.navbar-header,.container-fluid>.navbar-header,.container>.navbar-collapse,.container-fluid>.navbar-collapse{margin-right:0;margin-left:0}}.navbar-static-top{z-index:1000;border-width:0 0 1px}@media (min-width:768px){.navbar-static-top{border-radius:0}}.navbar-fixed-top,.navbar-fixed-bottom{position:fixed;right:0;left:0;z-index:1030}@media (min-width:768px){.navbar-fixed-top,.navbar-fixed-bottom{border-radius:0}}.navbar-fixed-top{top:0;border-width:0 0 1px}.navbar-fixed-bottom{bottom:0;margin-bottom:0;border-width:1px 0 0}.navbar-brand{float:left;padding:21.5px 15px;font-size:20px;line-height:22px;height:65px}.navbar-brand:hover,.navbar-brand:focus{text-decoration:none}.navbar-brand>img{display:block}@media (min-width:768px){.navbar>.container .navbar-brand,.navbar>.container-fluid .navbar-brand{margin-left:-15px}}.navbar-toggle{position:relative;float:right;margin-right:15px;padding:9px 10px;margin-top:15.5px;margin-bottom:15.5px;background-color:transparent;background-image:none;border:1px solid transparent;border-radius:4px}.navbar-toggle:focus{outline:0}.navbar-toggle .icon-bar{display:block;width:22px;height:2px;border-radius:1px}.navbar-toggle .icon-bar+.icon-bar{margin-top:4px}@media (min-width:768px){.navbar-toggle{display:none}}.navbar-nav{margin:10.75px -15px}.navbar-nav>li>a{padding-top:10px;padding-bottom:10px;line-height:22px}@media (max-width:767px){.navbar-nav .open .dropdown-menu{position:static;float:none;width:auto;margin-top:0;background-color:transparent;border:0;-webkit-box-shadow:none;box-shadow:none}.navbar-nav .open .dropdown-menu>li>a,.navbar-nav .open .dropdown-menu .dropdown-header{padding:5px 15px 5px 25px}.navbar-nav .open .dropdown-menu>li>a{line-height:22px}.navbar-nav .open .dropdown-menu>li>a:hover,.navbar-nav .open .dropdown-menu>li>a:focus{background-image:none}}@media (min-width:768px){.navbar-nav{float:left;margin:0}.navbar-nav>li{float:left}.navbar-nav>li>a{padding-top:21.5px;padding-bottom:21.5px}}.navbar-form{margin-left:-15px;margin-right:-15px;padding:10px 15px;border-top:1px solid transparent;border-bottom:1px solid transparent;-webkit-box-shadow:inset 0 1px 0 rgba(255,255,255,0.1),0 1px 0 rgba(255,255,255,0.1);box-shadow:inset 0 1px 0 rgba(255,255,255,0.1),0 1px 0 rgba(255,255,255,0.1);margin-top:12.5px;margin-bottom:12.5px}@media (min-width:768px){.navbar-form .form-group{display:inline-block;margin-bottom:0;vertical-align:middle}.navbar-form .form-control{display:inline-block;width:auto;vertical-align:middle}.navbar-form .form-control-static{display:inline-block}.navbar-form .input-group{display:inline-table;vertical-align:middle}.navbar-form .input-group .input-group-addon,.navbar-form .input-group .input-group-btn,.navbar-form .input-group .form-control{width:auto}.navbar-form .input-group>.form-control{width:100%}.navbar-form .control-label{margin-bottom:0;vertical-align:middle}.navbar-form .radio,.navbar-form .checkbox{display:inline-block;margin-top:0;margin-bottom:0;vertical-align:middle}.navbar-form .radio label,.navbar-form .checkbox label{padding-left:0}.navbar-form .radio input[type="radio"],.navbar-form .checkbox input[type="checkbox"]{position:relative;margin-left:0}.navbar-form .has-feedback .form-control-feedback{top:0}}@media (max-width:767px){.navbar-form .form-group{margin-bottom:5px}.navbar-form .form-group:last-child{margin-bottom:0}}@media (min-width:768px){.navbar-form{width:auto;border:0;margin-left:0;margin-right:0;padding-top:0;padding-bottom:0;-webkit-box-shadow:none;box-shadow:none}}.navbar-nav>li>.dropdown-menu{margin-top:0;border-top-right-radius:0;border-top-left-radius:0}.navbar-fixed-bottom .navbar-nav>li>.dropdown-menu{margin-bottom:0;border-top-right-radius:4px;border-top-left-radius:4px;border-bottom-right-radius:0;border-bottom-left-radius:0}.navbar-btn{margin-top:12.5px;margin-bottom:12.5px}.navbar-btn.btn-sm{margin-top:16px;margin-bottom:16px}.navbar-btn.btn-xs{margin-top:21.5px;margin-bottom:21.5px}.navbar-text{margin-top:21.5px;margin-bottom:21.5px}@media (min-width:768px){.navbar-text{float:left;margin-left:15px;margin-right:15px}}@media (min-width:768px){.navbar-left{float:left !important}.navbar-right{float:right !important;margin-right:-15px}.navbar-right~.navbar-right{margin-right:0}}.navbar-default{background-color:#ffffff;border-color:#dddddd}.navbar-default .navbar-brand{color:#4582ec}.navbar-default .navbar-brand:hover,.navbar-default .navbar-brand:focus{color:#4582ec;background-color:transparent}.navbar-default .navbar-text{color:#333333}.navbar-default .navbar-nav>li>a{color:#4582ec}.navbar-default .navbar-nav>li>a:hover,.navbar-default .navbar-nav>li>a:focus{color:#4582ec;background-color:transparent}.navbar-default .navbar-nav>.active>a,.navbar-default .navbar-nav>.active>a:hover,.navbar-default .navbar-nav>.active>a:focus{color:#4582ec;background-color:transparent}.navbar-default .navbar-nav>.disabled>a,.navbar-default .navbar-nav>.disabled>a:hover,.navbar-default .navbar-nav>.disabled>a:focus{color:#333333;background-color:transparent}.navbar-default .navbar-toggle{border-color:#dddddd}.navbar-default .navbar-toggle:hover,.navbar-default .navbar-toggle:focus{background-color:#dddddd}.navbar-default .navbar-toggle .icon-bar{background-color:#cccccc}.navbar-default .navbar-collapse,.navbar-default .navbar-form{border-color:#dddddd}.navbar-default .navbar-nav>.open>a,.navbar-default .navbar-nav>.open>a:hover,.navbar-default .navbar-nav>.open>a:focus{background-color:transparent;color:#4582ec}@media (max-width:767px){.navbar-default .navbar-nav .open .dropdown-menu>li>a{color:#4582ec}.navbar-default .navbar-nav .open .dropdown-menu>li>a:hover,.navbar-default .navbar-nav .open .dropdown-menu>li>a:focus{color:#4582ec;background-color:transparent}.navbar-default .navbar-nav .open .dropdown-menu>.active>a,.navbar-default .navbar-nav .open .dropdown-menu>.active>a:hover,.navbar-default .navbar-nav .open .dropdown-menu>.active>a:focus{color:#4582ec;background-color:transparent}.navbar-default .navbar-nav .open .dropdown-menu>.disabled>a,.navbar-default .navbar-nav .open .dropdown-menu>.disabled>a:hover,.navbar-default .navbar-nav .open .dropdown-menu>.disabled>a:focus{color:#333333;background-color:transparent}}.navbar-default .navbar-link{color:#4582ec}.navbar-default .navbar-link:hover{color:#4582ec}.navbar-default .btn-link{color:#4582ec}.navbar-default .btn-link:hover,.navbar-default .btn-link:focus{color:#4582ec}.navbar-default .btn-link[disabled]:hover,fieldset[disabled] .navbar-default .btn-link:hover,.navbar-default .btn-link[disabled]:focus,fieldset[disabled] .navbar-default .btn-link:focus{color:#333333}.navbar-inverse{background-color:#ffffff;border-color:#dddddd}.navbar-inverse .navbar-brand{color:#333333}.navbar-inverse .navbar-brand:hover,.navbar-inverse .navbar-brand:focus{color:#333333;background-color:transparent}.navbar-inverse .navbar-text{color:#333333}.navbar-inverse .navbar-nav>li>a{color:#333333}.navbar-inverse .navbar-nav>li>a:hover,.navbar-inverse .navbar-nav>li>a:focus{color:#333333;background-color:transparent}.navbar-inverse .navbar-nav>.active>a,.navbar-inverse .navbar-nav>.active>a:hover,.navbar-inverse .navbar-nav>.active>a:focus{color:#333333;background-color:transparent}.navbar-inverse .navbar-nav>.disabled>a,.navbar-inverse .navbar-nav>.disabled>a:hover,.navbar-inverse .navbar-nav>.disabled>a:focus{color:#cccccc;background-color:transparent}.navbar-inverse .navbar-toggle{border-color:#dddddd}.navbar-inverse .navbar-toggle:hover,.navbar-inverse .navbar-toggle:focus{background-color:#dddddd}.navbar-inverse .navbar-toggle .icon-bar{background-color:#cccccc}.navbar-inverse .navbar-collapse,.navbar-inverse .navbar-form{border-color:#ededed}.navbar-inverse .navbar-nav>.open>a,.navbar-inverse .navbar-nav>.open>a:hover,.navbar-inverse .navbar-nav>.open>a:focus{background-color:transparent;color:#333333}@media (max-width:767px){.navbar-inverse .navbar-nav .open .dropdown-menu>.dropdown-header{border-color:#dddddd}.navbar-inverse .navbar-nav .open .dropdown-menu .divider{background-color:#dddddd}.navbar-inverse .navbar-nav .open .dropdown-menu>li>a{color:#333333}.navbar-inverse .navbar-nav .open .dropdown-menu>li>a:hover,.navbar-inverse .navbar-nav .open .dropdown-menu>li>a:focus{color:#333333;background-color:transparent}.navbar-inverse .navbar-nav .open .dropdown-menu>.active>a,.navbar-inverse .navbar-nav .open .dropdown-menu>.active>a:hover,.navbar-inverse .navbar-nav .open .dropdown-menu>.active>a:focus{color:#333333;background-color:transparent}.navbar-inverse .navbar-nav .open .dropdown-menu>.disabled>a,.navbar-inverse .navbar-nav .open .dropdown-menu>.disabled>a:hover,.navbar-inverse .navbar-nav .open .dropdown-menu>.disabled>a:focus{color:#cccccc;background-color:transparent}}.navbar-inverse .navbar-link{color:#333333}.navbar-inverse .navbar-link:hover{color:#333333}.navbar-inverse .btn-link{color:#333333}.navbar-inverse .btn-link:hover,.navbar-inverse .btn-link:focus{color:#333333}.navbar-inverse .btn-link[disabled]:hover,fieldset[disabled] .navbar-inverse .btn-link:hover,.navbar-inverse .btn-link[disabled]:focus,fieldset[disabled] .navbar-inverse .btn-link:focus{color:#cccccc}.breadcrumb{padding:8px 15px;margin-bottom:22px;list-style:none;background-color:#f5f5f5;border-radius:4px}.breadcrumb>li{display:inline-block}.breadcrumb>li+li:before{content:"/\00a0";padding:0 5px;color:#cccccc}.breadcrumb>.active{color:#b3b3b3}.pagination{display:inline-block;padding-left:0;margin:22px 0;border-radius:4px}.pagination>li{display:inline}.pagination>li>a,.pagination>li>span{position:relative;float:left;padding:8px 12px;line-height:1.42857143;text-decoration:none;color:#333333;background-color:#ffffff;border:1px solid #dddddd;margin-left:-1px}.pagination>li:first-child>a,.pagination>li:first-child>span{margin-left:0;border-bottom-left-radius:4px;border-top-left-radius:4px}.pagination>li:last-child>a,.pagination>li:last-child>span{border-bottom-right-radius:4px;border-top-right-radius:4px}.pagination>li>a:hover,.pagination>li>span:hover,.pagination>li>a:focus,.pagination>li>span:focus{z-index:2;color:#ffffff;background-color:#4582ec;border-color:#4582ec}.pagination>.active>a,.pagination>.active>span,.pagination>.active>a:hover,.pagination>.active>span:hover,.pagination>.active>a:focus,.pagination>.active>span:focus{z-index:3;color:#ffffff;background-color:#4582ec;border-color:#4582ec;cursor:default}.pagination>.disabled>span,.pagination>.disabled>span:hover,.pagination>.disabled>span:focus,.pagination>.disabled>a,.pagination>.disabled>a:hover,.pagination>.disabled>a:focus{color:#b3b3b3;background-color:#ffffff;border-color:#dddddd;cursor:not-allowed}.pagination-lg>li>a,.pagination-lg>li>span{padding:14px 16px;font-size:20px;line-height:1.3333333}.pagination-lg>li:first-child>a,.pagination-lg>li:first-child>span{border-bottom-left-radius:6px;border-top-left-radius:6px}.pagination-lg>li:last-child>a,.pagination-lg>li:last-child>span{border-bottom-right-radius:6px;border-top-right-radius:6px}.pagination-sm>li>a,.pagination-sm>li>span{padding:5px 10px;font-size:14px;line-height:1.5}.pagination-sm>li:first-child>a,.pagination-sm>li:first-child>span{border-bottom-left-radius:3px;border-top-left-radius:3px}.pagination-sm>li:last-child>a,.pagination-sm>li:last-child>span{border-bottom-right-radius:3px;border-top-right-radius:3px}.pager{padding-left:0;margin:22px 0;list-style:none;text-align:center}.pager li{display:inline}.pager li>a,.pager li>span{display:inline-block;padding:5px 14px;background-color:#ffffff;border:1px solid #dddddd;border-radius:15px}.pager li>a:hover,.pager li>a:focus{text-decoration:none;background-color:#4582ec}.pager .next>a,.pager .next>span{float:right}.pager .previous>a,.pager .previous>span{float:left}.pager .disabled>a,.pager .disabled>a:hover,.pager .disabled>a:focus,.pager .disabled>span{color:#b3b3b3;background-color:#ffffff;cursor:not-allowed}.label{display:inline;padding:.2em .6em .3em;font-size:75%;font-weight:bold;line-height:1;color:#ffffff;text-align:center;white-space:nowrap;vertical-align:baseline;border-radius:.25em}a.label:hover,a.label:focus{color:#ffffff;text-decoration:none;cursor:pointer}.label:empty{display:none}.btn .label{position:relative;top:-1px}.label-default{background-color:#ffffff}.label-default[href]:hover,.label-default[href]:focus{background-color:#e6e6e6}.label-primary{background-color:#4582ec}.label-primary[href]:hover,.label-primary[href]:focus{background-color:#1863e6}.label-success{background-color:#3fad46}.label-success[href]:hover,.label-success[href]:focus{background-color:#318837}.label-info{background-color:#5bc0de}.label-info[href]:hover,.label-info[href]:focus{background-color:#31b0d5}.label-warning{background-color:#f0ad4e}.label-warning[href]:hover,.label-warning[href]:focus{background-color:#ec971f}.label-danger{background-color:#d9534f}.label-danger[href]:hover,.label-danger[href]:focus{background-color:#c9302c}.badge{display:inline-block;min-width:10px;padding:3px 7px;font-size:14px;font-weight:bold;color:#ffffff;line-height:1;vertical-align:middle;white-space:nowrap;text-align:center;background-color:#4582ec;border-radius:10px}.badge:empty{display:none}.btn .badge{position:relative;top:-1px}.btn-xs .badge,.btn-group-xs>.btn .badge{top:0;padding:1px 5px}a.badge:hover,a.badge:focus{color:#ffffff;text-decoration:none;cursor:pointer}.list-group-item.active>.badge,.nav-pills>.active>a>.badge{color:#4582ec;background-color:#ffffff}.list-group-item>.badge{float:right}.list-group-item>.badge+.badge{margin-right:5px}.nav-pills>li>a>.badge{margin-left:3px}.jumbotron{padding-top:30px;padding-bottom:30px;margin-bottom:30px;color:inherit;background-color:#f7f7f7}.jumbotron h1,.jumbotron .h1{color:inherit}.jumbotron p{margin-bottom:15px;font-size:24px;font-weight:200}.jumbotron>hr{border-top-color:#dedede}.container .jumbotron,.container-fluid .jumbotron{border-radius:6px;padding-left:15px;padding-right:15px}.jumbotron .container{max-width:100%}@media screen and (min-width:768px){.jumbotron{padding-top:48px;padding-bottom:48px}.container .jumbotron,.container-fluid .jumbotron{padding-left:60px;padding-right:60px}.jumbotron h1,.jumbotron .h1{font-size:72px}}.thumbnail{display:block;padding:4px;margin-bottom:22px;line-height:1.42857143;background-color:#ffffff;border:1px solid #dddddd;border-radius:4px;-webkit-transition:border .2s ease-in-out;-o-transition:border .2s ease-in-out;transition:border .2s ease-in-out}.thumbnail>img,.thumbnail a>img{margin-left:auto;margin-right:auto}a.thumbnail:hover,a.thumbnail:focus,a.thumbnail.active{border-color:#4582ec}.thumbnail .caption{padding:9px;color:#333333}.alert{padding:15px;margin-bottom:22px;border:1px solid transparent;border-radius:4px}.alert h4{margin-top:0;color:inherit}.alert .alert-link{font-weight:bold}.alert>p,.alert>ul{margin-bottom:0}.alert>p+p{margin-top:5px}.alert-dismissable,.alert-dismissible{padding-right:35px}.alert-dismissable .close,.alert-dismissible .close{position:relative;top:-2px;right:-21px;color:inherit}.alert-success{background-color:#3fad46;border-color:#3fad46;color:#ffffff}.alert-success hr{border-top-color:#389a3e}.alert-success .alert-link{color:#e6e6e6}.alert-info{background-color:#5bc0de;border-color:#5bc0de;color:#ffffff}.alert-info hr{border-top-color:#46b8da}.alert-info .alert-link{color:#e6e6e6}.alert-warning{background-color:#f0ad4e;border-color:#f0ad4e;color:#ffffff}.alert-warning hr{border-top-color:#eea236}.alert-warning .alert-link{color:#e6e6e6}.alert-danger{background-color:#d9534f;border-color:#d9534f;color:#ffffff}.alert-danger hr{border-top-color:#d43f3a}.alert-danger .alert-link{color:#e6e6e6}@-webkit-keyframes progress-bar-stripes{from{background-position:40px 0}to{background-position:0 0}}@-o-keyframes progress-bar-stripes{from{background-position:40px 0}to{background-position:0 0}}@keyframes progress-bar-stripes{from{background-position:40px 0}to{background-position:0 0}}.progress{overflow:hidden;height:22px;margin-bottom:22px;background-color:#f5f5f5;border-radius:4px;-webkit-box-shadow:inset 0 1px 2px rgba(0,0,0,0.1);box-shadow:inset 0 1px 2px rgba(0,0,0,0.1)}.progress-bar{float:left;width:0%;height:100%;font-size:14px;line-height:22px;color:#ffffff;text-align:center;background-color:#4582ec;-webkit-box-shadow:inset 0 -1px 0 rgba(0,0,0,0.15);box-shadow:inset 0 -1px 0 rgba(0,0,0,0.15);-webkit-transition:width 0.6s ease;-o-transition:width 0.6s ease;transition:width 0.6s ease}.progress-striped .progress-bar,.progress-bar-striped{background-image:-webkit-linear-gradient(45deg, rgba(255,255,255,0.15) 25%, transparent 25%, transparent 50%, rgba(255,255,255,0.15) 50%, rgba(255,255,255,0.15) 75%, transparent 75%, transparent);background-image:-o-linear-gradient(45deg, rgba(255,255,255,0.15) 25%, transparent 25%, transparent 50%, rgba(255,255,255,0.15) 50%, rgba(255,255,255,0.15) 75%, transparent 75%, transparent);background-image:linear-gradient(45deg, rgba(255,255,255,0.15) 25%, transparent 25%, transparent 50%, rgba(255,255,255,0.15) 50%, rgba(255,255,255,0.15) 75%, transparent 75%, transparent);-webkit-background-size:40px 40px;background-size:40px 40px}.progress.active .progress-bar,.progress-bar.active{-webkit-animation:progress-bar-stripes 2s linear infinite;-o-animation:progress-bar-stripes 2s linear infinite;animation:progress-bar-stripes 2s linear infinite}.progress-bar-success{background-color:#3fad46}.progress-striped .progress-bar-success{background-image:-webkit-linear-gradient(45deg, rgba(255,255,255,0.15) 25%, transparent 25%, transparent 50%, rgba(255,255,255,0.15) 50%, rgba(255,255,255,0.15) 75%, transparent 75%, transparent);background-image:-o-linear-gradient(45deg, rgba(255,255,255,0.15) 25%, transparent 25%, transparent 50%, rgba(255,255,255,0.15) 50%, rgba(255,255,255,0.15) 75%, transparent 75%, transparent);background-image:linear-gradient(45deg, rgba(255,255,255,0.15) 25%, transparent 25%, transparent 50%, rgba(255,255,255,0.15) 50%, rgba(255,255,255,0.15) 75%, transparent 75%, transparent)}.progress-bar-info{background-color:#5bc0de}.progress-striped .progress-bar-info{background-image:-webkit-linear-gradient(45deg, rgba(255,255,255,0.15) 25%, transparent 25%, transparent 50%, rgba(255,255,255,0.15) 50%, rgba(255,255,255,0.15) 75%, transparent 75%, transparent);background-image:-o-linear-gradient(45deg, rgba(255,255,255,0.15) 25%, transparent 25%, transparent 50%, rgba(255,255,255,0.15) 50%, rgba(255,255,255,0.15) 75%, transparent 75%, transparent);background-image:linear-gradient(45deg, rgba(255,255,255,0.15) 25%, transparent 25%, transparent 50%, rgba(255,255,255,0.15) 50%, rgba(255,255,255,0.15) 75%, transparent 75%, transparent)}.progress-bar-warning{background-color:#f0ad4e}.progress-striped .progress-bar-warning{background-image:-webkit-linear-gradient(45deg, rgba(255,255,255,0.15) 25%, transparent 25%, transparent 50%, rgba(255,255,255,0.15) 50%, rgba(255,255,255,0.15) 75%, transparent 75%, transparent);background-image:-o-linear-gradient(45deg, rgba(255,255,255,0.15) 25%, transparent 25%, transparent 50%, rgba(255,255,255,0.15) 50%, rgba(255,255,255,0.15) 75%, transparent 75%, transparent);background-image:linear-gradient(45deg, rgba(255,255,255,0.15) 25%, transparent 25%, transparent 50%, rgba(255,255,255,0.15) 50%, rgba(255,255,255,0.15) 75%, transparent 75%, transparent)}.progress-bar-danger{background-color:#d9534f}.progress-striped .progress-bar-danger{background-image:-webkit-linear-gradient(45deg, rgba(255,255,255,0.15) 25%, transparent 25%, transparent 50%, rgba(255,255,255,0.15) 50%, rgba(255,255,255,0.15) 75%, transparent 75%, transparent);background-image:-o-linear-gradient(45deg, rgba(255,255,255,0.15) 25%, transparent 25%, transparent 50%, rgba(255,255,255,0.15) 50%, rgba(255,255,255,0.15) 75%, transparent 75%, transparent);background-image:linear-gradient(45deg, rgba(255,255,255,0.15) 25%, transparent 25%, transparent 50%, rgba(255,255,255,0.15) 50%, rgba(255,255,255,0.15) 75%, transparent 75%, transparent)}.media{margin-top:15px}.media:first-child{margin-top:0}.media,.media-body{zoom:1;overflow:hidden}.media-body{width:10000px}.media-object{display:block}.media-object.img-thumbnail{max-width:none}.media-right,.media>.pull-right{padding-left:10px}.media-left,.media>.pull-left{padding-right:10px}.media-left,.media-right,.media-body{display:table-cell;vertical-align:top}.media-middle{vertical-align:middle}.media-bottom{vertical-align:bottom}.media-heading{margin-top:0;margin-bottom:5px}.media-list{padding-left:0;list-style:none}.list-group{margin-bottom:20px;padding-left:0}.list-group-item{position:relative;display:block;padding:10px 15px;margin-bottom:-1px;background-color:#ffffff;border:1px solid #dddddd}.list-group-item:first-child{border-top-right-radius:4px;border-top-left-radius:4px}.list-group-item:last-child{margin-bottom:0;border-bottom-right-radius:4px;border-bottom-left-radius:4px}a.list-group-item,button.list-group-item{color:#555555}a.list-group-item .list-group-item-heading,button.list-group-item .list-group-item-heading{color:#333333}a.list-group-item:hover,button.list-group-item:hover,a.list-group-item:focus,button.list-group-item:focus{text-decoration:none;color:#555555;background-color:#f5f5f5}button.list-group-item{width:100%;text-align:left}.list-group-item.disabled,.list-group-item.disabled:hover,.list-group-item.disabled:focus{background-color:#eeeeee;color:#b3b3b3;cursor:not-allowed}.list-group-item.disabled .list-group-item-heading,.list-group-item.disabled:hover .list-group-item-heading,.list-group-item.disabled:focus .list-group-item-heading{color:inherit}.list-group-item.disabled .list-group-item-text,.list-group-item.disabled:hover .list-group-item-text,.list-group-item.disabled:focus .list-group-item-text{color:#b3b3b3}.list-group-item.active,.list-group-item.active:hover,.list-group-item.active:focus{z-index:2;color:#ffffff;background-color:#4582ec;border-color:#4582ec}.list-group-item.active .list-group-item-heading,.list-group-item.active:hover .list-group-item-heading,.list-group-item.active:focus .list-group-item-heading,.list-group-item.active .list-group-item-heading>small,.list-group-item.active:hover .list-group-item-heading>small,.list-group-item.active:focus .list-group-item-heading>small,.list-group-item.active .list-group-item-heading>.small,.list-group-item.active:hover .list-group-item-heading>.small,.list-group-item.active:focus .list-group-item-heading>.small{color:inherit}.list-group-item.active .list-group-item-text,.list-group-item.active:hover .list-group-item-text,.list-group-item.active:focus .list-group-item-text{color:#fefeff}.list-group-item-success{color:#3fad46;background-color:#dff0d8}a.list-group-item-success,button.list-group-item-success{color:#3fad46}a.list-group-item-success .list-group-item-heading,button.list-group-item-success .list-group-item-heading{color:inherit}a.list-group-item-success:hover,button.list-group-item-success:hover,a.list-group-item-success:focus,button.list-group-item-success:focus{color:#3fad46;background-color:#d0e9c6}a.list-group-item-success.active,button.list-group-item-success.active,a.list-group-item-success.active:hover,button.list-group-item-success.active:hover,a.list-group-item-success.active:focus,button.list-group-item-success.active:focus{color:#fff;background-color:#3fad46;border-color:#3fad46}.list-group-item-info{color:#5bc0de;background-color:#d9edf7}a.list-group-item-info,button.list-group-item-info{color:#5bc0de}a.list-group-item-info .list-group-item-heading,button.list-group-item-info .list-group-item-heading{color:inherit}a.list-group-item-info:hover,button.list-group-item-info:hover,a.list-group-item-info:focus,button.list-group-item-info:focus{color:#5bc0de;background-color:#c4e3f3}a.list-group-item-info.active,button.list-group-item-info.active,a.list-group-item-info.active:hover,button.list-group-item-info.active:hover,a.list-group-item-info.active:focus,button.list-group-item-info.active:focus{color:#fff;background-color:#5bc0de;border-color:#5bc0de}.list-group-item-warning{color:#f0ad4e;background-color:#fcf8e3}a.list-group-item-warning,button.list-group-item-warning{color:#f0ad4e}a.list-group-item-warning .list-group-item-heading,button.list-group-item-warning .list-group-item-heading{color:inherit}a.list-group-item-warning:hover,button.list-group-item-warning:hover,a.list-group-item-warning:focus,button.list-group-item-warning:focus{color:#f0ad4e;background-color:#faf2cc}a.list-group-item-warning.active,button.list-group-item-warning.active,a.list-group-item-warning.active:hover,button.list-group-item-warning.active:hover,a.list-group-item-warning.active:focus,button.list-group-item-warning.active:focus{color:#fff;background-color:#f0ad4e;border-color:#f0ad4e}.list-group-item-danger{color:#d9534f;background-color:#f2dede}a.list-group-item-danger,button.list-group-item-danger{color:#d9534f}a.list-group-item-danger .list-group-item-heading,button.list-group-item-danger .list-group-item-heading{color:inherit}a.list-group-item-danger:hover,button.list-group-item-danger:hover,a.list-group-item-danger:focus,button.list-group-item-danger:focus{color:#d9534f;background-color:#ebcccc}a.list-group-item-danger.active,button.list-group-item-danger.active,a.list-group-item-danger.active:hover,button.list-group-item-danger.active:hover,a.list-group-item-danger.active:focus,button.list-group-item-danger.active:focus{color:#fff;background-color:#d9534f;border-color:#d9534f}.list-group-item-heading{margin-top:0;margin-bottom:5px}.list-group-item-text{margin-bottom:0;line-height:1.3}.panel{margin-bottom:22px;background-color:#ffffff;border:1px solid transparent;border-radius:4px;-webkit-box-shadow:0 1px 1px rgba(0,0,0,0.05);box-shadow:0 1px 1px rgba(0,0,0,0.05)}.panel-body{padding:15px}.panel-heading{padding:10px 15px;border-bottom:1px solid transparent;border-top-right-radius:3px;border-top-left-radius:3px}.panel-heading>.dropdown .dropdown-toggle{color:inherit}.panel-title{margin-top:0;margin-bottom:0;font-size:18px;color:inherit}.panel-title>a,.panel-title>small,.panel-title>.small,.panel-title>small>a,.panel-title>.small>a{color:inherit}.panel-footer{padding:10px 15px;background-color:#ffffff;border-top:1px solid #dddddd;border-bottom-right-radius:3px;border-bottom-left-radius:3px}.panel>.list-group,.panel>.panel-collapse>.list-group{margin-bottom:0}.panel>.list-group .list-group-item,.panel>.panel-collapse>.list-group .list-group-item{border-width:1px 0;border-radius:0}.panel>.list-group:first-child .list-group-item:first-child,.panel>.panel-collapse>.list-group:first-child .list-group-item:first-child{border-top:0;border-top-right-radius:3px;border-top-left-radius:3px}.panel>.list-group:last-child .list-group-item:last-child,.panel>.panel-collapse>.list-group:last-child .list-group-item:last-child{border-bottom:0;border-bottom-right-radius:3px;border-bottom-left-radius:3px}.panel>.panel-heading+.panel-collapse>.list-group .list-group-item:first-child{border-top-right-radius:0;border-top-left-radius:0}.panel-heading+.list-group .list-group-item:first-child{border-top-width:0}.list-group+.panel-footer{border-top-width:0}.panel>.table,.panel>.table-responsive>.table,.panel>.panel-collapse>.table{margin-bottom:0}.panel>.table caption,.panel>.table-responsive>.table caption,.panel>.panel-collapse>.table caption{padding-left:15px;padding-right:15px}.panel>.table:first-child,.panel>.table-responsive:first-child>.table:first-child{border-top-right-radius:3px;border-top-left-radius:3px}.panel>.table:first-child>thead:first-child>tr:first-child,.panel>.table-responsive:first-child>.table:first-child>thead:first-child>tr:first-child,.panel>.table:first-child>tbody:first-child>tr:first-child,.panel>.table-responsive:first-child>.table:first-child>tbody:first-child>tr:first-child{border-top-left-radius:3px;border-top-right-radius:3px}.panel>.table:first-child>thead:first-child>tr:first-child td:first-child,.panel>.table-responsive:first-child>.table:first-child>thead:first-child>tr:first-child td:first-child,.panel>.table:first-child>tbody:first-child>tr:first-child td:first-child,.panel>.table-responsive:first-child>.table:first-child>tbody:first-child>tr:first-child td:first-child,.panel>.table:first-child>thead:first-child>tr:first-child th:first-child,.panel>.table-responsive:first-child>.table:first-child>thead:first-child>tr:first-child th:first-child,.panel>.table:first-child>tbody:first-child>tr:first-child th:first-child,.panel>.table-responsive:first-child>.table:first-child>tbody:first-child>tr:first-child th:first-child{border-top-left-radius:3px}.panel>.table:first-child>thead:first-child>tr:first-child td:last-child,.panel>.table-responsive:first-child>.table:first-child>thead:first-child>tr:first-child td:last-child,.panel>.table:first-child>tbody:first-child>tr:first-child td:last-child,.panel>.table-responsive:first-child>.table:first-child>tbody:first-child>tr:first-child td:last-child,.panel>.table:first-child>thead:first-child>tr:first-child th:last-child,.panel>.table-responsive:first-child>.table:first-child>thead:first-child>tr:first-child th:last-child,.panel>.table:first-child>tbody:first-child>tr:first-child th:last-child,.panel>.table-responsive:first-child>.table:first-child>tbody:first-child>tr:first-child th:last-child{border-top-right-radius:3px}.panel>.table:last-child,.panel>.table-responsive:last-child>.table:last-child{border-bottom-right-radius:3px;border-bottom-left-radius:3px}.panel>.table:last-child>tbody:last-child>tr:last-child,.panel>.table-responsive:last-child>.table:last-child>tbody:last-child>tr:last-child,.panel>.table:last-child>tfoot:last-child>tr:last-child,.panel>.table-responsive:last-child>.table:last-child>tfoot:last-child>tr:last-child{border-bottom-left-radius:3px;border-bottom-right-radius:3px}.panel>.table:last-child>tbody:last-child>tr:last-child td:first-child,.panel>.table-responsive:last-child>.table:last-child>tbody:last-child>tr:last-child td:first-child,.panel>.table:last-child>tfoot:last-child>tr:last-child td:first-child,.panel>.table-responsive:last-child>.table:last-child>tfoot:last-child>tr:last-child td:first-child,.panel>.table:last-child>tbody:last-child>tr:last-child th:first-child,.panel>.table-responsive:last-child>.table:last-child>tbody:last-child>tr:last-child th:first-child,.panel>.table:last-child>tfoot:last-child>tr:last-child th:first-child,.panel>.table-responsive:last-child>.table:last-child>tfoot:last-child>tr:last-child th:first-child{border-bottom-left-radius:3px}.panel>.table:last-child>tbody:last-child>tr:last-child td:last-child,.panel>.table-responsive:last-child>.table:last-child>tbody:last-child>tr:last-child td:last-child,.panel>.table:last-child>tfoot:last-child>tr:last-child td:last-child,.panel>.table-responsive:last-child>.table:last-child>tfoot:last-child>tr:last-child td:last-child,.panel>.table:last-child>tbody:last-child>tr:last-child th:last-child,.panel>.table-responsive:last-child>.table:last-child>tbody:last-child>tr:last-child th:last-child,.panel>.table:last-child>tfoot:last-child>tr:last-child th:last-child,.panel>.table-responsive:last-child>.table:last-child>tfoot:last-child>tr:last-child th:last-child{border-bottom-right-radius:3px}.panel>.panel-body+.table,.panel>.panel-body+.table-responsive,.panel>.table+.panel-body,.panel>.table-responsive+.panel-body{border-top:1px solid #dddddd}.panel>.table>tbody:first-child>tr:first-child th,.panel>.table>tbody:first-child>tr:first-child td{border-top:0}.panel>.table-bordered,.panel>.table-responsive>.table-bordered{border:0}.panel>.table-bordered>thead>tr>th:first-child,.panel>.table-responsive>.table-bordered>thead>tr>th:first-child,.panel>.table-bordered>tbody>tr>th:first-child,.panel>.table-responsive>.table-bordered>tbody>tr>th:first-child,.panel>.table-bordered>tfoot>tr>th:first-child,.panel>.table-responsive>.table-bordered>tfoot>tr>th:first-child,.panel>.table-bordered>thead>tr>td:first-child,.panel>.table-responsive>.table-bordered>thead>tr>td:first-child,.panel>.table-bordered>tbody>tr>td:first-child,.panel>.table-responsive>.table-bordered>tbody>tr>td:first-child,.panel>.table-bordered>tfoot>tr>td:first-child,.panel>.table-responsive>.table-bordered>tfoot>tr>td:first-child{border-left:0}.panel>.table-bordered>thead>tr>th:last-child,.panel>.table-responsive>.table-bordered>thead>tr>th:last-child,.panel>.table-bordered>tbody>tr>th:last-child,.panel>.table-responsive>.table-bordered>tbody>tr>th:last-child,.panel>.table-bordered>tfoot>tr>th:last-child,.panel>.table-responsive>.table-bordered>tfoot>tr>th:last-child,.panel>.table-bordered>thead>tr>td:last-child,.panel>.table-responsive>.table-bordered>thead>tr>td:last-child,.panel>.table-bordered>tbody>tr>td:last-child,.panel>.table-responsive>.table-bordered>tbody>tr>td:last-child,.panel>.table-bordered>tfoot>tr>td:last-child,.panel>.table-responsive>.table-bordered>tfoot>tr>td:last-child{border-right:0}.panel>.table-bordered>thead>tr:first-child>td,.panel>.table-responsive>.table-bordered>thead>tr:first-child>td,.panel>.table-bordered>tbody>tr:first-child>td,.panel>.table-responsive>.table-bordered>tbody>tr:first-child>td,.panel>.table-bordered>thead>tr:first-child>th,.panel>.table-responsive>.table-bordered>thead>tr:first-child>th,.panel>.table-bordered>tbody>tr:first-child>th,.panel>.table-responsive>.table-bordered>tbody>tr:first-child>th{border-bottom:0}.panel>.table-bordered>tbody>tr:last-child>td,.panel>.table-responsive>.table-bordered>tbody>tr:last-child>td,.panel>.table-bordered>tfoot>tr:last-child>td,.panel>.table-responsive>.table-bordered>tfoot>tr:last-child>td,.panel>.table-bordered>tbody>tr:last-child>th,.panel>.table-responsive>.table-bordered>tbody>tr:last-child>th,.panel>.table-bordered>tfoot>tr:last-child>th,.panel>.table-responsive>.table-bordered>tfoot>tr:last-child>th{border-bottom:0}.panel>.table-responsive{border:0;margin-bottom:0}.panel-group{margin-bottom:22px}.panel-group .panel{margin-bottom:0;border-radius:4px}.panel-group .panel+.panel{margin-top:5px}.panel-group .panel-heading{border-bottom:0}.panel-group .panel-heading+.panel-collapse>.panel-body,.panel-group .panel-heading+.panel-collapse>.list-group{border-top:1px solid #dddddd}.panel-group .panel-footer{border-top:0}.panel-group .panel-footer+.panel-collapse .panel-body{border-bottom:1px solid #dddddd}.panel-default{border-color:#dddddd}.panel-default>.panel-heading{color:#333333;background-color:#f5f5f5;border-color:#dddddd}.panel-default>.panel-heading+.panel-collapse>.panel-body{border-top-color:#dddddd}.panel-default>.panel-heading .badge{color:#f5f5f5;background-color:#333333}.panel-default>.panel-footer+.panel-collapse>.panel-body{border-bottom-color:#dddddd}.panel-primary{border-color:#4582ec}.panel-primary>.panel-heading{color:#ffffff;background-color:#4582ec;border-color:#4582ec}.panel-primary>.panel-heading+.panel-collapse>.panel-body{border-top-color:#4582ec}.panel-primary>.panel-heading .badge{color:#4582ec;background-color:#ffffff}.panel-primary>.panel-footer+.panel-collapse>.panel-body{border-bottom-color:#4582ec}.panel-success{border-color:#3fad46}.panel-success>.panel-heading{color:#ffffff;background-color:#3fad46;border-color:#3fad46}.panel-success>.panel-heading+.panel-collapse>.panel-body{border-top-color:#3fad46}.panel-success>.panel-heading .badge{color:#3fad46;background-color:#ffffff}.panel-success>.panel-footer+.panel-collapse>.panel-body{border-bottom-color:#3fad46}.panel-info{border-color:#5bc0de}.panel-info>.panel-heading{color:#ffffff;background-color:#5bc0de;border-color:#5bc0de}.panel-info>.panel-heading+.panel-collapse>.panel-body{border-top-color:#5bc0de}.panel-info>.panel-heading .badge{color:#5bc0de;background-color:#ffffff}.panel-info>.panel-footer+.panel-collapse>.panel-body{border-bottom-color:#5bc0de}.panel-warning{border-color:#f0ad4e}.panel-warning>.panel-heading{color:#ffffff;background-color:#f0ad4e;border-color:#f0ad4e}.panel-warning>.panel-heading+.panel-collapse>.panel-body{border-top-color:#f0ad4e}.panel-warning>.panel-heading .badge{color:#f0ad4e;background-color:#ffffff}.panel-warning>.panel-footer+.panel-collapse>.panel-body{border-bottom-color:#f0ad4e}.panel-danger{border-color:#d9534f}.panel-danger>.panel-heading{color:#ffffff;background-color:#d9534f;border-color:#d9534f}.panel-danger>.panel-heading+.panel-collapse>.panel-body{border-top-color:#d9534f}.panel-danger>.panel-heading .badge{color:#d9534f;background-color:#ffffff}.panel-danger>.panel-footer+.panel-collapse>.panel-body{border-bottom-color:#d9534f}.embed-responsive{position:relative;display:block;height:0;padding:0;overflow:hidden}.embed-responsive .embed-responsive-item,.embed-responsive iframe,.embed-responsive embed,.embed-responsive object,.embed-responsive video{position:absolute;top:0;left:0;bottom:0;height:100%;width:100%;border:0}.embed-responsive-16by9{padding-bottom:56.25%}.embed-responsive-4by3{padding-bottom:75%}.well{min-height:20px;padding:19px;margin-bottom:20px;background-color:#f7f7f7;border:1px solid #e5e5e5;border-radius:4px;-webkit-box-shadow:inset 0 1px 1px rgba(0,0,0,0.05);box-shadow:inset 0 1px 1px rgba(0,0,0,0.05)}.well blockquote{border-color:#ddd;border-color:rgba(0,0,0,0.15)}.well-lg{padding:24px;border-radius:6px}.well-sm{padding:9px;border-radius:3px}.close{float:right;font-size:24px;font-weight:bold;line-height:1;color:#ffffff;text-shadow:0 1px 0 #ffffff;opacity:0.2;filter:alpha(opacity=20)}.close:hover,.close:focus{color:#ffffff;text-decoration:none;cursor:pointer;opacity:0.5;filter:alpha(opacity=50)}button.close{padding:0;cursor:pointer;background:transparent;border:0;-webkit-appearance:none}.modal-open{overflow:hidden}.modal{display:none;overflow:hidden;position:fixed;top:0;right:0;bottom:0;left:0;z-index:1050;-webkit-overflow-scrolling:touch;outline:0}.modal.fade .modal-dialog{-webkit-transform:translate(0, -25%);-ms-transform:translate(0, -25%);-o-transform:translate(0, -25%);transform:translate(0, -25%);-webkit-transition:-webkit-transform .3s ease-out;-o-transition:-o-transform .3s ease-out;transition:transform .3s ease-out}.modal.in .modal-dialog{-webkit-transform:translate(0, 0);-ms-transform:translate(0, 0);-o-transform:translate(0, 0);transform:translate(0, 0)}.modal-open .modal{overflow-x:hidden;overflow-y:auto}.modal-dialog{position:relative;width:auto;margin:10px}.modal-content{position:relative;background-color:#ffffff;border:1px solid #999999;border:1px solid rgba(0,0,0,0.2);border-radius:6px;-webkit-box-shadow:0 3px 9px rgba(0,0,0,0.5);box-shadow:0 3px 9px rgba(0,0,0,0.5);-webkit-background-clip:padding-box;background-clip:padding-box;outline:0}.modal-backdrop{position:fixed;top:0;right:0;bottom:0;left:0;z-index:1040;background-color:#000000}.modal-backdrop.fade{opacity:0;filter:alpha(opacity=0)}.modal-backdrop.in{opacity:0.5;filter:alpha(opacity=50)}.modal-header{padding:15px;border-bottom:1px solid #e5e5e5}.modal-header .close{margin-top:-2px}.modal-title{margin:0;line-height:1.42857143}.modal-body{position:relative;padding:20px}.modal-footer{padding:20px;text-align:right;border-top:1px solid #e5e5e5}.modal-footer .btn+.btn{margin-left:5px;margin-bottom:0}.modal-footer .btn-group .btn+.btn{margin-left:-1px}.modal-footer .btn-block+.btn-block{margin-left:0}.modal-scrollbar-measure{position:absolute;top:-9999px;width:50px;height:50px;overflow:scroll}@media (min-width:768px){.modal-dialog{width:600px;margin:30px auto}.modal-content{-webkit-box-shadow:0 5px 15px rgba(0,0,0,0.5);box-shadow:0 5px 15px rgba(0,0,0,0.5)}.modal-sm{width:300px}}@media (min-width:992px){.modal-lg{width:900px}}.tooltip{position:absolute;z-index:1070;display:block;font-family:Georgia,"Times New Roman",Times,serif;font-style:normal;font-weight:normal;letter-spacing:normal;line-break:auto;line-height:1.42857143;text-align:left;text-align:start;text-decoration:none;text-shadow:none;text-transform:none;white-space:normal;word-break:normal;word-spacing:normal;word-wrap:normal;font-size:14px;opacity:0;filter:alpha(opacity=0)}.tooltip.in{opacity:0.9;filter:alpha(opacity=90)}.tooltip.top{margin-top:-3px;padding:5px 0}.tooltip.right{margin-left:3px;padding:0 5px}.tooltip.bottom{margin-top:3px;padding:5px 0}.tooltip.left{margin-left:-3px;padding:0 5px}.tooltip-inner{max-width:200px;padding:3px 8px;color:#ffffff;text-align:center;background-color:#000000;border-radius:4px}.tooltip-arrow{position:absolute;width:0;height:0;border-color:transparent;border-style:solid}.tooltip.top .tooltip-arrow{bottom:0;left:50%;margin-left:-5px;border-width:5px 5px 0;border-top-color:#000000}.tooltip.top-left .tooltip-arrow{bottom:0;right:5px;margin-bottom:-5px;border-width:5px 5px 0;border-top-color:#000000}.tooltip.top-right .tooltip-arrow{bottom:0;left:5px;margin-bottom:-5px;border-width:5px 5px 0;border-top-color:#000000}.tooltip.right .tooltip-arrow{top:50%;left:0;margin-top:-5px;border-width:5px 5px 5px 0;border-right-color:#000000}.tooltip.left .tooltip-arrow{top:50%;right:0;margin-top:-5px;border-width:5px 0 5px 5px;border-left-color:#000000}.tooltip.bottom .tooltip-arrow{top:0;left:50%;margin-left:-5px;border-width:0 5px 5px;border-bottom-color:#000000}.tooltip.bottom-left .tooltip-arrow{top:0;right:5px;margin-top:-5px;border-width:0 5px 5px;border-bottom-color:#000000}.tooltip.bottom-right .tooltip-arrow{top:0;left:5px;margin-top:-5px;border-width:0 5px 5px;border-bottom-color:#000000}.popover{position:absolute;top:0;left:0;z-index:1060;display:none;max-width:276px;padding:1px;font-family:Georgia,"Times New Roman",Times,serif;font-style:normal;font-weight:normal;letter-spacing:normal;line-break:auto;line-height:1.42857143;text-align:left;text-align:start;text-decoration:none;text-shadow:none;text-transform:none;white-space:normal;word-break:normal;word-spacing:normal;word-wrap:normal;font-size:16px;background-color:#ffffff;-webkit-background-clip:padding-box;background-clip:padding-box;border:1px solid #cccccc;border:1px solid rgba(0,0,0,0.2);border-radius:6px;-webkit-box-shadow:0 5px 10px rgba(0,0,0,0.2);box-shadow:0 5px 10px rgba(0,0,0,0.2)}.popover.top{margin-top:-10px}.popover.right{margin-left:10px}.popover.bottom{margin-top:10px}.popover.left{margin-left:-10px}.popover-title{margin:0;padding:8px 14px;font-size:16px;background-color:#f7f7f7;border-bottom:1px solid #ebebeb;border-radius:5px 5px 0 0}.popover-content{padding:9px 14px}.popover>.arrow,.popover>.arrow:after{position:absolute;display:block;width:0;height:0;border-color:transparent;border-style:solid}.popover>.arrow{border-width:11px}.popover>.arrow:after{border-width:10px;content:""}.popover.top>.arrow{left:50%;margin-left:-11px;border-bottom-width:0;border-top-color:#999999;border-top-color:rgba(0,0,0,0.25);bottom:-11px}.popover.top>.arrow:after{content:" ";bottom:1px;margin-left:-10px;border-bottom-width:0;border-top-color:#ffffff}.popover.right>.arrow{top:50%;left:-11px;margin-top:-11px;border-left-width:0;border-right-color:#999999;border-right-color:rgba(0,0,0,0.25)}.popover.right>.arrow:after{content:" ";left:1px;bottom:-10px;border-left-width:0;border-right-color:#ffffff}.popover.bottom>.arrow{left:50%;margin-left:-11px;border-top-width:0;border-bottom-color:#999999;border-bottom-color:rgba(0,0,0,0.25);top:-11px}.popover.bottom>.arrow:after{content:" ";top:1px;margin-left:-10px;border-top-width:0;border-bottom-color:#ffffff}.popover.left>.arrow{top:50%;right:-11px;margin-top:-11px;border-right-width:0;border-left-color:#999999;border-left-color:rgba(0,0,0,0.25)}.popover.left>.arrow:after{content:" ";right:1px;border-right-width:0;border-left-color:#ffffff;bottom:-10px}.carousel{position:relative}.carousel-inner{position:relative;overflow:hidden;width:100%}.carousel-inner>.item{display:none;position:relative;-webkit-transition:.6s ease-in-out left;-o-transition:.6s ease-in-out left;transition:.6s ease-in-out left}.carousel-inner>.item>img,.carousel-inner>.item>a>img{line-height:1}@media all and (transform-3d),(-webkit-transform-3d){.carousel-inner>.item{-webkit-transition:-webkit-transform .6s ease-in-out;-o-transition:-o-transform .6s ease-in-out;transition:transform .6s ease-in-out;-webkit-backface-visibility:hidden;backface-visibility:hidden;-webkit-perspective:1000px;perspective:1000px}.carousel-inner>.item.next,.carousel-inner>.item.active.right{-webkit-transform:translate3d(100%, 0, 0);transform:translate3d(100%, 0, 0);left:0}.carousel-inner>.item.prev,.carousel-inner>.item.active.left{-webkit-transform:translate3d(-100%, 0, 0);transform:translate3d(-100%, 0, 0);left:0}.carousel-inner>.item.next.left,.carousel-inner>.item.prev.right,.carousel-inner>.item.active{-webkit-transform:translate3d(0, 0, 0);transform:translate3d(0, 0, 0);left:0}}.carousel-inner>.active,.carousel-inner>.next,.carousel-inner>.prev{display:block}.carousel-inner>.active{left:0}.carousel-inner>.next,.carousel-inner>.prev{position:absolute;top:0;width:100%}.carousel-inner>.next{left:100%}.carousel-inner>.prev{left:-100%}.carousel-inner>.next.left,.carousel-inner>.prev.right{left:0}.carousel-inner>.active.left{left:-100%}.carousel-inner>.active.right{left:100%}.carousel-control{position:absolute;top:0;left:0;bottom:0;width:15%;opacity:0.5;filter:alpha(opacity=50);font-size:20px;color:#ffffff;text-align:center;text-shadow:0 1px 2px rgba(0,0,0,0.6);background-color:rgba(0,0,0,0)}.carousel-control.left{background-image:-webkit-linear-gradient(left, rgba(0,0,0,0.5) 0, rgba(0,0,0,0.0001) 100%);background-image:-o-linear-gradient(left, rgba(0,0,0,0.5) 0, rgba(0,0,0,0.0001) 100%);background-image:-webkit-gradient(linear, left top, right top, from(rgba(0,0,0,0.5)), to(rgba(0,0,0,0.0001)));background-image:linear-gradient(to right, rgba(0,0,0,0.5) 0, rgba(0,0,0,0.0001) 100%);background-repeat:repeat-x;filter:progid:DXImageTransform.Microsoft.gradient(startColorstr='#80000000', endColorstr='#00000000', GradientType=1)}.carousel-control.right{left:auto;right:0;background-image:-webkit-linear-gradient(left, rgba(0,0,0,0.0001) 0, rgba(0,0,0,0.5) 100%);background-image:-o-linear-gradient(left, rgba(0,0,0,0.0001) 0, rgba(0,0,0,0.5) 100%);background-image:-webkit-gradient(linear, left top, right top, from(rgba(0,0,0,0.0001)), to(rgba(0,0,0,0.5)));background-image:linear-gradient(to right, rgba(0,0,0,0.0001) 0, rgba(0,0,0,0.5) 100%);background-repeat:repeat-x;filter:progid:DXImageTransform.Microsoft.gradient(startColorstr='#00000000', endColorstr='#80000000', GradientType=1)}.carousel-control:hover,.carousel-control:focus{outline:0;color:#ffffff;text-decoration:none;opacity:0.9;filter:alpha(opacity=90)}.carousel-control .icon-prev,.carousel-control .icon-next,.carousel-control .glyphicon-chevron-left,.carousel-control .glyphicon-chevron-right{position:absolute;top:50%;margin-top:-10px;z-index:5;display:inline-block}.carousel-control .icon-prev,.carousel-control .glyphicon-chevron-left{left:50%;margin-left:-10px}.carousel-control .icon-next,.carousel-control .glyphicon-chevron-right{right:50%;margin-right:-10px}.carousel-control .icon-prev,.carousel-control .icon-next{width:20px;height:20px;line-height:1;font-family:serif}.carousel-control .icon-prev:before{content:'\2039'}.carousel-control .icon-next:before{content:'\203a'}.carousel-indicators{position:absolute;bottom:10px;left:50%;z-index:15;width:60%;margin-left:-30%;padding-left:0;list-style:none;text-align:center}.carousel-indicators li{display:inline-block;width:10px;height:10px;margin:1px;text-indent:-999px;border:1px solid #ffffff;border-radius:10px;cursor:pointer;background-color:#000 \9;background-color:rgba(0,0,0,0)}.carousel-indicators .active{margin:0;width:12px;height:12px;background-color:#ffffff}.carousel-caption{position:absolute;left:15%;right:15%;bottom:20px;z-index:10;padding-top:20px;padding-bottom:20px;color:#ffffff;text-align:center;text-shadow:0 1px 2px rgba(0,0,0,0.6)}.carousel-caption .btn{text-shadow:none}@media screen and (min-width:768px){.carousel-control .glyphicon-chevron-left,.carousel-control .glyphicon-chevron-right,.carousel-control .icon-prev,.carousel-control .icon-next{width:30px;height:30px;margin-top:-10px;font-size:30px}.carousel-control .glyphicon-chevron-left,.carousel-control .icon-prev{margin-left:-10px}.carousel-control .glyphicon-chevron-right,.carousel-control .icon-next{margin-right:-10px}.carousel-caption{left:20%;right:20%;padding-bottom:30px}.carousel-indicators{bottom:20px}}.clearfix:before,.clearfix:after,.dl-horizontal dd:before,.dl-horizontal dd:after,.container:before,.container:after,.container-fluid:before,.container-fluid:after,.row:before,.row:after,.form-horizontal .form-group:before,.form-horizontal .form-group:after,.btn-toolbar:before,.btn-toolbar:after,.btn-group-vertical>.btn-group:before,.btn-group-vertical>.btn-group:after,.nav:before,.nav:after,.navbar:before,.navbar:after,.navbar-header:before,.navbar-header:after,.navbar-collapse:before,.navbar-collapse:after,.pager:before,.pager:after,.panel-body:before,.panel-body:after,.modal-header:before,.modal-header:after,.modal-footer:before,.modal-footer:after{content:" ";display:table}.clearfix:after,.dl-horizontal dd:after,.container:after,.container-fluid:after,.row:after,.form-horizontal .form-group:after,.btn-toolbar:after,.btn-group-vertical>.btn-group:after,.nav:after,.navbar:after,.navbar-header:after,.navbar-collapse:after,.pager:after,.panel-body:after,.modal-header:after,.modal-footer:after{clear:both}.center-block{display:block;margin-left:auto;margin-right:auto}.pull-right{float:right !important}.pull-left{float:left !important}.hide{display:none !important}.show{display:block !important}.invisible{visibility:hidden}.text-hide{font:0/0 a;color:transparent;text-shadow:none;background-color:transparent;border:0}.hidden{display:none !important}.affix{position:fixed}@-ms-viewport{width:device-width}.visible-xs,.visible-sm,.visible-md,.visible-lg{display:none !important}.visible-xs-block,.visible-xs-inline,.visible-xs-inline-block,.visible-sm-block,.visible-sm-inline,.visible-sm-inline-block,.visible-md-block,.visible-md-inline,.visible-md-inline-block,.visible-lg-block,.visible-lg-inline,.visible-lg-inline-block{display:none !important}@media (max-width:767px){.visible-xs{display:block !important}table.visible-xs{display:table !important}tr.visible-xs{display:table-row !important}th.visible-xs,td.visible-xs{display:table-cell !important}}@media (max-width:767px){.visible-xs-block{display:block !important}}@media (max-width:767px){.visible-xs-inline{display:inline !important}}@media (max-width:767px){.visible-xs-inline-block{display:inline-block !important}}@media (min-width:768px) and (max-width:991px){.visible-sm{display:block !important}table.visible-sm{display:table !important}tr.visible-sm{display:table-row !important}th.visible-sm,td.visible-sm{display:table-cell !important}}@media (min-width:768px) and (max-width:991px){.visible-sm-block{display:block !important}}@media (min-width:768px) and (max-width:991px){.visible-sm-inline{display:inline !important}}@media (min-width:768px) and (max-width:991px){.visible-sm-inline-block{display:inline-block !important}}@media (min-width:992px) and (max-width:1199px){.visible-md{display:block !important}table.visible-md{display:table !important}tr.visible-md{display:table-row !important}th.visible-md,td.visible-md{display:table-cell !important}}@media (min-width:992px) and (max-width:1199px){.visible-md-block{display:block !important}}@media (min-width:992px) and (max-width:1199px){.visible-md-inline{display:inline !important}}@media (min-width:992px) and (max-width:1199px){.visible-md-inline-block{display:inline-block !important}}@media (min-width:1200px){.visible-lg{display:block !important}table.visible-lg{display:table !important}tr.visible-lg{display:table-row !important}th.visible-lg,td.visible-lg{display:table-cell !important}}@media (min-width:1200px){.visible-lg-block{display:block !important}}@media (min-width:1200px){.visible-lg-inline{display:inline !important}}@media (min-width:1200px){.visible-lg-inline-block{display:inline-block !important}}@media (max-width:767px){.hidden-xs{display:none !important}}@media (min-width:768px) and (max-width:991px){.hidden-sm{display:none !important}}@media (min-width:992px) and (max-width:1199px){.hidden-md{display:none !important}}@media (min-width:1200px){.hidden-lg{display:none !important}}.visible-print{display:none !important}@media print{.visible-print{display:block !important}table.visible-print{display:table !important}tr.visible-print{display:table-row !important}th.visible-print,td.visible-print{display:table-cell !important}}.visible-print-block{display:none !important}@media print{.visible-print-block{display:block !important}}.visible-print-inline{display:none !important}@media print{.visible-print-inline{display:inline !important}}.visible-print-inline-block{display:none !important}@media print{.visible-print-inline-block{display:inline-block !important}}@media print{.hidden-print{display:none !important}}.navbar{font-family:"Raleway","Helvetica Neue",Helvetica,Arial,sans-serif}.navbar-nav,.navbar-form{margin-left:0;margin-right:0}.navbar-nav>li>a{margin:12.5px 6px;padding:8px 12px;border:1px solid transparent;border-radius:4px}.navbar-nav>li>a:hover{border:1px solid #ddd}.navbar-nav>.active>a,.navbar-nav>.active>a:hover{border:1px solid #ddd}.navbar-default .navbar-nav>.active>a:hover{color:#4582ec}.navbar-inverse .navbar-nav>.active>a:hover{color:#333333}.navbar-brand{padding-top:12.5px;padding-bottom:12.5px;line-height:1.9}@media (min-width:768px){.navbar .navbar-nav>li>a{padding:8px 12px}}@media (max-width:767px){.navbar .navbar-nav>li>a{margin:0}}.btn{font-family:"Raleway","Helvetica Neue",Helvetica,Arial,sans-serif}legend{font-family:"Raleway","Helvetica Neue",Helvetica,Arial,sans-serif}.input-group-addon{font-family:"Raleway","Helvetica Neue",Helvetica,Arial,sans-serif}.nav .open>a,.nav .open>a:hover,.nav .open>a:focus{border:1px solid #ddd}.pagination{font-family:"Raleway","Helvetica Neue",Helvetica,Arial,sans-serif}.pagination-lg>li>a,.pagination-lg>li>span{padding:14px 24px}.pager{font-family:"Raleway","Helvetica Neue",Helvetica,Arial,sans-serif}.pager a{color:#333333}.pager a:hover{border-color:transparent;color:#fff}.pager .disabled a{border-color:#dddddd}.close{color:#fff;text-decoration:none;text-shadow:none;opacity:0.4}.close:hover,.close:focus{color:#fff;opacity:1}.alert .alert-link{color:#ffffff;text-decoration:underline}.label{font-family:"Raleway","Helvetica Neue",Helvetica,Arial,sans-serif;font-weight:normal}.label-default{border:1px solid #ddd;color:#333333}.badge{padding:1px 7px 5px;vertical-align:2px;font-family:"Raleway","Helvetica Neue",Helvetica,Arial,sans-serif;font-weight:normal}.panel{-webkit-box-shadow:none;box-shadow:none}.panel-default .close{color:#333333}.modal .close{color:#333333}
+</style>
+<script>/*!
+ * Bootstrap v3.3.5 (http://getbootstrap.com)
+ * Copyright 2011-2015 Twitter, Inc.
+ * Licensed under the MIT license
+ */
+if("undefined"==typeof jQuery)throw new Error("Bootstrap's JavaScript requires jQuery");+function(a){"use strict";var b=a.fn.jquery.split(" ")[0].split(".");if(b[0]<2&&b[1]<9||1==b[0]&&9==b[1]&&b[2]<1)throw new Error("Bootstrap's JavaScript requires jQuery version 1.9.1 or higher")}(jQuery),+function(a){"use strict";function b(){var a=document.createElement("bootstrap"),b={WebkitTransition:"webkitTransitionEnd",MozTransition:"transitionend",OTransition:"oTransitionEnd otransitionend",transition:"transitionend"};for(var c in b)if(void 0!==a.style[c])return{end:b[c]};return!1}a.fn.emulateTransitionEnd=function(b){var c=!1,d=this;a(this).one("bsTransitionEnd",function(){c=!0});var e=function(){c||a(d).trigger(a.support.transition.end)};return setTimeout(e,b),this},a(function(){a.support.transition=b(),a.support.transition&&(a.event.special.bsTransitionEnd={bindType:a.support.transition.end,delegateType:a.support.transition.end,handle:function(b){return a(b.target).is(this)?b.handleObj.handler.apply(this,arguments):void 0}})})}(jQuery),+function(a){"use strict";function b(b){return this.each(function(){var c=a(this),e=c.data("bs.alert");e||c.data("bs.alert",e=new d(this)),"string"==typeof b&&e[b].call(c)})}var c='[data-dismiss="alert"]',d=function(b){a(b).on("click",c,this.close)};d.VERSION="3.3.5",d.TRANSITION_DURATION=150,d.prototype.close=function(b){function c(){g.detach().trigger("closed.bs.alert").remove()}var e=a(this),f=e.attr("data-target");f||(f=e.attr("href"),f=f&&f.replace(/.*(?=#[^\s]*$)/,""));var g=a(f);b&&b.preventDefault(),g.length||(g=e.closest(".alert")),g.trigger(b=a.Event("close.bs.alert")),b.isDefaultPrevented()||(g.removeClass("in"),a.support.transition&&g.hasClass("fade")?g.one("bsTransitionEnd",c).emulateTransitionEnd(d.TRANSITION_DURATION):c())};var e=a.fn.alert;a.fn.alert=b,a.fn.alert.Constructor=d,a.fn.alert.noConflict=function(){return a.fn.alert=e,this},a(document).on("click.bs.alert.data-api",c,d.prototype.close)}(jQuery),+function(a){"use strict";function b(b){return this.each(function(){var d=a(this),e=d.data("bs.button"),f="object"==typeof b&&b;e||d.data("bs.button",e=new c(this,f)),"toggle"==b?e.toggle():b&&e.setState(b)})}var c=function(b,d){this.$element=a(b),this.options=a.extend({},c.DEFAULTS,d),this.isLoading=!1};c.VERSION="3.3.5",c.DEFAULTS={loadingText:"loading..."},c.prototype.setState=function(b){var c="disabled",d=this.$element,e=d.is("input")?"val":"html",f=d.data();b+="Text",null==f.resetText&&d.data("resetText",d[e]()),setTimeout(a.proxy(function(){d[e](null==f[b]?this.options[b]:f[b]),"loadingText"==b?(this.isLoading=!0,d.addClass(c).attr(c,c)):this.isLoading&&(this.isLoading=!1,d.removeClass(c).removeAttr(c))},this),0)},c.prototype.toggle=function(){var a=!0,b=this.$element.closest('[data-toggle="buttons"]');if(b.length){var c=this.$element.find("input");"radio"==c.prop("type")?(c.prop("checked")&&(a=!1),b.find(".active").removeClass("active"),this.$element.addClass("active")):"checkbox"==c.prop("type")&&(c.prop("checked")!==this.$element.hasClass("active")&&(a=!1),this.$element.toggleClass("active")),c.prop("checked",this.$element.hasClass("active")),a&&c.trigger("change")}else this.$element.attr("aria-pressed",!this.$element.hasClass("active")),this.$element.toggleClass("active")};var d=a.fn.button;a.fn.button=b,a.fn.button.Constructor=c,a.fn.button.noConflict=function(){return a.fn.button=d,this},a(document).on("click.bs.button.data-api",'[data-toggle^="button"]',function(c){var d=a(c.target);d.hasClass("btn")||(d=d.closest(".btn")),b.call(d,"toggle"),a(c.target).is('input[type="radio"]')||a(c.target).is('input[type="checkbox"]')||c.preventDefault()}).on("focus.bs.button.data-api blur.bs.button.data-api",'[data-toggle^="button"]',function(b){a(b.target).closest(".btn").toggleClass("focus",/^focus(in)?$/.test(b.type))})}(jQuery),+function(a){"use strict";function b(b){return this.each(function(){var d=a(this),e=d.data("bs.carousel"),f=a.extend({},c.DEFAULTS,d.data(),"object"==typeof b&&b),g="string"==typeof b?b:f.slide;e||d.data("bs.carousel",e=new c(this,f)),"number"==typeof b?e.to(b):g?e[g]():f.interval&&e.pause().cycle()})}var c=function(b,c){this.$element=a(b),this.$indicators=this.$element.find(".carousel-indicators"),this.options=c,this.paused=null,this.sliding=null,this.interval=null,this.$active=null,this.$items=null,this.options.keyboard&&this.$element.on("keydown.bs.carousel",a.proxy(this.keydown,this)),"hover"==this.options.pause&&!("ontouchstart"in document.documentElement)&&this.$element.on("mouseenter.bs.carousel",a.proxy(this.pause,this)).on("mouseleave.bs.carousel",a.proxy(this.cycle,this))};c.VERSION="3.3.5",c.TRANSITION_DURATION=600,c.DEFAULTS={interval:5e3,pause:"hover",wrap:!0,keyboard:!0},c.prototype.keydown=function(a){if(!/input|textarea/i.test(a.target.tagName)){switch(a.which){case 37:this.prev();break;case 39:this.next();break;default:return}a.preventDefault()}},c.prototype.cycle=function(b){return b||(this.paused=!1),this.interval&&clearInterval(this.interval),this.options.interval&&!this.paused&&(this.interval=setInterval(a.proxy(this.next,this),this.options.interval)),this},c.prototype.getItemIndex=function(a){return this.$items=a.parent().children(".item"),this.$items.index(a||this.$active)},c.prototype.getItemForDirection=function(a,b){var c=this.getItemIndex(b),d="prev"==a&&0===c||"next"==a&&c==this.$items.length-1;if(d&&!this.options.wrap)return b;var e="prev"==a?-1:1,f=(c+e)%this.$items.length;return this.$items.eq(f)},c.prototype.to=function(a){var b=this,c=this.getItemIndex(this.$active=this.$element.find(".item.active"));return a>this.$items.length-1||0>a?void 0:this.sliding?this.$element.one("slid.bs.carousel",function(){b.to(a)}):c==a?this.pause().cycle():this.slide(a>c?"next":"prev",this.$items.eq(a))},c.prototype.pause=function(b){return b||(this.paused=!0),this.$element.find(".next, .prev").length&&a.support.transition&&(this.$element.trigger(a.support.transition.end),this.cycle(!0)),this.interval=clearInterval(this.interval),this},c.prototype.next=function(){return this.sliding?void 0:this.slide("next")},c.prototype.prev=function(){return this.sliding?void 0:this.slide("prev")},c.prototype.slide=function(b,d){var e=this.$element.find(".item.active"),f=d||this.getItemForDirection(b,e),g=this.interval,h="next"==b?"left":"right",i=this;if(f.hasClass("active"))return this.sliding=!1;var j=f[0],k=a.Event("slide.bs.carousel",{relatedTarget:j,direction:h});if(this.$element.trigger(k),!k.isDefaultPrevented()){if(this.sliding=!0,g&&this.pause(),this.$indicators.length){this.$indicators.find(".active").removeClass("active");var l=a(this.$indicators.children()[this.getItemIndex(f)]);l&&l.addClass("active")}var m=a.Event("slid.bs.carousel",{relatedTarget:j,direction:h});return a.support.transition&&this.$element.hasClass("slide")?(f.addClass(b),f[0].offsetWidth,e.addClass(h),f.addClass(h),e.one("bsTransitionEnd",function(){f.removeClass([b,h].join(" ")).addClass("active"),e.removeClass(["active",h].join(" ")),i.sliding=!1,setTimeout(function(){i.$element.trigger(m)},0)}).emulateTransitionEnd(c.TRANSITION_DURATION)):(e.removeClass("active"),f.addClass("active"),this.sliding=!1,this.$element.trigger(m)),g&&this.cycle(),this}};var d=a.fn.carousel;a.fn.carousel=b,a.fn.carousel.Constructor=c,a.fn.carousel.noConflict=function(){return a.fn.carousel=d,this};var e=function(c){var d,e=a(this),f=a(e.attr("data-target")||(d=e.attr("href"))&&d.replace(/.*(?=#[^\s]+$)/,""));if(f.hasClass("carousel")){var g=a.extend({},f.data(),e.data()),h=e.attr("data-slide-to");h&&(g.interval=!1),b.call(f,g),h&&f.data("bs.carousel").to(h),c.preventDefault()}};a(document).on("click.bs.carousel.data-api","[data-slide]",e).on("click.bs.carousel.data-api","[data-slide-to]",e),a(window).on("load",function(){a('[data-ride="carousel"]').each(function(){var c=a(this);b.call(c,c.data())})})}(jQuery),+function(a){"use strict";function b(b){var c,d=b.attr("data-target")||(c=b.attr("href"))&&c.replace(/.*(?=#[^\s]+$)/,"");return a(d)}function c(b){return this.each(function(){var c=a(this),e=c.data("bs.collapse"),f=a.extend({},d.DEFAULTS,c.data(),"object"==typeof b&&b);!e&&f.toggle&&/show|hide/.test(b)&&(f.toggle=!1),e||c.data("bs.collapse",e=new d(this,f)),"string"==typeof b&&e[b]()})}var d=function(b,c){this.$element=a(b),this.options=a.extend({},d.DEFAULTS,c),this.$trigger=a('[data-toggle="collapse"][href="#'+b.id+'"],[data-toggle="collapse"][data-target="#'+b.id+'"]'),this.transitioning=null,this.options.parent?this.$parent=this.getParent():this.addAriaAndCollapsedClass(this.$element,this.$trigger),this.options.toggle&&this.toggle()};d.VERSION="3.3.5",d.TRANSITION_DURATION=350,d.DEFAULTS={toggle:!0},d.prototype.dimension=function(){var a=this.$element.hasClass("width");return a?"width":"height"},d.prototype.show=function(){if(!this.transitioning&&!this.$element.hasClass("in")){var b,e=this.$parent&&this.$parent.children(".panel").children(".in, .collapsing");if(!(e&&e.length&&(b=e.data("bs.collapse"),b&&b.transitioning))){var f=a.Event("show.bs.collapse");if(this.$element.trigger(f),!f.isDefaultPrevented()){e&&e.length&&(c.call(e,"hide"),b||e.data("bs.collapse",null));var g=this.dimension();this.$element.removeClass("collapse").addClass("collapsing")[g](0).attr("aria-expanded",!0),this.$trigger.removeClass("collapsed").attr("aria-expanded",!0),this.transitioning=1;var h=function(){this.$element.removeClass("collapsing").addClass("collapse in")[g](""),this.transitioning=0,this.$element.trigger("shown.bs.collapse")};if(!a.support.transition)return h.call(this);var i=a.camelCase(["scroll",g].join("-"));this.$element.one("bsTransitionEnd",a.proxy(h,this)).emulateTransitionEnd(d.TRANSITION_DURATION)[g](this.$element[0][i])}}}},d.prototype.hide=function(){if(!this.transitioning&&this.$element.hasClass("in")){var b=a.Event("hide.bs.collapse");if(this.$element.trigger(b),!b.isDefaultPrevented()){var c=this.dimension();this.$element[c](this.$element[c]())[0].offsetHeight,this.$element.addClass("collapsing").removeClass("collapse in").attr("aria-expanded",!1),this.$trigger.addClass("collapsed").attr("aria-expanded",!1),this.transitioning=1;var e=function(){this.transitioning=0,this.$element.removeClass("collapsing").addClass("collapse").trigger("hidden.bs.collapse")};return a.support.transition?void this.$element[c](0).one("bsTransitionEnd",a.proxy(e,this)).emulateTransitionEnd(d.TRANSITION_DURATION):e.call(this)}}},d.prototype.toggle=function(){this[this.$element.hasClass("in")?"hide":"show"]()},d.prototype.getParent=function(){return a(this.options.parent).find('[data-toggle="collapse"][data-parent="'+this.options.parent+'"]').each(a.proxy(function(c,d){var e=a(d);this.addAriaAndCollapsedClass(b(e),e)},this)).end()},d.prototype.addAriaAndCollapsedClass=function(a,b){var c=a.hasClass("in");a.attr("aria-expanded",c),b.toggleClass("collapsed",!c).attr("aria-expanded",c)};var e=a.fn.collapse;a.fn.collapse=c,a.fn.collapse.Constructor=d,a.fn.collapse.noConflict=function(){return a.fn.collapse=e,this},a(document).on("click.bs.collapse.data-api",'[data-toggle="collapse"]',function(d){var e=a(this);e.attr("data-target")||d.preventDefault();var f=b(e),g=f.data("bs.collapse"),h=g?"toggle":e.data();c.call(f,h)})}(jQuery),+function(a){"use strict";function b(b){var c=b.attr("data-target");c||(c=b.attr("href"),c=c&&/#[A-Za-z]/.test(c)&&c.replace(/.*(?=#[^\s]*$)/,""));var d=c&&a(c);return d&&d.length?d:b.parent()}function c(c){c&&3===c.which||(a(e).remove(),a(f).each(function(){var d=a(this),e=b(d),f={relatedTarget:this};e.hasClass("open")&&(c&&"click"==c.type&&/input|textarea/i.test(c.target.tagName)&&a.contains(e[0],c.target)||(e.trigger(c=a.Event("hide.bs.dropdown",f)),c.isDefaultPrevented()||(d.attr("aria-expanded","false"),e.removeClass("open").trigger("hidden.bs.dropdown",f))))}))}function d(b){return this.each(function(){var c=a(this),d=c.data("bs.dropdown");d||c.data("bs.dropdown",d=new g(this)),"string"==typeof b&&d[b].call(c)})}var e=".dropdown-backdrop",f='[data-toggle="dropdown"]',g=function(b){a(b).on("click.bs.dropdown",this.toggle)};g.VERSION="3.3.5",g.prototype.toggle=function(d){var e=a(this);if(!e.is(".disabled, :disabled")){var f=b(e),g=f.hasClass("open");if(c(),!g){"ontouchstart"in document.documentElement&&!f.closest(".navbar-nav").length&&a(document.createElement("div")).addClass("dropdown-backdrop").insertAfter(a(this)).on("click",c);var h={relatedTarget:this};if(f.trigger(d=a.Event("show.bs.dropdown",h)),d.isDefaultPrevented())return;e.trigger("focus").attr("aria-expanded","true"),f.toggleClass("open").trigger("shown.bs.dropdown",h)}return!1}},g.prototype.keydown=function(c){if(/(38|40|27|32)/.test(c.which)&&!/input|textarea/i.test(c.target.tagName)){var d=a(this);if(c.preventDefault(),c.stopPropagation(),!d.is(".disabled, :disabled")){var e=b(d),g=e.hasClass("open");if(!g&&27!=c.which||g&&27==c.which)return 27==c.which&&e.find(f).trigger("focus"),d.trigger("click");var h=" li:not(.disabled):visible a",i=e.find(".dropdown-menu"+h);if(i.length){var j=i.index(c.target);38==c.which&&j>0&&j--,40==c.which&&j<i.length-1&&j++,~j||(j=0),i.eq(j).trigger("focus")}}}};var h=a.fn.dropdown;a.fn.dropdown=d,a.fn.dropdown.Constructor=g,a.fn.dropdown.noConflict=function(){return a.fn.dropdown=h,this},a(document).on("click.bs.dropdown.data-api",c).on("click.bs.dropdown.data-api",".dropdown form",function(a){a.stopPropagation()}).on("click.bs.dropdown.data-api",f,g.prototype.toggle).on("keydown.bs.dropdown.data-api",f,g.prototype.keydown).on("keydown.bs.dropdown.data-api",".dropdown-menu",g.prototype.keydown)}(jQuery),+function(a){"use strict";function b(b,d){return this.each(function(){var e=a(this),f=e.data("bs.modal"),g=a.extend({},c.DEFAULTS,e.data(),"object"==typeof b&&b);f||e.data("bs.modal",f=new c(this,g)),"string"==typeof b?f[b](d):g.show&&f.show(d)})}var c=function(b,c){this.options=c,this.$body=a(document.body),this.$element=a(b),this.$dialog=this.$element.find(".modal-dialog"),this.$backdrop=null,this.isShown=null,this.originalBodyPad=null,this.scrollbarWidth=0,this.ignoreBackdropClick=!1,this.options.remote&&this.$element.find(".modal-content").load(this.options.remote,a.proxy(function(){this.$element.trigger("loaded.bs.modal")},this))};c.VERSION="3.3.5",c.TRANSITION_DURATION=300,c.BACKDROP_TRANSITION_DURATION=150,c.DEFAULTS={backdrop:!0,keyboard:!0,show:!0},c.prototype.toggle=function(a){return this.isShown?this.hide():this.show(a)},c.prototype.show=function(b){var d=this,e=a.Event("show.bs.modal",{relatedTarget:b});this.$element.trigger(e),this.isShown||e.isDefaultPrevented()||(this.isShown=!0,this.checkScrollbar(),this.setScrollbar(),this.$body.addClass("modal-open"),this.escape(),this.resize(),this.$element.on("click.dismiss.bs.modal",'[data-dismiss="modal"]',a.proxy(this.hide,this)),this.$dialog.on("mousedown.dismiss.bs.modal",function(){d.$element.one("mouseup.dismiss.bs.modal",function(b){a(b.target).is(d.$element)&&(d.ignoreBackdropClick=!0)})}),this.backdrop(function(){var e=a.support.transition&&d.$element.hasClass("fade");d.$element.parent().length||d.$element.appendTo(d.$body),d.$element.show().scrollTop(0),d.adjustDialog(),e&&d.$element[0].offsetWidth,d.$element.addClass("in"),d.enforceFocus();var f=a.Event("shown.bs.modal",{relatedTarget:b});e?d.$dialog.one("bsTransitionEnd",function(){d.$element.trigger("focus").trigger(f)}).emulateTransitionEnd(c.TRANSITION_DURATION):d.$element.trigger("focus").trigger(f)}))},c.prototype.hide=function(b){b&&b.preventDefault(),b=a.Event("hide.bs.modal"),this.$element.trigger(b),this.isShown&&!b.isDefaultPrevented()&&(this.isShown=!1,this.escape(),this.resize(),a(document).off("focusin.bs.modal"),this.$element.removeClass("in").off("click.dismiss.bs.modal").off("mouseup.dismiss.bs.modal"),this.$dialog.off("mousedown.dismiss.bs.modal"),a.support.transition&&this.$element.hasClass("fade")?this.$element.one("bsTransitionEnd",a.proxy(this.hideModal,this)).emulateTransitionEnd(c.TRANSITION_DURATION):this.hideModal())},c.prototype.enforceFocus=function(){a(document).off("focusin.bs.modal").on("focusin.bs.modal",a.proxy(function(a){this.$element[0]===a.target||this.$element.has(a.target).length||this.$element.trigger("focus")},this))},c.prototype.escape=function(){this.isShown&&this.options.keyboard?this.$element.on("keydown.dismiss.bs.modal",a.proxy(function(a){27==a.which&&this.hide()},this)):this.isShown||this.$element.off("keydown.dismiss.bs.modal")},c.prototype.resize=function(){this.isShown?a(window).on("resize.bs.modal",a.proxy(this.handleUpdate,this)):a(window).off("resize.bs.modal")},c.prototype.hideModal=function(){var a=this;this.$element.hide(),this.backdrop(function(){a.$body.removeClass("modal-open"),a.resetAdjustments(),a.resetScrollbar(),a.$element.trigger("hidden.bs.modal")})},c.prototype.removeBackdrop=function(){this.$backdrop&&this.$backdrop.remove(),this.$backdrop=null},c.prototype.backdrop=function(b){var d=this,e=this.$element.hasClass("fade")?"fade":"";if(this.isShown&&this.options.backdrop){var f=a.support.transition&&e;if(this.$backdrop=a(document.createElement("div")).addClass("modal-backdrop "+e).appendTo(this.$body),this.$element.on("click.dismiss.bs.modal",a.proxy(function(a){return this.ignoreBackdropClick?void(this.ignoreBackdropClick=!1):void(a.target===a.currentTarget&&("static"==this.options.backdrop?this.$element[0].focus():this.hide()))},this)),f&&this.$backdrop[0].offsetWidth,this.$backdrop.addClass("in"),!b)return;f?this.$backdrop.one("bsTransitionEnd",b).emulateTransitionEnd(c.BACKDROP_TRANSITION_DURATION):b()}else if(!this.isShown&&this.$backdrop){this.$backdrop.removeClass("in");var g=function(){d.removeBackdrop(),b&&b()};a.support.transition&&this.$element.hasClass("fade")?this.$backdrop.one("bsTransitionEnd",g).emulateTransitionEnd(c.BACKDROP_TRANSITION_DURATION):g()}else b&&b()},c.prototype.handleUpdate=function(){this.adjustDialog()},c.prototype.adjustDialog=function(){var a=this.$element[0].scrollHeight>document.documentElement.clientHeight;this.$element.css({paddingLeft:!this.bodyIsOverflowing&&a?this.scrollbarWidth:"",paddingRight:this.bodyIsOverflowing&&!a?this.scrollbarWidth:""})},c.prototype.resetAdjustments=function(){this.$element.css({paddingLeft:"",paddingRight:""})},c.prototype.checkScrollbar=function(){var a=window.innerWidth;if(!a){var b=document.documentElement.getBoundingClientRect();a=b.right-Math.abs(b.left)}this.bodyIsOverflowing=document.body.clientWidth<a,this.scrollbarWidth=this.measureScrollbar()},c.prototype.setScrollbar=function(){var a=parseInt(this.$body.css("padding-right")||0,10);this.originalBodyPad=document.body.style.paddingRight||"",this.bodyIsOverflowing&&this.$body.css("padding-right",a+this.scrollbarWidth)},c.prototype.resetScrollbar=function(){this.$body.css("padding-right",this.originalBodyPad)},c.prototype.measureScrollbar=function(){var a=document.createElement("div");a.className="modal-scrollbar-measure",this.$body.append(a);var b=a.offsetWidth-a.clientWidth;return this.$body[0].removeChild(a),b};var d=a.fn.modal;a.fn.modal=b,a.fn.modal.Constructor=c,a.fn.modal.noConflict=function(){return a.fn.modal=d,this},a(document).on("click.bs.modal.data-api",'[data-toggle="modal"]',function(c){var d=a(this),e=d.attr("href"),f=a(d.attr("data-target")||e&&e.replace(/.*(?=#[^\s]+$)/,"")),g=f.data("bs.modal")?"toggle":a.extend({remote:!/#/.test(e)&&e},f.data(),d.data());d.is("a")&&c.preventDefault(),f.one("show.bs.modal",function(a){a.isDefaultPrevented()||f.one("hidden.bs.modal",function(){d.is(":visible")&&d.trigger("focus")})}),b.call(f,g,this)})}(jQuery),+function(a){"use strict";function b(b){return this.each(function(){var d=a(this),e=d.data("bs.tooltip"),f="object"==typeof b&&b;(e||!/destroy|hide/.test(b))&&(e||d.data("bs.tooltip",e=new c(this,f)),"string"==typeof b&&e[b]())})}var c=function(a,b){this.type=null,this.options=null,this.enabled=null,this.timeout=null,this.hoverState=null,this.$element=null,this.inState=null,this.init("tooltip",a,b)};c.VERSION="3.3.5",c.TRANSITION_DURATION=150,c.DEFAULTS={animation:!0,placement:"top",selector:!1,template:'<div class="tooltip" role="tooltip"><div class="tooltip-arrow"></div><div class="tooltip-inner"></div></div>',trigger:"hover focus",title:"",delay:0,html:!1,container:!1,viewport:{selector:"body",padding:0}},c.prototype.init=function(b,c,d){if(this.enabled=!0,this.type=b,this.$element=a(c),this.options=this.getOptions(d),this.$viewport=this.options.viewport&&a(a.isFunction(this.options.viewport)?this.options.viewport.call(this,this.$element):this.options.viewport.selector||this.options.viewport),this.inState={click:!1,hover:!1,focus:!1},this.$element[0]instanceof document.constructor&&!this.options.selector)throw new Error("`selector` option must be specified when initializing "+this.type+" on the window.document object!");for(var e=this.options.trigger.split(" "),f=e.length;f--;){var g=e[f];if("click"==g)this.$element.on("click."+this.type,this.options.selector,a.proxy(this.toggle,this));else if("manual"!=g){var h="hover"==g?"mouseenter":"focusin",i="hover"==g?"mouseleave":"focusout";this.$element.on(h+"."+this.type,this.options.selector,a.proxy(this.enter,this)),this.$element.on(i+"."+this.type,this.options.selector,a.proxy(this.leave,this))}}this.options.selector?this._options=a.extend({},this.options,{trigger:"manual",selector:""}):this.fixTitle()},c.prototype.getDefaults=function(){return c.DEFAULTS},c.prototype.getOptions=function(b){return b=a.extend({},this.getDefaults(),this.$element.data(),b),b.delay&&"number"==typeof b.delay&&(b.delay={show:b.delay,hide:b.delay}),b},c.prototype.getDelegateOptions=function(){var b={},c=this.getDefaults();return this._options&&a.each(this._options,function(a,d){c[a]!=d&&(b[a]=d)}),b},c.prototype.enter=function(b){var c=b instanceof this.constructor?b:a(b.currentTarget).data("bs."+this.type);return c||(c=new this.constructor(b.currentTarget,this.getDelegateOptions()),a(b.currentTarget).data("bs."+this.type,c)),b instanceof a.Event&&(c.inState["focusin"==b.type?"focus":"hover"]=!0),c.tip().hasClass("in")||"in"==c.hoverState?void(c.hoverState="in"):(clearTimeout(c.timeout),c.hoverState="in",c.options.delay&&c.options.delay.show?void(c.timeout=setTimeout(function(){"in"==c.hoverState&&c.show()},c.options.delay.show)):c.show())},c.prototype.isInStateTrue=function(){for(var a in this.inState)if(this.inState[a])return!0;return!1},c.prototype.leave=function(b){var c=b instanceof this.constructor?b:a(b.currentTarget).data("bs."+this.type);return c||(c=new this.constructor(b.currentTarget,this.getDelegateOptions()),a(b.currentTarget).data("bs."+this.type,c)),b instanceof a.Event&&(c.inState["focusout"==b.type?"focus":"hover"]=!1),c.isInStateTrue()?void 0:(clearTimeout(c.timeout),c.hoverState="out",c.options.delay&&c.options.delay.hide?void(c.timeout=setTimeout(function(){"out"==c.hoverState&&c.hide()},c.options.delay.hide)):c.hide())},c.prototype.show=function(){var b=a.Event("show.bs."+this.type);if(this.hasContent()&&this.enabled){this.$element.trigger(b);var d=a.contains(this.$element[0].ownerDocument.documentElement,this.$element[0]);if(b.isDefaultPrevented()||!d)return;var e=this,f=this.tip(),g=this.getUID(this.type);this.setContent(),f.attr("id",g),this.$element.attr("aria-describedby",g),this.options.animation&&f.addClass("fade");var h="function"==typeof this.options.placement?this.options.placement.call(this,f[0],this.$element[0]):this.options.placement,i=/\s?auto?\s?/i,j=i.test(h);j&&(h=h.replace(i,"")||"top"),f.detach().css({top:0,left:0,display:"block"}).addClass(h).data("bs."+this.type,this),this.options.container?f.appendTo(this.options.container):f.insertAfter(this.$element),this.$element.trigger("inserted.bs."+this.type);var k=this.getPosition(),l=f[0].offsetWidth,m=f[0].offsetHeight;if(j){var n=h,o=this.getPosition(this.$viewport);h="bottom"==h&&k.bottom+m>o.bottom?"top":"top"==h&&k.top-m<o.top?"bottom":"right"==h&&k.right+l>o.width?"left":"left"==h&&k.left-l<o.left?"right":h,f.removeClass(n).addClass(h)}var p=this.getCalculatedOffset(h,k,l,m);this.applyPlacement(p,h);var q=function(){var a=e.hoverState;e.$element.trigger("shown.bs."+e.type),e.hoverState=null,"out"==a&&e.leave(e)};a.support.transition&&this.$tip.hasClass("fade")?f.one("bsTransitionEnd",q).emulateTransitionEnd(c.TRANSITION_DURATION):q()}},c.prototype.applyPlacement=function(b,c){var d=this.tip(),e=d[0].offsetWidth,f=d[0].offsetHeight,g=parseInt(d.css("margin-top"),10),h=parseInt(d.css("margin-left"),10);isNaN(g)&&(g=0),isNaN(h)&&(h=0),b.top+=g,b.left+=h,a.offset.setOffset(d[0],a.extend({using:function(a){d.css({top:Math.round(a.top),left:Math.round(a.left)})}},b),0),d.addClass("in");var i=d[0].offsetWidth,j=d[0].offsetHeight;"top"==c&&j!=f&&(b.top=b.top+f-j);var k=this.getViewportAdjustedDelta(c,b,i,j);k.left?b.left+=k.left:b.top+=k.top;var l=/top|bottom/.test(c),m=l?2*k.left-e+i:2*k.top-f+j,n=l?"offsetWidth":"offsetHeight";d.offset(b),this.replaceArrow(m,d[0][n],l)},c.prototype.replaceArrow=function(a,b,c){this.arrow().css(c?"left":"top",50*(1-a/b)+"%").css(c?"top":"left","")},c.prototype.setContent=function(){var a=this.tip(),b=this.getTitle();a.find(".tooltip-inner")[this.options.html?"html":"text"](b),a.removeClass("fade in top bottom left right")},c.prototype.hide=function(b){function d(){"in"!=e.hoverState&&f.detach(),e.$element.removeAttr("aria-describedby").trigger("hidden.bs."+e.type),b&&b()}var e=this,f=a(this.$tip),g=a.Event("hide.bs."+this.type);return this.$element.trigger(g),g.isDefaultPrevented()?void 0:(f.removeClass("in"),a.support.transition&&f.hasClass("fade")?f.one("bsTransitionEnd",d).emulateTransitionEnd(c.TRANSITION_DURATION):d(),this.hoverState=null,this)},c.prototype.fixTitle=function(){var a=this.$element;(a.attr("title")||"string"!=typeof a.attr("data-original-title"))&&a.attr("data-original-title",a.attr("title")||"").attr("title","")},c.prototype.hasContent=function(){return this.getTitle()},c.prototype.getPosition=function(b){b=b||this.$element;var c=b[0],d="BODY"==c.tagName,e=c.getBoundingClientRect();null==e.width&&(e=a.extend({},e,{width:e.right-e.left,height:e.bottom-e.top}));var f=d?{top:0,left:0}:b.offset(),g={scroll:d?document.documentElement.scrollTop||document.body.scrollTop:b.scrollTop()},h=d?{width:a(window).width(),height:a(window).height()}:null;return a.extend({},e,g,h,f)},c.prototype.getCalculatedOffset=function(a,b,c,d){return"bottom"==a?{top:b.top+b.height,left:b.left+b.width/2-c/2}:"top"==a?{top:b.top-d,left:b.left+b.width/2-c/2}:"left"==a?{top:b.top+b.height/2-d/2,left:b.left-c}:{top:b.top+b.height/2-d/2,left:b.left+b.width}},c.prototype.getViewportAdjustedDelta=function(a,b,c,d){var e={top:0,left:0};if(!this.$viewport)return e;var f=this.options.viewport&&this.options.viewport.padding||0,g=this.getPosition(this.$viewport);if(/right|left/.test(a)){var h=b.top-f-g.scroll,i=b.top+f-g.scroll+d;h<g.top?e.top=g.top-h:i>g.top+g.height&&(e.top=g.top+g.height-i)}else{var j=b.left-f,k=b.left+f+c;j<g.left?e.left=g.left-j:k>g.right&&(e.left=g.left+g.width-k)}return e},c.prototype.getTitle=function(){var a,b=this.$element,c=this.options;return a=b.attr("data-original-title")||("function"==typeof c.title?c.title.call(b[0]):c.title)},c.prototype.getUID=function(a){do a+=~~(1e6*Math.random());while(document.getElementById(a));return a},c.prototype.tip=function(){if(!this.$tip&&(this.$tip=a(this.options.template),1!=this.$tip.length))throw new Error(this.type+" `template` option must consist of exactly 1 top-level element!");return this.$tip},c.prototype.arrow=function(){return this.$arrow=this.$arrow||this.tip().find(".tooltip-arrow")},c.prototype.enable=function(){this.enabled=!0},c.prototype.disable=function(){this.enabled=!1},c.prototype.toggleEnabled=function(){this.enabled=!this.enabled},c.prototype.toggle=function(b){var c=this;b&&(c=a(b.currentTarget).data("bs."+this.type),c||(c=new this.constructor(b.currentTarget,this.getDelegateOptions()),a(b.currentTarget).data("bs."+this.type,c))),b?(c.inState.click=!c.inState.click,c.isInStateTrue()?c.enter(c):c.leave(c)):c.tip().hasClass("in")?c.leave(c):c.enter(c)},c.prototype.destroy=function(){var a=this;clearTimeout(this.timeout),this.hide(function(){a.$element.off("."+a.type).removeData("bs."+a.type),a.$tip&&a.$tip.detach(),a.$tip=null,a.$arrow=null,a.$viewport=null})};var d=a.fn.tooltip;a.fn.tooltip=b,a.fn.tooltip.Constructor=c,a.fn.tooltip.noConflict=function(){return a.fn.tooltip=d,this}}(jQuery),+function(a){"use strict";function b(b){return this.each(function(){var d=a(this),e=d.data("bs.popover"),f="object"==typeof b&&b;(e||!/destroy|hide/.test(b))&&(e||d.data("bs.popover",e=new c(this,f)),"string"==typeof b&&e[b]())})}var c=function(a,b){this.init("popover",a,b)};if(!a.fn.tooltip)throw new Error("Popover requires tooltip.js");c.VERSION="3.3.5",c.DEFAULTS=a.extend({},a.fn.tooltip.Constructor.DEFAULTS,{placement:"right",trigger:"click",content:"",template:'<div class="popover" role="tooltip"><div class="arrow"></div><h3 class="popover-title"></h3><div class="popover-content"></div></div>'}),c.prototype=a.extend({},a.fn.tooltip.Constructor.prototype),c.prototype.constructor=c,c.prototype.getDefaults=function(){return c.DEFAULTS},c.prototype.setContent=function(){var a=this.tip(),b=this.getTitle(),c=this.getContent();a.find(".popover-title")[this.options.html?"html":"text"](b),a.find(".popover-content").children().detach().end()[this.options.html?"string"==typeof c?"html":"append":"text"](c),a.removeClass("fade top bottom left right in"),a.find(".popover-title").html()||a.find(".popover-title").hide()},c.prototype.hasContent=function(){return this.getTitle()||this.getContent()},c.prototype.getContent=function(){var a=this.$element,b=this.options;return a.attr("data-content")||("function"==typeof b.content?b.content.call(a[0]):b.content)},c.prototype.arrow=function(){return this.$arrow=this.$arrow||this.tip().find(".arrow")};var d=a.fn.popover;a.fn.popover=b,a.fn.popover.Constructor=c,a.fn.popover.noConflict=function(){return a.fn.popover=d,this}}(jQuery),+function(a){"use strict";function b(c,d){this.$body=a(document.body),this.$scrollElement=a(a(c).is(document.body)?window:c),this.options=a.extend({},b.DEFAULTS,d),this.selector=(this.options.target||"")+" .nav li > a",this.offsets=[],this.targets=[],this.activeTarget=null,this.scrollHeight=0,this.$scrollElement.on("scroll.bs.scrollspy",a.proxy(this.process,this)),this.refresh(),this.process()}function c(c){return this.each(function(){var d=a(this),e=d.data("bs.scrollspy"),f="object"==typeof c&&c;e||d.data("bs.scrollspy",e=new b(this,f)),"string"==typeof c&&e[c]()})}b.VERSION="3.3.5",b.DEFAULTS={offset:10},b.prototype.getScrollHeight=function(){return this.$scrollElement[0].scrollHeight||Math.max(this.$body[0].scrollHeight,document.documentElement.scrollHeight)},b.prototype.refresh=function(){var b=this,c="offset",d=0;this.offsets=[],this.targets=[],this.scrollHeight=this.getScrollHeight(),a.isWindow(this.$scrollElement[0])||(c="position",d=this.$scrollElement.scrollTop()),this.$body.find(this.selector).map(function(){var b=a(this),e=b.data("target")||b.attr("href"),f=/^#./.test(e)&&a(e);return f&&f.length&&f.is(":visible")&&[[f[c]().top+d,e]]||null}).sort(function(a,b){return a[0]-b[0]}).each(function(){b.offsets.push(this[0]),b.targets.push(this[1])})},b.prototype.process=function(){var a,b=this.$scrollElement.scrollTop()+this.options.offset,c=this.getScrollHeight(),d=this.options.offset+c-this.$scrollElement.height(),e=this.offsets,f=this.targets,g=this.activeTarget;if(this.scrollHeight!=c&&this.refresh(),b>=d)return g!=(a=f[f.length-1])&&this.activate(a);if(g&&b<e[0])return this.activeTarget=null,this.clear();for(a=e.length;a--;)g!=f[a]&&b>=e[a]&&(void 0===e[a+1]||b<e[a+1])&&this.activate(f[a])},b.prototype.activate=function(b){this.activeTarget=b,this.clear();var c=this.selector+'[data-target="'+b+'"],'+this.selector+'[href="'+b+'"]',d=a(c).parents("li").addClass("active");d.parent(".dropdown-menu").length&&(d=d.closest("li.dropdown").addClass("active")),
+d.trigger("activate.bs.scrollspy")},b.prototype.clear=function(){a(this.selector).parentsUntil(this.options.target,".active").removeClass("active")};var d=a.fn.scrollspy;a.fn.scrollspy=c,a.fn.scrollspy.Constructor=b,a.fn.scrollspy.noConflict=function(){return a.fn.scrollspy=d,this},a(window).on("load.bs.scrollspy.data-api",function(){a('[data-spy="scroll"]').each(function(){var b=a(this);c.call(b,b.data())})})}(jQuery),+function(a){"use strict";function b(b){return this.each(function(){var d=a(this),e=d.data("bs.tab");e||d.data("bs.tab",e=new c(this)),"string"==typeof b&&e[b]()})}var c=function(b){this.element=a(b)};c.VERSION="3.3.5",c.TRANSITION_DURATION=150,c.prototype.show=function(){var b=this.element,c=b.closest("ul:not(.dropdown-menu)"),d=b.data("target");if(d||(d=b.attr("href"),d=d&&d.replace(/.*(?=#[^\s]*$)/,"")),!b.parent("li").hasClass("active")){var e=c.find(".active:last a"),f=a.Event("hide.bs.tab",{relatedTarget:b[0]}),g=a.Event("show.bs.tab",{relatedTarget:e[0]});if(e.trigger(f),b.trigger(g),!g.isDefaultPrevented()&&!f.isDefaultPrevented()){var h=a(d);this.activate(b.closest("li"),c),this.activate(h,h.parent(),function(){e.trigger({type:"hidden.bs.tab",relatedTarget:b[0]}),b.trigger({type:"shown.bs.tab",relatedTarget:e[0]})})}}},c.prototype.activate=function(b,d,e){function f(){g.removeClass("active").find("> .dropdown-menu > .active").removeClass("active").end().find('[data-toggle="tab"]').attr("aria-expanded",!1),b.addClass("active").find('[data-toggle="tab"]').attr("aria-expanded",!0),h?(b[0].offsetWidth,b.addClass("in")):b.removeClass("fade"),b.parent(".dropdown-menu").length&&b.closest("li.dropdown").addClass("active").end().find('[data-toggle="tab"]').attr("aria-expanded",!0),e&&e()}var g=d.find("> .active"),h=e&&a.support.transition&&(g.length&&g.hasClass("fade")||!!d.find("> .fade").length);g.length&&h?g.one("bsTransitionEnd",f).emulateTransitionEnd(c.TRANSITION_DURATION):f(),g.removeClass("in")};var d=a.fn.tab;a.fn.tab=b,a.fn.tab.Constructor=c,a.fn.tab.noConflict=function(){return a.fn.tab=d,this};var e=function(c){c.preventDefault(),b.call(a(this),"show")};a(document).on("click.bs.tab.data-api",'[data-toggle="tab"]',e).on("click.bs.tab.data-api",'[data-toggle="pill"]',e)}(jQuery),+function(a){"use strict";function b(b){return this.each(function(){var d=a(this),e=d.data("bs.affix"),f="object"==typeof b&&b;e||d.data("bs.affix",e=new c(this,f)),"string"==typeof b&&e[b]()})}var c=function(b,d){this.options=a.extend({},c.DEFAULTS,d),this.$target=a(this.options.target).on("scroll.bs.affix.data-api",a.proxy(this.checkPosition,this)).on("click.bs.affix.data-api",a.proxy(this.checkPositionWithEventLoop,this)),this.$element=a(b),this.affixed=null,this.unpin=null,this.pinnedOffset=null,this.checkPosition()};c.VERSION="3.3.5",c.RESET="affix affix-top affix-bottom",c.DEFAULTS={offset:0,target:window},c.prototype.getState=function(a,b,c,d){var e=this.$target.scrollTop(),f=this.$element.offset(),g=this.$target.height();if(null!=c&&"top"==this.affixed)return c>e?"top":!1;if("bottom"==this.affixed)return null!=c?e+this.unpin<=f.top?!1:"bottom":a-d>=e+g?!1:"bottom";var h=null==this.affixed,i=h?e:f.top,j=h?g:b;return null!=c&&c>=e?"top":null!=d&&i+j>=a-d?"bottom":!1},c.prototype.getPinnedOffset=function(){if(this.pinnedOffset)return this.pinnedOffset;this.$element.removeClass(c.RESET).addClass("affix");var a=this.$target.scrollTop(),b=this.$element.offset();return this.pinnedOffset=b.top-a},c.prototype.checkPositionWithEventLoop=function(){setTimeout(a.proxy(this.checkPosition,this),1)},c.prototype.checkPosition=function(){if(this.$element.is(":visible")){var b=this.$element.height(),d=this.options.offset,e=d.top,f=d.bottom,g=Math.max(a(document).height(),a(document.body).height());"object"!=typeof d&&(f=e=d),"function"==typeof e&&(e=d.top(this.$element)),"function"==typeof f&&(f=d.bottom(this.$element));var h=this.getState(g,b,e,f);if(this.affixed!=h){null!=this.unpin&&this.$element.css("top","");var i="affix"+(h?"-"+h:""),j=a.Event(i+".bs.affix");if(this.$element.trigger(j),j.isDefaultPrevented())return;this.affixed=h,this.unpin="bottom"==h?this.getPinnedOffset():null,this.$element.removeClass(c.RESET).addClass(i).trigger(i.replace("affix","affixed")+".bs.affix")}"bottom"==h&&this.$element.offset({top:g-b-f})}};var d=a.fn.affix;a.fn.affix=b,a.fn.affix.Constructor=c,a.fn.affix.noConflict=function(){return a.fn.affix=d,this},a(window).on("load",function(){a('[data-spy="affix"]').each(function(){var c=a(this),d=c.data();d.offset=d.offset||{},null!=d.offsetBottom&&(d.offset.bottom=d.offsetBottom),null!=d.offsetTop&&(d.offset.top=d.offsetTop),b.call(c,d)})})}(jQuery);</script>
+<script>/**
+* @preserve HTML5 Shiv 3.7.2 | @afarkas @jdalton @jon_neal @rem | MIT/GPL2 Licensed
+*/
+// Only run this code in IE 8
+if (!!window.navigator.userAgent.match("MSIE 8")) {
+!function(a,b){function c(a,b){var c=a.createElement("p"),d=a.getElementsByTagName("head")[0]||a.documentElement;return c.innerHTML="x<style>"+b+"</style>",d.insertBefore(c.lastChild,d.firstChild)}function d(){var a=t.elements;return"string"==typeof a?a.split(" "):a}function e(a,b){var c=t.elements;"string"!=typeof c&&(c=c.join(" ")),"string"!=typeof a&&(a=a.join(" ")),t.elements=c+" "+a,j(b)}function f(a){var b=s[a[q]];return b||(b={},r++,a[q]=r,s[r]=b),b}function g(a,c,d){if(c||(c=b),l)return c.createElement(a);d||(d=f(c));var e;return e=d.cache[a]?d.cache[a].cloneNode():p.test(a)?(d.cache[a]=d.createElem(a)).cloneNode():d.createElem(a),!e.canHaveChildren||o.test(a)||e.tagUrn?e:d.frag.appendChild(e)}function h(a,c){if(a||(a=b),l)return a.createDocumentFragment();c=c||f(a);for(var e=c.frag.cloneNode(),g=0,h=d(),i=h.length;i>g;g++)e.createElement(h[g]);return e}function i(a,b){b.cache||(b.cache={},b.createElem=a.createElement,b.createFrag=a.createDocumentFragment,b.frag=b.createFrag()),a.createElement=function(c){return t.shivMethods?g(c,a,b):b.createElem(c)},a.createDocumentFragment=Function("h,f","return function(){var n=f.cloneNode(),c=n.createElement;h.shivMethods&&("+d().join().replace(/[\w\-:]+/g,function(a){return b.createElem(a),b.frag.createElement(a),'c("'+a+'")'})+");return n}")(t,b.frag)}function j(a){a||(a=b);var d=f(a);return!t.shivCSS||k||d.hasCSS||(d.hasCSS=!!c(a,"article,aside,dialog,figcaption,figure,footer,header,hgroup,main,nav,section{display:block}mark{background:#FF0;color:#000}template{display:none}")),l||i(a,d),a}var k,l,m="3.7.2",n=a.html5||{},o=/^<|^(?:button|map|select|textarea|object|iframe|option|optgroup)$/i,p=/^(?:a|b|code|div|fieldset|h1|h2|h3|h4|h5|h6|i|label|li|ol|p|q|span|strong|style|table|tbody|td|th|tr|ul)$/i,q="_html5shiv",r=0,s={};!function(){try{var a=b.createElement("a");a.innerHTML="<xyz></xyz>",k="hidden"in a,l=1==a.childNodes.length||function(){b.createElement("a");var a=b.createDocumentFragment();return"undefined"==typeof a.cloneNode||"undefined"==typeof a.createDocumentFragment||"undefined"==typeof a.createElement}()}catch(c){k=!0,l=!0}}();var t={elements:n.elements||"abbr article aside audio bdi canvas data datalist details dialog figcaption figure footer header hgroup main mark meter nav output picture progress section summary template time video",version:m,shivCSS:n.shivCSS!==!1,supportsUnknownElements:l,shivMethods:n.shivMethods!==!1,type:"default",shivDocument:j,createElement:g,createDocumentFragment:h,addElements:e};a.html5=t,j(b)}(this,document);
+};
+</script>
+<script>/*! Respond.js v1.4.2: min/max-width media query polyfill * Copyright 2013 Scott Jehl
+ * Licensed under https://github.com/scottjehl/Respond/blob/master/LICENSE-MIT
+ *  */
+
+// Only run this code in IE 8
+if (!!window.navigator.userAgent.match("MSIE 8")) {
+!function(a){"use strict";a.matchMedia=a.matchMedia||function(a){var b,c=a.documentElement,d=c.firstElementChild||c.firstChild,e=a.createElement("body"),f=a.createElement("div");return f.id="mq-test-1",f.style.cssText="position:absolute;top:-100em",e.style.background="none",e.appendChild(f),function(a){return f.innerHTML='&shy;<style media="'+a+'"> #mq-test-1 { width: 42px; }</style>',c.insertBefore(e,d),b=42===f.offsetWidth,c.removeChild(e),{matches:b,media:a}}}(a.document)}(this),function(a){"use strict";function b(){u(!0)}var c={};a.respond=c,c.update=function(){};var d=[],e=function(){var b=!1;try{b=new a.XMLHttpRequest}catch(c){b=new a.ActiveXObject("Microsoft.XMLHTTP")}return function(){return b}}(),f=function(a,b){var c=e();c&&(c.open("GET",a,!0),c.onreadystatechange=function(){4!==c.readyState||200!==c.status&&304!==c.status||b(c.responseText)},4!==c.readyState&&c.send(null))};if(c.ajax=f,c.queue=d,c.regex={media:/@media[^\{]+\{([^\{\}]*\{[^\}\{]*\})+/gi,keyframes:/@(?:\-(?:o|moz|webkit)\-)?keyframes[^\{]+\{(?:[^\{\}]*\{[^\}\{]*\})+[^\}]*\}/gi,urls:/(url\()['"]?([^\/\)'"][^:\)'"]+)['"]?(\))/g,findStyles:/@media *([^\{]+)\{([\S\s]+?)$/,only:/(only\s+)?([a-zA-Z]+)\s?/,minw:/\([\s]*min\-width\s*:[\s]*([\s]*[0-9\.]+)(px|em)[\s]*\)/,maxw:/\([\s]*max\-width\s*:[\s]*([\s]*[0-9\.]+)(px|em)[\s]*\)/},c.mediaQueriesSupported=a.matchMedia&&null!==a.matchMedia("only all")&&a.matchMedia("only all").matches,!c.mediaQueriesSupported){var g,h,i,j=a.document,k=j.documentElement,l=[],m=[],n=[],o={},p=30,q=j.getElementsByTagName("head")[0]||k,r=j.getElementsByTagName("base")[0],s=q.getElementsByTagName("link"),t=function(){var a,b=j.createElement("div"),c=j.body,d=k.style.fontSize,e=c&&c.style.fontSize,f=!1;return b.style.cssText="position:absolute;font-size:1em;width:1em",c||(c=f=j.createElement("body"),c.style.background="none"),k.style.fontSize="100%",c.style.fontSize="100%",c.appendChild(b),f&&k.insertBefore(c,k.firstChild),a=b.offsetWidth,f?k.removeChild(c):c.removeChild(b),k.style.fontSize=d,e&&(c.style.fontSize=e),a=i=parseFloat(a)},u=function(b){var c="clientWidth",d=k[c],e="CSS1Compat"===j.compatMode&&d||j.body[c]||d,f={},o=s[s.length-1],r=(new Date).getTime();if(b&&g&&p>r-g)return a.clearTimeout(h),h=a.setTimeout(u,p),void 0;g=r;for(var v in l)if(l.hasOwnProperty(v)){var w=l[v],x=w.minw,y=w.maxw,z=null===x,A=null===y,B="em";x&&(x=parseFloat(x)*(x.indexOf(B)>-1?i||t():1)),y&&(y=parseFloat(y)*(y.indexOf(B)>-1?i||t():1)),w.hasquery&&(z&&A||!(z||e>=x)||!(A||y>=e))||(f[w.media]||(f[w.media]=[]),f[w.media].push(m[w.rules]))}for(var C in n)n.hasOwnProperty(C)&&n[C]&&n[C].parentNode===q&&q.removeChild(n[C]);n.length=0;for(var D in f)if(f.hasOwnProperty(D)){var E=j.createElement("style"),F=f[D].join("\n");E.type="text/css",E.media=D,q.insertBefore(E,o.nextSibling),E.styleSheet?E.styleSheet.cssText=F:E.appendChild(j.createTextNode(F)),n.push(E)}},v=function(a,b,d){var e=a.replace(c.regex.keyframes,"").match(c.regex.media),f=e&&e.length||0;b=b.substring(0,b.lastIndexOf("/"));var g=function(a){return a.replace(c.regex.urls,"$1"+b+"$2$3")},h=!f&&d;b.length&&(b+="/"),h&&(f=1);for(var i=0;f>i;i++){var j,k,n,o;h?(j=d,m.push(g(a))):(j=e[i].match(c.regex.findStyles)&&RegExp.$1,m.push(RegExp.$2&&g(RegExp.$2))),n=j.split(","),o=n.length;for(var p=0;o>p;p++)k=n[p],l.push({media:k.split("(")[0].match(c.regex.only)&&RegExp.$2||"all",rules:m.length-1,hasquery:k.indexOf("(")>-1,minw:k.match(c.regex.minw)&&parseFloat(RegExp.$1)+(RegExp.$2||""),maxw:k.match(c.regex.maxw)&&parseFloat(RegExp.$1)+(RegExp.$2||"")})}u()},w=function(){if(d.length){var b=d.shift();f(b.href,function(c){v(c,b.href,b.media),o[b.href]=!0,a.setTimeout(function(){w()},0)})}},x=function(){for(var b=0;b<s.length;b++){var c=s[b],e=c.href,f=c.media,g=c.rel&&"stylesheet"===c.rel.toLowerCase();e&&g&&!o[e]&&(c.styleSheet&&c.styleSheet.rawCssText?(v(c.styleSheet.rawCssText,e,f),o[e]=!0):(!/^([a-zA-Z:]*\/\/)/.test(e)&&!r||e.replace(RegExp.$1,"").split("/")[0]===a.location.host)&&("//"===e.substring(0,2)&&(e=a.location.protocol+e),d.push({href:e,media:f})))}w()};x(),c.update=x,c.getEmValue=t,a.addEventListener?a.addEventListener("resize",b,!1):a.attachEvent&&a.attachEvent("onresize",b)}}(this);
+};
+</script>
+<script>/*! jQuery UI - v1.11.4 - 2016-01-05
+* http://jqueryui.com
+* Includes: core.js, widget.js, mouse.js, position.js, draggable.js, droppable.js, resizable.js, selectable.js, sortable.js, accordion.js, autocomplete.js, button.js, dialog.js, menu.js, progressbar.js, selectmenu.js, slider.js, spinner.js, tabs.js, tooltip.js, effect.js, effect-blind.js, effect-bounce.js, effect-clip.js, effect-drop.js, effect-explode.js, effect-fade.js, effect-fold.js, effect-highlight.js, effect-puff.js, effect-pulsate.js, effect-scale.js, effect-shake.js, effect-size.js, effect-slide.js, effect-transfer.js
+* Copyright jQuery Foundation and other contributors; Licensed MIT */
+
+(function(e){"function"==typeof define&&define.amd?define(["jquery"],e):e(jQuery)})(function(e){function t(t,s){var n,a,o,r=t.nodeName.toLowerCase();return"area"===r?(n=t.parentNode,a=n.name,t.href&&a&&"map"===n.nodeName.toLowerCase()?(o=e("img[usemap='#"+a+"']")[0],!!o&&i(o)):!1):(/^(input|select|textarea|button|object)$/.test(r)?!t.disabled:"a"===r?t.href||s:s)&&i(t)}function i(t){return e.expr.filters.visible(t)&&!e(t).parents().addBack().filter(function(){return"hidden"===e.css(this,"visibility")}).length}function s(e){return function(){var t=this.element.val();e.apply(this,arguments),this._refresh(),t!==this.element.val()&&this._trigger("change")}}e.ui=e.ui||{},e.extend(e.ui,{version:"1.11.4",keyCode:{BACKSPACE:8,COMMA:188,DELETE:46,DOWN:40,END:35,ENTER:13,ESCAPE:27,HOME:36,LEFT:37,PAGE_DOWN:34,PAGE_UP:33,PERIOD:190,RIGHT:39,SPACE:32,TAB:9,UP:38}}),e.fn.extend({scrollParent:function(t){var i=this.css("position"),s="absolute"===i,n=t?/(auto|scroll|hidden)/:/(auto|scroll)/,a=this.parents().filter(function(){var t=e(this);return s&&"static"===t.css("position")?!1:n.test(t.css("overflow")+t.css("overflow-y")+t.css("overflow-x"))}).eq(0);return"fixed"!==i&&a.length?a:e(this[0].ownerDocument||document)},uniqueId:function(){var e=0;return function(){return this.each(function(){this.id||(this.id="ui-id-"+ ++e)})}}(),removeUniqueId:function(){return this.each(function(){/^ui-id-\d+$/.test(this.id)&&e(this).removeAttr("id")})}}),e.extend(e.expr[":"],{data:e.expr.createPseudo?e.expr.createPseudo(function(t){return function(i){return!!e.data(i,t)}}):function(t,i,s){return!!e.data(t,s[3])},focusable:function(i){return t(i,!isNaN(e.attr(i,"tabindex")))},tabbable:function(i){var s=e.attr(i,"tabindex"),n=isNaN(s);return(n||s>=0)&&t(i,!n)}}),e("<a>").outerWidth(1).jquery||e.each(["Width","Height"],function(t,i){function s(t,i,s,a){return e.each(n,function(){i-=parseFloat(e.css(t,"padding"+this))||0,s&&(i-=parseFloat(e.css(t,"border"+this+"Width"))||0),a&&(i-=parseFloat(e.css(t,"margin"+this))||0)}),i}var n="Width"===i?["Left","Right"]:["Top","Bottom"],a=i.toLowerCase(),o={innerWidth:e.fn.innerWidth,innerHeight:e.fn.innerHeight,outerWidth:e.fn.outerWidth,outerHeight:e.fn.outerHeight};e.fn["inner"+i]=function(t){return void 0===t?o["inner"+i].call(this):this.each(function(){e(this).css(a,s(this,t)+"px")})},e.fn["outer"+i]=function(t,n){return"number"!=typeof t?o["outer"+i].call(this,t):this.each(function(){e(this).css(a,s(this,t,!0,n)+"px")})}}),e.fn.addBack||(e.fn.addBack=function(e){return this.add(null==e?this.prevObject:this.prevObject.filter(e))}),e("<a>").data("a-b","a").removeData("a-b").data("a-b")&&(e.fn.removeData=function(t){return function(i){return arguments.length?t.call(this,e.camelCase(i)):t.call(this)}}(e.fn.removeData)),e.ui.ie=!!/msie [\w.]+/.exec(navigator.userAgent.toLowerCase()),e.fn.extend({focus:function(t){return function(i,s){return"number"==typeof i?this.each(function(){var t=this;setTimeout(function(){e(t).focus(),s&&s.call(t)},i)}):t.apply(this,arguments)}}(e.fn.focus),disableSelection:function(){var e="onselectstart"in document.createElement("div")?"selectstart":"mousedown";return function(){return this.bind(e+".ui-disableSelection",function(e){e.preventDefault()})}}(),enableSelection:function(){return this.unbind(".ui-disableSelection")},zIndex:function(t){if(void 0!==t)return this.css("zIndex",t);if(this.length)for(var i,s,n=e(this[0]);n.length&&n[0]!==document;){if(i=n.css("position"),("absolute"===i||"relative"===i||"fixed"===i)&&(s=parseInt(n.css("zIndex"),10),!isNaN(s)&&0!==s))return s;n=n.parent()}return 0}}),e.ui.plugin={add:function(t,i,s){var n,a=e.ui[t].prototype;for(n in s)a.plugins[n]=a.plugins[n]||[],a.plugins[n].push([i,s[n]])},call:function(e,t,i,s){var n,a=e.plugins[t];if(a&&(s||e.element[0].parentNode&&11!==e.element[0].parentNode.nodeType))for(n=0;a.length>n;n++)e.options[a[n][0]]&&a[n][1].apply(e.element,i)}};var n=0,a=Array.prototype.slice;e.cleanData=function(t){return function(i){var s,n,a;for(a=0;null!=(n=i[a]);a++)try{s=e._data(n,"events"),s&&s.remove&&e(n).triggerHandler("remove")}catch(o){}t(i)}}(e.cleanData),e.widget=function(t,i,s){var n,a,o,r,h={},l=t.split(".")[0];return t=t.split(".")[1],n=l+"-"+t,s||(s=i,i=e.Widget),e.expr[":"][n.toLowerCase()]=function(t){return!!e.data(t,n)},e[l]=e[l]||{},a=e[l][t],o=e[l][t]=function(e,t){return this._createWidget?(arguments.length&&this._createWidget(e,t),void 0):new o(e,t)},e.extend(o,a,{version:s.version,_proto:e.extend({},s),_childConstructors:[]}),r=new i,r.options=e.widget.extend({},r.options),e.each(s,function(t,s){return e.isFunction(s)?(h[t]=function(){var e=function(){return i.prototype[t].apply(this,arguments)},n=function(e){return i.prototype[t].apply(this,e)};return function(){var t,i=this._super,a=this._superApply;return this._super=e,this._superApply=n,t=s.apply(this,arguments),this._super=i,this._superApply=a,t}}(),void 0):(h[t]=s,void 0)}),o.prototype=e.widget.extend(r,{widgetEventPrefix:a?r.widgetEventPrefix||t:t},h,{constructor:o,namespace:l,widgetName:t,widgetFullName:n}),a?(e.each(a._childConstructors,function(t,i){var s=i.prototype;e.widget(s.namespace+"."+s.widgetName,o,i._proto)}),delete a._childConstructors):i._childConstructors.push(o),e.widget.bridge(t,o),o},e.widget.extend=function(t){for(var i,s,n=a.call(arguments,1),o=0,r=n.length;r>o;o++)for(i in n[o])s=n[o][i],n[o].hasOwnProperty(i)&&void 0!==s&&(t[i]=e.isPlainObject(s)?e.isPlainObject(t[i])?e.widget.extend({},t[i],s):e.widget.extend({},s):s);return t},e.widget.bridge=function(t,i){var s=i.prototype.widgetFullName||t;e.fn[t]=function(n){var o="string"==typeof n,r=a.call(arguments,1),h=this;return o?this.each(function(){var i,a=e.data(this,s);return"instance"===n?(h=a,!1):a?e.isFunction(a[n])&&"_"!==n.charAt(0)?(i=a[n].apply(a,r),i!==a&&void 0!==i?(h=i&&i.jquery?h.pushStack(i.get()):i,!1):void 0):e.error("no such method '"+n+"' for "+t+" widget instance"):e.error("cannot call methods on "+t+" prior to initialization; "+"attempted to call method '"+n+"'")}):(r.length&&(n=e.widget.extend.apply(null,[n].concat(r))),this.each(function(){var t=e.data(this,s);t?(t.option(n||{}),t._init&&t._init()):e.data(this,s,new i(n,this))})),h}},e.Widget=function(){},e.Widget._childConstructors=[],e.Widget.prototype={widgetName:"widget",widgetEventPrefix:"",defaultElement:"<div>",options:{disabled:!1,create:null},_createWidget:function(t,i){i=e(i||this.defaultElement||this)[0],this.element=e(i),this.uuid=n++,this.eventNamespace="."+this.widgetName+this.uuid,this.bindings=e(),this.hoverable=e(),this.focusable=e(),i!==this&&(e.data(i,this.widgetFullName,this),this._on(!0,this.element,{remove:function(e){e.target===i&&this.destroy()}}),this.document=e(i.style?i.ownerDocument:i.document||i),this.window=e(this.document[0].defaultView||this.document[0].parentWindow)),this.options=e.widget.extend({},this.options,this._getCreateOptions(),t),this._create(),this._trigger("create",null,this._getCreateEventData()),this._init()},_getCreateOptions:e.noop,_getCreateEventData:e.noop,_create:e.noop,_init:e.noop,destroy:function(){this._destroy(),this.element.unbind(this.eventNamespace).removeData(this.widgetFullName).removeData(e.camelCase(this.widgetFullName)),this.widget().unbind(this.eventNamespace).removeAttr("aria-disabled").removeClass(this.widgetFullName+"-disabled "+"ui-state-disabled"),this.bindings.unbind(this.eventNamespace),this.hoverable.removeClass("ui-state-hover"),this.focusable.removeClass("ui-state-focus")},_destroy:e.noop,widget:function(){return this.element},option:function(t,i){var s,n,a,o=t;if(0===arguments.length)return e.widget.extend({},this.options);if("string"==typeof t)if(o={},s=t.split("."),t=s.shift(),s.length){for(n=o[t]=e.widget.extend({},this.options[t]),a=0;s.length-1>a;a++)n[s[a]]=n[s[a]]||{},n=n[s[a]];if(t=s.pop(),1===arguments.length)return void 0===n[t]?null:n[t];n[t]=i}else{if(1===arguments.length)return void 0===this.options[t]?null:this.options[t];o[t]=i}return this._setOptions(o),this},_setOptions:function(e){var t;for(t in e)this._setOption(t,e[t]);return this},_setOption:function(e,t){return this.options[e]=t,"disabled"===e&&(this.widget().toggleClass(this.widgetFullName+"-disabled",!!t),t&&(this.hoverable.removeClass("ui-state-hover"),this.focusable.removeClass("ui-state-focus"))),this},enable:function(){return this._setOptions({disabled:!1})},disable:function(){return this._setOptions({disabled:!0})},_on:function(t,i,s){var n,a=this;"boolean"!=typeof t&&(s=i,i=t,t=!1),s?(i=n=e(i),this.bindings=this.bindings.add(i)):(s=i,i=this.element,n=this.widget()),e.each(s,function(s,o){function r(){return t||a.options.disabled!==!0&&!e(this).hasClass("ui-state-disabled")?("string"==typeof o?a[o]:o).apply(a,arguments):void 0}"string"!=typeof o&&(r.guid=o.guid=o.guid||r.guid||e.guid++);var h=s.match(/^([\w:-]*)\s*(.*)$/),l=h[1]+a.eventNamespace,u=h[2];u?n.delegate(u,l,r):i.bind(l,r)})},_off:function(t,i){i=(i||"").split(" ").join(this.eventNamespace+" ")+this.eventNamespace,t.unbind(i).undelegate(i),this.bindings=e(this.bindings.not(t).get()),this.focusable=e(this.focusable.not(t).get()),this.hoverable=e(this.hoverable.not(t).get())},_delay:function(e,t){function i(){return("string"==typeof e?s[e]:e).apply(s,arguments)}var s=this;return setTimeout(i,t||0)},_hoverable:function(t){this.hoverable=this.hoverable.add(t),this._on(t,{mouseenter:function(t){e(t.currentTarget).addClass("ui-state-hover")},mouseleave:function(t){e(t.currentTarget).removeClass("ui-state-hover")}})},_focusable:function(t){this.focusable=this.focusable.add(t),this._on(t,{focusin:function(t){e(t.currentTarget).addClass("ui-state-focus")},focusout:function(t){e(t.currentTarget).removeClass("ui-state-focus")}})},_trigger:function(t,i,s){var n,a,o=this.options[t];if(s=s||{},i=e.Event(i),i.type=(t===this.widgetEventPrefix?t:this.widgetEventPrefix+t).toLowerCase(),i.target=this.element[0],a=i.originalEvent)for(n in a)n in i||(i[n]=a[n]);return this.element.trigger(i,s),!(e.isFunction(o)&&o.apply(this.element[0],[i].concat(s))===!1||i.isDefaultPrevented())}},e.each({show:"fadeIn",hide:"fadeOut"},function(t,i){e.Widget.prototype["_"+t]=function(s,n,a){"string"==typeof n&&(n={effect:n});var o,r=n?n===!0||"number"==typeof n?i:n.effect||i:t;n=n||{},"number"==typeof n&&(n={duration:n}),o=!e.isEmptyObject(n),n.complete=a,n.delay&&s.delay(n.delay),o&&e.effects&&e.effects.effect[r]?s[t](n):r!==t&&s[r]?s[r](n.duration,n.easing,a):s.queue(function(i){e(this)[t](),a&&a.call(s[0]),i()})}}),e.widget;var o=!1;e(document).mouseup(function(){o=!1}),e.widget("ui.mouse",{version:"1.11.4",options:{cancel:"input,textarea,button,select,option",distance:1,delay:0},_mouseInit:function(){var t=this;this.element.bind("mousedown."+this.widgetName,function(e){return t._mouseDown(e)}).bind("click."+this.widgetName,function(i){return!0===e.data(i.target,t.widgetName+".preventClickEvent")?(e.removeData(i.target,t.widgetName+".preventClickEvent"),i.stopImmediatePropagation(),!1):void 0}),this.started=!1},_mouseDestroy:function(){this.element.unbind("."+this.widgetName),this._mouseMoveDelegate&&this.document.unbind("mousemove."+this.widgetName,this._mouseMoveDelegate).unbind("mouseup."+this.widgetName,this._mouseUpDelegate)},_mouseDown:function(t){if(!o){this._mouseMoved=!1,this._mouseStarted&&this._mouseUp(t),this._mouseDownEvent=t;var i=this,s=1===t.which,n="string"==typeof this.options.cancel&&t.target.nodeName?e(t.target).closest(this.options.cancel).length:!1;return s&&!n&&this._mouseCapture(t)?(this.mouseDelayMet=!this.options.delay,this.mouseDelayMet||(this._mouseDelayTimer=setTimeout(function(){i.mouseDelayMet=!0},this.options.delay)),this._mouseDistanceMet(t)&&this._mouseDelayMet(t)&&(this._mouseStarted=this._mouseStart(t)!==!1,!this._mouseStarted)?(t.preventDefault(),!0):(!0===e.data(t.target,this.widgetName+".preventClickEvent")&&e.removeData(t.target,this.widgetName+".preventClickEvent"),this._mouseMoveDelegate=function(e){return i._mouseMove(e)},this._mouseUpDelegate=function(e){return i._mouseUp(e)},this.document.bind("mousemove."+this.widgetName,this._mouseMoveDelegate).bind("mouseup."+this.widgetName,this._mouseUpDelegate),t.preventDefault(),o=!0,!0)):!0}},_mouseMove:function(t){if(this._mouseMoved){if(e.ui.ie&&(!document.documentMode||9>document.documentMode)&&!t.button)return this._mouseUp(t);if(!t.which)return this._mouseUp(t)}return(t.which||t.button)&&(this._mouseMoved=!0),this._mouseStarted?(this._mouseDrag(t),t.preventDefault()):(this._mouseDistanceMet(t)&&this._mouseDelayMet(t)&&(this._mouseStarted=this._mouseStart(this._mouseDownEvent,t)!==!1,this._mouseStarted?this._mouseDrag(t):this._mouseUp(t)),!this._mouseStarted)},_mouseUp:function(t){return this.document.unbind("mousemove."+this.widgetName,this._mouseMoveDelegate).unbind("mouseup."+this.widgetName,this._mouseUpDelegate),this._mouseStarted&&(this._mouseStarted=!1,t.target===this._mouseDownEvent.target&&e.data(t.target,this.widgetName+".preventClickEvent",!0),this._mouseStop(t)),o=!1,!1},_mouseDistanceMet:function(e){return Math.max(Math.abs(this._mouseDownEvent.pageX-e.pageX),Math.abs(this._mouseDownEvent.pageY-e.pageY))>=this.options.distance},_mouseDelayMet:function(){return this.mouseDelayMet},_mouseStart:function(){},_mouseDrag:function(){},_mouseStop:function(){},_mouseCapture:function(){return!0}}),function(){function t(e,t,i){return[parseFloat(e[0])*(p.test(e[0])?t/100:1),parseFloat(e[1])*(p.test(e[1])?i/100:1)]}function i(t,i){return parseInt(e.css(t,i),10)||0}function s(t){var i=t[0];return 9===i.nodeType?{width:t.width(),height:t.height(),offset:{top:0,left:0}}:e.isWindow(i)?{width:t.width(),height:t.height(),offset:{top:t.scrollTop(),left:t.scrollLeft()}}:i.preventDefault?{width:0,height:0,offset:{top:i.pageY,left:i.pageX}}:{width:t.outerWidth(),height:t.outerHeight(),offset:t.offset()}}e.ui=e.ui||{};var n,a,o=Math.max,r=Math.abs,h=Math.round,l=/left|center|right/,u=/top|center|bottom/,d=/[\+\-]\d+(\.[\d]+)?%?/,c=/^\w+/,p=/%$/,f=e.fn.position;e.position={scrollbarWidth:function(){if(void 0!==n)return n;var t,i,s=e("<div style='display:block;position:absolute;width:50px;height:50px;overflow:hidden;'><div style='height:100px;width:auto;'></div></div>"),a=s.children()[0];return e("body").append(s),t=a.offsetWidth,s.css("overflow","scroll"),i=a.offsetWidth,t===i&&(i=s[0].clientWidth),s.remove(),n=t-i},getScrollInfo:function(t){var i=t.isWindow||t.isDocument?"":t.element.css("overflow-x"),s=t.isWindow||t.isDocument?"":t.element.css("overflow-y"),n="scroll"===i||"auto"===i&&t.width<t.element[0].scrollWidth,a="scroll"===s||"auto"===s&&t.height<t.element[0].scrollHeight;return{width:a?e.position.scrollbarWidth():0,height:n?e.position.scrollbarWidth():0}},getWithinInfo:function(t){var i=e(t||window),s=e.isWindow(i[0]),n=!!i[0]&&9===i[0].nodeType;return{element:i,isWindow:s,isDocument:n,offset:i.offset()||{left:0,top:0},scrollLeft:i.scrollLeft(),scrollTop:i.scrollTop(),width:s||n?i.width():i.outerWidth(),height:s||n?i.height():i.outerHeight()}}},e.fn.position=function(n){if(!n||!n.of)return f.apply(this,arguments);n=e.extend({},n);var p,m,g,v,y,b,_=e(n.of),x=e.position.getWithinInfo(n.within),w=e.position.getScrollInfo(x),k=(n.collision||"flip").split(" "),T={};return b=s(_),_[0].preventDefault&&(n.at="left top"),m=b.width,g=b.height,v=b.offset,y=e.extend({},v),e.each(["my","at"],function(){var e,t,i=(n[this]||"").split(" ");1===i.length&&(i=l.test(i[0])?i.concat(["center"]):u.test(i[0])?["center"].concat(i):["center","center"]),i[0]=l.test(i[0])?i[0]:"center",i[1]=u.test(i[1])?i[1]:"center",e=d.exec(i[0]),t=d.exec(i[1]),T[this]=[e?e[0]:0,t?t[0]:0],n[this]=[c.exec(i[0])[0],c.exec(i[1])[0]]}),1===k.length&&(k[1]=k[0]),"right"===n.at[0]?y.left+=m:"center"===n.at[0]&&(y.left+=m/2),"bottom"===n.at[1]?y.top+=g:"center"===n.at[1]&&(y.top+=g/2),p=t(T.at,m,g),y.left+=p[0],y.top+=p[1],this.each(function(){var s,l,u=e(this),d=u.outerWidth(),c=u.outerHeight(),f=i(this,"marginLeft"),b=i(this,"marginTop"),D=d+f+i(this,"marginRight")+w.width,S=c+b+i(this,"marginBottom")+w.height,N=e.extend({},y),M=t(T.my,u.outerWidth(),u.outerHeight());"right"===n.my[0]?N.left-=d:"center"===n.my[0]&&(N.left-=d/2),"bottom"===n.my[1]?N.top-=c:"center"===n.my[1]&&(N.top-=c/2),N.left+=M[0],N.top+=M[1],a||(N.left=h(N.left),N.top=h(N.top)),s={marginLeft:f,marginTop:b},e.each(["left","top"],function(t,i){e.ui.position[k[t]]&&e.ui.position[k[t]][i](N,{targetWidth:m,targetHeight:g,elemWidth:d,elemHeight:c,collisionPosition:s,collisionWidth:D,collisionHeight:S,offset:[p[0]+M[0],p[1]+M[1]],my:n.my,at:n.at,within:x,elem:u})}),n.using&&(l=function(e){var t=v.left-N.left,i=t+m-d,s=v.top-N.top,a=s+g-c,h={target:{element:_,left:v.left,top:v.top,width:m,height:g},element:{element:u,left:N.left,top:N.top,width:d,height:c},horizontal:0>i?"left":t>0?"right":"center",vertical:0>a?"top":s>0?"bottom":"middle"};d>m&&m>r(t+i)&&(h.horizontal="center"),c>g&&g>r(s+a)&&(h.vertical="middle"),h.important=o(r(t),r(i))>o(r(s),r(a))?"horizontal":"vertical",n.using.call(this,e,h)}),u.offset(e.extend(N,{using:l}))})},e.ui.position={fit:{left:function(e,t){var i,s=t.within,n=s.isWindow?s.scrollLeft:s.offset.left,a=s.width,r=e.left-t.collisionPosition.marginLeft,h=n-r,l=r+t.collisionWidth-a-n;t.collisionWidth>a?h>0&&0>=l?(i=e.left+h+t.collisionWidth-a-n,e.left+=h-i):e.left=l>0&&0>=h?n:h>l?n+a-t.collisionWidth:n:h>0?e.left+=h:l>0?e.left-=l:e.left=o(e.left-r,e.left)},top:function(e,t){var i,s=t.within,n=s.isWindow?s.scrollTop:s.offset.top,a=t.within.height,r=e.top-t.collisionPosition.marginTop,h=n-r,l=r+t.collisionHeight-a-n;t.collisionHeight>a?h>0&&0>=l?(i=e.top+h+t.collisionHeight-a-n,e.top+=h-i):e.top=l>0&&0>=h?n:h>l?n+a-t.collisionHeight:n:h>0?e.top+=h:l>0?e.top-=l:e.top=o(e.top-r,e.top)}},flip:{left:function(e,t){var i,s,n=t.within,a=n.offset.left+n.scrollLeft,o=n.width,h=n.isWindow?n.scrollLeft:n.offset.left,l=e.left-t.collisionPosition.marginLeft,u=l-h,d=l+t.collisionWidth-o-h,c="left"===t.my[0]?-t.elemWidth:"right"===t.my[0]?t.elemWidth:0,p="left"===t.at[0]?t.targetWidth:"right"===t.at[0]?-t.targetWidth:0,f=-2*t.offset[0];0>u?(i=e.left+c+p+f+t.collisionWidth-o-a,(0>i||r(u)>i)&&(e.left+=c+p+f)):d>0&&(s=e.left-t.collisionPosition.marginLeft+c+p+f-h,(s>0||d>r(s))&&(e.left+=c+p+f))},top:function(e,t){var i,s,n=t.within,a=n.offset.top+n.scrollTop,o=n.height,h=n.isWindow?n.scrollTop:n.offset.top,l=e.top-t.collisionPosition.marginTop,u=l-h,d=l+t.collisionHeight-o-h,c="top"===t.my[1],p=c?-t.elemHeight:"bottom"===t.my[1]?t.elemHeight:0,f="top"===t.at[1]?t.targetHeight:"bottom"===t.at[1]?-t.targetHeight:0,m=-2*t.offset[1];0>u?(s=e.top+p+f+m+t.collisionHeight-o-a,(0>s||r(u)>s)&&(e.top+=p+f+m)):d>0&&(i=e.top-t.collisionPosition.marginTop+p+f+m-h,(i>0||d>r(i))&&(e.top+=p+f+m))}},flipfit:{left:function(){e.ui.position.flip.left.apply(this,arguments),e.ui.position.fit.left.apply(this,arguments)},top:function(){e.ui.position.flip.top.apply(this,arguments),e.ui.position.fit.top.apply(this,arguments)}}},function(){var t,i,s,n,o,r=document.getElementsByTagName("body")[0],h=document.createElement("div");t=document.createElement(r?"div":"body"),s={visibility:"hidden",width:0,height:0,border:0,margin:0,background:"none"},r&&e.extend(s,{position:"absolute",left:"-1000px",top:"-1000px"});for(o in s)t.style[o]=s[o];t.appendChild(h),i=r||document.documentElement,i.insertBefore(t,i.firstChild),h.style.cssText="position: absolute; left: 10.7432222px;",n=e(h).offset().left,a=n>10&&11>n,t.innerHTML="",i.removeChild(t)}()}(),e.ui.position,e.widget("ui.draggable",e.ui.mouse,{version:"1.11.4",widgetEventPrefix:"drag",options:{addClasses:!0,appendTo:"parent",axis:!1,connectToSortable:!1,containment:!1,cursor:"auto",cursorAt:!1,grid:!1,handle:!1,helper:"original",iframeFix:!1,opacity:!1,refreshPositions:!1,revert:!1,revertDuration:500,scope:"default",scroll:!0,scrollSensitivity:20,scrollSpeed:20,snap:!1,snapMode:"both",snapTolerance:20,stack:!1,zIndex:!1,drag:null,start:null,stop:null},_create:function(){"original"===this.options.helper&&this._setPositionRelative(),this.options.addClasses&&this.element.addClass("ui-draggable"),this.options.disabled&&this.element.addClass("ui-draggable-disabled"),this._setHandleClassName(),this._mouseInit()},_setOption:function(e,t){this._super(e,t),"handle"===e&&(this._removeHandleClassName(),this._setHandleClassName())},_destroy:function(){return(this.helper||this.element).is(".ui-draggable-dragging")?(this.destroyOnClear=!0,void 0):(this.element.removeClass("ui-draggable ui-draggable-dragging ui-draggable-disabled"),this._removeHandleClassName(),this._mouseDestroy(),void 0)},_mouseCapture:function(t){var i=this.options;return this._blurActiveElement(t),this.helper||i.disabled||e(t.target).closest(".ui-resizable-handle").length>0?!1:(this.handle=this._getHandle(t),this.handle?(this._blockFrames(i.iframeFix===!0?"iframe":i.iframeFix),!0):!1)},_blockFrames:function(t){this.iframeBlocks=this.document.find(t).map(function(){var t=e(this);return e("<div>").css("position","absolute").appendTo(t.parent()).outerWidth(t.outerWidth()).outerHeight(t.outerHeight()).offset(t.offset())[0]})},_unblockFrames:function(){this.iframeBlocks&&(this.iframeBlocks.remove(),delete this.iframeBlocks)},_blurActiveElement:function(t){var i=this.document[0];if(this.handleElement.is(t.target))try{i.activeElement&&"body"!==i.activeElement.nodeName.toLowerCase()&&e(i.activeElement).blur()}catch(s){}},_mouseStart:function(t){var i=this.options;return this.helper=this._createHelper(t),this.helper.addClass("ui-draggable-dragging"),this._cacheHelperProportions(),e.ui.ddmanager&&(e.ui.ddmanager.current=this),this._cacheMargins(),this.cssPosition=this.helper.css("position"),this.scrollParent=this.helper.scrollParent(!0),this.offsetParent=this.helper.offsetParent(),this.hasFixedAncestor=this.helper.parents().filter(function(){return"fixed"===e(this).css("position")}).length>0,this.positionAbs=this.element.offset(),this._refreshOffsets(t),this.originalPosition=this.position=this._generatePosition(t,!1),this.originalPageX=t.pageX,this.originalPageY=t.pageY,i.cursorAt&&this._adjustOffsetFromHelper(i.cursorAt),this._setContainment(),this._trigger("start",t)===!1?(this._clear(),!1):(this._cacheHelperProportions(),e.ui.ddmanager&&!i.dropBehaviour&&e.ui.ddmanager.prepareOffsets(this,t),this._normalizeRightBottom(),this._mouseDrag(t,!0),e.ui.ddmanager&&e.ui.ddmanager.dragStart(this,t),!0)},_refreshOffsets:function(e){this.offset={top:this.positionAbs.top-this.margins.top,left:this.positionAbs.left-this.margins.left,scroll:!1,parent:this._getParentOffset(),relative:this._getRelativeOffset()},this.offset.click={left:e.pageX-this.offset.left,top:e.pageY-this.offset.top}},_mouseDrag:function(t,i){if(this.hasFixedAncestor&&(this.offset.parent=this._getParentOffset()),this.position=this._generatePosition(t,!0),this.positionAbs=this._convertPositionTo("absolute"),!i){var s=this._uiHash();if(this._trigger("drag",t,s)===!1)return this._mouseUp({}),!1;this.position=s.position}return this.helper[0].style.left=this.position.left+"px",this.helper[0].style.top=this.position.top+"px",e.ui.ddmanager&&e.ui.ddmanager.drag(this,t),!1},_mouseStop:function(t){var i=this,s=!1;return e.ui.ddmanager&&!this.options.dropBehaviour&&(s=e.ui.ddmanager.drop(this,t)),this.dropped&&(s=this.dropped,this.dropped=!1),"invalid"===this.options.revert&&!s||"valid"===this.options.revert&&s||this.options.revert===!0||e.isFunction(this.options.revert)&&this.options.revert.call(this.element,s)?e(this.helper).animate(this.originalPosition,parseInt(this.options.revertDuration,10),function(){i._trigger("stop",t)!==!1&&i._clear()}):this._trigger("stop",t)!==!1&&this._clear(),!1},_mouseUp:function(t){return this._unblockFrames(),e.ui.ddmanager&&e.ui.ddmanager.dragStop(this,t),this.handleElement.is(t.target)&&this.element.focus(),e.ui.mouse.prototype._mouseUp.call(this,t)},cancel:function(){return this.helper.is(".ui-draggable-dragging")?this._mouseUp({}):this._clear(),this},_getHandle:function(t){return this.options.handle?!!e(t.target).closest(this.element.find(this.options.handle)).length:!0},_setHandleClassName:function(){this.handleElement=this.options.handle?this.element.find(this.options.handle):this.element,this.handleElement.addClass("ui-draggable-handle")},_removeHandleClassName:function(){this.handleElement.removeClass("ui-draggable-handle")},_createHelper:function(t){var i=this.options,s=e.isFunction(i.helper),n=s?e(i.helper.apply(this.element[0],[t])):"clone"===i.helper?this.element.clone().removeAttr("id"):this.element;return n.parents("body").length||n.appendTo("parent"===i.appendTo?this.element[0].parentNode:i.appendTo),s&&n[0]===this.element[0]&&this._setPositionRelative(),n[0]===this.element[0]||/(fixed|absolute)/.test(n.css("position"))||n.css("position","absolute"),n},_setPositionRelative:function(){/^(?:r|a|f)/.test(this.element.css("position"))||(this.element[0].style.position="relative")},_adjustOffsetFromHelper:function(t){"string"==typeof t&&(t=t.split(" ")),e.isArray(t)&&(t={left:+t[0],top:+t[1]||0}),"left"in t&&(this.offset.click.left=t.left+this.margins.left),"right"in t&&(this.offset.click.left=this.helperProportions.width-t.right+this.margins.left),"top"in t&&(this.offset.click.top=t.top+this.margins.top),"bottom"in t&&(this.offset.click.top=this.helperProportions.height-t.bottom+this.margins.top)},_isRootNode:function(e){return/(html|body)/i.test(e.tagName)||e===this.document[0]},_getParentOffset:function(){var t=this.offsetParent.offset(),i=this.document[0];return"absolute"===this.cssPosition&&this.scrollParent[0]!==i&&e.contains(this.scrollParent[0],this.offsetParent[0])&&(t.left+=this.scrollParent.scrollLeft(),t.top+=this.scrollParent.scrollTop()),this._isRootNode(this.offsetParent[0])&&(t={top:0,left:0}),{top:t.top+(parseInt(this.offsetParent.css("borderTopWidth"),10)||0),left:t.left+(parseInt(this.offsetParent.css("borderLeftWidth"),10)||0)}},_getRelativeOffset:function(){if("relative"!==this.cssPosition)return{top:0,left:0};var e=this.element.position(),t=this._isRootNode(this.scrollParent[0]);return{top:e.top-(parseInt(this.helper.css("top"),10)||0)+(t?0:this.scrollParent.scrollTop()),left:e.left-(parseInt(this.helper.css("left"),10)||0)+(t?0:this.scrollParent.scrollLeft())}},_cacheMargins:function(){this.margins={left:parseInt(this.element.css("marginLeft"),10)||0,top:parseInt(this.element.css("marginTop"),10)||0,right:parseInt(this.element.css("marginRight"),10)||0,bottom:parseInt(this.element.css("marginBottom"),10)||0}},_cacheHelperProportions:function(){this.helperProportions={width:this.helper.outerWidth(),height:this.helper.outerHeight()}},_setContainment:function(){var t,i,s,n=this.options,a=this.document[0];return this.relativeContainer=null,n.containment?"window"===n.containment?(this.containment=[e(window).scrollLeft()-this.offset.relative.left-this.offset.parent.left,e(window).scrollTop()-this.offset.relative.top-this.offset.parent.top,e(window).scrollLeft()+e(window).width()-this.helperProportions.width-this.margins.left,e(window).scrollTop()+(e(window).height()||a.body.parentNode.scrollHeight)-this.helperProportions.height-this.margins.top],void 0):"document"===n.containment?(this.containment=[0,0,e(a).width()-this.helperProportions.width-this.margins.left,(e(a).height()||a.body.parentNode.scrollHeight)-this.helperProportions.height-this.margins.top],void 0):n.containment.constructor===Array?(this.containment=n.containment,void 0):("parent"===n.containment&&(n.containment=this.helper[0].parentNode),i=e(n.containment),s=i[0],s&&(t=/(scroll|auto)/.test(i.css("overflow")),this.containment=[(parseInt(i.css("borderLeftWidth"),10)||0)+(parseInt(i.css("paddingLeft"),10)||0),(parseInt(i.css("borderTopWidth"),10)||0)+(parseInt(i.css("paddingTop"),10)||0),(t?Math.max(s.scrollWidth,s.offsetWidth):s.offsetWidth)-(parseInt(i.css("borderRightWidth"),10)||0)-(parseInt(i.css("paddingRight"),10)||0)-this.helperProportions.width-this.margins.left-this.margins.right,(t?Math.max(s.scrollHeight,s.offsetHeight):s.offsetHeight)-(parseInt(i.css("borderBottomWidth"),10)||0)-(parseInt(i.css("paddingBottom"),10)||0)-this.helperProportions.height-this.margins.top-this.margins.bottom],this.relativeContainer=i),void 0):(this.containment=null,void 0)},_convertPositionTo:function(e,t){t||(t=this.position);var i="absolute"===e?1:-1,s=this._isRootNode(this.scrollParent[0]);return{top:t.top+this.offset.relative.top*i+this.offset.parent.top*i-("fixed"===this.cssPosition?-this.offset.scroll.top:s?0:this.offset.scroll.top)*i,left:t.left+this.offset.relative.left*i+this.offset.parent.left*i-("fixed"===this.cssPosition?-this.offset.scroll.left:s?0:this.offset.scroll.left)*i}},_generatePosition:function(e,t){var i,s,n,a,o=this.options,r=this._isRootNode(this.scrollParent[0]),h=e.pageX,l=e.pageY;return r&&this.offset.scroll||(this.offset.scroll={top:this.scrollParent.scrollTop(),left:this.scrollParent.scrollLeft()}),t&&(this.containment&&(this.relativeContainer?(s=this.relativeContainer.offset(),i=[this.containment[0]+s.left,this.containment[1]+s.top,this.containment[2]+s.left,this.containment[3]+s.top]):i=this.containment,e.pageX-this.offset.click.left<i[0]&&(h=i[0]+this.offset.click.left),e.pageY-this.offset.click.top<i[1]&&(l=i[1]+this.offset.click.top),e.pageX-this.offset.click.left>i[2]&&(h=i[2]+this.offset.click.left),e.pageY-this.offset.click.top>i[3]&&(l=i[3]+this.offset.click.top)),o.grid&&(n=o.grid[1]?this.originalPageY+Math.round((l-this.originalPageY)/o.grid[1])*o.grid[1]:this.originalPageY,l=i?n-this.offset.click.top>=i[1]||n-this.offset.click.top>i[3]?n:n-this.offset.click.top>=i[1]?n-o.grid[1]:n+o.grid[1]:n,a=o.grid[0]?this.originalPageX+Math.round((h-this.originalPageX)/o.grid[0])*o.grid[0]:this.originalPageX,h=i?a-this.offset.click.left>=i[0]||a-this.offset.click.left>i[2]?a:a-this.offset.click.left>=i[0]?a-o.grid[0]:a+o.grid[0]:a),"y"===o.axis&&(h=this.originalPageX),"x"===o.axis&&(l=this.originalPageY)),{top:l-this.offset.click.top-this.offset.relative.top-this.offset.parent.top+("fixed"===this.cssPosition?-this.offset.scroll.top:r?0:this.offset.scroll.top),left:h-this.offset.click.left-this.offset.relative.left-this.offset.parent.left+("fixed"===this.cssPosition?-this.offset.scroll.left:r?0:this.offset.scroll.left)}},_clear:function(){this.helper.removeClass("ui-draggable-dragging"),this.helper[0]===this.element[0]||this.cancelHelperRemoval||this.helper.remove(),this.helper=null,this.cancelHelperRemoval=!1,this.destroyOnClear&&this.destroy()},_normalizeRightBottom:function(){"y"!==this.options.axis&&"auto"!==this.helper.css("right")&&(this.helper.width(this.helper.width()),this.helper.css("right","auto")),"x"!==this.options.axis&&"auto"!==this.helper.css("bottom")&&(this.helper.height(this.helper.height()),this.helper.css("bottom","auto"))},_trigger:function(t,i,s){return s=s||this._uiHash(),e.ui.plugin.call(this,t,[i,s,this],!0),/^(drag|start|stop)/.test(t)&&(this.positionAbs=this._convertPositionTo("absolute"),s.offset=this.positionAbs),e.Widget.prototype._trigger.call(this,t,i,s)},plugins:{},_uiHash:function(){return{helper:this.helper,position:this.position,originalPosition:this.originalPosition,offset:this.positionAbs}}}),e.ui.plugin.add("draggable","connectToSortable",{start:function(t,i,s){var n=e.extend({},i,{item:s.element});s.sortables=[],e(s.options.connectToSortable).each(function(){var i=e(this).sortable("instance");i&&!i.options.disabled&&(s.sortables.push(i),i.refreshPositions(),i._trigger("activate",t,n))})},stop:function(t,i,s){var n=e.extend({},i,{item:s.element});s.cancelHelperRemoval=!1,e.each(s.sortables,function(){var e=this;e.isOver?(e.isOver=0,s.cancelHelperRemoval=!0,e.cancelHelperRemoval=!1,e._storedCSS={position:e.placeholder.css("position"),top:e.placeholder.css("top"),left:e.placeholder.css("left")},e._mouseStop(t),e.options.helper=e.options._helper):(e.cancelHelperRemoval=!0,e._trigger("deactivate",t,n))})},drag:function(t,i,s){e.each(s.sortables,function(){var n=!1,a=this;a.positionAbs=s.positionAbs,a.helperProportions=s.helperProportions,a.offset.click=s.offset.click,a._intersectsWith(a.containerCache)&&(n=!0,e.each(s.sortables,function(){return this.positionAbs=s.positionAbs,this.helperProportions=s.helperProportions,this.offset.click=s.offset.click,this!==a&&this._intersectsWith(this.containerCache)&&e.contains(a.element[0],this.element[0])&&(n=!1),n
+})),n?(a.isOver||(a.isOver=1,s._parent=i.helper.parent(),a.currentItem=i.helper.appendTo(a.element).data("ui-sortable-item",!0),a.options._helper=a.options.helper,a.options.helper=function(){return i.helper[0]},t.target=a.currentItem[0],a._mouseCapture(t,!0),a._mouseStart(t,!0,!0),a.offset.click.top=s.offset.click.top,a.offset.click.left=s.offset.click.left,a.offset.parent.left-=s.offset.parent.left-a.offset.parent.left,a.offset.parent.top-=s.offset.parent.top-a.offset.parent.top,s._trigger("toSortable",t),s.dropped=a.element,e.each(s.sortables,function(){this.refreshPositions()}),s.currentItem=s.element,a.fromOutside=s),a.currentItem&&(a._mouseDrag(t),i.position=a.position)):a.isOver&&(a.isOver=0,a.cancelHelperRemoval=!0,a.options._revert=a.options.revert,a.options.revert=!1,a._trigger("out",t,a._uiHash(a)),a._mouseStop(t,!0),a.options.revert=a.options._revert,a.options.helper=a.options._helper,a.placeholder&&a.placeholder.remove(),i.helper.appendTo(s._parent),s._refreshOffsets(t),i.position=s._generatePosition(t,!0),s._trigger("fromSortable",t),s.dropped=!1,e.each(s.sortables,function(){this.refreshPositions()}))})}}),e.ui.plugin.add("draggable","cursor",{start:function(t,i,s){var n=e("body"),a=s.options;n.css("cursor")&&(a._cursor=n.css("cursor")),n.css("cursor",a.cursor)},stop:function(t,i,s){var n=s.options;n._cursor&&e("body").css("cursor",n._cursor)}}),e.ui.plugin.add("draggable","opacity",{start:function(t,i,s){var n=e(i.helper),a=s.options;n.css("opacity")&&(a._opacity=n.css("opacity")),n.css("opacity",a.opacity)},stop:function(t,i,s){var n=s.options;n._opacity&&e(i.helper).css("opacity",n._opacity)}}),e.ui.plugin.add("draggable","scroll",{start:function(e,t,i){i.scrollParentNotHidden||(i.scrollParentNotHidden=i.helper.scrollParent(!1)),i.scrollParentNotHidden[0]!==i.document[0]&&"HTML"!==i.scrollParentNotHidden[0].tagName&&(i.overflowOffset=i.scrollParentNotHidden.offset())},drag:function(t,i,s){var n=s.options,a=!1,o=s.scrollParentNotHidden[0],r=s.document[0];o!==r&&"HTML"!==o.tagName?(n.axis&&"x"===n.axis||(s.overflowOffset.top+o.offsetHeight-t.pageY<n.scrollSensitivity?o.scrollTop=a=o.scrollTop+n.scrollSpeed:t.pageY-s.overflowOffset.top<n.scrollSensitivity&&(o.scrollTop=a=o.scrollTop-n.scrollSpeed)),n.axis&&"y"===n.axis||(s.overflowOffset.left+o.offsetWidth-t.pageX<n.scrollSensitivity?o.scrollLeft=a=o.scrollLeft+n.scrollSpeed:t.pageX-s.overflowOffset.left<n.scrollSensitivity&&(o.scrollLeft=a=o.scrollLeft-n.scrollSpeed))):(n.axis&&"x"===n.axis||(t.pageY-e(r).scrollTop()<n.scrollSensitivity?a=e(r).scrollTop(e(r).scrollTop()-n.scrollSpeed):e(window).height()-(t.pageY-e(r).scrollTop())<n.scrollSensitivity&&(a=e(r).scrollTop(e(r).scrollTop()+n.scrollSpeed))),n.axis&&"y"===n.axis||(t.pageX-e(r).scrollLeft()<n.scrollSensitivity?a=e(r).scrollLeft(e(r).scrollLeft()-n.scrollSpeed):e(window).width()-(t.pageX-e(r).scrollLeft())<n.scrollSensitivity&&(a=e(r).scrollLeft(e(r).scrollLeft()+n.scrollSpeed)))),a!==!1&&e.ui.ddmanager&&!n.dropBehaviour&&e.ui.ddmanager.prepareOffsets(s,t)}}),e.ui.plugin.add("draggable","snap",{start:function(t,i,s){var n=s.options;s.snapElements=[],e(n.snap.constructor!==String?n.snap.items||":data(ui-draggable)":n.snap).each(function(){var t=e(this),i=t.offset();this!==s.element[0]&&s.snapElements.push({item:this,width:t.outerWidth(),height:t.outerHeight(),top:i.top,left:i.left})})},drag:function(t,i,s){var n,a,o,r,h,l,u,d,c,p,f=s.options,m=f.snapTolerance,g=i.offset.left,v=g+s.helperProportions.width,y=i.offset.top,b=y+s.helperProportions.height;for(c=s.snapElements.length-1;c>=0;c--)h=s.snapElements[c].left-s.margins.left,l=h+s.snapElements[c].width,u=s.snapElements[c].top-s.margins.top,d=u+s.snapElements[c].height,h-m>v||g>l+m||u-m>b||y>d+m||!e.contains(s.snapElements[c].item.ownerDocument,s.snapElements[c].item)?(s.snapElements[c].snapping&&s.options.snap.release&&s.options.snap.release.call(s.element,t,e.extend(s._uiHash(),{snapItem:s.snapElements[c].item})),s.snapElements[c].snapping=!1):("inner"!==f.snapMode&&(n=m>=Math.abs(u-b),a=m>=Math.abs(d-y),o=m>=Math.abs(h-v),r=m>=Math.abs(l-g),n&&(i.position.top=s._convertPositionTo("relative",{top:u-s.helperProportions.height,left:0}).top),a&&(i.position.top=s._convertPositionTo("relative",{top:d,left:0}).top),o&&(i.position.left=s._convertPositionTo("relative",{top:0,left:h-s.helperProportions.width}).left),r&&(i.position.left=s._convertPositionTo("relative",{top:0,left:l}).left)),p=n||a||o||r,"outer"!==f.snapMode&&(n=m>=Math.abs(u-y),a=m>=Math.abs(d-b),o=m>=Math.abs(h-g),r=m>=Math.abs(l-v),n&&(i.position.top=s._convertPositionTo("relative",{top:u,left:0}).top),a&&(i.position.top=s._convertPositionTo("relative",{top:d-s.helperProportions.height,left:0}).top),o&&(i.position.left=s._convertPositionTo("relative",{top:0,left:h}).left),r&&(i.position.left=s._convertPositionTo("relative",{top:0,left:l-s.helperProportions.width}).left)),!s.snapElements[c].snapping&&(n||a||o||r||p)&&s.options.snap.snap&&s.options.snap.snap.call(s.element,t,e.extend(s._uiHash(),{snapItem:s.snapElements[c].item})),s.snapElements[c].snapping=n||a||o||r||p)}}),e.ui.plugin.add("draggable","stack",{start:function(t,i,s){var n,a=s.options,o=e.makeArray(e(a.stack)).sort(function(t,i){return(parseInt(e(t).css("zIndex"),10)||0)-(parseInt(e(i).css("zIndex"),10)||0)});o.length&&(n=parseInt(e(o[0]).css("zIndex"),10)||0,e(o).each(function(t){e(this).css("zIndex",n+t)}),this.css("zIndex",n+o.length))}}),e.ui.plugin.add("draggable","zIndex",{start:function(t,i,s){var n=e(i.helper),a=s.options;n.css("zIndex")&&(a._zIndex=n.css("zIndex")),n.css("zIndex",a.zIndex)},stop:function(t,i,s){var n=s.options;n._zIndex&&e(i.helper).css("zIndex",n._zIndex)}}),e.ui.draggable,e.widget("ui.droppable",{version:"1.11.4",widgetEventPrefix:"drop",options:{accept:"*",activeClass:!1,addClasses:!0,greedy:!1,hoverClass:!1,scope:"default",tolerance:"intersect",activate:null,deactivate:null,drop:null,out:null,over:null},_create:function(){var t,i=this.options,s=i.accept;this.isover=!1,this.isout=!0,this.accept=e.isFunction(s)?s:function(e){return e.is(s)},this.proportions=function(){return arguments.length?(t=arguments[0],void 0):t?t:t={width:this.element[0].offsetWidth,height:this.element[0].offsetHeight}},this._addToManager(i.scope),i.addClasses&&this.element.addClass("ui-droppable")},_addToManager:function(t){e.ui.ddmanager.droppables[t]=e.ui.ddmanager.droppables[t]||[],e.ui.ddmanager.droppables[t].push(this)},_splice:function(e){for(var t=0;e.length>t;t++)e[t]===this&&e.splice(t,1)},_destroy:function(){var t=e.ui.ddmanager.droppables[this.options.scope];this._splice(t),this.element.removeClass("ui-droppable ui-droppable-disabled")},_setOption:function(t,i){if("accept"===t)this.accept=e.isFunction(i)?i:function(e){return e.is(i)};else if("scope"===t){var s=e.ui.ddmanager.droppables[this.options.scope];this._splice(s),this._addToManager(i)}this._super(t,i)},_activate:function(t){var i=e.ui.ddmanager.current;this.options.activeClass&&this.element.addClass(this.options.activeClass),i&&this._trigger("activate",t,this.ui(i))},_deactivate:function(t){var i=e.ui.ddmanager.current;this.options.activeClass&&this.element.removeClass(this.options.activeClass),i&&this._trigger("deactivate",t,this.ui(i))},_over:function(t){var i=e.ui.ddmanager.current;i&&(i.currentItem||i.element)[0]!==this.element[0]&&this.accept.call(this.element[0],i.currentItem||i.element)&&(this.options.hoverClass&&this.element.addClass(this.options.hoverClass),this._trigger("over",t,this.ui(i)))},_out:function(t){var i=e.ui.ddmanager.current;i&&(i.currentItem||i.element)[0]!==this.element[0]&&this.accept.call(this.element[0],i.currentItem||i.element)&&(this.options.hoverClass&&this.element.removeClass(this.options.hoverClass),this._trigger("out",t,this.ui(i)))},_drop:function(t,i){var s=i||e.ui.ddmanager.current,n=!1;return s&&(s.currentItem||s.element)[0]!==this.element[0]?(this.element.find(":data(ui-droppable)").not(".ui-draggable-dragging").each(function(){var i=e(this).droppable("instance");return i.options.greedy&&!i.options.disabled&&i.options.scope===s.options.scope&&i.accept.call(i.element[0],s.currentItem||s.element)&&e.ui.intersect(s,e.extend(i,{offset:i.element.offset()}),i.options.tolerance,t)?(n=!0,!1):void 0}),n?!1:this.accept.call(this.element[0],s.currentItem||s.element)?(this.options.activeClass&&this.element.removeClass(this.options.activeClass),this.options.hoverClass&&this.element.removeClass(this.options.hoverClass),this._trigger("drop",t,this.ui(s)),this.element):!1):!1},ui:function(e){return{draggable:e.currentItem||e.element,helper:e.helper,position:e.position,offset:e.positionAbs}}}),e.ui.intersect=function(){function e(e,t,i){return e>=t&&t+i>e}return function(t,i,s,n){if(!i.offset)return!1;var a=(t.positionAbs||t.position.absolute).left+t.margins.left,o=(t.positionAbs||t.position.absolute).top+t.margins.top,r=a+t.helperProportions.width,h=o+t.helperProportions.height,l=i.offset.left,u=i.offset.top,d=l+i.proportions().width,c=u+i.proportions().height;switch(s){case"fit":return a>=l&&d>=r&&o>=u&&c>=h;case"intersect":return a+t.helperProportions.width/2>l&&d>r-t.helperProportions.width/2&&o+t.helperProportions.height/2>u&&c>h-t.helperProportions.height/2;case"pointer":return e(n.pageY,u,i.proportions().height)&&e(n.pageX,l,i.proportions().width);case"touch":return(o>=u&&c>=o||h>=u&&c>=h||u>o&&h>c)&&(a>=l&&d>=a||r>=l&&d>=r||l>a&&r>d);default:return!1}}}(),e.ui.ddmanager={current:null,droppables:{"default":[]},prepareOffsets:function(t,i){var s,n,a=e.ui.ddmanager.droppables[t.options.scope]||[],o=i?i.type:null,r=(t.currentItem||t.element).find(":data(ui-droppable)").addBack();e:for(s=0;a.length>s;s++)if(!(a[s].options.disabled||t&&!a[s].accept.call(a[s].element[0],t.currentItem||t.element))){for(n=0;r.length>n;n++)if(r[n]===a[s].element[0]){a[s].proportions().height=0;continue e}a[s].visible="none"!==a[s].element.css("display"),a[s].visible&&("mousedown"===o&&a[s]._activate.call(a[s],i),a[s].offset=a[s].element.offset(),a[s].proportions({width:a[s].element[0].offsetWidth,height:a[s].element[0].offsetHeight}))}},drop:function(t,i){var s=!1;return e.each((e.ui.ddmanager.droppables[t.options.scope]||[]).slice(),function(){this.options&&(!this.options.disabled&&this.visible&&e.ui.intersect(t,this,this.options.tolerance,i)&&(s=this._drop.call(this,i)||s),!this.options.disabled&&this.visible&&this.accept.call(this.element[0],t.currentItem||t.element)&&(this.isout=!0,this.isover=!1,this._deactivate.call(this,i)))}),s},dragStart:function(t,i){t.element.parentsUntil("body").bind("scroll.droppable",function(){t.options.refreshPositions||e.ui.ddmanager.prepareOffsets(t,i)})},drag:function(t,i){t.options.refreshPositions&&e.ui.ddmanager.prepareOffsets(t,i),e.each(e.ui.ddmanager.droppables[t.options.scope]||[],function(){if(!this.options.disabled&&!this.greedyChild&&this.visible){var s,n,a,o=e.ui.intersect(t,this,this.options.tolerance,i),r=!o&&this.isover?"isout":o&&!this.isover?"isover":null;r&&(this.options.greedy&&(n=this.options.scope,a=this.element.parents(":data(ui-droppable)").filter(function(){return e(this).droppable("instance").options.scope===n}),a.length&&(s=e(a[0]).droppable("instance"),s.greedyChild="isover"===r)),s&&"isover"===r&&(s.isover=!1,s.isout=!0,s._out.call(s,i)),this[r]=!0,this["isout"===r?"isover":"isout"]=!1,this["isover"===r?"_over":"_out"].call(this,i),s&&"isout"===r&&(s.isout=!1,s.isover=!0,s._over.call(s,i)))}})},dragStop:function(t,i){t.element.parentsUntil("body").unbind("scroll.droppable"),t.options.refreshPositions||e.ui.ddmanager.prepareOffsets(t,i)}},e.ui.droppable,e.widget("ui.resizable",e.ui.mouse,{version:"1.11.4",widgetEventPrefix:"resize",options:{alsoResize:!1,animate:!1,animateDuration:"slow",animateEasing:"swing",aspectRatio:!1,autoHide:!1,containment:!1,ghost:!1,grid:!1,handles:"e,s,se",helper:!1,maxHeight:null,maxWidth:null,minHeight:10,minWidth:10,zIndex:90,resize:null,start:null,stop:null},_num:function(e){return parseInt(e,10)||0},_isNumber:function(e){return!isNaN(parseInt(e,10))},_hasScroll:function(t,i){if("hidden"===e(t).css("overflow"))return!1;var s=i&&"left"===i?"scrollLeft":"scrollTop",n=!1;return t[s]>0?!0:(t[s]=1,n=t[s]>0,t[s]=0,n)},_create:function(){var t,i,s,n,a,o=this,r=this.options;if(this.element.addClass("ui-resizable"),e.extend(this,{_aspectRatio:!!r.aspectRatio,aspectRatio:r.aspectRatio,originalElement:this.element,_proportionallyResizeElements:[],_helper:r.helper||r.ghost||r.animate?r.helper||"ui-resizable-helper":null}),this.element[0].nodeName.match(/^(canvas|textarea|input|select|button|img)$/i)&&(this.element.wrap(e("<div class='ui-wrapper' style='overflow: hidden;'></div>").css({position:this.element.css("position"),width:this.element.outerWidth(),height:this.element.outerHeight(),top:this.element.css("top"),left:this.element.css("left")})),this.element=this.element.parent().data("ui-resizable",this.element.resizable("instance")),this.elementIsWrapper=!0,this.element.css({marginLeft:this.originalElement.css("marginLeft"),marginTop:this.originalElement.css("marginTop"),marginRight:this.originalElement.css("marginRight"),marginBottom:this.originalElement.css("marginBottom")}),this.originalElement.css({marginLeft:0,marginTop:0,marginRight:0,marginBottom:0}),this.originalResizeStyle=this.originalElement.css("resize"),this.originalElement.css("resize","none"),this._proportionallyResizeElements.push(this.originalElement.css({position:"static",zoom:1,display:"block"})),this.originalElement.css({margin:this.originalElement.css("margin")}),this._proportionallyResize()),this.handles=r.handles||(e(".ui-resizable-handle",this.element).length?{n:".ui-resizable-n",e:".ui-resizable-e",s:".ui-resizable-s",w:".ui-resizable-w",se:".ui-resizable-se",sw:".ui-resizable-sw",ne:".ui-resizable-ne",nw:".ui-resizable-nw"}:"e,s,se"),this._handles=e(),this.handles.constructor===String)for("all"===this.handles&&(this.handles="n,e,s,w,se,sw,ne,nw"),t=this.handles.split(","),this.handles={},i=0;t.length>i;i++)s=e.trim(t[i]),a="ui-resizable-"+s,n=e("<div class='ui-resizable-handle "+a+"'></div>"),n.css({zIndex:r.zIndex}),"se"===s&&n.addClass("ui-icon ui-icon-gripsmall-diagonal-se"),this.handles[s]=".ui-resizable-"+s,this.element.append(n);this._renderAxis=function(t){var i,s,n,a;t=t||this.element;for(i in this.handles)this.handles[i].constructor===String?this.handles[i]=this.element.children(this.handles[i]).first().show():(this.handles[i].jquery||this.handles[i].nodeType)&&(this.handles[i]=e(this.handles[i]),this._on(this.handles[i],{mousedown:o._mouseDown})),this.elementIsWrapper&&this.originalElement[0].nodeName.match(/^(textarea|input|select|button)$/i)&&(s=e(this.handles[i],this.element),a=/sw|ne|nw|se|n|s/.test(i)?s.outerHeight():s.outerWidth(),n=["padding",/ne|nw|n/.test(i)?"Top":/se|sw|s/.test(i)?"Bottom":/^e$/.test(i)?"Right":"Left"].join(""),t.css(n,a),this._proportionallyResize()),this._handles=this._handles.add(this.handles[i])},this._renderAxis(this.element),this._handles=this._handles.add(this.element.find(".ui-resizable-handle")),this._handles.disableSelection(),this._handles.mouseover(function(){o.resizing||(this.className&&(n=this.className.match(/ui-resizable-(se|sw|ne|nw|n|e|s|w)/i)),o.axis=n&&n[1]?n[1]:"se")}),r.autoHide&&(this._handles.hide(),e(this.element).addClass("ui-resizable-autohide").mouseenter(function(){r.disabled||(e(this).removeClass("ui-resizable-autohide"),o._handles.show())}).mouseleave(function(){r.disabled||o.resizing||(e(this).addClass("ui-resizable-autohide"),o._handles.hide())})),this._mouseInit()},_destroy:function(){this._mouseDestroy();var t,i=function(t){e(t).removeClass("ui-resizable ui-resizable-disabled ui-resizable-resizing").removeData("resizable").removeData("ui-resizable").unbind(".resizable").find(".ui-resizable-handle").remove()};return this.elementIsWrapper&&(i(this.element),t=this.element,this.originalElement.css({position:t.css("position"),width:t.outerWidth(),height:t.outerHeight(),top:t.css("top"),left:t.css("left")}).insertAfter(t),t.remove()),this.originalElement.css("resize",this.originalResizeStyle),i(this.originalElement),this},_mouseCapture:function(t){var i,s,n=!1;for(i in this.handles)s=e(this.handles[i])[0],(s===t.target||e.contains(s,t.target))&&(n=!0);return!this.options.disabled&&n},_mouseStart:function(t){var i,s,n,a=this.options,o=this.element;return this.resizing=!0,this._renderProxy(),i=this._num(this.helper.css("left")),s=this._num(this.helper.css("top")),a.containment&&(i+=e(a.containment).scrollLeft()||0,s+=e(a.containment).scrollTop()||0),this.offset=this.helper.offset(),this.position={left:i,top:s},this.size=this._helper?{width:this.helper.width(),height:this.helper.height()}:{width:o.width(),height:o.height()},this.originalSize=this._helper?{width:o.outerWidth(),height:o.outerHeight()}:{width:o.width(),height:o.height()},this.sizeDiff={width:o.outerWidth()-o.width(),height:o.outerHeight()-o.height()},this.originalPosition={left:i,top:s},this.originalMousePosition={left:t.pageX,top:t.pageY},this.aspectRatio="number"==typeof a.aspectRatio?a.aspectRatio:this.originalSize.width/this.originalSize.height||1,n=e(".ui-resizable-"+this.axis).css("cursor"),e("body").css("cursor","auto"===n?this.axis+"-resize":n),o.addClass("ui-resizable-resizing"),this._propagate("start",t),!0},_mouseDrag:function(t){var i,s,n=this.originalMousePosition,a=this.axis,o=t.pageX-n.left||0,r=t.pageY-n.top||0,h=this._change[a];return this._updatePrevProperties(),h?(i=h.apply(this,[t,o,r]),this._updateVirtualBoundaries(t.shiftKey),(this._aspectRatio||t.shiftKey)&&(i=this._updateRatio(i,t)),i=this._respectSize(i,t),this._updateCache(i),this._propagate("resize",t),s=this._applyChanges(),!this._helper&&this._proportionallyResizeElements.length&&this._proportionallyResize(),e.isEmptyObject(s)||(this._updatePrevProperties(),this._trigger("resize",t,this.ui()),this._applyChanges()),!1):!1},_mouseStop:function(t){this.resizing=!1;var i,s,n,a,o,r,h,l=this.options,u=this;return this._helper&&(i=this._proportionallyResizeElements,s=i.length&&/textarea/i.test(i[0].nodeName),n=s&&this._hasScroll(i[0],"left")?0:u.sizeDiff.height,a=s?0:u.sizeDiff.width,o={width:u.helper.width()-a,height:u.helper.height()-n},r=parseInt(u.element.css("left"),10)+(u.position.left-u.originalPosition.left)||null,h=parseInt(u.element.css("top"),10)+(u.position.top-u.originalPosition.top)||null,l.animate||this.element.css(e.extend(o,{top:h,left:r})),u.helper.height(u.size.height),u.helper.width(u.size.width),this._helper&&!l.animate&&this._proportionallyResize()),e("body").css("cursor","auto"),this.element.removeClass("ui-resizable-resizing"),this._propagate("stop",t),this._helper&&this.helper.remove(),!1},_updatePrevProperties:function(){this.prevPosition={top:this.position.top,left:this.position.left},this.prevSize={width:this.size.width,height:this.size.height}},_applyChanges:function(){var e={};return this.position.top!==this.prevPosition.top&&(e.top=this.position.top+"px"),this.position.left!==this.prevPosition.left&&(e.left=this.position.left+"px"),this.size.width!==this.prevSize.width&&(e.width=this.size.width+"px"),this.size.height!==this.prevSize.height&&(e.height=this.size.height+"px"),this.helper.css(e),e},_updateVirtualBoundaries:function(e){var t,i,s,n,a,o=this.options;a={minWidth:this._isNumber(o.minWidth)?o.minWidth:0,maxWidth:this._isNumber(o.maxWidth)?o.maxWidth:1/0,minHeight:this._isNumber(o.minHeight)?o.minHeight:0,maxHeight:this._isNumber(o.maxHeight)?o.maxHeight:1/0},(this._aspectRatio||e)&&(t=a.minHeight*this.aspectRatio,s=a.minWidth/this.aspectRatio,i=a.maxHeight*this.aspectRatio,n=a.maxWidth/this.aspectRatio,t>a.minWidth&&(a.minWidth=t),s>a.minHeight&&(a.minHeight=s),a.maxWidth>i&&(a.maxWidth=i),a.maxHeight>n&&(a.maxHeight=n)),this._vBoundaries=a},_updateCache:function(e){this.offset=this.helper.offset(),this._isNumber(e.left)&&(this.position.left=e.left),this._isNumber(e.top)&&(this.position.top=e.top),this._isNumber(e.height)&&(this.size.height=e.height),this._isNumber(e.width)&&(this.size.width=e.width)},_updateRatio:function(e){var t=this.position,i=this.size,s=this.axis;return this._isNumber(e.height)?e.width=e.height*this.aspectRatio:this._isNumber(e.width)&&(e.height=e.width/this.aspectRatio),"sw"===s&&(e.left=t.left+(i.width-e.width),e.top=null),"nw"===s&&(e.top=t.top+(i.height-e.height),e.left=t.left+(i.width-e.width)),e},_respectSize:function(e){var t=this._vBoundaries,i=this.axis,s=this._isNumber(e.width)&&t.maxWidth&&t.maxWidth<e.width,n=this._isNumber(e.height)&&t.maxHeight&&t.maxHeight<e.height,a=this._isNumber(e.width)&&t.minWidth&&t.minWidth>e.width,o=this._isNumber(e.height)&&t.minHeight&&t.minHeight>e.height,r=this.originalPosition.left+this.originalSize.width,h=this.position.top+this.size.height,l=/sw|nw|w/.test(i),u=/nw|ne|n/.test(i);return a&&(e.width=t.minWidth),o&&(e.height=t.minHeight),s&&(e.width=t.maxWidth),n&&(e.height=t.maxHeight),a&&l&&(e.left=r-t.minWidth),s&&l&&(e.left=r-t.maxWidth),o&&u&&(e.top=h-t.minHeight),n&&u&&(e.top=h-t.maxHeight),e.width||e.height||e.left||!e.top?e.width||e.height||e.top||!e.left||(e.left=null):e.top=null,e},_getPaddingPlusBorderDimensions:function(e){for(var t=0,i=[],s=[e.css("borderTopWidth"),e.css("borderRightWidth"),e.css("borderBottomWidth"),e.css("borderLeftWidth")],n=[e.css("paddingTop"),e.css("paddingRight"),e.css("paddingBottom"),e.css("paddingLeft")];4>t;t++)i[t]=parseInt(s[t],10)||0,i[t]+=parseInt(n[t],10)||0;return{height:i[0]+i[2],width:i[1]+i[3]}},_proportionallyResize:function(){if(this._proportionallyResizeElements.length)for(var e,t=0,i=this.helper||this.element;this._proportionallyResizeElements.length>t;t++)e=this._proportionallyResizeElements[t],this.outerDimensions||(this.outerDimensions=this._getPaddingPlusBorderDimensions(e)),e.css({height:i.height()-this.outerDimensions.height||0,width:i.width()-this.outerDimensions.width||0})},_renderProxy:function(){var t=this.element,i=this.options;this.elementOffset=t.offset(),this._helper?(this.helper=this.helper||e("<div style='overflow:hidden;'></div>"),this.helper.addClass(this._helper).css({width:this.element.outerWidth()-1,height:this.element.outerHeight()-1,position:"absolute",left:this.elementOffset.left+"px",top:this.elementOffset.top+"px",zIndex:++i.zIndex}),this.helper.appendTo("body").disableSelection()):this.helper=this.element},_change:{e:function(e,t){return{width:this.originalSize.width+t}},w:function(e,t){var i=this.originalSize,s=this.originalPosition;return{left:s.left+t,width:i.width-t}},n:function(e,t,i){var s=this.originalSize,n=this.originalPosition;return{top:n.top+i,height:s.height-i}},s:function(e,t,i){return{height:this.originalSize.height+i}},se:function(t,i,s){return e.extend(this._change.s.apply(this,arguments),this._change.e.apply(this,[t,i,s]))},sw:function(t,i,s){return e.extend(this._change.s.apply(this,arguments),this._change.w.apply(this,[t,i,s]))},ne:function(t,i,s){return e.extend(this._change.n.apply(this,arguments),this._change.e.apply(this,[t,i,s]))},nw:function(t,i,s){return e.extend(this._change.n.apply(this,arguments),this._change.w.apply(this,[t,i,s]))}},_propagate:function(t,i){e.ui.plugin.call(this,t,[i,this.ui()]),"resize"!==t&&this._trigger(t,i,this.ui())},plugins:{},ui:function(){return{originalElement:this.originalElement,element:this.element,helper:this.helper,position:this.position,size:this.size,originalSize:this.originalSize,originalPosition:this.originalPosition}}}),e.ui.plugin.add("resizable","animate",{stop:function(t){var i=e(this).resizable("instance"),s=i.options,n=i._proportionallyResizeElements,a=n.length&&/textarea/i.test(n[0].nodeName),o=a&&i._hasScroll(n[0],"left")?0:i.sizeDiff.height,r=a?0:i.sizeDiff.width,h={width:i.size.width-r,height:i.size.height-o},l=parseInt(i.element.css("left"),10)+(i.position.left-i.originalPosition.left)||null,u=parseInt(i.element.css("top"),10)+(i.position.top-i.originalPosition.top)||null;i.element.animate(e.extend(h,u&&l?{top:u,left:l}:{}),{duration:s.animateDuration,easing:s.animateEasing,step:function(){var s={width:parseInt(i.element.css("width"),10),height:parseInt(i.element.css("height"),10),top:parseInt(i.element.css("top"),10),left:parseInt(i.element.css("left"),10)};n&&n.length&&e(n[0]).css({width:s.width,height:s.height}),i._updateCache(s),i._propagate("resize",t)}})}}),e.ui.plugin.add("resizable","containment",{start:function(){var t,i,s,n,a,o,r,h=e(this).resizable("instance"),l=h.options,u=h.element,d=l.containment,c=d instanceof e?d.get(0):/parent/.test(d)?u.parent().get(0):d;c&&(h.containerElement=e(c),/document/.test(d)||d===document?(h.containerOffset={left:0,top:0},h.containerPosition={left:0,top:0},h.parentData={element:e(document),left:0,top:0,width:e(document).width(),height:e(document).height()||document.body.parentNode.scrollHeight}):(t=e(c),i=[],e(["Top","Right","Left","Bottom"]).each(function(e,s){i[e]=h._num(t.css("padding"+s))}),h.containerOffset=t.offset(),h.containerPosition=t.position(),h.containerSize={height:t.innerHeight()-i[3],width:t.innerWidth()-i[1]},s=h.containerOffset,n=h.containerSize.height,a=h.containerSize.width,o=h._hasScroll(c,"left")?c.scrollWidth:a,r=h._hasScroll(c)?c.scrollHeight:n,h.parentData={element:c,left:s.left,top:s.top,width:o,height:r}))},resize:function(t){var i,s,n,a,o=e(this).resizable("instance"),r=o.options,h=o.containerOffset,l=o.position,u=o._aspectRatio||t.shiftKey,d={top:0,left:0},c=o.containerElement,p=!0;c[0]!==document&&/static/.test(c.css("position"))&&(d=h),l.left<(o._helper?h.left:0)&&(o.size.width=o.size.width+(o._helper?o.position.left-h.left:o.position.left-d.left),u&&(o.size.height=o.size.width/o.aspectRatio,p=!1),o.position.left=r.helper?h.left:0),l.top<(o._helper?h.top:0)&&(o.size.height=o.size.height+(o._helper?o.position.top-h.top:o.position.top),u&&(o.size.width=o.size.height*o.aspectRatio,p=!1),o.position.top=o._helper?h.top:0),n=o.containerElement.get(0)===o.element.parent().get(0),a=/relative|absolute/.test(o.containerElement.css("position")),n&&a?(o.offset.left=o.parentData.left+o.position.left,o.offset.top=o.parentData.top+o.position.top):(o.offset.left=o.element.offset().left,o.offset.top=o.element.offset().top),i=Math.abs(o.sizeDiff.width+(o._helper?o.offset.left-d.left:o.offset.left-h.left)),s=Math.abs(o.sizeDiff.height+(o._helper?o.offset.top-d.top:o.offset.top-h.top)),i+o.size.width>=o.parentData.width&&(o.size.width=o.parentData.width-i,u&&(o.size.height=o.size.width/o.aspectRatio,p=!1)),s+o.size.height>=o.parentData.height&&(o.size.height=o.parentData.height-s,u&&(o.size.width=o.size.height*o.aspectRatio,p=!1)),p||(o.position.left=o.prevPosition.left,o.position.top=o.prevPosition.top,o.size.width=o.prevSize.width,o.size.height=o.prevSize.height)},stop:function(){var t=e(this).resizable("instance"),i=t.options,s=t.containerOffset,n=t.containerPosition,a=t.containerElement,o=e(t.helper),r=o.offset(),h=o.outerWidth()-t.sizeDiff.width,l=o.outerHeight()-t.sizeDiff.height;t._helper&&!i.animate&&/relative/.test(a.css("position"))&&e(this).css({left:r.left-n.left-s.left,width:h,height:l}),t._helper&&!i.animate&&/static/.test(a.css("position"))&&e(this).css({left:r.left-n.left-s.left,width:h,height:l})}}),e.ui.plugin.add("resizable","alsoResize",{start:function(){var t=e(this).resizable("instance"),i=t.options;e(i.alsoResize).each(function(){var t=e(this);t.data("ui-resizable-alsoresize",{width:parseInt(t.width(),10),height:parseInt(t.height(),10),left:parseInt(t.css("left"),10),top:parseInt(t.css("top"),10)})})},resize:function(t,i){var s=e(this).resizable("instance"),n=s.options,a=s.originalSize,o=s.originalPosition,r={height:s.size.height-a.height||0,width:s.size.width-a.width||0,top:s.position.top-o.top||0,left:s.position.left-o.left||0};e(n.alsoResize).each(function(){var t=e(this),s=e(this).data("ui-resizable-alsoresize"),n={},a=t.parents(i.originalElement[0]).length?["width","height"]:["width","height","top","left"];e.each(a,function(e,t){var i=(s[t]||0)+(r[t]||0);i&&i>=0&&(n[t]=i||null)}),t.css(n)})},stop:function(){e(this).removeData("resizable-alsoresize")}}),e.ui.plugin.add("resizable","ghost",{start:function(){var t=e(this).resizable("instance"),i=t.options,s=t.size;t.ghost=t.originalElement.clone(),t.ghost.css({opacity:.25,display:"block",position:"relative",height:s.height,width:s.width,margin:0,left:0,top:0}).addClass("ui-resizable-ghost").addClass("string"==typeof i.ghost?i.ghost:""),t.ghost.appendTo(t.helper)},resize:function(){var t=e(this).resizable("instance");t.ghost&&t.ghost.css({position:"relative",height:t.size.height,width:t.size.width})},stop:function(){var t=e(this).resizable("instance");t.ghost&&t.helper&&t.helper.get(0).removeChild(t.ghost.get(0))}}),e.ui.plugin.add("resizable","grid",{resize:function(){var t,i=e(this).resizable("instance"),s=i.options,n=i.size,a=i.originalSize,o=i.originalPosition,r=i.axis,h="number"==typeof s.grid?[s.grid,s.grid]:s.grid,l=h[0]||1,u=h[1]||1,d=Math.round((n.width-a.width)/l)*l,c=Math.round((n.height-a.height)/u)*u,p=a.width+d,f=a.height+c,m=s.maxWidth&&p>s.maxWidth,g=s.maxHeight&&f>s.maxHeight,v=s.minWidth&&s.minWidth>p,y=s.minHeight&&s.minHeight>f;s.grid=h,v&&(p+=l),y&&(f+=u),m&&(p-=l),g&&(f-=u),/^(se|s|e)$/.test(r)?(i.size.width=p,i.size.height=f):/^(ne)$/.test(r)?(i.size.width=p,i.size.height=f,i.position.top=o.top-c):/^(sw)$/.test(r)?(i.size.width=p,i.size.height=f,i.position.left=o.left-d):((0>=f-u||0>=p-l)&&(t=i._getPaddingPlusBorderDimensions(this)),f-u>0?(i.size.height=f,i.position.top=o.top-c):(f=u-t.height,i.size.height=f,i.position.top=o.top+a.height-f),p-l>0?(i.size.width=p,i.position.left=o.left-d):(p=l-t.width,i.size.width=p,i.position.left=o.left+a.width-p))}}),e.ui.resizable,e.widget("ui.selectable",e.ui.mouse,{version:"1.11.4",options:{appendTo:"body",autoRefresh:!0,distance:0,filter:"*",tolerance:"touch",selected:null,selecting:null,start:null,stop:null,unselected:null,unselecting:null},_create:function(){var t,i=this;this.element.addClass("ui-selectable"),this.dragged=!1,this.refresh=function(){t=e(i.options.filter,i.element[0]),t.addClass("ui-selectee"),t.each(function(){var t=e(this),i=t.offset();e.data(this,"selectable-item",{element:this,$element:t,left:i.left,top:i.top,right:i.left+t.outerWidth(),bottom:i.top+t.outerHeight(),startselected:!1,selected:t.hasClass("ui-selected"),selecting:t.hasClass("ui-selecting"),unselecting:t.hasClass("ui-unselecting")})})},this.refresh(),this.selectees=t.addClass("ui-selectee"),this._mouseInit(),this.helper=e("<div class='ui-selectable-helper'></div>")},_destroy:function(){this.selectees.removeClass("ui-selectee").removeData("selectable-item"),this.element.removeClass("ui-selectable ui-selectable-disabled"),this._mouseDestroy()},_mouseStart:function(t){var i=this,s=this.options;this.opos=[t.pageX,t.pageY],this.options.disabled||(this.selectees=e(s.filter,this.element[0]),this._trigger("start",t),e(s.appendTo).append(this.helper),this.helper.css({left:t.pageX,top:t.pageY,width:0,height:0}),s.autoRefresh&&this.refresh(),this.selectees.filter(".ui-selected").each(function(){var s=e.data(this,"selectable-item");s.startselected=!0,t.metaKey||t.ctrlKey||(s.$element.removeClass("ui-selected"),s.selected=!1,s.$element.addClass("ui-unselecting"),s.unselecting=!0,i._trigger("unselecting",t,{unselecting:s.element}))}),e(t.target).parents().addBack().each(function(){var s,n=e.data(this,"selectable-item");return n?(s=!t.metaKey&&!t.ctrlKey||!n.$element.hasClass("ui-selected"),n.$element.removeClass(s?"ui-unselecting":"ui-selected").addClass(s?"ui-selecting":"ui-unselecting"),n.unselecting=!s,n.selecting=s,n.selected=s,s?i._trigger("selecting",t,{selecting:n.element}):i._trigger("unselecting",t,{unselecting:n.element}),!1):void 0}))},_mouseDrag:function(t){if(this.dragged=!0,!this.options.disabled){var i,s=this,n=this.options,a=this.opos[0],o=this.opos[1],r=t.pageX,h=t.pageY;return a>r&&(i=r,r=a,a=i),o>h&&(i=h,h=o,o=i),this.helper.css({left:a,top:o,width:r-a,height:h-o}),this.selectees.each(function(){var i=e.data(this,"selectable-item"),l=!1;
+i&&i.element!==s.element[0]&&("touch"===n.tolerance?l=!(i.left>r||a>i.right||i.top>h||o>i.bottom):"fit"===n.tolerance&&(l=i.left>a&&r>i.right&&i.top>o&&h>i.bottom),l?(i.selected&&(i.$element.removeClass("ui-selected"),i.selected=!1),i.unselecting&&(i.$element.removeClass("ui-unselecting"),i.unselecting=!1),i.selecting||(i.$element.addClass("ui-selecting"),i.selecting=!0,s._trigger("selecting",t,{selecting:i.element}))):(i.selecting&&((t.metaKey||t.ctrlKey)&&i.startselected?(i.$element.removeClass("ui-selecting"),i.selecting=!1,i.$element.addClass("ui-selected"),i.selected=!0):(i.$element.removeClass("ui-selecting"),i.selecting=!1,i.startselected&&(i.$element.addClass("ui-unselecting"),i.unselecting=!0),s._trigger("unselecting",t,{unselecting:i.element}))),i.selected&&(t.metaKey||t.ctrlKey||i.startselected||(i.$element.removeClass("ui-selected"),i.selected=!1,i.$element.addClass("ui-unselecting"),i.unselecting=!0,s._trigger("unselecting",t,{unselecting:i.element})))))}),!1}},_mouseStop:function(t){var i=this;return this.dragged=!1,e(".ui-unselecting",this.element[0]).each(function(){var s=e.data(this,"selectable-item");s.$element.removeClass("ui-unselecting"),s.unselecting=!1,s.startselected=!1,i._trigger("unselected",t,{unselected:s.element})}),e(".ui-selecting",this.element[0]).each(function(){var s=e.data(this,"selectable-item");s.$element.removeClass("ui-selecting").addClass("ui-selected"),s.selecting=!1,s.selected=!0,s.startselected=!0,i._trigger("selected",t,{selected:s.element})}),this._trigger("stop",t),this.helper.remove(),!1}}),e.widget("ui.sortable",e.ui.mouse,{version:"1.11.4",widgetEventPrefix:"sort",ready:!1,options:{appendTo:"parent",axis:!1,connectWith:!1,containment:!1,cursor:"auto",cursorAt:!1,dropOnEmpty:!0,forcePlaceholderSize:!1,forceHelperSize:!1,grid:!1,handle:!1,helper:"original",items:"> *",opacity:!1,placeholder:!1,revert:!1,scroll:!0,scrollSensitivity:20,scrollSpeed:20,scope:"default",tolerance:"intersect",zIndex:1e3,activate:null,beforeStop:null,change:null,deactivate:null,out:null,over:null,receive:null,remove:null,sort:null,start:null,stop:null,update:null},_isOverAxis:function(e,t,i){return e>=t&&t+i>e},_isFloating:function(e){return/left|right/.test(e.css("float"))||/inline|table-cell/.test(e.css("display"))},_create:function(){this.containerCache={},this.element.addClass("ui-sortable"),this.refresh(),this.offset=this.element.offset(),this._mouseInit(),this._setHandleClassName(),this.ready=!0},_setOption:function(e,t){this._super(e,t),"handle"===e&&this._setHandleClassName()},_setHandleClassName:function(){this.element.find(".ui-sortable-handle").removeClass("ui-sortable-handle"),e.each(this.items,function(){(this.instance.options.handle?this.item.find(this.instance.options.handle):this.item).addClass("ui-sortable-handle")})},_destroy:function(){this.element.removeClass("ui-sortable ui-sortable-disabled").find(".ui-sortable-handle").removeClass("ui-sortable-handle"),this._mouseDestroy();for(var e=this.items.length-1;e>=0;e--)this.items[e].item.removeData(this.widgetName+"-item");return this},_mouseCapture:function(t,i){var s=null,n=!1,a=this;return this.reverting?!1:this.options.disabled||"static"===this.options.type?!1:(this._refreshItems(t),e(t.target).parents().each(function(){return e.data(this,a.widgetName+"-item")===a?(s=e(this),!1):void 0}),e.data(t.target,a.widgetName+"-item")===a&&(s=e(t.target)),s?!this.options.handle||i||(e(this.options.handle,s).find("*").addBack().each(function(){this===t.target&&(n=!0)}),n)?(this.currentItem=s,this._removeCurrentsFromItems(),!0):!1:!1)},_mouseStart:function(t,i,s){var n,a,o=this.options;if(this.currentContainer=this,this.refreshPositions(),this.helper=this._createHelper(t),this._cacheHelperProportions(),this._cacheMargins(),this.scrollParent=this.helper.scrollParent(),this.offset=this.currentItem.offset(),this.offset={top:this.offset.top-this.margins.top,left:this.offset.left-this.margins.left},e.extend(this.offset,{click:{left:t.pageX-this.offset.left,top:t.pageY-this.offset.top},parent:this._getParentOffset(),relative:this._getRelativeOffset()}),this.helper.css("position","absolute"),this.cssPosition=this.helper.css("position"),this.originalPosition=this._generatePosition(t),this.originalPageX=t.pageX,this.originalPageY=t.pageY,o.cursorAt&&this._adjustOffsetFromHelper(o.cursorAt),this.domPosition={prev:this.currentItem.prev()[0],parent:this.currentItem.parent()[0]},this.helper[0]!==this.currentItem[0]&&this.currentItem.hide(),this._createPlaceholder(),o.containment&&this._setContainment(),o.cursor&&"auto"!==o.cursor&&(a=this.document.find("body"),this.storedCursor=a.css("cursor"),a.css("cursor",o.cursor),this.storedStylesheet=e("<style>*{ cursor: "+o.cursor+" !important; }</style>").appendTo(a)),o.opacity&&(this.helper.css("opacity")&&(this._storedOpacity=this.helper.css("opacity")),this.helper.css("opacity",o.opacity)),o.zIndex&&(this.helper.css("zIndex")&&(this._storedZIndex=this.helper.css("zIndex")),this.helper.css("zIndex",o.zIndex)),this.scrollParent[0]!==this.document[0]&&"HTML"!==this.scrollParent[0].tagName&&(this.overflowOffset=this.scrollParent.offset()),this._trigger("start",t,this._uiHash()),this._preserveHelperProportions||this._cacheHelperProportions(),!s)for(n=this.containers.length-1;n>=0;n--)this.containers[n]._trigger("activate",t,this._uiHash(this));return e.ui.ddmanager&&(e.ui.ddmanager.current=this),e.ui.ddmanager&&!o.dropBehaviour&&e.ui.ddmanager.prepareOffsets(this,t),this.dragging=!0,this.helper.addClass("ui-sortable-helper"),this._mouseDrag(t),!0},_mouseDrag:function(t){var i,s,n,a,o=this.options,r=!1;for(this.position=this._generatePosition(t),this.positionAbs=this._convertPositionTo("absolute"),this.lastPositionAbs||(this.lastPositionAbs=this.positionAbs),this.options.scroll&&(this.scrollParent[0]!==this.document[0]&&"HTML"!==this.scrollParent[0].tagName?(this.overflowOffset.top+this.scrollParent[0].offsetHeight-t.pageY<o.scrollSensitivity?this.scrollParent[0].scrollTop=r=this.scrollParent[0].scrollTop+o.scrollSpeed:t.pageY-this.overflowOffset.top<o.scrollSensitivity&&(this.scrollParent[0].scrollTop=r=this.scrollParent[0].scrollTop-o.scrollSpeed),this.overflowOffset.left+this.scrollParent[0].offsetWidth-t.pageX<o.scrollSensitivity?this.scrollParent[0].scrollLeft=r=this.scrollParent[0].scrollLeft+o.scrollSpeed:t.pageX-this.overflowOffset.left<o.scrollSensitivity&&(this.scrollParent[0].scrollLeft=r=this.scrollParent[0].scrollLeft-o.scrollSpeed)):(t.pageY-this.document.scrollTop()<o.scrollSensitivity?r=this.document.scrollTop(this.document.scrollTop()-o.scrollSpeed):this.window.height()-(t.pageY-this.document.scrollTop())<o.scrollSensitivity&&(r=this.document.scrollTop(this.document.scrollTop()+o.scrollSpeed)),t.pageX-this.document.scrollLeft()<o.scrollSensitivity?r=this.document.scrollLeft(this.document.scrollLeft()-o.scrollSpeed):this.window.width()-(t.pageX-this.document.scrollLeft())<o.scrollSensitivity&&(r=this.document.scrollLeft(this.document.scrollLeft()+o.scrollSpeed))),r!==!1&&e.ui.ddmanager&&!o.dropBehaviour&&e.ui.ddmanager.prepareOffsets(this,t)),this.positionAbs=this._convertPositionTo("absolute"),this.options.axis&&"y"===this.options.axis||(this.helper[0].style.left=this.position.left+"px"),this.options.axis&&"x"===this.options.axis||(this.helper[0].style.top=this.position.top+"px"),i=this.items.length-1;i>=0;i--)if(s=this.items[i],n=s.item[0],a=this._intersectsWithPointer(s),a&&s.instance===this.currentContainer&&n!==this.currentItem[0]&&this.placeholder[1===a?"next":"prev"]()[0]!==n&&!e.contains(this.placeholder[0],n)&&("semi-dynamic"===this.options.type?!e.contains(this.element[0],n):!0)){if(this.direction=1===a?"down":"up","pointer"!==this.options.tolerance&&!this._intersectsWithSides(s))break;this._rearrange(t,s),this._trigger("change",t,this._uiHash());break}return this._contactContainers(t),e.ui.ddmanager&&e.ui.ddmanager.drag(this,t),this._trigger("sort",t,this._uiHash()),this.lastPositionAbs=this.positionAbs,!1},_mouseStop:function(t,i){if(t){if(e.ui.ddmanager&&!this.options.dropBehaviour&&e.ui.ddmanager.drop(this,t),this.options.revert){var s=this,n=this.placeholder.offset(),a=this.options.axis,o={};a&&"x"!==a||(o.left=n.left-this.offset.parent.left-this.margins.left+(this.offsetParent[0]===this.document[0].body?0:this.offsetParent[0].scrollLeft)),a&&"y"!==a||(o.top=n.top-this.offset.parent.top-this.margins.top+(this.offsetParent[0]===this.document[0].body?0:this.offsetParent[0].scrollTop)),this.reverting=!0,e(this.helper).animate(o,parseInt(this.options.revert,10)||500,function(){s._clear(t)})}else this._clear(t,i);return!1}},cancel:function(){if(this.dragging){this._mouseUp({target:null}),"original"===this.options.helper?this.currentItem.css(this._storedCSS).removeClass("ui-sortable-helper"):this.currentItem.show();for(var t=this.containers.length-1;t>=0;t--)this.containers[t]._trigger("deactivate",null,this._uiHash(this)),this.containers[t].containerCache.over&&(this.containers[t]._trigger("out",null,this._uiHash(this)),this.containers[t].containerCache.over=0)}return this.placeholder&&(this.placeholder[0].parentNode&&this.placeholder[0].parentNode.removeChild(this.placeholder[0]),"original"!==this.options.helper&&this.helper&&this.helper[0].parentNode&&this.helper.remove(),e.extend(this,{helper:null,dragging:!1,reverting:!1,_noFinalSort:null}),this.domPosition.prev?e(this.domPosition.prev).after(this.currentItem):e(this.domPosition.parent).prepend(this.currentItem)),this},serialize:function(t){var i=this._getItemsAsjQuery(t&&t.connected),s=[];return t=t||{},e(i).each(function(){var i=(e(t.item||this).attr(t.attribute||"id")||"").match(t.expression||/(.+)[\-=_](.+)/);i&&s.push((t.key||i[1]+"[]")+"="+(t.key&&t.expression?i[1]:i[2]))}),!s.length&&t.key&&s.push(t.key+"="),s.join("&")},toArray:function(t){var i=this._getItemsAsjQuery(t&&t.connected),s=[];return t=t||{},i.each(function(){s.push(e(t.item||this).attr(t.attribute||"id")||"")}),s},_intersectsWith:function(e){var t=this.positionAbs.left,i=t+this.helperProportions.width,s=this.positionAbs.top,n=s+this.helperProportions.height,a=e.left,o=a+e.width,r=e.top,h=r+e.height,l=this.offset.click.top,u=this.offset.click.left,d="x"===this.options.axis||s+l>r&&h>s+l,c="y"===this.options.axis||t+u>a&&o>t+u,p=d&&c;return"pointer"===this.options.tolerance||this.options.forcePointerForContainers||"pointer"!==this.options.tolerance&&this.helperProportions[this.floating?"width":"height"]>e[this.floating?"width":"height"]?p:t+this.helperProportions.width/2>a&&o>i-this.helperProportions.width/2&&s+this.helperProportions.height/2>r&&h>n-this.helperProportions.height/2},_intersectsWithPointer:function(e){var t="x"===this.options.axis||this._isOverAxis(this.positionAbs.top+this.offset.click.top,e.top,e.height),i="y"===this.options.axis||this._isOverAxis(this.positionAbs.left+this.offset.click.left,e.left,e.width),s=t&&i,n=this._getDragVerticalDirection(),a=this._getDragHorizontalDirection();return s?this.floating?a&&"right"===a||"down"===n?2:1:n&&("down"===n?2:1):!1},_intersectsWithSides:function(e){var t=this._isOverAxis(this.positionAbs.top+this.offset.click.top,e.top+e.height/2,e.height),i=this._isOverAxis(this.positionAbs.left+this.offset.click.left,e.left+e.width/2,e.width),s=this._getDragVerticalDirection(),n=this._getDragHorizontalDirection();return this.floating&&n?"right"===n&&i||"left"===n&&!i:s&&("down"===s&&t||"up"===s&&!t)},_getDragVerticalDirection:function(){var e=this.positionAbs.top-this.lastPositionAbs.top;return 0!==e&&(e>0?"down":"up")},_getDragHorizontalDirection:function(){var e=this.positionAbs.left-this.lastPositionAbs.left;return 0!==e&&(e>0?"right":"left")},refresh:function(e){return this._refreshItems(e),this._setHandleClassName(),this.refreshPositions(),this},_connectWith:function(){var e=this.options;return e.connectWith.constructor===String?[e.connectWith]:e.connectWith},_getItemsAsjQuery:function(t){function i(){r.push(this)}var s,n,a,o,r=[],h=[],l=this._connectWith();if(l&&t)for(s=l.length-1;s>=0;s--)for(a=e(l[s],this.document[0]),n=a.length-1;n>=0;n--)o=e.data(a[n],this.widgetFullName),o&&o!==this&&!o.options.disabled&&h.push([e.isFunction(o.options.items)?o.options.items.call(o.element):e(o.options.items,o.element).not(".ui-sortable-helper").not(".ui-sortable-placeholder"),o]);for(h.push([e.isFunction(this.options.items)?this.options.items.call(this.element,null,{options:this.options,item:this.currentItem}):e(this.options.items,this.element).not(".ui-sortable-helper").not(".ui-sortable-placeholder"),this]),s=h.length-1;s>=0;s--)h[s][0].each(i);return e(r)},_removeCurrentsFromItems:function(){var t=this.currentItem.find(":data("+this.widgetName+"-item)");this.items=e.grep(this.items,function(e){for(var i=0;t.length>i;i++)if(t[i]===e.item[0])return!1;return!0})},_refreshItems:function(t){this.items=[],this.containers=[this];var i,s,n,a,o,r,h,l,u=this.items,d=[[e.isFunction(this.options.items)?this.options.items.call(this.element[0],t,{item:this.currentItem}):e(this.options.items,this.element),this]],c=this._connectWith();if(c&&this.ready)for(i=c.length-1;i>=0;i--)for(n=e(c[i],this.document[0]),s=n.length-1;s>=0;s--)a=e.data(n[s],this.widgetFullName),a&&a!==this&&!a.options.disabled&&(d.push([e.isFunction(a.options.items)?a.options.items.call(a.element[0],t,{item:this.currentItem}):e(a.options.items,a.element),a]),this.containers.push(a));for(i=d.length-1;i>=0;i--)for(o=d[i][1],r=d[i][0],s=0,l=r.length;l>s;s++)h=e(r[s]),h.data(this.widgetName+"-item",o),u.push({item:h,instance:o,width:0,height:0,left:0,top:0})},refreshPositions:function(t){this.floating=this.items.length?"x"===this.options.axis||this._isFloating(this.items[0].item):!1,this.offsetParent&&this.helper&&(this.offset.parent=this._getParentOffset());var i,s,n,a;for(i=this.items.length-1;i>=0;i--)s=this.items[i],s.instance!==this.currentContainer&&this.currentContainer&&s.item[0]!==this.currentItem[0]||(n=this.options.toleranceElement?e(this.options.toleranceElement,s.item):s.item,t||(s.width=n.outerWidth(),s.height=n.outerHeight()),a=n.offset(),s.left=a.left,s.top=a.top);if(this.options.custom&&this.options.custom.refreshContainers)this.options.custom.refreshContainers.call(this);else for(i=this.containers.length-1;i>=0;i--)a=this.containers[i].element.offset(),this.containers[i].containerCache.left=a.left,this.containers[i].containerCache.top=a.top,this.containers[i].containerCache.width=this.containers[i].element.outerWidth(),this.containers[i].containerCache.height=this.containers[i].element.outerHeight();return this},_createPlaceholder:function(t){t=t||this;var i,s=t.options;s.placeholder&&s.placeholder.constructor!==String||(i=s.placeholder,s.placeholder={element:function(){var s=t.currentItem[0].nodeName.toLowerCase(),n=e("<"+s+">",t.document[0]).addClass(i||t.currentItem[0].className+" ui-sortable-placeholder").removeClass("ui-sortable-helper");return"tbody"===s?t._createTrPlaceholder(t.currentItem.find("tr").eq(0),e("<tr>",t.document[0]).appendTo(n)):"tr"===s?t._createTrPlaceholder(t.currentItem,n):"img"===s&&n.attr("src",t.currentItem.attr("src")),i||n.css("visibility","hidden"),n},update:function(e,n){(!i||s.forcePlaceholderSize)&&(n.height()||n.height(t.currentItem.innerHeight()-parseInt(t.currentItem.css("paddingTop")||0,10)-parseInt(t.currentItem.css("paddingBottom")||0,10)),n.width()||n.width(t.currentItem.innerWidth()-parseInt(t.currentItem.css("paddingLeft")||0,10)-parseInt(t.currentItem.css("paddingRight")||0,10)))}}),t.placeholder=e(s.placeholder.element.call(t.element,t.currentItem)),t.currentItem.after(t.placeholder),s.placeholder.update(t,t.placeholder)},_createTrPlaceholder:function(t,i){var s=this;t.children().each(function(){e("<td>&#160;</td>",s.document[0]).attr("colspan",e(this).attr("colspan")||1).appendTo(i)})},_contactContainers:function(t){var i,s,n,a,o,r,h,l,u,d,c=null,p=null;for(i=this.containers.length-1;i>=0;i--)if(!e.contains(this.currentItem[0],this.containers[i].element[0]))if(this._intersectsWith(this.containers[i].containerCache)){if(c&&e.contains(this.containers[i].element[0],c.element[0]))continue;c=this.containers[i],p=i}else this.containers[i].containerCache.over&&(this.containers[i]._trigger("out",t,this._uiHash(this)),this.containers[i].containerCache.over=0);if(c)if(1===this.containers.length)this.containers[p].containerCache.over||(this.containers[p]._trigger("over",t,this._uiHash(this)),this.containers[p].containerCache.over=1);else{for(n=1e4,a=null,u=c.floating||this._isFloating(this.currentItem),o=u?"left":"top",r=u?"width":"height",d=u?"clientX":"clientY",s=this.items.length-1;s>=0;s--)e.contains(this.containers[p].element[0],this.items[s].item[0])&&this.items[s].item[0]!==this.currentItem[0]&&(h=this.items[s].item.offset()[o],l=!1,t[d]-h>this.items[s][r]/2&&(l=!0),n>Math.abs(t[d]-h)&&(n=Math.abs(t[d]-h),a=this.items[s],this.direction=l?"up":"down"));if(!a&&!this.options.dropOnEmpty)return;if(this.currentContainer===this.containers[p])return this.currentContainer.containerCache.over||(this.containers[p]._trigger("over",t,this._uiHash()),this.currentContainer.containerCache.over=1),void 0;a?this._rearrange(t,a,null,!0):this._rearrange(t,null,this.containers[p].element,!0),this._trigger("change",t,this._uiHash()),this.containers[p]._trigger("change",t,this._uiHash(this)),this.currentContainer=this.containers[p],this.options.placeholder.update(this.currentContainer,this.placeholder),this.containers[p]._trigger("over",t,this._uiHash(this)),this.containers[p].containerCache.over=1}},_createHelper:function(t){var i=this.options,s=e.isFunction(i.helper)?e(i.helper.apply(this.element[0],[t,this.currentItem])):"clone"===i.helper?this.currentItem.clone():this.currentItem;return s.parents("body").length||e("parent"!==i.appendTo?i.appendTo:this.currentItem[0].parentNode)[0].appendChild(s[0]),s[0]===this.currentItem[0]&&(this._storedCSS={width:this.currentItem[0].style.width,height:this.currentItem[0].style.height,position:this.currentItem.css("position"),top:this.currentItem.css("top"),left:this.currentItem.css("left")}),(!s[0].style.width||i.forceHelperSize)&&s.width(this.currentItem.width()),(!s[0].style.height||i.forceHelperSize)&&s.height(this.currentItem.height()),s},_adjustOffsetFromHelper:function(t){"string"==typeof t&&(t=t.split(" ")),e.isArray(t)&&(t={left:+t[0],top:+t[1]||0}),"left"in t&&(this.offset.click.left=t.left+this.margins.left),"right"in t&&(this.offset.click.left=this.helperProportions.width-t.right+this.margins.left),"top"in t&&(this.offset.click.top=t.top+this.margins.top),"bottom"in t&&(this.offset.click.top=this.helperProportions.height-t.bottom+this.margins.top)},_getParentOffset:function(){this.offsetParent=this.helper.offsetParent();var t=this.offsetParent.offset();return"absolute"===this.cssPosition&&this.scrollParent[0]!==this.document[0]&&e.contains(this.scrollParent[0],this.offsetParent[0])&&(t.left+=this.scrollParent.scrollLeft(),t.top+=this.scrollParent.scrollTop()),(this.offsetParent[0]===this.document[0].body||this.offsetParent[0].tagName&&"html"===this.offsetParent[0].tagName.toLowerCase()&&e.ui.ie)&&(t={top:0,left:0}),{top:t.top+(parseInt(this.offsetParent.css("borderTopWidth"),10)||0),left:t.left+(parseInt(this.offsetParent.css("borderLeftWidth"),10)||0)}},_getRelativeOffset:function(){if("relative"===this.cssPosition){var e=this.currentItem.position();return{top:e.top-(parseInt(this.helper.css("top"),10)||0)+this.scrollParent.scrollTop(),left:e.left-(parseInt(this.helper.css("left"),10)||0)+this.scrollParent.scrollLeft()}}return{top:0,left:0}},_cacheMargins:function(){this.margins={left:parseInt(this.currentItem.css("marginLeft"),10)||0,top:parseInt(this.currentItem.css("marginTop"),10)||0}},_cacheHelperProportions:function(){this.helperProportions={width:this.helper.outerWidth(),height:this.helper.outerHeight()}},_setContainment:function(){var t,i,s,n=this.options;"parent"===n.containment&&(n.containment=this.helper[0].parentNode),("document"===n.containment||"window"===n.containment)&&(this.containment=[0-this.offset.relative.left-this.offset.parent.left,0-this.offset.relative.top-this.offset.parent.top,"document"===n.containment?this.document.width():this.window.width()-this.helperProportions.width-this.margins.left,("document"===n.containment?this.document.width():this.window.height()||this.document[0].body.parentNode.scrollHeight)-this.helperProportions.height-this.margins.top]),/^(document|window|parent)$/.test(n.containment)||(t=e(n.containment)[0],i=e(n.containment).offset(),s="hidden"!==e(t).css("overflow"),this.containment=[i.left+(parseInt(e(t).css("borderLeftWidth"),10)||0)+(parseInt(e(t).css("paddingLeft"),10)||0)-this.margins.left,i.top+(parseInt(e(t).css("borderTopWidth"),10)||0)+(parseInt(e(t).css("paddingTop"),10)||0)-this.margins.top,i.left+(s?Math.max(t.scrollWidth,t.offsetWidth):t.offsetWidth)-(parseInt(e(t).css("borderLeftWidth"),10)||0)-(parseInt(e(t).css("paddingRight"),10)||0)-this.helperProportions.width-this.margins.left,i.top+(s?Math.max(t.scrollHeight,t.offsetHeight):t.offsetHeight)-(parseInt(e(t).css("borderTopWidth"),10)||0)-(parseInt(e(t).css("paddingBottom"),10)||0)-this.helperProportions.height-this.margins.top])},_convertPositionTo:function(t,i){i||(i=this.position);var s="absolute"===t?1:-1,n="absolute"!==this.cssPosition||this.scrollParent[0]!==this.document[0]&&e.contains(this.scrollParent[0],this.offsetParent[0])?this.scrollParent:this.offsetParent,a=/(html|body)/i.test(n[0].tagName);return{top:i.top+this.offset.relative.top*s+this.offset.parent.top*s-("fixed"===this.cssPosition?-this.scrollParent.scrollTop():a?0:n.scrollTop())*s,left:i.left+this.offset.relative.left*s+this.offset.parent.left*s-("fixed"===this.cssPosition?-this.scrollParent.scrollLeft():a?0:n.scrollLeft())*s}},_generatePosition:function(t){var i,s,n=this.options,a=t.pageX,o=t.pageY,r="absolute"!==this.cssPosition||this.scrollParent[0]!==this.document[0]&&e.contains(this.scrollParent[0],this.offsetParent[0])?this.scrollParent:this.offsetParent,h=/(html|body)/i.test(r[0].tagName);return"relative"!==this.cssPosition||this.scrollParent[0]!==this.document[0]&&this.scrollParent[0]!==this.offsetParent[0]||(this.offset.relative=this._getRelativeOffset()),this.originalPosition&&(this.containment&&(t.pageX-this.offset.click.left<this.containment[0]&&(a=this.containment[0]+this.offset.click.left),t.pageY-this.offset.click.top<this.containment[1]&&(o=this.containment[1]+this.offset.click.top),t.pageX-this.offset.click.left>this.containment[2]&&(a=this.containment[2]+this.offset.click.left),t.pageY-this.offset.click.top>this.containment[3]&&(o=this.containment[3]+this.offset.click.top)),n.grid&&(i=this.originalPageY+Math.round((o-this.originalPageY)/n.grid[1])*n.grid[1],o=this.containment?i-this.offset.click.top>=this.containment[1]&&i-this.offset.click.top<=this.containment[3]?i:i-this.offset.click.top>=this.containment[1]?i-n.grid[1]:i+n.grid[1]:i,s=this.originalPageX+Math.round((a-this.originalPageX)/n.grid[0])*n.grid[0],a=this.containment?s-this.offset.click.left>=this.containment[0]&&s-this.offset.click.left<=this.containment[2]?s:s-this.offset.click.left>=this.containment[0]?s-n.grid[0]:s+n.grid[0]:s)),{top:o-this.offset.click.top-this.offset.relative.top-this.offset.parent.top+("fixed"===this.cssPosition?-this.scrollParent.scrollTop():h?0:r.scrollTop()),left:a-this.offset.click.left-this.offset.relative.left-this.offset.parent.left+("fixed"===this.cssPosition?-this.scrollParent.scrollLeft():h?0:r.scrollLeft())}},_rearrange:function(e,t,i,s){i?i[0].appendChild(this.placeholder[0]):t.item[0].parentNode.insertBefore(this.placeholder[0],"down"===this.direction?t.item[0]:t.item[0].nextSibling),this.counter=this.counter?++this.counter:1;var n=this.counter;this._delay(function(){n===this.counter&&this.refreshPositions(!s)})},_clear:function(e,t){function i(e,t,i){return function(s){i._trigger(e,s,t._uiHash(t))}}this.reverting=!1;var s,n=[];if(!this._noFinalSort&&this.currentItem.parent().length&&this.placeholder.before(this.currentItem),this._noFinalSort=null,this.helper[0]===this.currentItem[0]){for(s in this._storedCSS)("auto"===this._storedCSS[s]||"static"===this._storedCSS[s])&&(this._storedCSS[s]="");this.currentItem.css(this._storedCSS).removeClass("ui-sortable-helper")}else this.currentItem.show();for(this.fromOutside&&!t&&n.push(function(e){this._trigger("receive",e,this._uiHash(this.fromOutside))}),!this.fromOutside&&this.domPosition.prev===this.currentItem.prev().not(".ui-sortable-helper")[0]&&this.domPosition.parent===this.currentItem.parent()[0]||t||n.push(function(e){this._trigger("update",e,this._uiHash())}),this!==this.currentContainer&&(t||(n.push(function(e){this._trigger("remove",e,this._uiHash())}),n.push(function(e){return function(t){e._trigger("receive",t,this._uiHash(this))}}.call(this,this.currentContainer)),n.push(function(e){return function(t){e._trigger("update",t,this._uiHash(this))}}.call(this,this.currentContainer)))),s=this.containers.length-1;s>=0;s--)t||n.push(i("deactivate",this,this.containers[s])),this.containers[s].containerCache.over&&(n.push(i("out",this,this.containers[s])),this.containers[s].containerCache.over=0);if(this.storedCursor&&(this.document.find("body").css("cursor",this.storedCursor),this.storedStylesheet.remove()),this._storedOpacity&&this.helper.css("opacity",this._storedOpacity),this._storedZIndex&&this.helper.css("zIndex","auto"===this._storedZIndex?"":this._storedZIndex),this.dragging=!1,t||this._trigger("beforeStop",e,this._uiHash()),this.placeholder[0].parentNode.removeChild(this.placeholder[0]),this.cancelHelperRemoval||(this.helper[0]!==this.currentItem[0]&&this.helper.remove(),this.helper=null),!t){for(s=0;n.length>s;s++)n[s].call(this,e);this._trigger("stop",e,this._uiHash())}return this.fromOutside=!1,!this.cancelHelperRemoval},_trigger:function(){e.Widget.prototype._trigger.apply(this,arguments)===!1&&this.cancel()},_uiHash:function(t){var i=t||this;return{helper:i.helper,placeholder:i.placeholder||e([]),position:i.position,originalPosition:i.originalPosition,offset:i.positionAbs,item:i.currentItem,sender:t?t.element:null}}}),e.widget("ui.accordion",{version:"1.11.4",options:{active:0,animate:{},collapsible:!1,event:"click",header:"> li > :first-child,> :not(li):even",heightStyle:"auto",icons:{activeHeader:"ui-icon-triangle-1-s",header:"ui-icon-triangle-1-e"},activate:null,beforeActivate:null},hideProps:{borderTopWidth:"hide",borderBottomWidth:"hide",paddingTop:"hide",paddingBottom:"hide",height:"hide"},showProps:{borderTopWidth:"show",borderBottomWidth:"show",paddingTop:"show",paddingBottom:"show",height:"show"},_create:function(){var t=this.options;this.prevShow=this.prevHide=e(),this.element.addClass("ui-accordion ui-widget ui-helper-reset").attr("role","tablist"),t.collapsible||t.active!==!1&&null!=t.active||(t.active=0),this._processPanels(),0>t.active&&(t.active+=this.headers.length),this._refresh()},_getCreateEventData:function(){return{header:this.active,panel:this.active.length?this.active.next():e()}},_createIcons:function(){var t=this.options.icons;t&&(e("<span>").addClass("ui-accordion-header-icon ui-icon "+t.header).prependTo(this.headers),this.active.children(".ui-accordion-header-icon").removeClass(t.header).addClass(t.activeHeader),this.headers.addClass("ui-accordion-icons"))},_destroyIcons:function(){this.headers.removeClass("ui-accordion-icons").children(".ui-accordion-header-icon").remove()},_destroy:function(){var e;this.element.removeClass("ui-accordion ui-widget ui-helper-reset").removeAttr("role"),this.headers.removeClass("ui-accordion-header ui-accordion-header-active ui-state-default ui-corner-all ui-state-active ui-state-disabled ui-corner-top").removeAttr("role").removeAttr("aria-expanded").removeAttr("aria-selected").removeAttr("aria-controls").removeAttr("tabIndex").removeUniqueId(),this._destroyIcons(),e=this.headers.next().removeClass("ui-helper-reset ui-widget-content ui-corner-bottom ui-accordion-content ui-accordion-content-active ui-state-disabled").css("display","").removeAttr("role").removeAttr("aria-hidden").removeAttr("aria-labelledby").removeUniqueId(),"content"!==this.options.heightStyle&&e.css("height","")},_setOption:function(e,t){return"active"===e?(this._activate(t),void 0):("event"===e&&(this.options.event&&this._off(this.headers,this.options.event),this._setupEvents(t)),this._super(e,t),"collapsible"!==e||t||this.options.active!==!1||this._activate(0),"icons"===e&&(this._destroyIcons(),t&&this._createIcons()),"disabled"===e&&(this.element.toggleClass("ui-state-disabled",!!t).attr("aria-disabled",t),this.headers.add(this.headers.next()).toggleClass("ui-state-disabled",!!t)),void 0)},_keydown:function(t){if(!t.altKey&&!t.ctrlKey){var i=e.ui.keyCode,s=this.headers.length,n=this.headers.index(t.target),a=!1;switch(t.keyCode){case i.RIGHT:case i.DOWN:a=this.headers[(n+1)%s];break;case i.LEFT:case i.UP:a=this.headers[(n-1+s)%s];break;case i.SPACE:case i.ENTER:this._eventHandler(t);break;case i.HOME:a=this.headers[0];break;case i.END:a=this.headers[s-1]}a&&(e(t.target).attr("tabIndex",-1),e(a).attr("tabIndex",0),a.focus(),t.preventDefault())}},_panelKeyDown:function(t){t.keyCode===e.ui.keyCode.UP&&t.ctrlKey&&e(t.currentTarget).prev().focus()},refresh:function(){var t=this.options;this._processPanels(),t.active===!1&&t.collapsible===!0||!this.headers.length?(t.active=!1,this.active=e()):t.active===!1?this._activate(0):this.active.length&&!e.contains(this.element[0],this.active[0])?this.headers.length===this.headers.find(".ui-state-disabled").length?(t.active=!1,this.active=e()):this._activate(Math.max(0,t.active-1)):t.active=this.headers.index(this.active),this._destroyIcons(),this._refresh()},_processPanels:function(){var e=this.headers,t=this.panels;this.headers=this.element.find(this.options.header).addClass("ui-accordion-header ui-state-default ui-corner-all"),this.panels=this.headers.next().addClass("ui-accordion-content ui-helper-reset ui-widget-content ui-corner-bottom").filter(":not(.ui-accordion-content-active)").hide(),t&&(this._off(e.not(this.headers)),this._off(t.not(this.panels)))},_refresh:function(){var t,i=this.options,s=i.heightStyle,n=this.element.parent();this.active=this._findActive(i.active).addClass("ui-accordion-header-active ui-state-active ui-corner-top").removeClass("ui-corner-all"),this.active.next().addClass("ui-accordion-content-active").show(),this.headers.attr("role","tab").each(function(){var t=e(this),i=t.uniqueId().attr("id"),s=t.next(),n=s.uniqueId().attr("id");t.attr("aria-controls",n),s.attr("aria-labelledby",i)}).next().attr("role","tabpanel"),this.headers.not(this.active).attr({"aria-selected":"false","aria-expanded":"false",tabIndex:-1}).next().attr({"aria-hidden":"true"}).hide(),this.active.length?this.active.attr({"aria-selected":"true","aria-expanded":"true",tabIndex:0}).next().attr({"aria-hidden":"false"}):this.headers.eq(0).attr("tabIndex",0),this._createIcons(),this._setupEvents(i.event),"fill"===s?(t=n.height(),this.element.siblings(":visible").each(function(){var i=e(this),s=i.css("position");"absolute"!==s&&"fixed"!==s&&(t-=i.outerHeight(!0))}),this.headers.each(function(){t-=e(this).outerHeight(!0)}),this.headers.next().each(function(){e(this).height(Math.max(0,t-e(this).innerHeight()+e(this).height()))}).css("overflow","auto")):"auto"===s&&(t=0,this.headers.next().each(function(){t=Math.max(t,e(this).css("height","").height())}).height(t))},_activate:function(t){var i=this._findActive(t)[0];i!==this.active[0]&&(i=i||this.active[0],this._eventHandler({target:i,currentTarget:i,preventDefault:e.noop}))},_findActive:function(t){return"number"==typeof t?this.headers.eq(t):e()},_setupEvents:function(t){var i={keydown:"_keydown"};t&&e.each(t.split(" "),function(e,t){i[t]="_eventHandler"}),this._off(this.headers.add(this.headers.next())),this._on(this.headers,i),this._on(this.headers.next(),{keydown:"_panelKeyDown"}),this._hoverable(this.headers),this._focusable(this.headers)},_eventHandler:function(t){var i=this.options,s=this.active,n=e(t.currentTarget),a=n[0]===s[0],o=a&&i.collapsible,r=o?e():n.next(),h=s.next(),l={oldHeader:s,oldPanel:h,newHeader:o?e():n,newPanel:r};
+t.preventDefault(),a&&!i.collapsible||this._trigger("beforeActivate",t,l)===!1||(i.active=o?!1:this.headers.index(n),this.active=a?e():n,this._toggle(l),s.removeClass("ui-accordion-header-active ui-state-active"),i.icons&&s.children(".ui-accordion-header-icon").removeClass(i.icons.activeHeader).addClass(i.icons.header),a||(n.removeClass("ui-corner-all").addClass("ui-accordion-header-active ui-state-active ui-corner-top"),i.icons&&n.children(".ui-accordion-header-icon").removeClass(i.icons.header).addClass(i.icons.activeHeader),n.next().addClass("ui-accordion-content-active")))},_toggle:function(t){var i=t.newPanel,s=this.prevShow.length?this.prevShow:t.oldPanel;this.prevShow.add(this.prevHide).stop(!0,!0),this.prevShow=i,this.prevHide=s,this.options.animate?this._animate(i,s,t):(s.hide(),i.show(),this._toggleComplete(t)),s.attr({"aria-hidden":"true"}),s.prev().attr({"aria-selected":"false","aria-expanded":"false"}),i.length&&s.length?s.prev().attr({tabIndex:-1,"aria-expanded":"false"}):i.length&&this.headers.filter(function(){return 0===parseInt(e(this).attr("tabIndex"),10)}).attr("tabIndex",-1),i.attr("aria-hidden","false").prev().attr({"aria-selected":"true","aria-expanded":"true",tabIndex:0})},_animate:function(e,t,i){var s,n,a,o=this,r=0,h=e.css("box-sizing"),l=e.length&&(!t.length||e.index()<t.index()),u=this.options.animate||{},d=l&&u.down||u,c=function(){o._toggleComplete(i)};return"number"==typeof d&&(a=d),"string"==typeof d&&(n=d),n=n||d.easing||u.easing,a=a||d.duration||u.duration,t.length?e.length?(s=e.show().outerHeight(),t.animate(this.hideProps,{duration:a,easing:n,step:function(e,t){t.now=Math.round(e)}}),e.hide().animate(this.showProps,{duration:a,easing:n,complete:c,step:function(e,i){i.now=Math.round(e),"height"!==i.prop?"content-box"===h&&(r+=i.now):"content"!==o.options.heightStyle&&(i.now=Math.round(s-t.outerHeight()-r),r=0)}}),void 0):t.animate(this.hideProps,a,n,c):e.animate(this.showProps,a,n,c)},_toggleComplete:function(e){var t=e.oldPanel;t.removeClass("ui-accordion-content-active").prev().removeClass("ui-corner-top").addClass("ui-corner-all"),t.length&&(t.parent()[0].className=t.parent()[0].className),this._trigger("activate",null,e)}}),e.widget("ui.menu",{version:"1.11.4",defaultElement:"<ul>",delay:300,options:{icons:{submenu:"ui-icon-carat-1-e"},items:"> *",menus:"ul",position:{my:"left-1 top",at:"right top"},role:"menu",blur:null,focus:null,select:null},_create:function(){this.activeMenu=this.element,this.mouseHandled=!1,this.element.uniqueId().addClass("ui-menu ui-widget ui-widget-content").toggleClass("ui-menu-icons",!!this.element.find(".ui-icon").length).attr({role:this.options.role,tabIndex:0}),this.options.disabled&&this.element.addClass("ui-state-disabled").attr("aria-disabled","true"),this._on({"mousedown .ui-menu-item":function(e){e.preventDefault()},"click .ui-menu-item":function(t){var i=e(t.target);!this.mouseHandled&&i.not(".ui-state-disabled").length&&(this.select(t),t.isPropagationStopped()||(this.mouseHandled=!0),i.has(".ui-menu").length?this.expand(t):!this.element.is(":focus")&&e(this.document[0].activeElement).closest(".ui-menu").length&&(this.element.trigger("focus",[!0]),this.active&&1===this.active.parents(".ui-menu").length&&clearTimeout(this.timer)))},"mouseenter .ui-menu-item":function(t){if(!this.previousFilter){var i=e(t.currentTarget);i.siblings(".ui-state-active").removeClass("ui-state-active"),this.focus(t,i)}},mouseleave:"collapseAll","mouseleave .ui-menu":"collapseAll",focus:function(e,t){var i=this.active||this.element.find(this.options.items).eq(0);t||this.focus(e,i)},blur:function(t){this._delay(function(){e.contains(this.element[0],this.document[0].activeElement)||this.collapseAll(t)})},keydown:"_keydown"}),this.refresh(),this._on(this.document,{click:function(e){this._closeOnDocumentClick(e)&&this.collapseAll(e),this.mouseHandled=!1}})},_destroy:function(){this.element.removeAttr("aria-activedescendant").find(".ui-menu").addBack().removeClass("ui-menu ui-widget ui-widget-content ui-menu-icons ui-front").removeAttr("role").removeAttr("tabIndex").removeAttr("aria-labelledby").removeAttr("aria-expanded").removeAttr("aria-hidden").removeAttr("aria-disabled").removeUniqueId().show(),this.element.find(".ui-menu-item").removeClass("ui-menu-item").removeAttr("role").removeAttr("aria-disabled").removeUniqueId().removeClass("ui-state-hover").removeAttr("tabIndex").removeAttr("role").removeAttr("aria-haspopup").children().each(function(){var t=e(this);t.data("ui-menu-submenu-carat")&&t.remove()}),this.element.find(".ui-menu-divider").removeClass("ui-menu-divider ui-widget-content")},_keydown:function(t){var i,s,n,a,o=!0;switch(t.keyCode){case e.ui.keyCode.PAGE_UP:this.previousPage(t);break;case e.ui.keyCode.PAGE_DOWN:this.nextPage(t);break;case e.ui.keyCode.HOME:this._move("first","first",t);break;case e.ui.keyCode.END:this._move("last","last",t);break;case e.ui.keyCode.UP:this.previous(t);break;case e.ui.keyCode.DOWN:this.next(t);break;case e.ui.keyCode.LEFT:this.collapse(t);break;case e.ui.keyCode.RIGHT:this.active&&!this.active.is(".ui-state-disabled")&&this.expand(t);break;case e.ui.keyCode.ENTER:case e.ui.keyCode.SPACE:this._activate(t);break;case e.ui.keyCode.ESCAPE:this.collapse(t);break;default:o=!1,s=this.previousFilter||"",n=String.fromCharCode(t.keyCode),a=!1,clearTimeout(this.filterTimer),n===s?a=!0:n=s+n,i=this._filterMenuItems(n),i=a&&-1!==i.index(this.active.next())?this.active.nextAll(".ui-menu-item"):i,i.length||(n=String.fromCharCode(t.keyCode),i=this._filterMenuItems(n)),i.length?(this.focus(t,i),this.previousFilter=n,this.filterTimer=this._delay(function(){delete this.previousFilter},1e3)):delete this.previousFilter}o&&t.preventDefault()},_activate:function(e){this.active.is(".ui-state-disabled")||(this.active.is("[aria-haspopup='true']")?this.expand(e):this.select(e))},refresh:function(){var t,i,s=this,n=this.options.icons.submenu,a=this.element.find(this.options.menus);this.element.toggleClass("ui-menu-icons",!!this.element.find(".ui-icon").length),a.filter(":not(.ui-menu)").addClass("ui-menu ui-widget ui-widget-content ui-front").hide().attr({role:this.options.role,"aria-hidden":"true","aria-expanded":"false"}).each(function(){var t=e(this),i=t.parent(),s=e("<span>").addClass("ui-menu-icon ui-icon "+n).data("ui-menu-submenu-carat",!0);i.attr("aria-haspopup","true").prepend(s),t.attr("aria-labelledby",i.attr("id"))}),t=a.add(this.element),i=t.find(this.options.items),i.not(".ui-menu-item").each(function(){var t=e(this);s._isDivider(t)&&t.addClass("ui-widget-content ui-menu-divider")}),i.not(".ui-menu-item, .ui-menu-divider").addClass("ui-menu-item").uniqueId().attr({tabIndex:-1,role:this._itemRole()}),i.filter(".ui-state-disabled").attr("aria-disabled","true"),this.active&&!e.contains(this.element[0],this.active[0])&&this.blur()},_itemRole:function(){return{menu:"menuitem",listbox:"option"}[this.options.role]},_setOption:function(e,t){"icons"===e&&this.element.find(".ui-menu-icon").removeClass(this.options.icons.submenu).addClass(t.submenu),"disabled"===e&&this.element.toggleClass("ui-state-disabled",!!t).attr("aria-disabled",t),this._super(e,t)},focus:function(e,t){var i,s;this.blur(e,e&&"focus"===e.type),this._scrollIntoView(t),this.active=t.first(),s=this.active.addClass("ui-state-focus").removeClass("ui-state-active"),this.options.role&&this.element.attr("aria-activedescendant",s.attr("id")),this.active.parent().closest(".ui-menu-item").addClass("ui-state-active"),e&&"keydown"===e.type?this._close():this.timer=this._delay(function(){this._close()},this.delay),i=t.children(".ui-menu"),i.length&&e&&/^mouse/.test(e.type)&&this._startOpening(i),this.activeMenu=t.parent(),this._trigger("focus",e,{item:t})},_scrollIntoView:function(t){var i,s,n,a,o,r;this._hasScroll()&&(i=parseFloat(e.css(this.activeMenu[0],"borderTopWidth"))||0,s=parseFloat(e.css(this.activeMenu[0],"paddingTop"))||0,n=t.offset().top-this.activeMenu.offset().top-i-s,a=this.activeMenu.scrollTop(),o=this.activeMenu.height(),r=t.outerHeight(),0>n?this.activeMenu.scrollTop(a+n):n+r>o&&this.activeMenu.scrollTop(a+n-o+r))},blur:function(e,t){t||clearTimeout(this.timer),this.active&&(this.active.removeClass("ui-state-focus"),this.active=null,this._trigger("blur",e,{item:this.active}))},_startOpening:function(e){clearTimeout(this.timer),"true"===e.attr("aria-hidden")&&(this.timer=this._delay(function(){this._close(),this._open(e)},this.delay))},_open:function(t){var i=e.extend({of:this.active},this.options.position);clearTimeout(this.timer),this.element.find(".ui-menu").not(t.parents(".ui-menu")).hide().attr("aria-hidden","true"),t.show().removeAttr("aria-hidden").attr("aria-expanded","true").position(i)},collapseAll:function(t,i){clearTimeout(this.timer),this.timer=this._delay(function(){var s=i?this.element:e(t&&t.target).closest(this.element.find(".ui-menu"));s.length||(s=this.element),this._close(s),this.blur(t),this.activeMenu=s},this.delay)},_close:function(e){e||(e=this.active?this.active.parent():this.element),e.find(".ui-menu").hide().attr("aria-hidden","true").attr("aria-expanded","false").end().find(".ui-state-active").not(".ui-state-focus").removeClass("ui-state-active")},_closeOnDocumentClick:function(t){return!e(t.target).closest(".ui-menu").length},_isDivider:function(e){return!/[^\-\u2014\u2013\s]/.test(e.text())},collapse:function(e){var t=this.active&&this.active.parent().closest(".ui-menu-item",this.element);t&&t.length&&(this._close(),this.focus(e,t))},expand:function(e){var t=this.active&&this.active.children(".ui-menu ").find(this.options.items).first();t&&t.length&&(this._open(t.parent()),this._delay(function(){this.focus(e,t)}))},next:function(e){this._move("next","first",e)},previous:function(e){this._move("prev","last",e)},isFirstItem:function(){return this.active&&!this.active.prevAll(".ui-menu-item").length},isLastItem:function(){return this.active&&!this.active.nextAll(".ui-menu-item").length},_move:function(e,t,i){var s;this.active&&(s="first"===e||"last"===e?this.active["first"===e?"prevAll":"nextAll"](".ui-menu-item").eq(-1):this.active[e+"All"](".ui-menu-item").eq(0)),s&&s.length&&this.active||(s=this.activeMenu.find(this.options.items)[t]()),this.focus(i,s)},nextPage:function(t){var i,s,n;return this.active?(this.isLastItem()||(this._hasScroll()?(s=this.active.offset().top,n=this.element.height(),this.active.nextAll(".ui-menu-item").each(function(){return i=e(this),0>i.offset().top-s-n}),this.focus(t,i)):this.focus(t,this.activeMenu.find(this.options.items)[this.active?"last":"first"]())),void 0):(this.next(t),void 0)},previousPage:function(t){var i,s,n;return this.active?(this.isFirstItem()||(this._hasScroll()?(s=this.active.offset().top,n=this.element.height(),this.active.prevAll(".ui-menu-item").each(function(){return i=e(this),i.offset().top-s+n>0}),this.focus(t,i)):this.focus(t,this.activeMenu.find(this.options.items).first())),void 0):(this.next(t),void 0)},_hasScroll:function(){return this.element.outerHeight()<this.element.prop("scrollHeight")},select:function(t){this.active=this.active||e(t.target).closest(".ui-menu-item");var i={item:this.active};this.active.has(".ui-menu").length||this.collapseAll(t,!0),this._trigger("select",t,i)},_filterMenuItems:function(t){var i=t.replace(/[\-\[\]{}()*+?.,\\\^$|#\s]/g,"\\$&"),s=RegExp("^"+i,"i");return this.activeMenu.find(this.options.items).filter(".ui-menu-item").filter(function(){return s.test(e.trim(e(this).text()))})}}),e.widget("ui.autocomplete",{version:"1.11.4",defaultElement:"<input>",options:{appendTo:null,autoFocus:!1,delay:300,minLength:1,position:{my:"left top",at:"left bottom",collision:"none"},source:null,change:null,close:null,focus:null,open:null,response:null,search:null,select:null},requestIndex:0,pending:0,_create:function(){var t,i,s,n=this.element[0].nodeName.toLowerCase(),a="textarea"===n,o="input"===n;this.isMultiLine=a?!0:o?!1:this.element.prop("isContentEditable"),this.valueMethod=this.element[a||o?"val":"text"],this.isNewMenu=!0,this.element.addClass("ui-autocomplete-input").attr("autocomplete","off"),this._on(this.element,{keydown:function(n){if(this.element.prop("readOnly"))return t=!0,s=!0,i=!0,void 0;t=!1,s=!1,i=!1;var a=e.ui.keyCode;switch(n.keyCode){case a.PAGE_UP:t=!0,this._move("previousPage",n);break;case a.PAGE_DOWN:t=!0,this._move("nextPage",n);break;case a.UP:t=!0,this._keyEvent("previous",n);break;case a.DOWN:t=!0,this._keyEvent("next",n);break;case a.ENTER:this.menu.active&&(t=!0,n.preventDefault(),this.menu.select(n));break;case a.TAB:this.menu.active&&this.menu.select(n);break;case a.ESCAPE:this.menu.element.is(":visible")&&(this.isMultiLine||this._value(this.term),this.close(n),n.preventDefault());break;default:i=!0,this._searchTimeout(n)}},keypress:function(s){if(t)return t=!1,(!this.isMultiLine||this.menu.element.is(":visible"))&&s.preventDefault(),void 0;if(!i){var n=e.ui.keyCode;switch(s.keyCode){case n.PAGE_UP:this._move("previousPage",s);break;case n.PAGE_DOWN:this._move("nextPage",s);break;case n.UP:this._keyEvent("previous",s);break;case n.DOWN:this._keyEvent("next",s)}}},input:function(e){return s?(s=!1,e.preventDefault(),void 0):(this._searchTimeout(e),void 0)},focus:function(){this.selectedItem=null,this.previous=this._value()},blur:function(e){return this.cancelBlur?(delete this.cancelBlur,void 0):(clearTimeout(this.searching),this.close(e),this._change(e),void 0)}}),this._initSource(),this.menu=e("<ul>").addClass("ui-autocomplete ui-front").appendTo(this._appendTo()).menu({role:null}).hide().menu("instance"),this._on(this.menu.element,{mousedown:function(t){t.preventDefault(),this.cancelBlur=!0,this._delay(function(){delete this.cancelBlur});var i=this.menu.element[0];e(t.target).closest(".ui-menu-item").length||this._delay(function(){var t=this;this.document.one("mousedown",function(s){s.target===t.element[0]||s.target===i||e.contains(i,s.target)||t.close()})})},menufocus:function(t,i){var s,n;return this.isNewMenu&&(this.isNewMenu=!1,t.originalEvent&&/^mouse/.test(t.originalEvent.type))?(this.menu.blur(),this.document.one("mousemove",function(){e(t.target).trigger(t.originalEvent)}),void 0):(n=i.item.data("ui-autocomplete-item"),!1!==this._trigger("focus",t,{item:n})&&t.originalEvent&&/^key/.test(t.originalEvent.type)&&this._value(n.value),s=i.item.attr("aria-label")||n.value,s&&e.trim(s).length&&(this.liveRegion.children().hide(),e("<div>").text(s).appendTo(this.liveRegion)),void 0)},menuselect:function(e,t){var i=t.item.data("ui-autocomplete-item"),s=this.previous;this.element[0]!==this.document[0].activeElement&&(this.element.focus(),this.previous=s,this._delay(function(){this.previous=s,this.selectedItem=i})),!1!==this._trigger("select",e,{item:i})&&this._value(i.value),this.term=this._value(),this.close(e),this.selectedItem=i}}),this.liveRegion=e("<span>",{role:"status","aria-live":"assertive","aria-relevant":"additions"}).addClass("ui-helper-hidden-accessible").appendTo(this.document[0].body),this._on(this.window,{beforeunload:function(){this.element.removeAttr("autocomplete")}})},_destroy:function(){clearTimeout(this.searching),this.element.removeClass("ui-autocomplete-input").removeAttr("autocomplete"),this.menu.element.remove(),this.liveRegion.remove()},_setOption:function(e,t){this._super(e,t),"source"===e&&this._initSource(),"appendTo"===e&&this.menu.element.appendTo(this._appendTo()),"disabled"===e&&t&&this.xhr&&this.xhr.abort()},_appendTo:function(){var t=this.options.appendTo;return t&&(t=t.jquery||t.nodeType?e(t):this.document.find(t).eq(0)),t&&t[0]||(t=this.element.closest(".ui-front")),t.length||(t=this.document[0].body),t},_initSource:function(){var t,i,s=this;e.isArray(this.options.source)?(t=this.options.source,this.source=function(i,s){s(e.ui.autocomplete.filter(t,i.term))}):"string"==typeof this.options.source?(i=this.options.source,this.source=function(t,n){s.xhr&&s.xhr.abort(),s.xhr=e.ajax({url:i,data:t,dataType:"json",success:function(e){n(e)},error:function(){n([])}})}):this.source=this.options.source},_searchTimeout:function(e){clearTimeout(this.searching),this.searching=this._delay(function(){var t=this.term===this._value(),i=this.menu.element.is(":visible"),s=e.altKey||e.ctrlKey||e.metaKey||e.shiftKey;(!t||t&&!i&&!s)&&(this.selectedItem=null,this.search(null,e))},this.options.delay)},search:function(e,t){return e=null!=e?e:this._value(),this.term=this._value(),e.length<this.options.minLength?this.close(t):this._trigger("search",t)!==!1?this._search(e):void 0},_search:function(e){this.pending++,this.element.addClass("ui-autocomplete-loading"),this.cancelSearch=!1,this.source({term:e},this._response())},_response:function(){var t=++this.requestIndex;return e.proxy(function(e){t===this.requestIndex&&this.__response(e),this.pending--,this.pending||this.element.removeClass("ui-autocomplete-loading")},this)},__response:function(e){e&&(e=this._normalize(e)),this._trigger("response",null,{content:e}),!this.options.disabled&&e&&e.length&&!this.cancelSearch?(this._suggest(e),this._trigger("open")):this._close()},close:function(e){this.cancelSearch=!0,this._close(e)},_close:function(e){this.menu.element.is(":visible")&&(this.menu.element.hide(),this.menu.blur(),this.isNewMenu=!0,this._trigger("close",e))},_change:function(e){this.previous!==this._value()&&this._trigger("change",e,{item:this.selectedItem})},_normalize:function(t){return t.length&&t[0].label&&t[0].value?t:e.map(t,function(t){return"string"==typeof t?{label:t,value:t}:e.extend({},t,{label:t.label||t.value,value:t.value||t.label})})},_suggest:function(t){var i=this.menu.element.empty();this._renderMenu(i,t),this.isNewMenu=!0,this.menu.refresh(),i.show(),this._resizeMenu(),i.position(e.extend({of:this.element},this.options.position)),this.options.autoFocus&&this.menu.next()},_resizeMenu:function(){var e=this.menu.element;e.outerWidth(Math.max(e.width("").outerWidth()+1,this.element.outerWidth()))},_renderMenu:function(t,i){var s=this;e.each(i,function(e,i){s._renderItemData(t,i)})},_renderItemData:function(e,t){return this._renderItem(e,t).data("ui-autocomplete-item",t)},_renderItem:function(t,i){return e("<li>").text(i.label).appendTo(t)},_move:function(e,t){return this.menu.element.is(":visible")?this.menu.isFirstItem()&&/^previous/.test(e)||this.menu.isLastItem()&&/^next/.test(e)?(this.isMultiLine||this._value(this.term),this.menu.blur(),void 0):(this.menu[e](t),void 0):(this.search(null,t),void 0)},widget:function(){return this.menu.element},_value:function(){return this.valueMethod.apply(this.element,arguments)},_keyEvent:function(e,t){(!this.isMultiLine||this.menu.element.is(":visible"))&&(this._move(e,t),t.preventDefault())}}),e.extend(e.ui.autocomplete,{escapeRegex:function(e){return e.replace(/[\-\[\]{}()*+?.,\\\^$|#\s]/g,"\\$&")},filter:function(t,i){var s=RegExp(e.ui.autocomplete.escapeRegex(i),"i");return e.grep(t,function(e){return s.test(e.label||e.value||e)})}}),e.widget("ui.autocomplete",e.ui.autocomplete,{options:{messages:{noResults:"No search results.",results:function(e){return e+(e>1?" results are":" result is")+" available, use up and down arrow keys to navigate."}}},__response:function(t){var i;this._superApply(arguments),this.options.disabled||this.cancelSearch||(i=t&&t.length?this.options.messages.results(t.length):this.options.messages.noResults,this.liveRegion.children().hide(),e("<div>").text(i).appendTo(this.liveRegion))}}),e.ui.autocomplete;var r,h="ui-button ui-widget ui-state-default ui-corner-all",l="ui-button-icons-only ui-button-icon-only ui-button-text-icons ui-button-text-icon-primary ui-button-text-icon-secondary ui-button-text-only",u=function(){var t=e(this);setTimeout(function(){t.find(":ui-button").button("refresh")},1)},d=function(t){var i=t.name,s=t.form,n=e([]);return i&&(i=i.replace(/'/g,"\\'"),n=s?e(s).find("[name='"+i+"'][type=radio]"):e("[name='"+i+"'][type=radio]",t.ownerDocument).filter(function(){return!this.form})),n};e.widget("ui.button",{version:"1.11.4",defaultElement:"<button>",options:{disabled:null,text:!0,label:null,icons:{primary:null,secondary:null}},_create:function(){this.element.closest("form").unbind("reset"+this.eventNamespace).bind("reset"+this.eventNamespace,u),"boolean"!=typeof this.options.disabled?this.options.disabled=!!this.element.prop("disabled"):this.element.prop("disabled",this.options.disabled),this._determineButtonType(),this.hasTitle=!!this.buttonElement.attr("title");var t=this,i=this.options,s="checkbox"===this.type||"radio"===this.type,n=s?"":"ui-state-active";null===i.label&&(i.label="input"===this.type?this.buttonElement.val():this.buttonElement.html()),this._hoverable(this.buttonElement),this.buttonElement.addClass(h).attr("role","button").bind("mouseenter"+this.eventNamespace,function(){i.disabled||this===r&&e(this).addClass("ui-state-active")}).bind("mouseleave"+this.eventNamespace,function(){i.disabled||e(this).removeClass(n)}).bind("click"+this.eventNamespace,function(e){i.disabled&&(e.preventDefault(),e.stopImmediatePropagation())}),this._on({focus:function(){this.buttonElement.addClass("ui-state-focus")},blur:function(){this.buttonElement.removeClass("ui-state-focus")}}),s&&this.element.bind("change"+this.eventNamespace,function(){t.refresh()}),"checkbox"===this.type?this.buttonElement.bind("click"+this.eventNamespace,function(){return i.disabled?!1:void 0}):"radio"===this.type?this.buttonElement.bind("click"+this.eventNamespace,function(){if(i.disabled)return!1;e(this).addClass("ui-state-active"),t.buttonElement.attr("aria-pressed","true");var s=t.element[0];d(s).not(s).map(function(){return e(this).button("widget")[0]}).removeClass("ui-state-active").attr("aria-pressed","false")}):(this.buttonElement.bind("mousedown"+this.eventNamespace,function(){return i.disabled?!1:(e(this).addClass("ui-state-active"),r=this,t.document.one("mouseup",function(){r=null}),void 0)}).bind("mouseup"+this.eventNamespace,function(){return i.disabled?!1:(e(this).removeClass("ui-state-active"),void 0)}).bind("keydown"+this.eventNamespace,function(t){return i.disabled?!1:((t.keyCode===e.ui.keyCode.SPACE||t.keyCode===e.ui.keyCode.ENTER)&&e(this).addClass("ui-state-active"),void 0)}).bind("keyup"+this.eventNamespace+" blur"+this.eventNamespace,function(){e(this).removeClass("ui-state-active")}),this.buttonElement.is("a")&&this.buttonElement.keyup(function(t){t.keyCode===e.ui.keyCode.SPACE&&e(this).click()})),this._setOption("disabled",i.disabled),this._resetButton()},_determineButtonType:function(){var e,t,i;this.type=this.element.is("[type=checkbox]")?"checkbox":this.element.is("[type=radio]")?"radio":this.element.is("input")?"input":"button","checkbox"===this.type||"radio"===this.type?(e=this.element.parents().last(),t="label[for='"+this.element.attr("id")+"']",this.buttonElement=e.find(t),this.buttonElement.length||(e=e.length?e.siblings():this.element.siblings(),this.buttonElement=e.filter(t),this.buttonElement.length||(this.buttonElement=e.find(t))),this.element.addClass("ui-helper-hidden-accessible"),i=this.element.is(":checked"),i&&this.buttonElement.addClass("ui-state-active"),this.buttonElement.prop("aria-pressed",i)):this.buttonElement=this.element},widget:function(){return this.buttonElement},_destroy:function(){this.element.removeClass("ui-helper-hidden-accessible"),this.buttonElement.removeClass(h+" ui-state-active "+l).removeAttr("role").removeAttr("aria-pressed").html(this.buttonElement.find(".ui-button-text").html()),this.hasTitle||this.buttonElement.removeAttr("title")},_setOption:function(e,t){return this._super(e,t),"disabled"===e?(this.widget().toggleClass("ui-state-disabled",!!t),this.element.prop("disabled",!!t),t&&("checkbox"===this.type||"radio"===this.type?this.buttonElement.removeClass("ui-state-focus"):this.buttonElement.removeClass("ui-state-focus ui-state-active")),void 0):(this._resetButton(),void 0)},refresh:function(){var t=this.element.is("input, button")?this.element.is(":disabled"):this.element.hasClass("ui-button-disabled");t!==this.options.disabled&&this._setOption("disabled",t),"radio"===this.type?d(this.element[0]).each(function(){e(this).is(":checked")?e(this).button("widget").addClass("ui-state-active").attr("aria-pressed","true"):e(this).button("widget").removeClass("ui-state-active").attr("aria-pressed","false")}):"checkbox"===this.type&&(this.element.is(":checked")?this.buttonElement.addClass("ui-state-active").attr("aria-pressed","true"):this.buttonElement.removeClass("ui-state-active").attr("aria-pressed","false"))},_resetButton:function(){if("input"===this.type)return this.options.label&&this.element.val(this.options.label),void 0;var t=this.buttonElement.removeClass(l),i=e("<span></span>",this.document[0]).addClass("ui-button-text").html(this.options.label).appendTo(t.empty()).text(),s=this.options.icons,n=s.primary&&s.secondary,a=[];s.primary||s.secondary?(this.options.text&&a.push("ui-button-text-icon"+(n?"s":s.primary?"-primary":"-secondary")),s.primary&&t.prepend("<span class='ui-button-icon-primary ui-icon "+s.primary+"'></span>"),s.secondary&&t.append("<span class='ui-button-icon-secondary ui-icon "+s.secondary+"'></span>"),this.options.text||(a.push(n?"ui-button-icons-only":"ui-button-icon-only"),this.hasTitle||t.attr("title",e.trim(i)))):a.push("ui-button-text-only"),t.addClass(a.join(" "))}}),e.widget("ui.buttonset",{version:"1.11.4",options:{items:"button, input[type=button], input[type=submit], input[type=reset], input[type=checkbox], input[type=radio], a, :data(ui-button)"},_create:function(){this.element.addClass("ui-buttonset")},_init:function(){this.refresh()},_setOption:function(e,t){"disabled"===e&&this.buttons.button("option",e,t),this._super(e,t)},refresh:function(){var t="rtl"===this.element.css("direction"),i=this.element.find(this.options.items),s=i.filter(":ui-button");i.not(":ui-button").button(),s.button("refresh"),this.buttons=i.map(function(){return e(this).button("widget")[0]}).removeClass("ui-corner-all ui-corner-left ui-corner-right").filter(":first").addClass(t?"ui-corner-right":"ui-corner-left").end().filter(":last").addClass(t?"ui-corner-left":"ui-corner-right").end().end()},_destroy:function(){this.element.removeClass("ui-buttonset"),this.buttons.map(function(){return e(this).button("widget")[0]}).removeClass("ui-corner-left ui-corner-right").end().button("destroy")}}),e.ui.button,e.widget("ui.dialog",{version:"1.11.4",options:{appendTo:"body",autoOpen:!0,buttons:[],closeOnEscape:!0,closeText:"Close",dialogClass:"",draggable:!0,hide:null,height:"auto",maxHeight:null,maxWidth:null,minHeight:150,minWidth:150,modal:!1,position:{my:"center",at:"center",of:window,collision:"fit",using:function(t){var i=e(this).css(t).offset().top;0>i&&e(this).css("top",t.top-i)}},resizable:!0,show:null,title:null,width:300,beforeClose:null,close:null,drag:null,dragStart:null,dragStop:null,focus:null,open:null,resize:null,resizeStart:null,resizeStop:null},sizeRelatedOptions:{buttons:!0,height:!0,maxHeight:!0,maxWidth:!0,minHeight:!0,minWidth:!0,width:!0},resizableRelatedOptions:{maxHeight:!0,maxWidth:!0,minHeight:!0,minWidth:!0},_create:function(){this.originalCss={display:this.element[0].style.display,width:this.element[0].style.width,minHeight:this.element[0].style.minHeight,maxHeight:this.element[0].style.maxHeight,height:this.element[0].style.height},this.originalPosition={parent:this.element.parent(),index:this.element.parent().children().index(this.element)},this.originalTitle=this.element.attr("title"),this.options.title=this.options.title||this.originalTitle,this._createWrapper(),this.element.show().removeAttr("title").addClass("ui-dialog-content ui-widget-content").appendTo(this.uiDialog),this._createTitlebar(),this._createButtonPane(),this.options.draggable&&e.fn.draggable&&this._makeDraggable(),this.options.resizable&&e.fn.resizable&&this._makeResizable(),this._isOpen=!1,this._trackFocus()},_init:function(){this.options.autoOpen&&this.open()},_appendTo:function(){var t=this.options.appendTo;return t&&(t.jquery||t.nodeType)?e(t):this.document.find(t||"body").eq(0)},_destroy:function(){var e,t=this.originalPosition;this._untrackInstance(),this._destroyOverlay(),this.element.removeUniqueId().removeClass("ui-dialog-content ui-widget-content").css(this.originalCss).detach(),this.uiDialog.stop(!0,!0).remove(),this.originalTitle&&this.element.attr("title",this.originalTitle),e=t.parent.children().eq(t.index),e.length&&e[0]!==this.element[0]?e.before(this.element):t.parent.append(this.element)},widget:function(){return this.uiDialog},disable:e.noop,enable:e.noop,close:function(t){var i,s=this;if(this._isOpen&&this._trigger("beforeClose",t)!==!1){if(this._isOpen=!1,this._focusedElement=null,this._destroyOverlay(),this._untrackInstance(),!this.opener.filter(":focusable").focus().length)try{i=this.document[0].activeElement,i&&"body"!==i.nodeName.toLowerCase()&&e(i).blur()}catch(n){}this._hide(this.uiDialog,this.options.hide,function(){s._trigger("close",t)})}},isOpen:function(){return this._isOpen},moveToTop:function(){this._moveToTop()},_moveToTop:function(t,i){var s=!1,n=this.uiDialog.siblings(".ui-front:visible").map(function(){return+e(this).css("z-index")}).get(),a=Math.max.apply(null,n);return a>=+this.uiDialog.css("z-index")&&(this.uiDialog.css("z-index",a+1),s=!0),s&&!i&&this._trigger("focus",t),s},open:function(){var t=this;return this._isOpen?(this._moveToTop()&&this._focusTabbable(),void 0):(this._isOpen=!0,this.opener=e(this.document[0].activeElement),this._size(),this._position(),this._createOverlay(),this._moveToTop(null,!0),this.overlay&&this.overlay.css("z-index",this.uiDialog.css("z-index")-1),this._show(this.uiDialog,this.options.show,function(){t._focusTabbable(),t._trigger("focus")}),this._makeFocusTarget(),this._trigger("open"),void 0)},_focusTabbable:function(){var e=this._focusedElement;e||(e=this.element.find("[autofocus]")),e.length||(e=this.element.find(":tabbable")),e.length||(e=this.uiDialogButtonPane.find(":tabbable")),e.length||(e=this.uiDialogTitlebarClose.filter(":tabbable")),e.length||(e=this.uiDialog),e.eq(0).focus()},_keepFocus:function(t){function i(){var t=this.document[0].activeElement,i=this.uiDialog[0]===t||e.contains(this.uiDialog[0],t);i||this._focusTabbable()}t.preventDefault(),i.call(this),this._delay(i)},_createWrapper:function(){this.uiDialog=e("<div>").addClass("ui-dialog ui-widget ui-widget-content ui-corner-all ui-front "+this.options.dialogClass).hide().attr({tabIndex:-1,role:"dialog"}).appendTo(this._appendTo()),this._on(this.uiDialog,{keydown:function(t){if(this.options.closeOnEscape&&!t.isDefaultPrevented()&&t.keyCode&&t.keyCode===e.ui.keyCode.ESCAPE)return t.preventDefault(),this.close(t),void 0;if(t.keyCode===e.ui.keyCode.TAB&&!t.isDefaultPrevented()){var i=this.uiDialog.find(":tabbable"),s=i.filter(":first"),n=i.filter(":last");t.target!==n[0]&&t.target!==this.uiDialog[0]||t.shiftKey?t.target!==s[0]&&t.target!==this.uiDialog[0]||!t.shiftKey||(this._delay(function(){n.focus()}),t.preventDefault()):(this._delay(function(){s.focus()}),t.preventDefault())}},mousedown:function(e){this._moveToTop(e)&&this._focusTabbable()}}),this.element.find("[aria-describedby]").length||this.uiDialog.attr({"aria-describedby":this.element.uniqueId().attr("id")})},_createTitlebar:function(){var t;this.uiDialogTitlebar=e("<div>").addClass("ui-dialog-titlebar ui-widget-header ui-corner-all ui-helper-clearfix").prependTo(this.uiDialog),this._on(this.uiDialogTitlebar,{mousedown:function(t){e(t.target).closest(".ui-dialog-titlebar-close")||this.uiDialog.focus()}}),this.uiDialogTitlebarClose=e("<button type='button'></button>").button({label:this.options.closeText,icons:{primary:"ui-icon-closethick"},text:!1}).addClass("ui-dialog-titlebar-close").appendTo(this.uiDialogTitlebar),this._on(this.uiDialogTitlebarClose,{click:function(e){e.preventDefault(),this.close(e)}}),t=e("<span>").uniqueId().addClass("ui-dialog-title").prependTo(this.uiDialogTitlebar),this._title(t),this.uiDialog.attr({"aria-labelledby":t.attr("id")})},_title:function(e){this.options.title||e.html("&#160;"),e.text(this.options.title)
+},_createButtonPane:function(){this.uiDialogButtonPane=e("<div>").addClass("ui-dialog-buttonpane ui-widget-content ui-helper-clearfix"),this.uiButtonSet=e("<div>").addClass("ui-dialog-buttonset").appendTo(this.uiDialogButtonPane),this._createButtons()},_createButtons:function(){var t=this,i=this.options.buttons;return this.uiDialogButtonPane.remove(),this.uiButtonSet.empty(),e.isEmptyObject(i)||e.isArray(i)&&!i.length?(this.uiDialog.removeClass("ui-dialog-buttons"),void 0):(e.each(i,function(i,s){var n,a;s=e.isFunction(s)?{click:s,text:i}:s,s=e.extend({type:"button"},s),n=s.click,s.click=function(){n.apply(t.element[0],arguments)},a={icons:s.icons,text:s.showText},delete s.icons,delete s.showText,e("<button></button>",s).button(a).appendTo(t.uiButtonSet)}),this.uiDialog.addClass("ui-dialog-buttons"),this.uiDialogButtonPane.appendTo(this.uiDialog),void 0)},_makeDraggable:function(){function t(e){return{position:e.position,offset:e.offset}}var i=this,s=this.options;this.uiDialog.draggable({cancel:".ui-dialog-content, .ui-dialog-titlebar-close",handle:".ui-dialog-titlebar",containment:"document",start:function(s,n){e(this).addClass("ui-dialog-dragging"),i._blockFrames(),i._trigger("dragStart",s,t(n))},drag:function(e,s){i._trigger("drag",e,t(s))},stop:function(n,a){var o=a.offset.left-i.document.scrollLeft(),r=a.offset.top-i.document.scrollTop();s.position={my:"left top",at:"left"+(o>=0?"+":"")+o+" "+"top"+(r>=0?"+":"")+r,of:i.window},e(this).removeClass("ui-dialog-dragging"),i._unblockFrames(),i._trigger("dragStop",n,t(a))}})},_makeResizable:function(){function t(e){return{originalPosition:e.originalPosition,originalSize:e.originalSize,position:e.position,size:e.size}}var i=this,s=this.options,n=s.resizable,a=this.uiDialog.css("position"),o="string"==typeof n?n:"n,e,s,w,se,sw,ne,nw";this.uiDialog.resizable({cancel:".ui-dialog-content",containment:"document",alsoResize:this.element,maxWidth:s.maxWidth,maxHeight:s.maxHeight,minWidth:s.minWidth,minHeight:this._minHeight(),handles:o,start:function(s,n){e(this).addClass("ui-dialog-resizing"),i._blockFrames(),i._trigger("resizeStart",s,t(n))},resize:function(e,s){i._trigger("resize",e,t(s))},stop:function(n,a){var o=i.uiDialog.offset(),r=o.left-i.document.scrollLeft(),h=o.top-i.document.scrollTop();s.height=i.uiDialog.height(),s.width=i.uiDialog.width(),s.position={my:"left top",at:"left"+(r>=0?"+":"")+r+" "+"top"+(h>=0?"+":"")+h,of:i.window},e(this).removeClass("ui-dialog-resizing"),i._unblockFrames(),i._trigger("resizeStop",n,t(a))}}).css("position",a)},_trackFocus:function(){this._on(this.widget(),{focusin:function(t){this._makeFocusTarget(),this._focusedElement=e(t.target)}})},_makeFocusTarget:function(){this._untrackInstance(),this._trackingInstances().unshift(this)},_untrackInstance:function(){var t=this._trackingInstances(),i=e.inArray(this,t);-1!==i&&t.splice(i,1)},_trackingInstances:function(){var e=this.document.data("ui-dialog-instances");return e||(e=[],this.document.data("ui-dialog-instances",e)),e},_minHeight:function(){var e=this.options;return"auto"===e.height?e.minHeight:Math.min(e.minHeight,e.height)},_position:function(){var e=this.uiDialog.is(":visible");e||this.uiDialog.show(),this.uiDialog.position(this.options.position),e||this.uiDialog.hide()},_setOptions:function(t){var i=this,s=!1,n={};e.each(t,function(e,t){i._setOption(e,t),e in i.sizeRelatedOptions&&(s=!0),e in i.resizableRelatedOptions&&(n[e]=t)}),s&&(this._size(),this._position()),this.uiDialog.is(":data(ui-resizable)")&&this.uiDialog.resizable("option",n)},_setOption:function(e,t){var i,s,n=this.uiDialog;"dialogClass"===e&&n.removeClass(this.options.dialogClass).addClass(t),"disabled"!==e&&(this._super(e,t),"appendTo"===e&&this.uiDialog.appendTo(this._appendTo()),"buttons"===e&&this._createButtons(),"closeText"===e&&this.uiDialogTitlebarClose.button({label:""+t}),"draggable"===e&&(i=n.is(":data(ui-draggable)"),i&&!t&&n.draggable("destroy"),!i&&t&&this._makeDraggable()),"position"===e&&this._position(),"resizable"===e&&(s=n.is(":data(ui-resizable)"),s&&!t&&n.resizable("destroy"),s&&"string"==typeof t&&n.resizable("option","handles",t),s||t===!1||this._makeResizable()),"title"===e&&this._title(this.uiDialogTitlebar.find(".ui-dialog-title")))},_size:function(){var e,t,i,s=this.options;this.element.show().css({width:"auto",minHeight:0,maxHeight:"none",height:0}),s.minWidth>s.width&&(s.width=s.minWidth),e=this.uiDialog.css({height:"auto",width:s.width}).outerHeight(),t=Math.max(0,s.minHeight-e),i="number"==typeof s.maxHeight?Math.max(0,s.maxHeight-e):"none","auto"===s.height?this.element.css({minHeight:t,maxHeight:i,height:"auto"}):this.element.height(Math.max(0,s.height-e)),this.uiDialog.is(":data(ui-resizable)")&&this.uiDialog.resizable("option","minHeight",this._minHeight())},_blockFrames:function(){this.iframeBlocks=this.document.find("iframe").map(function(){var t=e(this);return e("<div>").css({position:"absolute",width:t.outerWidth(),height:t.outerHeight()}).appendTo(t.parent()).offset(t.offset())[0]})},_unblockFrames:function(){this.iframeBlocks&&(this.iframeBlocks.remove(),delete this.iframeBlocks)},_allowInteraction:function(t){return e(t.target).closest(".ui-dialog").length?!0:!!e(t.target).closest(".ui-datepicker").length},_createOverlay:function(){if(this.options.modal){var t=!0;this._delay(function(){t=!1}),this.document.data("ui-dialog-overlays")||this._on(this.document,{focusin:function(e){t||this._allowInteraction(e)||(e.preventDefault(),this._trackingInstances()[0]._focusTabbable())}}),this.overlay=e("<div>").addClass("ui-widget-overlay ui-front").appendTo(this._appendTo()),this._on(this.overlay,{mousedown:"_keepFocus"}),this.document.data("ui-dialog-overlays",(this.document.data("ui-dialog-overlays")||0)+1)}},_destroyOverlay:function(){if(this.options.modal&&this.overlay){var e=this.document.data("ui-dialog-overlays")-1;e?this.document.data("ui-dialog-overlays",e):this.document.unbind("focusin").removeData("ui-dialog-overlays"),this.overlay.remove(),this.overlay=null}}}),e.widget("ui.progressbar",{version:"1.11.4",options:{max:100,value:0,change:null,complete:null},min:0,_create:function(){this.oldValue=this.options.value=this._constrainedValue(),this.element.addClass("ui-progressbar ui-widget ui-widget-content ui-corner-all").attr({role:"progressbar","aria-valuemin":this.min}),this.valueDiv=e("<div class='ui-progressbar-value ui-widget-header ui-corner-left'></div>").appendTo(this.element),this._refreshValue()},_destroy:function(){this.element.removeClass("ui-progressbar ui-widget ui-widget-content ui-corner-all").removeAttr("role").removeAttr("aria-valuemin").removeAttr("aria-valuemax").removeAttr("aria-valuenow"),this.valueDiv.remove()},value:function(e){return void 0===e?this.options.value:(this.options.value=this._constrainedValue(e),this._refreshValue(),void 0)},_constrainedValue:function(e){return void 0===e&&(e=this.options.value),this.indeterminate=e===!1,"number"!=typeof e&&(e=0),this.indeterminate?!1:Math.min(this.options.max,Math.max(this.min,e))},_setOptions:function(e){var t=e.value;delete e.value,this._super(e),this.options.value=this._constrainedValue(t),this._refreshValue()},_setOption:function(e,t){"max"===e&&(t=Math.max(this.min,t)),"disabled"===e&&this.element.toggleClass("ui-state-disabled",!!t).attr("aria-disabled",t),this._super(e,t)},_percentage:function(){return this.indeterminate?100:100*(this.options.value-this.min)/(this.options.max-this.min)},_refreshValue:function(){var t=this.options.value,i=this._percentage();this.valueDiv.toggle(this.indeterminate||t>this.min).toggleClass("ui-corner-right",t===this.options.max).width(i.toFixed(0)+"%"),this.element.toggleClass("ui-progressbar-indeterminate",this.indeterminate),this.indeterminate?(this.element.removeAttr("aria-valuenow"),this.overlayDiv||(this.overlayDiv=e("<div class='ui-progressbar-overlay'></div>").appendTo(this.valueDiv))):(this.element.attr({"aria-valuemax":this.options.max,"aria-valuenow":t}),this.overlayDiv&&(this.overlayDiv.remove(),this.overlayDiv=null)),this.oldValue!==t&&(this.oldValue=t,this._trigger("change")),t===this.options.max&&this._trigger("complete")}}),e.widget("ui.selectmenu",{version:"1.11.4",defaultElement:"<select>",options:{appendTo:null,disabled:null,icons:{button:"ui-icon-triangle-1-s"},position:{my:"left top",at:"left bottom",collision:"none"},width:null,change:null,close:null,focus:null,open:null,select:null},_create:function(){var e=this.element.uniqueId().attr("id");this.ids={element:e,button:e+"-button",menu:e+"-menu"},this._drawButton(),this._drawMenu(),this.options.disabled&&this.disable()},_drawButton:function(){var t=this;this.label=e("label[for='"+this.ids.element+"']").attr("for",this.ids.button),this._on(this.label,{click:function(e){this.button.focus(),e.preventDefault()}}),this.element.hide(),this.button=e("<span>",{"class":"ui-selectmenu-button ui-widget ui-state-default ui-corner-all",tabindex:this.options.disabled?-1:0,id:this.ids.button,role:"combobox","aria-expanded":"false","aria-autocomplete":"list","aria-owns":this.ids.menu,"aria-haspopup":"true"}).insertAfter(this.element),e("<span>",{"class":"ui-icon "+this.options.icons.button}).prependTo(this.button),this.buttonText=e("<span>",{"class":"ui-selectmenu-text"}).appendTo(this.button),this._setText(this.buttonText,this.element.find("option:selected").text()),this._resizeButton(),this._on(this.button,this._buttonEvents),this.button.one("focusin",function(){t.menuItems||t._refreshMenu()}),this._hoverable(this.button),this._focusable(this.button)},_drawMenu:function(){var t=this;this.menu=e("<ul>",{"aria-hidden":"true","aria-labelledby":this.ids.button,id:this.ids.menu}),this.menuWrap=e("<div>",{"class":"ui-selectmenu-menu ui-front"}).append(this.menu).appendTo(this._appendTo()),this.menuInstance=this.menu.menu({role:"listbox",select:function(e,i){e.preventDefault(),t._setSelection(),t._select(i.item.data("ui-selectmenu-item"),e)},focus:function(e,i){var s=i.item.data("ui-selectmenu-item");null!=t.focusIndex&&s.index!==t.focusIndex&&(t._trigger("focus",e,{item:s}),t.isOpen||t._select(s,e)),t.focusIndex=s.index,t.button.attr("aria-activedescendant",t.menuItems.eq(s.index).attr("id"))}}).menu("instance"),this.menu.addClass("ui-corner-bottom").removeClass("ui-corner-all"),this.menuInstance._off(this.menu,"mouseleave"),this.menuInstance._closeOnDocumentClick=function(){return!1},this.menuInstance._isDivider=function(){return!1}},refresh:function(){this._refreshMenu(),this._setText(this.buttonText,this._getSelectedItem().text()),this.options.width||this._resizeButton()},_refreshMenu:function(){this.menu.empty();var e,t=this.element.find("option");t.length&&(this._parseOptions(t),this._renderMenu(this.menu,this.items),this.menuInstance.refresh(),this.menuItems=this.menu.find("li").not(".ui-selectmenu-optgroup"),e=this._getSelectedItem(),this.menuInstance.focus(null,e),this._setAria(e.data("ui-selectmenu-item")),this._setOption("disabled",this.element.prop("disabled")))},open:function(e){this.options.disabled||(this.menuItems?(this.menu.find(".ui-state-focus").removeClass("ui-state-focus"),this.menuInstance.focus(null,this._getSelectedItem())):this._refreshMenu(),this.isOpen=!0,this._toggleAttr(),this._resizeMenu(),this._position(),this._on(this.document,this._documentClick),this._trigger("open",e))},_position:function(){this.menuWrap.position(e.extend({of:this.button},this.options.position))},close:function(e){this.isOpen&&(this.isOpen=!1,this._toggleAttr(),this.range=null,this._off(this.document),this._trigger("close",e))},widget:function(){return this.button},menuWidget:function(){return this.menu},_renderMenu:function(t,i){var s=this,n="";e.each(i,function(i,a){a.optgroup!==n&&(e("<li>",{"class":"ui-selectmenu-optgroup ui-menu-divider"+(a.element.parent("optgroup").prop("disabled")?" ui-state-disabled":""),text:a.optgroup}).appendTo(t),n=a.optgroup),s._renderItemData(t,a)})},_renderItemData:function(e,t){return this._renderItem(e,t).data("ui-selectmenu-item",t)},_renderItem:function(t,i){var s=e("<li>");return i.disabled&&s.addClass("ui-state-disabled"),this._setText(s,i.label),s.appendTo(t)},_setText:function(e,t){t?e.text(t):e.html("&#160;")},_move:function(e,t){var i,s,n=".ui-menu-item";this.isOpen?i=this.menuItems.eq(this.focusIndex):(i=this.menuItems.eq(this.element[0].selectedIndex),n+=":not(.ui-state-disabled)"),s="first"===e||"last"===e?i["first"===e?"prevAll":"nextAll"](n).eq(-1):i[e+"All"](n).eq(0),s.length&&this.menuInstance.focus(t,s)},_getSelectedItem:function(){return this.menuItems.eq(this.element[0].selectedIndex)},_toggle:function(e){this[this.isOpen?"close":"open"](e)},_setSelection:function(){var e;this.range&&(window.getSelection?(e=window.getSelection(),e.removeAllRanges(),e.addRange(this.range)):this.range.select(),this.button.focus())},_documentClick:{mousedown:function(t){this.isOpen&&(e(t.target).closest(".ui-selectmenu-menu, #"+this.ids.button).length||this.close(t))}},_buttonEvents:{mousedown:function(){var e;window.getSelection?(e=window.getSelection(),e.rangeCount&&(this.range=e.getRangeAt(0))):this.range=document.selection.createRange()},click:function(e){this._setSelection(),this._toggle(e)},keydown:function(t){var i=!0;switch(t.keyCode){case e.ui.keyCode.TAB:case e.ui.keyCode.ESCAPE:this.close(t),i=!1;break;case e.ui.keyCode.ENTER:this.isOpen&&this._selectFocusedItem(t);break;case e.ui.keyCode.UP:t.altKey?this._toggle(t):this._move("prev",t);break;case e.ui.keyCode.DOWN:t.altKey?this._toggle(t):this._move("next",t);break;case e.ui.keyCode.SPACE:this.isOpen?this._selectFocusedItem(t):this._toggle(t);break;case e.ui.keyCode.LEFT:this._move("prev",t);break;case e.ui.keyCode.RIGHT:this._move("next",t);break;case e.ui.keyCode.HOME:case e.ui.keyCode.PAGE_UP:this._move("first",t);break;case e.ui.keyCode.END:case e.ui.keyCode.PAGE_DOWN:this._move("last",t);break;default:this.menu.trigger(t),i=!1}i&&t.preventDefault()}},_selectFocusedItem:function(e){var t=this.menuItems.eq(this.focusIndex);t.hasClass("ui-state-disabled")||this._select(t.data("ui-selectmenu-item"),e)},_select:function(e,t){var i=this.element[0].selectedIndex;this.element[0].selectedIndex=e.index,this._setText(this.buttonText,e.label),this._setAria(e),this._trigger("select",t,{item:e}),e.index!==i&&this._trigger("change",t,{item:e}),this.close(t)},_setAria:function(e){var t=this.menuItems.eq(e.index).attr("id");this.button.attr({"aria-labelledby":t,"aria-activedescendant":t}),this.menu.attr("aria-activedescendant",t)},_setOption:function(e,t){"icons"===e&&this.button.find("span.ui-icon").removeClass(this.options.icons.button).addClass(t.button),this._super(e,t),"appendTo"===e&&this.menuWrap.appendTo(this._appendTo()),"disabled"===e&&(this.menuInstance.option("disabled",t),this.button.toggleClass("ui-state-disabled",t).attr("aria-disabled",t),this.element.prop("disabled",t),t?(this.button.attr("tabindex",-1),this.close()):this.button.attr("tabindex",0)),"width"===e&&this._resizeButton()},_appendTo:function(){var t=this.options.appendTo;return t&&(t=t.jquery||t.nodeType?e(t):this.document.find(t).eq(0)),t&&t[0]||(t=this.element.closest(".ui-front")),t.length||(t=this.document[0].body),t},_toggleAttr:function(){this.button.toggleClass("ui-corner-top",this.isOpen).toggleClass("ui-corner-all",!this.isOpen).attr("aria-expanded",this.isOpen),this.menuWrap.toggleClass("ui-selectmenu-open",this.isOpen),this.menu.attr("aria-hidden",!this.isOpen)},_resizeButton:function(){var e=this.options.width;e||(e=this.element.show().outerWidth(),this.element.hide()),this.button.outerWidth(e)},_resizeMenu:function(){this.menu.outerWidth(Math.max(this.button.outerWidth(),this.menu.width("").outerWidth()+1))},_getCreateOptions:function(){return{disabled:this.element.prop("disabled")}},_parseOptions:function(t){var i=[];t.each(function(t,s){var n=e(s),a=n.parent("optgroup");i.push({element:n,index:t,value:n.val(),label:n.text(),optgroup:a.attr("label")||"",disabled:a.prop("disabled")||n.prop("disabled")})}),this.items=i},_destroy:function(){this.menuWrap.remove(),this.button.remove(),this.element.show(),this.element.removeUniqueId(),this.label.attr("for",this.ids.element)}}),e.widget("ui.slider",e.ui.mouse,{version:"1.11.4",widgetEventPrefix:"slide",options:{animate:!1,distance:0,max:100,min:0,orientation:"horizontal",range:!1,step:1,value:0,values:null,change:null,slide:null,start:null,stop:null},numPages:5,_create:function(){this._keySliding=!1,this._mouseSliding=!1,this._animateOff=!0,this._handleIndex=null,this._detectOrientation(),this._mouseInit(),this._calculateNewMax(),this.element.addClass("ui-slider ui-slider-"+this.orientation+" ui-widget"+" ui-widget-content"+" ui-corner-all"),this._refresh(),this._setOption("disabled",this.options.disabled),this._animateOff=!1},_refresh:function(){this._createRange(),this._createHandles(),this._setupEvents(),this._refreshValue()},_createHandles:function(){var t,i,s=this.options,n=this.element.find(".ui-slider-handle").addClass("ui-state-default ui-corner-all"),a="<span class='ui-slider-handle ui-state-default ui-corner-all' tabindex='0'></span>",o=[];for(i=s.values&&s.values.length||1,n.length>i&&(n.slice(i).remove(),n=n.slice(0,i)),t=n.length;i>t;t++)o.push(a);this.handles=n.add(e(o.join("")).appendTo(this.element)),this.handle=this.handles.eq(0),this.handles.each(function(t){e(this).data("ui-slider-handle-index",t)})},_createRange:function(){var t=this.options,i="";t.range?(t.range===!0&&(t.values?t.values.length&&2!==t.values.length?t.values=[t.values[0],t.values[0]]:e.isArray(t.values)&&(t.values=t.values.slice(0)):t.values=[this._valueMin(),this._valueMin()]),this.range&&this.range.length?this.range.removeClass("ui-slider-range-min ui-slider-range-max").css({left:"",bottom:""}):(this.range=e("<div></div>").appendTo(this.element),i="ui-slider-range ui-widget-header ui-corner-all"),this.range.addClass(i+("min"===t.range||"max"===t.range?" ui-slider-range-"+t.range:""))):(this.range&&this.range.remove(),this.range=null)},_setupEvents:function(){this._off(this.handles),this._on(this.handles,this._handleEvents),this._hoverable(this.handles),this._focusable(this.handles)},_destroy:function(){this.handles.remove(),this.range&&this.range.remove(),this.element.removeClass("ui-slider ui-slider-horizontal ui-slider-vertical ui-widget ui-widget-content ui-corner-all"),this._mouseDestroy()},_mouseCapture:function(t){var i,s,n,a,o,r,h,l,u=this,d=this.options;return d.disabled?!1:(this.elementSize={width:this.element.outerWidth(),height:this.element.outerHeight()},this.elementOffset=this.element.offset(),i={x:t.pageX,y:t.pageY},s=this._normValueFromMouse(i),n=this._valueMax()-this._valueMin()+1,this.handles.each(function(t){var i=Math.abs(s-u.values(t));(n>i||n===i&&(t===u._lastChangedValue||u.values(t)===d.min))&&(n=i,a=e(this),o=t)}),r=this._start(t,o),r===!1?!1:(this._mouseSliding=!0,this._handleIndex=o,a.addClass("ui-state-active").focus(),h=a.offset(),l=!e(t.target).parents().addBack().is(".ui-slider-handle"),this._clickOffset=l?{left:0,top:0}:{left:t.pageX-h.left-a.width()/2,top:t.pageY-h.top-a.height()/2-(parseInt(a.css("borderTopWidth"),10)||0)-(parseInt(a.css("borderBottomWidth"),10)||0)+(parseInt(a.css("marginTop"),10)||0)},this.handles.hasClass("ui-state-hover")||this._slide(t,o,s),this._animateOff=!0,!0))},_mouseStart:function(){return!0},_mouseDrag:function(e){var t={x:e.pageX,y:e.pageY},i=this._normValueFromMouse(t);return this._slide(e,this._handleIndex,i),!1},_mouseStop:function(e){return this.handles.removeClass("ui-state-active"),this._mouseSliding=!1,this._stop(e,this._handleIndex),this._change(e,this._handleIndex),this._handleIndex=null,this._clickOffset=null,this._animateOff=!1,!1},_detectOrientation:function(){this.orientation="vertical"===this.options.orientation?"vertical":"horizontal"},_normValueFromMouse:function(e){var t,i,s,n,a;return"horizontal"===this.orientation?(t=this.elementSize.width,i=e.x-this.elementOffset.left-(this._clickOffset?this._clickOffset.left:0)):(t=this.elementSize.height,i=e.y-this.elementOffset.top-(this._clickOffset?this._clickOffset.top:0)),s=i/t,s>1&&(s=1),0>s&&(s=0),"vertical"===this.orientation&&(s=1-s),n=this._valueMax()-this._valueMin(),a=this._valueMin()+s*n,this._trimAlignValue(a)},_start:function(e,t){var i={handle:this.handles[t],value:this.value()};return this.options.values&&this.options.values.length&&(i.value=this.values(t),i.values=this.values()),this._trigger("start",e,i)},_slide:function(e,t,i){var s,n,a;this.options.values&&this.options.values.length?(s=this.values(t?0:1),2===this.options.values.length&&this.options.range===!0&&(0===t&&i>s||1===t&&s>i)&&(i=s),i!==this.values(t)&&(n=this.values(),n[t]=i,a=this._trigger("slide",e,{handle:this.handles[t],value:i,values:n}),s=this.values(t?0:1),a!==!1&&this.values(t,i))):i!==this.value()&&(a=this._trigger("slide",e,{handle:this.handles[t],value:i}),a!==!1&&this.value(i))},_stop:function(e,t){var i={handle:this.handles[t],value:this.value()};this.options.values&&this.options.values.length&&(i.value=this.values(t),i.values=this.values()),this._trigger("stop",e,i)},_change:function(e,t){if(!this._keySliding&&!this._mouseSliding){var i={handle:this.handles[t],value:this.value()};this.options.values&&this.options.values.length&&(i.value=this.values(t),i.values=this.values()),this._lastChangedValue=t,this._trigger("change",e,i)}},value:function(e){return arguments.length?(this.options.value=this._trimAlignValue(e),this._refreshValue(),this._change(null,0),void 0):this._value()},values:function(t,i){var s,n,a;if(arguments.length>1)return this.options.values[t]=this._trimAlignValue(i),this._refreshValue(),this._change(null,t),void 0;if(!arguments.length)return this._values();if(!e.isArray(arguments[0]))return this.options.values&&this.options.values.length?this._values(t):this.value();for(s=this.options.values,n=arguments[0],a=0;s.length>a;a+=1)s[a]=this._trimAlignValue(n[a]),this._change(null,a);this._refreshValue()},_setOption:function(t,i){var s,n=0;switch("range"===t&&this.options.range===!0&&("min"===i?(this.options.value=this._values(0),this.options.values=null):"max"===i&&(this.options.value=this._values(this.options.values.length-1),this.options.values=null)),e.isArray(this.options.values)&&(n=this.options.values.length),"disabled"===t&&this.element.toggleClass("ui-state-disabled",!!i),this._super(t,i),t){case"orientation":this._detectOrientation(),this.element.removeClass("ui-slider-horizontal ui-slider-vertical").addClass("ui-slider-"+this.orientation),this._refreshValue(),this.handles.css("horizontal"===i?"bottom":"left","");break;case"value":this._animateOff=!0,this._refreshValue(),this._change(null,0),this._animateOff=!1;break;case"values":for(this._animateOff=!0,this._refreshValue(),s=0;n>s;s+=1)this._change(null,s);this._animateOff=!1;break;case"step":case"min":case"max":this._animateOff=!0,this._calculateNewMax(),this._refreshValue(),this._animateOff=!1;break;case"range":this._animateOff=!0,this._refresh(),this._animateOff=!1}},_value:function(){var e=this.options.value;return e=this._trimAlignValue(e)},_values:function(e){var t,i,s;if(arguments.length)return t=this.options.values[e],t=this._trimAlignValue(t);if(this.options.values&&this.options.values.length){for(i=this.options.values.slice(),s=0;i.length>s;s+=1)i[s]=this._trimAlignValue(i[s]);return i}return[]},_trimAlignValue:function(e){if(this._valueMin()>=e)return this._valueMin();if(e>=this._valueMax())return this._valueMax();var t=this.options.step>0?this.options.step:1,i=(e-this._valueMin())%t,s=e-i;return 2*Math.abs(i)>=t&&(s+=i>0?t:-t),parseFloat(s.toFixed(5))},_calculateNewMax:function(){var e=this.options.max,t=this._valueMin(),i=this.options.step,s=Math.floor(+(e-t).toFixed(this._precision())/i)*i;e=s+t,this.max=parseFloat(e.toFixed(this._precision()))},_precision:function(){var e=this._precisionOf(this.options.step);return null!==this.options.min&&(e=Math.max(e,this._precisionOf(this.options.min))),e},_precisionOf:function(e){var t=""+e,i=t.indexOf(".");return-1===i?0:t.length-i-1},_valueMin:function(){return this.options.min},_valueMax:function(){return this.max},_refreshValue:function(){var t,i,s,n,a,o=this.options.range,r=this.options,h=this,l=this._animateOff?!1:r.animate,u={};this.options.values&&this.options.values.length?this.handles.each(function(s){i=100*((h.values(s)-h._valueMin())/(h._valueMax()-h._valueMin())),u["horizontal"===h.orientation?"left":"bottom"]=i+"%",e(this).stop(1,1)[l?"animate":"css"](u,r.animate),h.options.range===!0&&("horizontal"===h.orientation?(0===s&&h.range.stop(1,1)[l?"animate":"css"]({left:i+"%"},r.animate),1===s&&h.range[l?"animate":"css"]({width:i-t+"%"},{queue:!1,duration:r.animate})):(0===s&&h.range.stop(1,1)[l?"animate":"css"]({bottom:i+"%"},r.animate),1===s&&h.range[l?"animate":"css"]({height:i-t+"%"},{queue:!1,duration:r.animate}))),t=i}):(s=this.value(),n=this._valueMin(),a=this._valueMax(),i=a!==n?100*((s-n)/(a-n)):0,u["horizontal"===this.orientation?"left":"bottom"]=i+"%",this.handle.stop(1,1)[l?"animate":"css"](u,r.animate),"min"===o&&"horizontal"===this.orientation&&this.range.stop(1,1)[l?"animate":"css"]({width:i+"%"},r.animate),"max"===o&&"horizontal"===this.orientation&&this.range[l?"animate":"css"]({width:100-i+"%"},{queue:!1,duration:r.animate}),"min"===o&&"vertical"===this.orientation&&this.range.stop(1,1)[l?"animate":"css"]({height:i+"%"},r.animate),"max"===o&&"vertical"===this.orientation&&this.range[l?"animate":"css"]({height:100-i+"%"},{queue:!1,duration:r.animate}))},_handleEvents:{keydown:function(t){var i,s,n,a,o=e(t.target).data("ui-slider-handle-index");switch(t.keyCode){case e.ui.keyCode.HOME:case e.ui.keyCode.END:case e.ui.keyCode.PAGE_UP:case e.ui.keyCode.PAGE_DOWN:case e.ui.keyCode.UP:case e.ui.keyCode.RIGHT:case e.ui.keyCode.DOWN:case e.ui.keyCode.LEFT:if(t.preventDefault(),!this._keySliding&&(this._keySliding=!0,e(t.target).addClass("ui-state-active"),i=this._start(t,o),i===!1))return}switch(a=this.options.step,s=n=this.options.values&&this.options.values.length?this.values(o):this.value(),t.keyCode){case e.ui.keyCode.HOME:n=this._valueMin();break;case e.ui.keyCode.END:n=this._valueMax();break;case e.ui.keyCode.PAGE_UP:n=this._trimAlignValue(s+(this._valueMax()-this._valueMin())/this.numPages);break;case e.ui.keyCode.PAGE_DOWN:n=this._trimAlignValue(s-(this._valueMax()-this._valueMin())/this.numPages);break;case e.ui.keyCode.UP:case e.ui.keyCode.RIGHT:if(s===this._valueMax())return;n=this._trimAlignValue(s+a);break;case e.ui.keyCode.DOWN:case e.ui.keyCode.LEFT:if(s===this._valueMin())return;n=this._trimAlignValue(s-a)}this._slide(t,o,n)},keyup:function(t){var i=e(t.target).data("ui-slider-handle-index");this._keySliding&&(this._keySliding=!1,this._stop(t,i),this._change(t,i),e(t.target).removeClass("ui-state-active"))}}}),e.widget("ui.spinner",{version:"1.11.4",defaultElement:"<input>",widgetEventPrefix:"spin",options:{culture:null,icons:{down:"ui-icon-triangle-1-s",up:"ui-icon-triangle-1-n"},incremental:!0,max:null,min:null,numberFormat:null,page:10,step:1,change:null,spin:null,start:null,stop:null},_create:function(){this._setOption("max",this.options.max),this._setOption("min",this.options.min),this._setOption("step",this.options.step),""!==this.value()&&this._value(this.element.val(),!0),this._draw(),this._on(this._events),this._refresh(),this._on(this.window,{beforeunload:function(){this.element.removeAttr("autocomplete")}})},_getCreateOptions:function(){var t={},i=this.element;return e.each(["min","max","step"],function(e,s){var n=i.attr(s);void 0!==n&&n.length&&(t[s]=n)}),t},_events:{keydown:function(e){this._start(e)&&this._keydown(e)&&e.preventDefault()},keyup:"_stop",focus:function(){this.previous=this.element.val()},blur:function(e){return this.cancelBlur?(delete this.cancelBlur,void 0):(this._stop(),this._refresh(),this.previous!==this.element.val()&&this._trigger("change",e),void 0)},mousewheel:function(e,t){if(t){if(!this.spinning&&!this._start(e))return!1;this._spin((t>0?1:-1)*this.options.step,e),clearTimeout(this.mousewheelTimer),this.mousewheelTimer=this._delay(function(){this.spinning&&this._stop(e)},100),e.preventDefault()}},"mousedown .ui-spinner-button":function(t){function i(){var e=this.element[0]===this.document[0].activeElement;e||(this.element.focus(),this.previous=s,this._delay(function(){this.previous=s}))}var s;s=this.element[0]===this.document[0].activeElement?this.previous:this.element.val(),t.preventDefault(),i.call(this),this.cancelBlur=!0,this._delay(function(){delete this.cancelBlur,i.call(this)}),this._start(t)!==!1&&this._repeat(null,e(t.currentTarget).hasClass("ui-spinner-up")?1:-1,t)},"mouseup .ui-spinner-button":"_stop","mouseenter .ui-spinner-button":function(t){return e(t.currentTarget).hasClass("ui-state-active")?this._start(t)===!1?!1:(this._repeat(null,e(t.currentTarget).hasClass("ui-spinner-up")?1:-1,t),void 0):void 0},"mouseleave .ui-spinner-button":"_stop"},_draw:function(){var e=this.uiSpinner=this.element.addClass("ui-spinner-input").attr("autocomplete","off").wrap(this._uiSpinnerHtml()).parent().append(this._buttonHtml());this.element.attr("role","spinbutton"),this.buttons=e.find(".ui-spinner-button").attr("tabIndex",-1).button().removeClass("ui-corner-all"),this.buttons.height()>Math.ceil(.5*e.height())&&e.height()>0&&e.height(e.height()),this.options.disabled&&this.disable()},_keydown:function(t){var i=this.options,s=e.ui.keyCode;switch(t.keyCode){case s.UP:return this._repeat(null,1,t),!0;case s.DOWN:return this._repeat(null,-1,t),!0;case s.PAGE_UP:return this._repeat(null,i.page,t),!0;case s.PAGE_DOWN:return this._repeat(null,-i.page,t),!0}return!1},_uiSpinnerHtml:function(){return"<span class='ui-spinner ui-widget ui-widget-content ui-corner-all'></span>"},_buttonHtml:function(){return"<a class='ui-spinner-button ui-spinner-up ui-corner-tr'><span class='ui-icon "+this.options.icons.up+"'>&#9650;</span>"+"</a>"+"<a class='ui-spinner-button ui-spinner-down ui-corner-br'>"+"<span class='ui-icon "+this.options.icons.down+"'>&#9660;</span>"+"</a>"},_start:function(e){return this.spinning||this._trigger("start",e)!==!1?(this.counter||(this.counter=1),this.spinning=!0,!0):!1},_repeat:function(e,t,i){e=e||500,clearTimeout(this.timer),this.timer=this._delay(function(){this._repeat(40,t,i)},e),this._spin(t*this.options.step,i)},_spin:function(e,t){var i=this.value()||0;this.counter||(this.counter=1),i=this._adjustValue(i+e*this._increment(this.counter)),this.spinning&&this._trigger("spin",t,{value:i})===!1||(this._value(i),this.counter++)},_increment:function(t){var i=this.options.incremental;return i?e.isFunction(i)?i(t):Math.floor(t*t*t/5e4-t*t/500+17*t/200+1):1},_precision:function(){var e=this._precisionOf(this.options.step);return null!==this.options.min&&(e=Math.max(e,this._precisionOf(this.options.min))),e},_precisionOf:function(e){var t=""+e,i=t.indexOf(".");return-1===i?0:t.length-i-1},_adjustValue:function(e){var t,i,s=this.options;return t=null!==s.min?s.min:0,i=e-t,i=Math.round(i/s.step)*s.step,e=t+i,e=parseFloat(e.toFixed(this._precision())),null!==s.max&&e>s.max?s.max:null!==s.min&&s.min>e?s.min:e},_stop:function(e){this.spinning&&(clearTimeout(this.timer),clearTimeout(this.mousewheelTimer),this.counter=0,this.spinning=!1,this._trigger("stop",e))},_setOption:function(e,t){if("culture"===e||"numberFormat"===e){var i=this._parse(this.element.val());return this.options[e]=t,this.element.val(this._format(i)),void 0}("max"===e||"min"===e||"step"===e)&&"string"==typeof t&&(t=this._parse(t)),"icons"===e&&(this.buttons.first().find(".ui-icon").removeClass(this.options.icons.up).addClass(t.up),this.buttons.last().find(".ui-icon").removeClass(this.options.icons.down).addClass(t.down)),this._super(e,t),"disabled"===e&&(this.widget().toggleClass("ui-state-disabled",!!t),this.element.prop("disabled",!!t),this.buttons.button(t?"disable":"enable"))},_setOptions:s(function(e){this._super(e)}),_parse:function(e){return"string"==typeof e&&""!==e&&(e=window.Globalize&&this.options.numberFormat?Globalize.parseFloat(e,10,this.options.culture):+e),""===e||isNaN(e)?null:e
+},_format:function(e){return""===e?"":window.Globalize&&this.options.numberFormat?Globalize.format(e,this.options.numberFormat,this.options.culture):e},_refresh:function(){this.element.attr({"aria-valuemin":this.options.min,"aria-valuemax":this.options.max,"aria-valuenow":this._parse(this.element.val())})},isValid:function(){var e=this.value();return null===e?!1:e===this._adjustValue(e)},_value:function(e,t){var i;""!==e&&(i=this._parse(e),null!==i&&(t||(i=this._adjustValue(i)),e=this._format(i))),this.element.val(e),this._refresh()},_destroy:function(){this.element.removeClass("ui-spinner-input").prop("disabled",!1).removeAttr("autocomplete").removeAttr("role").removeAttr("aria-valuemin").removeAttr("aria-valuemax").removeAttr("aria-valuenow"),this.uiSpinner.replaceWith(this.element)},stepUp:s(function(e){this._stepUp(e)}),_stepUp:function(e){this._start()&&(this._spin((e||1)*this.options.step),this._stop())},stepDown:s(function(e){this._stepDown(e)}),_stepDown:function(e){this._start()&&(this._spin((e||1)*-this.options.step),this._stop())},pageUp:s(function(e){this._stepUp((e||1)*this.options.page)}),pageDown:s(function(e){this._stepDown((e||1)*this.options.page)}),value:function(e){return arguments.length?(s(this._value).call(this,e),void 0):this._parse(this.element.val())},widget:function(){return this.uiSpinner}}),e.widget("ui.tabs",{version:"1.11.4",delay:300,options:{active:null,collapsible:!1,event:"click",heightStyle:"content",hide:null,show:null,activate:null,beforeActivate:null,beforeLoad:null,load:null},_isLocal:function(){var e=/#.*$/;return function(t){var i,s;t=t.cloneNode(!1),i=t.href.replace(e,""),s=location.href.replace(e,"");try{i=decodeURIComponent(i)}catch(n){}try{s=decodeURIComponent(s)}catch(n){}return t.hash.length>1&&i===s}}(),_create:function(){var t=this,i=this.options;this.running=!1,this.element.addClass("ui-tabs ui-widget ui-widget-content ui-corner-all").toggleClass("ui-tabs-collapsible",i.collapsible),this._processTabs(),i.active=this._initialActive(),e.isArray(i.disabled)&&(i.disabled=e.unique(i.disabled.concat(e.map(this.tabs.filter(".ui-state-disabled"),function(e){return t.tabs.index(e)}))).sort()),this.active=this.options.active!==!1&&this.anchors.length?this._findActive(i.active):e(),this._refresh(),this.active.length&&this.load(i.active)},_initialActive:function(){var t=this.options.active,i=this.options.collapsible,s=location.hash.substring(1);return null===t&&(s&&this.tabs.each(function(i,n){return e(n).attr("aria-controls")===s?(t=i,!1):void 0}),null===t&&(t=this.tabs.index(this.tabs.filter(".ui-tabs-active"))),(null===t||-1===t)&&(t=this.tabs.length?0:!1)),t!==!1&&(t=this.tabs.index(this.tabs.eq(t)),-1===t&&(t=i?!1:0)),!i&&t===!1&&this.anchors.length&&(t=0),t},_getCreateEventData:function(){return{tab:this.active,panel:this.active.length?this._getPanelForTab(this.active):e()}},_tabKeydown:function(t){var i=e(this.document[0].activeElement).closest("li"),s=this.tabs.index(i),n=!0;if(!this._handlePageNav(t)){switch(t.keyCode){case e.ui.keyCode.RIGHT:case e.ui.keyCode.DOWN:s++;break;case e.ui.keyCode.UP:case e.ui.keyCode.LEFT:n=!1,s--;break;case e.ui.keyCode.END:s=this.anchors.length-1;break;case e.ui.keyCode.HOME:s=0;break;case e.ui.keyCode.SPACE:return t.preventDefault(),clearTimeout(this.activating),this._activate(s),void 0;case e.ui.keyCode.ENTER:return t.preventDefault(),clearTimeout(this.activating),this._activate(s===this.options.active?!1:s),void 0;default:return}t.preventDefault(),clearTimeout(this.activating),s=this._focusNextTab(s,n),t.ctrlKey||t.metaKey||(i.attr("aria-selected","false"),this.tabs.eq(s).attr("aria-selected","true"),this.activating=this._delay(function(){this.option("active",s)},this.delay))}},_panelKeydown:function(t){this._handlePageNav(t)||t.ctrlKey&&t.keyCode===e.ui.keyCode.UP&&(t.preventDefault(),this.active.focus())},_handlePageNav:function(t){return t.altKey&&t.keyCode===e.ui.keyCode.PAGE_UP?(this._activate(this._focusNextTab(this.options.active-1,!1)),!0):t.altKey&&t.keyCode===e.ui.keyCode.PAGE_DOWN?(this._activate(this._focusNextTab(this.options.active+1,!0)),!0):void 0},_findNextTab:function(t,i){function s(){return t>n&&(t=0),0>t&&(t=n),t}for(var n=this.tabs.length-1;-1!==e.inArray(s(),this.options.disabled);)t=i?t+1:t-1;return t},_focusNextTab:function(e,t){return e=this._findNextTab(e,t),this.tabs.eq(e).focus(),e},_setOption:function(e,t){return"active"===e?(this._activate(t),void 0):"disabled"===e?(this._setupDisabled(t),void 0):(this._super(e,t),"collapsible"===e&&(this.element.toggleClass("ui-tabs-collapsible",t),t||this.options.active!==!1||this._activate(0)),"event"===e&&this._setupEvents(t),"heightStyle"===e&&this._setupHeightStyle(t),void 0)},_sanitizeSelector:function(e){return e?e.replace(/[!"$%&'()*+,.\/:;<=>?@\[\]\^`{|}~]/g,"\\$&"):""},refresh:function(){var t=this.options,i=this.tablist.children(":has(a[href])");t.disabled=e.map(i.filter(".ui-state-disabled"),function(e){return i.index(e)}),this._processTabs(),t.active!==!1&&this.anchors.length?this.active.length&&!e.contains(this.tablist[0],this.active[0])?this.tabs.length===t.disabled.length?(t.active=!1,this.active=e()):this._activate(this._findNextTab(Math.max(0,t.active-1),!1)):t.active=this.tabs.index(this.active):(t.active=!1,this.active=e()),this._refresh()},_refresh:function(){this._setupDisabled(this.options.disabled),this._setupEvents(this.options.event),this._setupHeightStyle(this.options.heightStyle),this.tabs.not(this.active).attr({"aria-selected":"false","aria-expanded":"false",tabIndex:-1}),this.panels.not(this._getPanelForTab(this.active)).hide().attr({"aria-hidden":"true"}),this.active.length?(this.active.addClass("ui-tabs-active ui-state-active").attr({"aria-selected":"true","aria-expanded":"true",tabIndex:0}),this._getPanelForTab(this.active).show().attr({"aria-hidden":"false"})):this.tabs.eq(0).attr("tabIndex",0)},_processTabs:function(){var t=this,i=this.tabs,s=this.anchors,n=this.panels;this.tablist=this._getList().addClass("ui-tabs-nav ui-helper-reset ui-helper-clearfix ui-widget-header ui-corner-all").attr("role","tablist").delegate("> li","mousedown"+this.eventNamespace,function(t){e(this).is(".ui-state-disabled")&&t.preventDefault()}).delegate(".ui-tabs-anchor","focus"+this.eventNamespace,function(){e(this).closest("li").is(".ui-state-disabled")&&this.blur()}),this.tabs=this.tablist.find("> li:has(a[href])").addClass("ui-state-default ui-corner-top").attr({role:"tab",tabIndex:-1}),this.anchors=this.tabs.map(function(){return e("a",this)[0]}).addClass("ui-tabs-anchor").attr({role:"presentation",tabIndex:-1}),this.panels=e(),this.anchors.each(function(i,s){var n,a,o,r=e(s).uniqueId().attr("id"),h=e(s).closest("li"),l=h.attr("aria-controls");t._isLocal(s)?(n=s.hash,o=n.substring(1),a=t.element.find(t._sanitizeSelector(n))):(o=h.attr("aria-controls")||e({}).uniqueId()[0].id,n="#"+o,a=t.element.find(n),a.length||(a=t._createPanel(o),a.insertAfter(t.panels[i-1]||t.tablist)),a.attr("aria-live","polite")),a.length&&(t.panels=t.panels.add(a)),l&&h.data("ui-tabs-aria-controls",l),h.attr({"aria-controls":o,"aria-labelledby":r}),a.attr("aria-labelledby",r)}),this.panels.addClass("ui-tabs-panel ui-widget-content ui-corner-bottom").attr("role","tabpanel"),i&&(this._off(i.not(this.tabs)),this._off(s.not(this.anchors)),this._off(n.not(this.panels)))},_getList:function(){return this.tablist||this.element.find("ol,ul").eq(0)},_createPanel:function(t){return e("<div>").attr("id",t).addClass("ui-tabs-panel ui-widget-content ui-corner-bottom").data("ui-tabs-destroy",!0)},_setupDisabled:function(t){e.isArray(t)&&(t.length?t.length===this.anchors.length&&(t=!0):t=!1);for(var i,s=0;i=this.tabs[s];s++)t===!0||-1!==e.inArray(s,t)?e(i).addClass("ui-state-disabled").attr("aria-disabled","true"):e(i).removeClass("ui-state-disabled").removeAttr("aria-disabled");this.options.disabled=t},_setupEvents:function(t){var i={};t&&e.each(t.split(" "),function(e,t){i[t]="_eventHandler"}),this._off(this.anchors.add(this.tabs).add(this.panels)),this._on(!0,this.anchors,{click:function(e){e.preventDefault()}}),this._on(this.anchors,i),this._on(this.tabs,{keydown:"_tabKeydown"}),this._on(this.panels,{keydown:"_panelKeydown"}),this._focusable(this.tabs),this._hoverable(this.tabs)},_setupHeightStyle:function(t){var i,s=this.element.parent();"fill"===t?(i=s.height(),i-=this.element.outerHeight()-this.element.height(),this.element.siblings(":visible").each(function(){var t=e(this),s=t.css("position");"absolute"!==s&&"fixed"!==s&&(i-=t.outerHeight(!0))}),this.element.children().not(this.panels).each(function(){i-=e(this).outerHeight(!0)}),this.panels.each(function(){e(this).height(Math.max(0,i-e(this).innerHeight()+e(this).height()))}).css("overflow","auto")):"auto"===t&&(i=0,this.panels.each(function(){i=Math.max(i,e(this).height("").height())}).height(i))},_eventHandler:function(t){var i=this.options,s=this.active,n=e(t.currentTarget),a=n.closest("li"),o=a[0]===s[0],r=o&&i.collapsible,h=r?e():this._getPanelForTab(a),l=s.length?this._getPanelForTab(s):e(),u={oldTab:s,oldPanel:l,newTab:r?e():a,newPanel:h};t.preventDefault(),a.hasClass("ui-state-disabled")||a.hasClass("ui-tabs-loading")||this.running||o&&!i.collapsible||this._trigger("beforeActivate",t,u)===!1||(i.active=r?!1:this.tabs.index(a),this.active=o?e():a,this.xhr&&this.xhr.abort(),l.length||h.length||e.error("jQuery UI Tabs: Mismatching fragment identifier."),h.length&&this.load(this.tabs.index(a),t),this._toggle(t,u))},_toggle:function(t,i){function s(){a.running=!1,a._trigger("activate",t,i)}function n(){i.newTab.closest("li").addClass("ui-tabs-active ui-state-active"),o.length&&a.options.show?a._show(o,a.options.show,s):(o.show(),s())}var a=this,o=i.newPanel,r=i.oldPanel;this.running=!0,r.length&&this.options.hide?this._hide(r,this.options.hide,function(){i.oldTab.closest("li").removeClass("ui-tabs-active ui-state-active"),n()}):(i.oldTab.closest("li").removeClass("ui-tabs-active ui-state-active"),r.hide(),n()),r.attr("aria-hidden","true"),i.oldTab.attr({"aria-selected":"false","aria-expanded":"false"}),o.length&&r.length?i.oldTab.attr("tabIndex",-1):o.length&&this.tabs.filter(function(){return 0===e(this).attr("tabIndex")}).attr("tabIndex",-1),o.attr("aria-hidden","false"),i.newTab.attr({"aria-selected":"true","aria-expanded":"true",tabIndex:0})},_activate:function(t){var i,s=this._findActive(t);s[0]!==this.active[0]&&(s.length||(s=this.active),i=s.find(".ui-tabs-anchor")[0],this._eventHandler({target:i,currentTarget:i,preventDefault:e.noop}))},_findActive:function(t){return t===!1?e():this.tabs.eq(t)},_getIndex:function(e){return"string"==typeof e&&(e=this.anchors.index(this.anchors.filter("[href$='"+e+"']"))),e},_destroy:function(){this.xhr&&this.xhr.abort(),this.element.removeClass("ui-tabs ui-widget ui-widget-content ui-corner-all ui-tabs-collapsible"),this.tablist.removeClass("ui-tabs-nav ui-helper-reset ui-helper-clearfix ui-widget-header ui-corner-all").removeAttr("role"),this.anchors.removeClass("ui-tabs-anchor").removeAttr("role").removeAttr("tabIndex").removeUniqueId(),this.tablist.unbind(this.eventNamespace),this.tabs.add(this.panels).each(function(){e.data(this,"ui-tabs-destroy")?e(this).remove():e(this).removeClass("ui-state-default ui-state-active ui-state-disabled ui-corner-top ui-corner-bottom ui-widget-content ui-tabs-active ui-tabs-panel").removeAttr("tabIndex").removeAttr("aria-live").removeAttr("aria-busy").removeAttr("aria-selected").removeAttr("aria-labelledby").removeAttr("aria-hidden").removeAttr("aria-expanded").removeAttr("role")}),this.tabs.each(function(){var t=e(this),i=t.data("ui-tabs-aria-controls");i?t.attr("aria-controls",i).removeData("ui-tabs-aria-controls"):t.removeAttr("aria-controls")}),this.panels.show(),"content"!==this.options.heightStyle&&this.panels.css("height","")},enable:function(t){var i=this.options.disabled;i!==!1&&(void 0===t?i=!1:(t=this._getIndex(t),i=e.isArray(i)?e.map(i,function(e){return e!==t?e:null}):e.map(this.tabs,function(e,i){return i!==t?i:null})),this._setupDisabled(i))},disable:function(t){var i=this.options.disabled;if(i!==!0){if(void 0===t)i=!0;else{if(t=this._getIndex(t),-1!==e.inArray(t,i))return;i=e.isArray(i)?e.merge([t],i).sort():[t]}this._setupDisabled(i)}},load:function(t,i){t=this._getIndex(t);var s=this,n=this.tabs.eq(t),a=n.find(".ui-tabs-anchor"),o=this._getPanelForTab(n),r={tab:n,panel:o},h=function(e,t){"abort"===t&&s.panels.stop(!1,!0),n.removeClass("ui-tabs-loading"),o.removeAttr("aria-busy"),e===s.xhr&&delete s.xhr};this._isLocal(a[0])||(this.xhr=e.ajax(this._ajaxSettings(a,i,r)),this.xhr&&"canceled"!==this.xhr.statusText&&(n.addClass("ui-tabs-loading"),o.attr("aria-busy","true"),this.xhr.done(function(e,t,n){setTimeout(function(){o.html(e),s._trigger("load",i,r),h(n,t)},1)}).fail(function(e,t){setTimeout(function(){h(e,t)},1)})))},_ajaxSettings:function(t,i,s){var n=this;return{url:t.attr("href"),beforeSend:function(t,a){return n._trigger("beforeLoad",i,e.extend({jqXHR:t,ajaxSettings:a},s))}}},_getPanelForTab:function(t){var i=e(t).attr("aria-controls");return this.element.find(this._sanitizeSelector("#"+i))}}),e.widget("ui.tooltip",{version:"1.11.4",options:{content:function(){var t=e(this).attr("title")||"";return e("<a>").text(t).html()},hide:!0,items:"[title]:not([disabled])",position:{my:"left top+15",at:"left bottom",collision:"flipfit flip"},show:!0,tooltipClass:null,track:!1,close:null,open:null},_addDescribedBy:function(t,i){var s=(t.attr("aria-describedby")||"").split(/\s+/);s.push(i),t.data("ui-tooltip-id",i).attr("aria-describedby",e.trim(s.join(" ")))},_removeDescribedBy:function(t){var i=t.data("ui-tooltip-id"),s=(t.attr("aria-describedby")||"").split(/\s+/),n=e.inArray(i,s);-1!==n&&s.splice(n,1),t.removeData("ui-tooltip-id"),s=e.trim(s.join(" ")),s?t.attr("aria-describedby",s):t.removeAttr("aria-describedby")},_create:function(){this._on({mouseover:"open",focusin:"open"}),this.tooltips={},this.parents={},this.options.disabled&&this._disable(),this.liveRegion=e("<div>").attr({role:"log","aria-live":"assertive","aria-relevant":"additions"}).addClass("ui-helper-hidden-accessible").appendTo(this.document[0].body)},_setOption:function(t,i){var s=this;return"disabled"===t?(this[i?"_disable":"_enable"](),this.options[t]=i,void 0):(this._super(t,i),"content"===t&&e.each(this.tooltips,function(e,t){s._updateContent(t.element)}),void 0)},_disable:function(){var t=this;e.each(this.tooltips,function(i,s){var n=e.Event("blur");n.target=n.currentTarget=s.element[0],t.close(n,!0)}),this.element.find(this.options.items).addBack().each(function(){var t=e(this);t.is("[title]")&&t.data("ui-tooltip-title",t.attr("title")).removeAttr("title")})},_enable:function(){this.element.find(this.options.items).addBack().each(function(){var t=e(this);t.data("ui-tooltip-title")&&t.attr("title",t.data("ui-tooltip-title"))})},open:function(t){var i=this,s=e(t?t.target:this.element).closest(this.options.items);s.length&&!s.data("ui-tooltip-id")&&(s.attr("title")&&s.data("ui-tooltip-title",s.attr("title")),s.data("ui-tooltip-open",!0),t&&"mouseover"===t.type&&s.parents().each(function(){var t,s=e(this);s.data("ui-tooltip-open")&&(t=e.Event("blur"),t.target=t.currentTarget=this,i.close(t,!0)),s.attr("title")&&(s.uniqueId(),i.parents[this.id]={element:this,title:s.attr("title")},s.attr("title",""))}),this._registerCloseHandlers(t,s),this._updateContent(s,t))},_updateContent:function(e,t){var i,s=this.options.content,n=this,a=t?t.type:null;return"string"==typeof s?this._open(t,e,s):(i=s.call(e[0],function(i){n._delay(function(){e.data("ui-tooltip-open")&&(t&&(t.type=a),this._open(t,e,i))})}),i&&this._open(t,e,i),void 0)},_open:function(t,i,s){function n(e){l.of=e,o.is(":hidden")||o.position(l)}var a,o,r,h,l=e.extend({},this.options.position);if(s){if(a=this._find(i))return a.tooltip.find(".ui-tooltip-content").html(s),void 0;i.is("[title]")&&(t&&"mouseover"===t.type?i.attr("title",""):i.removeAttr("title")),a=this._tooltip(i),o=a.tooltip,this._addDescribedBy(i,o.attr("id")),o.find(".ui-tooltip-content").html(s),this.liveRegion.children().hide(),s.clone?(h=s.clone(),h.removeAttr("id").find("[id]").removeAttr("id")):h=s,e("<div>").html(h).appendTo(this.liveRegion),this.options.track&&t&&/^mouse/.test(t.type)?(this._on(this.document,{mousemove:n}),n(t)):o.position(e.extend({of:i},this.options.position)),o.hide(),this._show(o,this.options.show),this.options.show&&this.options.show.delay&&(r=this.delayedShow=setInterval(function(){o.is(":visible")&&(n(l.of),clearInterval(r))},e.fx.interval)),this._trigger("open",t,{tooltip:o})}},_registerCloseHandlers:function(t,i){var s={keyup:function(t){if(t.keyCode===e.ui.keyCode.ESCAPE){var s=e.Event(t);s.currentTarget=i[0],this.close(s,!0)}}};i[0]!==this.element[0]&&(s.remove=function(){this._removeTooltip(this._find(i).tooltip)}),t&&"mouseover"!==t.type||(s.mouseleave="close"),t&&"focusin"!==t.type||(s.focusout="close"),this._on(!0,i,s)},close:function(t){var i,s=this,n=e(t?t.currentTarget:this.element),a=this._find(n);return a?(i=a.tooltip,a.closing||(clearInterval(this.delayedShow),n.data("ui-tooltip-title")&&!n.attr("title")&&n.attr("title",n.data("ui-tooltip-title")),this._removeDescribedBy(n),a.hiding=!0,i.stop(!0),this._hide(i,this.options.hide,function(){s._removeTooltip(e(this))}),n.removeData("ui-tooltip-open"),this._off(n,"mouseleave focusout keyup"),n[0]!==this.element[0]&&this._off(n,"remove"),this._off(this.document,"mousemove"),t&&"mouseleave"===t.type&&e.each(this.parents,function(t,i){e(i.element).attr("title",i.title),delete s.parents[t]}),a.closing=!0,this._trigger("close",t,{tooltip:i}),a.hiding||(a.closing=!1)),void 0):(n.removeData("ui-tooltip-open"),void 0)},_tooltip:function(t){var i=e("<div>").attr("role","tooltip").addClass("ui-tooltip ui-widget ui-corner-all ui-widget-content "+(this.options.tooltipClass||"")),s=i.uniqueId().attr("id");return e("<div>").addClass("ui-tooltip-content").appendTo(i),i.appendTo(this.document[0].body),this.tooltips[s]={element:t,tooltip:i}},_find:function(e){var t=e.data("ui-tooltip-id");return t?this.tooltips[t]:null},_removeTooltip:function(e){e.remove(),delete this.tooltips[e.attr("id")]},_destroy:function(){var t=this;e.each(this.tooltips,function(i,s){var n=e.Event("blur"),a=s.element;n.target=n.currentTarget=a[0],t.close(n,!0),e("#"+i).remove(),a.data("ui-tooltip-title")&&(a.attr("title")||a.attr("title",a.data("ui-tooltip-title")),a.removeData("ui-tooltip-title"))}),this.liveRegion.remove()}});var c="ui-effects-",p=e;e.effects={effect:{}},function(e,t){function i(e,t,i){var s=d[t.type]||{};return null==e?i||!t.def?null:t.def:(e=s.floor?~~e:parseFloat(e),isNaN(e)?t.def:s.mod?(e+s.mod)%s.mod:0>e?0:e>s.max?s.max:e)}function s(i){var s=l(),n=s._rgba=[];return i=i.toLowerCase(),f(h,function(e,a){var o,r=a.re.exec(i),h=r&&a.parse(r),l=a.space||"rgba";return h?(o=s[l](h),s[u[l].cache]=o[u[l].cache],n=s._rgba=o._rgba,!1):t}),n.length?("0,0,0,0"===n.join()&&e.extend(n,a.transparent),s):a[i]}function n(e,t,i){return i=(i+1)%1,1>6*i?e+6*(t-e)*i:1>2*i?t:2>3*i?e+6*(t-e)*(2/3-i):e}var a,o="backgroundColor borderBottomColor borderLeftColor borderRightColor borderTopColor color columnRuleColor outlineColor textDecorationColor textEmphasisColor",r=/^([\-+])=\s*(\d+\.?\d*)/,h=[{re:/rgba?\(\s*(\d{1,3})\s*,\s*(\d{1,3})\s*,\s*(\d{1,3})\s*(?:,\s*(\d?(?:\.\d+)?)\s*)?\)/,parse:function(e){return[e[1],e[2],e[3],e[4]]}},{re:/rgba?\(\s*(\d+(?:\.\d+)?)\%\s*,\s*(\d+(?:\.\d+)?)\%\s*,\s*(\d+(?:\.\d+)?)\%\s*(?:,\s*(\d?(?:\.\d+)?)\s*)?\)/,parse:function(e){return[2.55*e[1],2.55*e[2],2.55*e[3],e[4]]}},{re:/#([a-f0-9]{2})([a-f0-9]{2})([a-f0-9]{2})/,parse:function(e){return[parseInt(e[1],16),parseInt(e[2],16),parseInt(e[3],16)]}},{re:/#([a-f0-9])([a-f0-9])([a-f0-9])/,parse:function(e){return[parseInt(e[1]+e[1],16),parseInt(e[2]+e[2],16),parseInt(e[3]+e[3],16)]}},{re:/hsla?\(\s*(\d+(?:\.\d+)?)\s*,\s*(\d+(?:\.\d+)?)\%\s*,\s*(\d+(?:\.\d+)?)\%\s*(?:,\s*(\d?(?:\.\d+)?)\s*)?\)/,space:"hsla",parse:function(e){return[e[1],e[2]/100,e[3]/100,e[4]]}}],l=e.Color=function(t,i,s,n){return new e.Color.fn.parse(t,i,s,n)},u={rgba:{props:{red:{idx:0,type:"byte"},green:{idx:1,type:"byte"},blue:{idx:2,type:"byte"}}},hsla:{props:{hue:{idx:0,type:"degrees"},saturation:{idx:1,type:"percent"},lightness:{idx:2,type:"percent"}}}},d={"byte":{floor:!0,max:255},percent:{max:1},degrees:{mod:360,floor:!0}},c=l.support={},p=e("<p>")[0],f=e.each;p.style.cssText="background-color:rgba(1,1,1,.5)",c.rgba=p.style.backgroundColor.indexOf("rgba")>-1,f(u,function(e,t){t.cache="_"+e,t.props.alpha={idx:3,type:"percent",def:1}}),l.fn=e.extend(l.prototype,{parse:function(n,o,r,h){if(n===t)return this._rgba=[null,null,null,null],this;(n.jquery||n.nodeType)&&(n=e(n).css(o),o=t);var d=this,c=e.type(n),p=this._rgba=[];return o!==t&&(n=[n,o,r,h],c="array"),"string"===c?this.parse(s(n)||a._default):"array"===c?(f(u.rgba.props,function(e,t){p[t.idx]=i(n[t.idx],t)}),this):"object"===c?(n instanceof l?f(u,function(e,t){n[t.cache]&&(d[t.cache]=n[t.cache].slice())}):f(u,function(t,s){var a=s.cache;f(s.props,function(e,t){if(!d[a]&&s.to){if("alpha"===e||null==n[e])return;d[a]=s.to(d._rgba)}d[a][t.idx]=i(n[e],t,!0)}),d[a]&&0>e.inArray(null,d[a].slice(0,3))&&(d[a][3]=1,s.from&&(d._rgba=s.from(d[a])))}),this):t},is:function(e){var i=l(e),s=!0,n=this;return f(u,function(e,a){var o,r=i[a.cache];return r&&(o=n[a.cache]||a.to&&a.to(n._rgba)||[],f(a.props,function(e,i){return null!=r[i.idx]?s=r[i.idx]===o[i.idx]:t})),s}),s},_space:function(){var e=[],t=this;return f(u,function(i,s){t[s.cache]&&e.push(i)}),e.pop()},transition:function(e,t){var s=l(e),n=s._space(),a=u[n],o=0===this.alpha()?l("transparent"):this,r=o[a.cache]||a.to(o._rgba),h=r.slice();return s=s[a.cache],f(a.props,function(e,n){var a=n.idx,o=r[a],l=s[a],u=d[n.type]||{};null!==l&&(null===o?h[a]=l:(u.mod&&(l-o>u.mod/2?o+=u.mod:o-l>u.mod/2&&(o-=u.mod)),h[a]=i((l-o)*t+o,n)))}),this[n](h)},blend:function(t){if(1===this._rgba[3])return this;var i=this._rgba.slice(),s=i.pop(),n=l(t)._rgba;return l(e.map(i,function(e,t){return(1-s)*n[t]+s*e}))},toRgbaString:function(){var t="rgba(",i=e.map(this._rgba,function(e,t){return null==e?t>2?1:0:e});return 1===i[3]&&(i.pop(),t="rgb("),t+i.join()+")"},toHslaString:function(){var t="hsla(",i=e.map(this.hsla(),function(e,t){return null==e&&(e=t>2?1:0),t&&3>t&&(e=Math.round(100*e)+"%"),e});return 1===i[3]&&(i.pop(),t="hsl("),t+i.join()+")"},toHexString:function(t){var i=this._rgba.slice(),s=i.pop();return t&&i.push(~~(255*s)),"#"+e.map(i,function(e){return e=(e||0).toString(16),1===e.length?"0"+e:e}).join("")},toString:function(){return 0===this._rgba[3]?"transparent":this.toRgbaString()}}),l.fn.parse.prototype=l.fn,u.hsla.to=function(e){if(null==e[0]||null==e[1]||null==e[2])return[null,null,null,e[3]];var t,i,s=e[0]/255,n=e[1]/255,a=e[2]/255,o=e[3],r=Math.max(s,n,a),h=Math.min(s,n,a),l=r-h,u=r+h,d=.5*u;return t=h===r?0:s===r?60*(n-a)/l+360:n===r?60*(a-s)/l+120:60*(s-n)/l+240,i=0===l?0:.5>=d?l/u:l/(2-u),[Math.round(t)%360,i,d,null==o?1:o]},u.hsla.from=function(e){if(null==e[0]||null==e[1]||null==e[2])return[null,null,null,e[3]];var t=e[0]/360,i=e[1],s=e[2],a=e[3],o=.5>=s?s*(1+i):s+i-s*i,r=2*s-o;return[Math.round(255*n(r,o,t+1/3)),Math.round(255*n(r,o,t)),Math.round(255*n(r,o,t-1/3)),a]},f(u,function(s,n){var a=n.props,o=n.cache,h=n.to,u=n.from;l.fn[s]=function(s){if(h&&!this[o]&&(this[o]=h(this._rgba)),s===t)return this[o].slice();var n,r=e.type(s),d="array"===r||"object"===r?s:arguments,c=this[o].slice();return f(a,function(e,t){var s=d["object"===r?e:t.idx];null==s&&(s=c[t.idx]),c[t.idx]=i(s,t)}),u?(n=l(u(c)),n[o]=c,n):l(c)},f(a,function(t,i){l.fn[t]||(l.fn[t]=function(n){var a,o=e.type(n),h="alpha"===t?this._hsla?"hsla":"rgba":s,l=this[h](),u=l[i.idx];return"undefined"===o?u:("function"===o&&(n=n.call(this,u),o=e.type(n)),null==n&&i.empty?this:("string"===o&&(a=r.exec(n),a&&(n=u+parseFloat(a[2])*("+"===a[1]?1:-1))),l[i.idx]=n,this[h](l)))})})}),l.hook=function(t){var i=t.split(" ");f(i,function(t,i){e.cssHooks[i]={set:function(t,n){var a,o,r="";if("transparent"!==n&&("string"!==e.type(n)||(a=s(n)))){if(n=l(a||n),!c.rgba&&1!==n._rgba[3]){for(o="backgroundColor"===i?t.parentNode:t;(""===r||"transparent"===r)&&o&&o.style;)try{r=e.css(o,"backgroundColor"),o=o.parentNode}catch(h){}n=n.blend(r&&"transparent"!==r?r:"_default")}n=n.toRgbaString()}try{t.style[i]=n}catch(h){}}},e.fx.step[i]=function(t){t.colorInit||(t.start=l(t.elem,i),t.end=l(t.end),t.colorInit=!0),e.cssHooks[i].set(t.elem,t.start.transition(t.end,t.pos))}})},l.hook(o),e.cssHooks.borderColor={expand:function(e){var t={};return f(["Top","Right","Bottom","Left"],function(i,s){t["border"+s+"Color"]=e}),t}},a=e.Color.names={aqua:"#00ffff",black:"#000000",blue:"#0000ff",fuchsia:"#ff00ff",gray:"#808080",green:"#008000",lime:"#00ff00",maroon:"#800000",navy:"#000080",olive:"#808000",purple:"#800080",red:"#ff0000",silver:"#c0c0c0",teal:"#008080",white:"#ffffff",yellow:"#ffff00",transparent:[null,null,null,0],_default:"#ffffff"}}(p),function(){function t(t){var i,s,n=t.ownerDocument.defaultView?t.ownerDocument.defaultView.getComputedStyle(t,null):t.currentStyle,a={};if(n&&n.length&&n[0]&&n[n[0]])for(s=n.length;s--;)i=n[s],"string"==typeof n[i]&&(a[e.camelCase(i)]=n[i]);else for(i in n)"string"==typeof n[i]&&(a[i]=n[i]);return a}function i(t,i){var s,a,o={};for(s in i)a=i[s],t[s]!==a&&(n[s]||(e.fx.step[s]||!isNaN(parseFloat(a)))&&(o[s]=a));return o}var s=["add","remove","toggle"],n={border:1,borderBottom:1,borderColor:1,borderLeft:1,borderRight:1,borderTop:1,borderWidth:1,margin:1,padding:1};e.each(["borderLeftStyle","borderRightStyle","borderBottomStyle","borderTopStyle"],function(t,i){e.fx.step[i]=function(e){("none"!==e.end&&!e.setAttr||1===e.pos&&!e.setAttr)&&(p.style(e.elem,i,e.end),e.setAttr=!0)}}),e.fn.addBack||(e.fn.addBack=function(e){return this.add(null==e?this.prevObject:this.prevObject.filter(e))}),e.effects.animateClass=function(n,a,o,r){var h=e.speed(a,o,r);return this.queue(function(){var a,o=e(this),r=o.attr("class")||"",l=h.children?o.find("*").addBack():o;l=l.map(function(){var i=e(this);return{el:i,start:t(this)}}),a=function(){e.each(s,function(e,t){n[t]&&o[t+"Class"](n[t])})},a(),l=l.map(function(){return this.end=t(this.el[0]),this.diff=i(this.start,this.end),this}),o.attr("class",r),l=l.map(function(){var t=this,i=e.Deferred(),s=e.extend({},h,{queue:!1,complete:function(){i.resolve(t)}});return this.el.animate(this.diff,s),i.promise()}),e.when.apply(e,l.get()).done(function(){a(),e.each(arguments,function(){var t=this.el;e.each(this.diff,function(e){t.css(e,"")})}),h.complete.call(o[0])})})},e.fn.extend({addClass:function(t){return function(i,s,n,a){return s?e.effects.animateClass.call(this,{add:i},s,n,a):t.apply(this,arguments)}}(e.fn.addClass),removeClass:function(t){return function(i,s,n,a){return arguments.length>1?e.effects.animateClass.call(this,{remove:i},s,n,a):t.apply(this,arguments)}}(e.fn.removeClass),toggleClass:function(t){return function(i,s,n,a,o){return"boolean"==typeof s||void 0===s?n?e.effects.animateClass.call(this,s?{add:i}:{remove:i},n,a,o):t.apply(this,arguments):e.effects.animateClass.call(this,{toggle:i},s,n,a)}}(e.fn.toggleClass),switchClass:function(t,i,s,n,a){return e.effects.animateClass.call(this,{add:i,remove:t},s,n,a)}})}(),function(){function t(t,i,s,n){return e.isPlainObject(t)&&(i=t,t=t.effect),t={effect:t},null==i&&(i={}),e.isFunction(i)&&(n=i,s=null,i={}),("number"==typeof i||e.fx.speeds[i])&&(n=s,s=i,i={}),e.isFunction(s)&&(n=s,s=null),i&&e.extend(t,i),s=s||i.duration,t.duration=e.fx.off?0:"number"==typeof s?s:s in e.fx.speeds?e.fx.speeds[s]:e.fx.speeds._default,t.complete=n||i.complete,t}function i(t){return!t||"number"==typeof t||e.fx.speeds[t]?!0:"string"!=typeof t||e.effects.effect[t]?e.isFunction(t)?!0:"object"!=typeof t||t.effect?!1:!0:!0}e.extend(e.effects,{version:"1.11.4",save:function(e,t){for(var i=0;t.length>i;i++)null!==t[i]&&e.data(c+t[i],e[0].style[t[i]])},restore:function(e,t){var i,s;for(s=0;t.length>s;s++)null!==t[s]&&(i=e.data(c+t[s]),void 0===i&&(i=""),e.css(t[s],i))},setMode:function(e,t){return"toggle"===t&&(t=e.is(":hidden")?"show":"hide"),t},getBaseline:function(e,t){var i,s;switch(e[0]){case"top":i=0;break;case"middle":i=.5;break;case"bottom":i=1;break;default:i=e[0]/t.height}switch(e[1]){case"left":s=0;break;case"center":s=.5;break;case"right":s=1;break;default:s=e[1]/t.width}return{x:s,y:i}},createWrapper:function(t){if(t.parent().is(".ui-effects-wrapper"))return t.parent();var i={width:t.outerWidth(!0),height:t.outerHeight(!0),"float":t.css("float")},s=e("<div></div>").addClass("ui-effects-wrapper").css({fontSize:"100%",background:"transparent",border:"none",margin:0,padding:0}),n={width:t.width(),height:t.height()},a=document.activeElement;try{a.id}catch(o){a=document.body}return t.wrap(s),(t[0]===a||e.contains(t[0],a))&&e(a).focus(),s=t.parent(),"static"===t.css("position")?(s.css({position:"relative"}),t.css({position:"relative"})):(e.extend(i,{position:t.css("position"),zIndex:t.css("z-index")}),e.each(["top","left","bottom","right"],function(e,s){i[s]=t.css(s),isNaN(parseInt(i[s],10))&&(i[s]="auto")}),t.css({position:"relative",top:0,left:0,right:"auto",bottom:"auto"})),t.css(n),s.css(i).show()},removeWrapper:function(t){var i=document.activeElement;return t.parent().is(".ui-effects-wrapper")&&(t.parent().replaceWith(t),(t[0]===i||e.contains(t[0],i))&&e(i).focus()),t},setTransition:function(t,i,s,n){return n=n||{},e.each(i,function(e,i){var a=t.cssUnit(i);a[0]>0&&(n[i]=a[0]*s+a[1])}),n}}),e.fn.extend({effect:function(){function i(t){function i(){e.isFunction(a)&&a.call(n[0]),e.isFunction(t)&&t()}var n=e(this),a=s.complete,r=s.mode;(n.is(":hidden")?"hide"===r:"show"===r)?(n[r](),i()):o.call(n[0],s,i)}var s=t.apply(this,arguments),n=s.mode,a=s.queue,o=e.effects.effect[s.effect];return e.fx.off||!o?n?this[n](s.duration,s.complete):this.each(function(){s.complete&&s.complete.call(this)}):a===!1?this.each(i):this.queue(a||"fx",i)},show:function(e){return function(s){if(i(s))return e.apply(this,arguments);var n=t.apply(this,arguments);return n.mode="show",this.effect.call(this,n)}}(e.fn.show),hide:function(e){return function(s){if(i(s))return e.apply(this,arguments);var n=t.apply(this,arguments);return n.mode="hide",this.effect.call(this,n)}}(e.fn.hide),toggle:function(e){return function(s){if(i(s)||"boolean"==typeof s)return e.apply(this,arguments);var n=t.apply(this,arguments);return n.mode="toggle",this.effect.call(this,n)}}(e.fn.toggle),cssUnit:function(t){var i=this.css(t),s=[];return e.each(["em","px","%","pt"],function(e,t){i.indexOf(t)>0&&(s=[parseFloat(i),t])}),s}})}(),function(){var t={};e.each(["Quad","Cubic","Quart","Quint","Expo"],function(e,i){t[i]=function(t){return Math.pow(t,e+2)}}),e.extend(t,{Sine:function(e){return 1-Math.cos(e*Math.PI/2)},Circ:function(e){return 1-Math.sqrt(1-e*e)},Elastic:function(e){return 0===e||1===e?e:-Math.pow(2,8*(e-1))*Math.sin((80*(e-1)-7.5)*Math.PI/15)},Back:function(e){return e*e*(3*e-2)},Bounce:function(e){for(var t,i=4;((t=Math.pow(2,--i))-1)/11>e;);return 1/Math.pow(4,3-i)-7.5625*Math.pow((3*t-2)/22-e,2)}}),e.each(t,function(t,i){e.easing["easeIn"+t]=i,e.easing["easeOut"+t]=function(e){return 1-i(1-e)},e.easing["easeInOut"+t]=function(e){return.5>e?i(2*e)/2:1-i(-2*e+2)/2}})}(),e.effects,e.effects.effect.blind=function(t,i){var s,n,a,o=e(this),r=/up|down|vertical/,h=/up|left|vertical|horizontal/,l=["position","top","bottom","left","right","height","width"],u=e.effects.setMode(o,t.mode||"hide"),d=t.direction||"up",c=r.test(d),p=c?"height":"width",f=c?"top":"left",m=h.test(d),g={},v="show"===u;o.parent().is(".ui-effects-wrapper")?e.effects.save(o.parent(),l):e.effects.save(o,l),o.show(),s=e.effects.createWrapper(o).css({overflow:"hidden"}),n=s[p](),a=parseFloat(s.css(f))||0,g[p]=v?n:0,m||(o.css(c?"bottom":"right",0).css(c?"top":"left","auto").css({position:"absolute"}),g[f]=v?a:n+a),v&&(s.css(p,0),m||s.css(f,a+n)),s.animate(g,{duration:t.duration,easing:t.easing,queue:!1,complete:function(){"hide"===u&&o.hide(),e.effects.restore(o,l),e.effects.removeWrapper(o),i()
+}})},e.effects.effect.bounce=function(t,i){var s,n,a,o=e(this),r=["position","top","bottom","left","right","height","width"],h=e.effects.setMode(o,t.mode||"effect"),l="hide"===h,u="show"===h,d=t.direction||"up",c=t.distance,p=t.times||5,f=2*p+(u||l?1:0),m=t.duration/f,g=t.easing,v="up"===d||"down"===d?"top":"left",y="up"===d||"left"===d,b=o.queue(),_=b.length;for((u||l)&&r.push("opacity"),e.effects.save(o,r),o.show(),e.effects.createWrapper(o),c||(c=o["top"===v?"outerHeight":"outerWidth"]()/3),u&&(a={opacity:1},a[v]=0,o.css("opacity",0).css(v,y?2*-c:2*c).animate(a,m,g)),l&&(c/=Math.pow(2,p-1)),a={},a[v]=0,s=0;p>s;s++)n={},n[v]=(y?"-=":"+=")+c,o.animate(n,m,g).animate(a,m,g),c=l?2*c:c/2;l&&(n={opacity:0},n[v]=(y?"-=":"+=")+c,o.animate(n,m,g)),o.queue(function(){l&&o.hide(),e.effects.restore(o,r),e.effects.removeWrapper(o),i()}),_>1&&b.splice.apply(b,[1,0].concat(b.splice(_,f+1))),o.dequeue()},e.effects.effect.clip=function(t,i){var s,n,a,o=e(this),r=["position","top","bottom","left","right","height","width"],h=e.effects.setMode(o,t.mode||"hide"),l="show"===h,u=t.direction||"vertical",d="vertical"===u,c=d?"height":"width",p=d?"top":"left",f={};e.effects.save(o,r),o.show(),s=e.effects.createWrapper(o).css({overflow:"hidden"}),n="IMG"===o[0].tagName?s:o,a=n[c](),l&&(n.css(c,0),n.css(p,a/2)),f[c]=l?a:0,f[p]=l?0:a/2,n.animate(f,{queue:!1,duration:t.duration,easing:t.easing,complete:function(){l||o.hide(),e.effects.restore(o,r),e.effects.removeWrapper(o),i()}})},e.effects.effect.drop=function(t,i){var s,n=e(this),a=["position","top","bottom","left","right","opacity","height","width"],o=e.effects.setMode(n,t.mode||"hide"),r="show"===o,h=t.direction||"left",l="up"===h||"down"===h?"top":"left",u="up"===h||"left"===h?"pos":"neg",d={opacity:r?1:0};e.effects.save(n,a),n.show(),e.effects.createWrapper(n),s=t.distance||n["top"===l?"outerHeight":"outerWidth"](!0)/2,r&&n.css("opacity",0).css(l,"pos"===u?-s:s),d[l]=(r?"pos"===u?"+=":"-=":"pos"===u?"-=":"+=")+s,n.animate(d,{queue:!1,duration:t.duration,easing:t.easing,complete:function(){"hide"===o&&n.hide(),e.effects.restore(n,a),e.effects.removeWrapper(n),i()}})},e.effects.effect.explode=function(t,i){function s(){b.push(this),b.length===d*c&&n()}function n(){p.css({visibility:"visible"}),e(b).remove(),m||p.hide(),i()}var a,o,r,h,l,u,d=t.pieces?Math.round(Math.sqrt(t.pieces)):3,c=d,p=e(this),f=e.effects.setMode(p,t.mode||"hide"),m="show"===f,g=p.show().css("visibility","hidden").offset(),v=Math.ceil(p.outerWidth()/c),y=Math.ceil(p.outerHeight()/d),b=[];for(a=0;d>a;a++)for(h=g.top+a*y,u=a-(d-1)/2,o=0;c>o;o++)r=g.left+o*v,l=o-(c-1)/2,p.clone().appendTo("body").wrap("<div></div>").css({position:"absolute",visibility:"visible",left:-o*v,top:-a*y}).parent().addClass("ui-effects-explode").css({position:"absolute",overflow:"hidden",width:v,height:y,left:r+(m?l*v:0),top:h+(m?u*y:0),opacity:m?0:1}).animate({left:r+(m?0:l*v),top:h+(m?0:u*y),opacity:m?1:0},t.duration||500,t.easing,s)},e.effects.effect.fade=function(t,i){var s=e(this),n=e.effects.setMode(s,t.mode||"toggle");s.animate({opacity:n},{queue:!1,duration:t.duration,easing:t.easing,complete:i})},e.effects.effect.fold=function(t,i){var s,n,a=e(this),o=["position","top","bottom","left","right","height","width"],r=e.effects.setMode(a,t.mode||"hide"),h="show"===r,l="hide"===r,u=t.size||15,d=/([0-9]+)%/.exec(u),c=!!t.horizFirst,p=h!==c,f=p?["width","height"]:["height","width"],m=t.duration/2,g={},v={};e.effects.save(a,o),a.show(),s=e.effects.createWrapper(a).css({overflow:"hidden"}),n=p?[s.width(),s.height()]:[s.height(),s.width()],d&&(u=parseInt(d[1],10)/100*n[l?0:1]),h&&s.css(c?{height:0,width:u}:{height:u,width:0}),g[f[0]]=h?n[0]:u,v[f[1]]=h?n[1]:0,s.animate(g,m,t.easing).animate(v,m,t.easing,function(){l&&a.hide(),e.effects.restore(a,o),e.effects.removeWrapper(a),i()})},e.effects.effect.highlight=function(t,i){var s=e(this),n=["backgroundImage","backgroundColor","opacity"],a=e.effects.setMode(s,t.mode||"show"),o={backgroundColor:s.css("backgroundColor")};"hide"===a&&(o.opacity=0),e.effects.save(s,n),s.show().css({backgroundImage:"none",backgroundColor:t.color||"#ffff99"}).animate(o,{queue:!1,duration:t.duration,easing:t.easing,complete:function(){"hide"===a&&s.hide(),e.effects.restore(s,n),i()}})},e.effects.effect.size=function(t,i){var s,n,a,o=e(this),r=["position","top","bottom","left","right","width","height","overflow","opacity"],h=["position","top","bottom","left","right","overflow","opacity"],l=["width","height","overflow"],u=["fontSize"],d=["borderTopWidth","borderBottomWidth","paddingTop","paddingBottom"],c=["borderLeftWidth","borderRightWidth","paddingLeft","paddingRight"],p=e.effects.setMode(o,t.mode||"effect"),f=t.restore||"effect"!==p,m=t.scale||"both",g=t.origin||["middle","center"],v=o.css("position"),y=f?r:h,b={height:0,width:0,outerHeight:0,outerWidth:0};"show"===p&&o.show(),s={height:o.height(),width:o.width(),outerHeight:o.outerHeight(),outerWidth:o.outerWidth()},"toggle"===t.mode&&"show"===p?(o.from=t.to||b,o.to=t.from||s):(o.from=t.from||("show"===p?b:s),o.to=t.to||("hide"===p?b:s)),a={from:{y:o.from.height/s.height,x:o.from.width/s.width},to:{y:o.to.height/s.height,x:o.to.width/s.width}},("box"===m||"both"===m)&&(a.from.y!==a.to.y&&(y=y.concat(d),o.from=e.effects.setTransition(o,d,a.from.y,o.from),o.to=e.effects.setTransition(o,d,a.to.y,o.to)),a.from.x!==a.to.x&&(y=y.concat(c),o.from=e.effects.setTransition(o,c,a.from.x,o.from),o.to=e.effects.setTransition(o,c,a.to.x,o.to))),("content"===m||"both"===m)&&a.from.y!==a.to.y&&(y=y.concat(u).concat(l),o.from=e.effects.setTransition(o,u,a.from.y,o.from),o.to=e.effects.setTransition(o,u,a.to.y,o.to)),e.effects.save(o,y),o.show(),e.effects.createWrapper(o),o.css("overflow","hidden").css(o.from),g&&(n=e.effects.getBaseline(g,s),o.from.top=(s.outerHeight-o.outerHeight())*n.y,o.from.left=(s.outerWidth-o.outerWidth())*n.x,o.to.top=(s.outerHeight-o.to.outerHeight)*n.y,o.to.left=(s.outerWidth-o.to.outerWidth)*n.x),o.css(o.from),("content"===m||"both"===m)&&(d=d.concat(["marginTop","marginBottom"]).concat(u),c=c.concat(["marginLeft","marginRight"]),l=r.concat(d).concat(c),o.find("*[width]").each(function(){var i=e(this),s={height:i.height(),width:i.width(),outerHeight:i.outerHeight(),outerWidth:i.outerWidth()};f&&e.effects.save(i,l),i.from={height:s.height*a.from.y,width:s.width*a.from.x,outerHeight:s.outerHeight*a.from.y,outerWidth:s.outerWidth*a.from.x},i.to={height:s.height*a.to.y,width:s.width*a.to.x,outerHeight:s.height*a.to.y,outerWidth:s.width*a.to.x},a.from.y!==a.to.y&&(i.from=e.effects.setTransition(i,d,a.from.y,i.from),i.to=e.effects.setTransition(i,d,a.to.y,i.to)),a.from.x!==a.to.x&&(i.from=e.effects.setTransition(i,c,a.from.x,i.from),i.to=e.effects.setTransition(i,c,a.to.x,i.to)),i.css(i.from),i.animate(i.to,t.duration,t.easing,function(){f&&e.effects.restore(i,l)})})),o.animate(o.to,{queue:!1,duration:t.duration,easing:t.easing,complete:function(){0===o.to.opacity&&o.css("opacity",o.from.opacity),"hide"===p&&o.hide(),e.effects.restore(o,y),f||("static"===v?o.css({position:"relative",top:o.to.top,left:o.to.left}):e.each(["top","left"],function(e,t){o.css(t,function(t,i){var s=parseInt(i,10),n=e?o.to.left:o.to.top;return"auto"===i?n+"px":s+n+"px"})})),e.effects.removeWrapper(o),i()}})},e.effects.effect.scale=function(t,i){var s=e(this),n=e.extend(!0,{},t),a=e.effects.setMode(s,t.mode||"effect"),o=parseInt(t.percent,10)||(0===parseInt(t.percent,10)?0:"hide"===a?0:100),r=t.direction||"both",h=t.origin,l={height:s.height(),width:s.width(),outerHeight:s.outerHeight(),outerWidth:s.outerWidth()},u={y:"horizontal"!==r?o/100:1,x:"vertical"!==r?o/100:1};n.effect="size",n.queue=!1,n.complete=i,"effect"!==a&&(n.origin=h||["middle","center"],n.restore=!0),n.from=t.from||("show"===a?{height:0,width:0,outerHeight:0,outerWidth:0}:l),n.to={height:l.height*u.y,width:l.width*u.x,outerHeight:l.outerHeight*u.y,outerWidth:l.outerWidth*u.x},n.fade&&("show"===a&&(n.from.opacity=0,n.to.opacity=1),"hide"===a&&(n.from.opacity=1,n.to.opacity=0)),s.effect(n)},e.effects.effect.puff=function(t,i){var s=e(this),n=e.effects.setMode(s,t.mode||"hide"),a="hide"===n,o=parseInt(t.percent,10)||150,r=o/100,h={height:s.height(),width:s.width(),outerHeight:s.outerHeight(),outerWidth:s.outerWidth()};e.extend(t,{effect:"scale",queue:!1,fade:!0,mode:n,complete:i,percent:a?o:100,from:a?h:{height:h.height*r,width:h.width*r,outerHeight:h.outerHeight*r,outerWidth:h.outerWidth*r}}),s.effect(t)},e.effects.effect.pulsate=function(t,i){var s,n=e(this),a=e.effects.setMode(n,t.mode||"show"),o="show"===a,r="hide"===a,h=o||"hide"===a,l=2*(t.times||5)+(h?1:0),u=t.duration/l,d=0,c=n.queue(),p=c.length;for((o||!n.is(":visible"))&&(n.css("opacity",0).show(),d=1),s=1;l>s;s++)n.animate({opacity:d},u,t.easing),d=1-d;n.animate({opacity:d},u,t.easing),n.queue(function(){r&&n.hide(),i()}),p>1&&c.splice.apply(c,[1,0].concat(c.splice(p,l+1))),n.dequeue()},e.effects.effect.shake=function(t,i){var s,n=e(this),a=["position","top","bottom","left","right","height","width"],o=e.effects.setMode(n,t.mode||"effect"),r=t.direction||"left",h=t.distance||20,l=t.times||3,u=2*l+1,d=Math.round(t.duration/u),c="up"===r||"down"===r?"top":"left",p="up"===r||"left"===r,f={},m={},g={},v=n.queue(),y=v.length;for(e.effects.save(n,a),n.show(),e.effects.createWrapper(n),f[c]=(p?"-=":"+=")+h,m[c]=(p?"+=":"-=")+2*h,g[c]=(p?"-=":"+=")+2*h,n.animate(f,d,t.easing),s=1;l>s;s++)n.animate(m,d,t.easing).animate(g,d,t.easing);n.animate(m,d,t.easing).animate(f,d/2,t.easing).queue(function(){"hide"===o&&n.hide(),e.effects.restore(n,a),e.effects.removeWrapper(n),i()}),y>1&&v.splice.apply(v,[1,0].concat(v.splice(y,u+1))),n.dequeue()},e.effects.effect.slide=function(t,i){var s,n=e(this),a=["position","top","bottom","left","right","width","height"],o=e.effects.setMode(n,t.mode||"show"),r="show"===o,h=t.direction||"left",l="up"===h||"down"===h?"top":"left",u="up"===h||"left"===h,d={};e.effects.save(n,a),n.show(),s=t.distance||n["top"===l?"outerHeight":"outerWidth"](!0),e.effects.createWrapper(n).css({overflow:"hidden"}),r&&n.css(l,u?isNaN(s)?"-"+s:-s:s),d[l]=(r?u?"+=":"-=":u?"-=":"+=")+s,n.animate(d,{queue:!1,duration:t.duration,easing:t.easing,complete:function(){"hide"===o&&n.hide(),e.effects.restore(n,a),e.effects.removeWrapper(n),i()}})},e.effects.effect.transfer=function(t,i){var s=e(this),n=e(t.to),a="fixed"===n.css("position"),o=e("body"),r=a?o.scrollTop():0,h=a?o.scrollLeft():0,l=n.offset(),u={top:l.top-r,left:l.left-h,height:n.innerHeight(),width:n.innerWidth()},d=s.offset(),c=e("<div class='ui-effects-transfer'></div>").appendTo(document.body).addClass(t.className).css({top:d.top-r,left:d.left-h,height:s.innerHeight(),width:s.innerWidth(),position:a?"fixed":"absolute"}).animate(u,t.duration,t.easing,function(){c.remove(),i()})}});</script>
+<style type="text/css">
+
+.tocify {
+width: 20%;
+max-height: 90%;
+overflow: auto;
+margin-left: 2%;
+position: fixed;
+border: 1px solid #ccc;
+border-radius: 6px;
+}
+
+.tocify ul, .tocify li {
+list-style: none;
+margin: 0;
+padding: 0;
+border: none;
+line-height: 30px;
+}
+
+.tocify-header {
+text-indent: 10px;
+}
+
+.tocify-subheader {
+text-indent: 20px;
+display: none;
+}
+
+.tocify-subheader li {
+font-size: 12px;
+}
+
+.tocify-subheader .tocify-subheader {
+text-indent: 30px;
+}
+.tocify-subheader .tocify-subheader .tocify-subheader {
+text-indent: 40px;
+}
+.tocify-subheader .tocify-subheader .tocify-subheader .tocify-subheader {
+text-indent: 50px;
+}
+.tocify-subheader .tocify-subheader .tocify-subheader .tocify-subheader .tocify-subheader {
+text-indent: 60px;
+}
+
+.tocify .tocify-item > a, .tocify .nav-list .nav-header {
+margin: 0px;
+}
+
+.tocify .tocify-item a, .tocify .list-group-item {
+padding: 5px;
+}
+.tocify .nav-pills > li {
+float: none;
+}
+
+
+</style>
+<script>/* jquery Tocify - v1.9.1 - 2013-10-22
+ * http://www.gregfranko.com/jquery.tocify.js/
+ * Copyright (c) 2013 Greg Franko; Licensed MIT */
+
+// Immediately-Invoked Function Expression (IIFE) [Ben Alman Blog Post](http://benalman.com/news/2010/11/immediately-invoked-function-expression/) that calls another IIFE that contains all of the plugin logic.  I used this pattern so that anyone viewing this code would not have to scroll to the bottom of the page to view the local parameters that were passed to the main IIFE.
+(function(tocify) {
+
+    // ECMAScript 5 Strict Mode: [John Resig Blog Post](http://ejohn.org/blog/ecmascript-5-strict-mode-json-and-more/)
+    "use strict";
+
+    // Calls the second IIFE and locally passes in the global jQuery, window, and document objects
+    tocify(window.jQuery, window, document);
+
+  }
+
+  // Locally passes in `jQuery`, the `window` object, the `document` object, and an `undefined` variable.  The `jQuery`, `window` and `document` objects are passed in locally, to improve performance, since javascript first searches for a variable match within the local variables set before searching the global variables set.  All of the global variables are also passed in locally to be minifier friendly. `undefined` can be passed in locally, because it is not a reserved word in JavaScript.
+  (function($, window, document, undefined) {
+
+    // ECMAScript 5 Strict Mode: [John Resig Blog Post](http://ejohn.org/blog/ecmascript-5-strict-mode-json-and-more/)
+    "use strict";
+
+    var tocClassName = "tocify",
+      tocClass = "." + tocClassName,
+      tocFocusClassName = "tocify-focus",
+      tocHoverClassName = "tocify-hover",
+      hideTocClassName = "tocify-hide",
+      hideTocClass = "." + hideTocClassName,
+      headerClassName = "tocify-header",
+      headerClass = "." + headerClassName,
+      subheaderClassName = "tocify-subheader",
+      subheaderClass = "." + subheaderClassName,
+      itemClassName = "tocify-item",
+      itemClass = "." + itemClassName,
+      extendPageClassName = "tocify-extend-page",
+      extendPageClass = "." + extendPageClassName;
+
+    // Calling the jQueryUI Widget Factory Method
+    $.widget("toc.tocify", {
+
+      //Plugin version
+      version: "1.9.1",
+
+      // These options will be used as defaults
+      options: {
+
+        // **context**: Accepts String: Any jQuery selector
+        // The container element that holds all of the elements used to generate the table of contents
+        context: "body",
+
+        // **ignoreSelector**: Accepts String: Any jQuery selector
+        // A selector to any element that would be matched by selectors that you wish to be ignored
+        ignoreSelector: null,
+
+        // **selectors**: Accepts an Array of Strings: Any jQuery selectors
+        // The element's used to generate the table of contents.  The order is very important since it will determine the table of content's nesting structure
+        selectors: "h1, h2, h3",
+
+        // **showAndHide**: Accepts a boolean: true or false
+        // Used to determine if elements should be shown and hidden
+        showAndHide: true,
+
+        // **showEffect**: Accepts String: "none", "fadeIn", "show", or "slideDown"
+        // Used to display any of the table of contents nested items
+        showEffect: "slideDown",
+
+        // **showEffectSpeed**: Accepts Number (milliseconds) or String: "slow", "medium", or "fast"
+        // The time duration of the show animation
+        showEffectSpeed: "medium",
+
+        // **hideEffect**: Accepts String: "none", "fadeOut", "hide", or "slideUp"
+        // Used to hide any of the table of contents nested items
+        hideEffect: "slideUp",
+
+        // **hideEffectSpeed**: Accepts Number (milliseconds) or String: "slow", "medium", or "fast"
+        // The time duration of the hide animation
+        hideEffectSpeed: "medium",
+
+        // **smoothScroll**: Accepts a boolean: true or false
+        // Determines if a jQuery animation should be used to scroll to specific table of contents items on the page
+        smoothScroll: true,
+
+        // **smoothScrollSpeed**: Accepts Number (milliseconds) or String: "slow", "medium", or "fast"
+        // The time duration of the smoothScroll animation
+        smoothScrollSpeed: "medium",
+
+        // **scrollTo**: Accepts Number (pixels)
+        // The amount of space between the top of page and the selected table of contents item after the page has been scrolled
+        scrollTo: 0,
+
+        // **showAndHideOnScroll**: Accepts a boolean: true or false
+        // Determines if table of contents nested items should be shown and hidden while scrolling
+        showAndHideOnScroll: true,
+
+        // **highlightOnScroll**: Accepts a boolean: true or false
+        // Determines if table of contents nested items should be highlighted (set to a different color) while scrolling
+        highlightOnScroll: true,
+
+        // **highlightOffset**: Accepts a number
+        // The offset distance in pixels to trigger the next active table of contents item
+        highlightOffset: 40,
+
+        // **theme**: Accepts a string: "bootstrap", "jqueryui", or "none"
+        // Determines if Twitter Bootstrap, jQueryUI, or Tocify classes should be added to the table of contents
+        theme: "bootstrap",
+
+        // **extendPage**: Accepts a boolean: true or false
+        // If a user scrolls to the bottom of the page and the page is not tall enough to scroll to the last table of contents item, then the page height is increased
+        extendPage: true,
+
+        // **extendPageOffset**: Accepts a number: pixels
+        // How close to the bottom of the page a user must scroll before the page is extended
+        extendPageOffset: 100,
+
+        // **history**: Accepts a boolean: true or false
+        // Adds a hash to the page url to maintain history
+        history: true,
+
+        // **scrollHistory**: Accepts a boolean: true or false
+        // Adds a hash to the page url, to maintain history, when scrolling to a TOC item
+        scrollHistory: false,
+
+        // **hashGenerator**: How the hash value (the anchor segment of the URL, following the
+        // # character) will be generated.
+        //
+        // "compact" (default) - #CompressesEverythingTogether
+        // "pretty" - #looks-like-a-nice-url-and-is-easily-readable
+        // function(text, element){} - Your own hash generation function that accepts the text as an
+        // argument, and returns the hash value.
+        hashGenerator: "compact",
+
+        // **highlightDefault**: Accepts a boolean: true or false
+        // Set's the first TOC item as active if no other TOC item is active.
+        highlightDefault: true
+
+      },
+
+      // _Create
+      // -------
+      //      Constructs the plugin.  Only called once.
+      _create: function() {
+
+        var self = this;
+
+        self.extendPageScroll = true;
+
+        // Internal array that keeps track of all TOC items (Helps to recognize if there are duplicate TOC item strings)
+        self.items = [];
+
+        // Generates the HTML for the dynamic table of contents
+        self._generateToc();
+
+        // Adds CSS classes to the newly generated table of contents HTML
+        self._addCSSClasses();
+
+        self.webkit = (function() {
+
+          for (var prop in window) {
+
+            if (prop) {
+
+              if (prop.toLowerCase().indexOf("webkit") !== -1) {
+
+                return true;
+
+              }
+
+            }
+
+          }
+
+          return false;
+
+        }());
+
+        // Adds jQuery event handlers to the newly generated table of contents
+        self._setEventHandlers();
+
+        // Binding to the Window load event to make sure the correct scrollTop is calculated
+        $(window).load(function() {
+
+          // Sets the active TOC item
+          self._setActiveElement(true);
+
+          // Once all animations on the page are complete, this callback function will be called
+          $("html, body").promise().done(function() {
+
+            setTimeout(function() {
+
+              self.extendPageScroll = false;
+
+            }, 0);
+
+          });
+
+        });
+
+      },
+
+      // _generateToc
+      // ------------
+      //      Generates the HTML for the dynamic table of contents
+      _generateToc: function() {
+
+        // _Local variables_
+
+        // Stores the plugin context in the self variable
+        var self = this,
+
+          // All of the HTML tags found within the context provided (i.e. body) that match the top level jQuery selector above
+          firstElem,
+
+          // Instantiated variable that will store the top level newly created unordered list DOM element
+          ul,
+          ignoreSelector = self.options.ignoreSelector;
+
+
+        // Determine the element to start the toc with
+        // get all the top level selectors
+        firstElem = [];
+        var selectors = this.options.selectors.replace(/ /g, "").split(",");
+        // find the first set that have at least one non-ignored element
+        for(var i = 0; i < selectors.length; i++) {
+          var foundSelectors = $(this.options.context).find(selectors[i]);
+          for (var s = 0; s < foundSelectors.length; s++) {
+            if (!$(foundSelectors[s]).is(ignoreSelector)) {
+              firstElem = foundSelectors;
+              break;
+            }
+          }
+          if (firstElem.length> 0)
+            break;
+        }
+
+        if (!firstElem.length) {
+
+          self.element.addClass(hideTocClassName);
+
+          return;
+
+        }
+
+        self.element.addClass(tocClassName);
+
+        // Loops through each top level selector
+        firstElem.each(function(index) {
+
+          //If the element matches the ignoreSelector then we skip it
+          if ($(this).is(ignoreSelector)) {
+            return;
+          }
+
+          // Creates an unordered list HTML element and adds a dynamic ID and standard class name
+          ul = $("<ul/>", {
+            "id": headerClassName + index,
+            "class": headerClassName
+          }).
+
+          // Appends a top level list item HTML element to the previously created HTML header
+          append(self._nestElements($(this), index));
+
+          // Add the created unordered list element to the HTML element calling the plugin
+          self.element.append(ul);
+
+          // Finds all of the HTML tags between the header and subheader elements
+          $(this).nextUntil(this.nodeName.toLowerCase()).each(function() {
+
+            // If there are no nested subheader elemements
+            if ($(this).find(self.options.selectors).length === 0) {
+
+              // Loops through all of the subheader elements
+              $(this).filter(self.options.selectors).each(function() {
+
+                //If the element matches the ignoreSelector then we skip it
+                if ($(this).is(ignoreSelector)) {
+                  return;
+                }
+
+                self._appendSubheaders.call(this, self, ul);
+
+              });
+
+            }
+
+            // If there are nested subheader elements
+            else {
+
+              // Loops through all of the subheader elements
+              $(this).find(self.options.selectors).each(function() {
+
+                //If the element matches the ignoreSelector then we skip it
+                if ($(this).is(ignoreSelector)) {
+                  return;
+                }
+
+                self._appendSubheaders.call(this, self, ul);
+
+              });
+
+            }
+
+          });
+
+        });
+
+      },
+
+      _setActiveElement: function(pageload) {
+
+        var self = this,
+
+          hash = window.location.hash.substring(1),
+
+          elem = self.element.find('li[data-unique="' + hash + '"]');
+
+        if (hash.length) {
+
+          // Removes highlighting from all of the list item's
+          self.element.find("." + self.focusClass).removeClass(self.focusClass);
+
+          // Highlights the current list item that was clicked
+          elem.addClass(self.focusClass);
+
+          // Triggers the click event on the currently focused TOC item
+          elem.click();
+
+        } else {
+
+          // Removes highlighting from all of the list item's
+          self.element.find("." + self.focusClass).removeClass(self.focusClass);
+
+          if (!hash.length && pageload && self.options.highlightDefault) {
+
+            // Highlights the first TOC item if no other items are highlighted
+            self.element.find(itemClass).first().addClass(self.focusClass);
+
+          }
+
+        }
+
+        return self;
+
+      },
+
+      // _nestElements
+      // -------------
+      //      Helps create the table of contents list by appending nested list items
+      _nestElements: function(self, index) {
+
+        var arr, item, hashValue;
+
+        arr = $.grep(this.items, function(item) {
+
+          return item === self.text();
+
+        });
+
+        // If there is already a duplicate TOC item
+        if (arr.length) {
+
+          // Adds the current TOC item text and index (for slight randomization) to the internal array
+          this.items.push(self.text() + index);
+
+        }
+
+        // If there not a duplicate TOC item
+        else {
+
+          // Adds the current TOC item text to the internal array
+          this.items.push(self.text());
+
+        }
+
+        hashValue = this._generateHashValue(arr, self, index);
+
+        // Appends a list item HTML element to the last unordered list HTML element found within the HTML element calling the plugin
+        item = $("<li/>", {
+
+          // Sets a common class name to the list item
+          "class": itemClassName,
+
+          "data-unique": hashValue
+
+        });
+
+        if (this.options.theme !== "bootstrap3") {
+
+          item.append($("<a/>", {
+
+            "html": self.html()
+
+          }));
+
+        } else {
+
+          item.html(self.html());
+
+        }
+
+        // Adds an HTML anchor tag before the currently traversed HTML element
+        self.before($("<div/>", {
+
+          // Sets a name attribute on the anchor tag to the text of the currently traversed HTML element (also making sure that all whitespace is replaced with an underscore)
+          "name": hashValue,
+
+          "data-unique": hashValue
+
+        }));
+
+        return item;
+
+      },
+
+      // _generateHashValue
+      // ------------------
+      //      Generates the hash value that will be used to refer to each item.
+      _generateHashValue: function(arr, self, index) {
+
+        var hashValue = "",
+          hashGeneratorOption = this.options.hashGenerator;
+
+        if (hashGeneratorOption === "pretty") {
+
+          // prettify the text
+          hashValue = self.text().toLowerCase().replace(/\s/g, "-");
+
+          // fix double hyphens
+          while (hashValue.indexOf("--") > -1) {
+            hashValue = hashValue.replace(/--/g, "-");
+          }
+
+          // fix colon-space instances
+          while (hashValue.indexOf(":-") > -1) {
+            hashValue = hashValue.replace(/:-/g, "-");
+          }
+
+        } else if (typeof hashGeneratorOption === "function") {
+
+          // call the function
+          hashValue = hashGeneratorOption(self.text(), self);
+
+        } else {
+
+          // compact - the default
+          hashValue = self.text().replace(/\s/g, "");
+
+        }
+
+        // add the index if we need to
+        if (arr.length) {
+          hashValue += "" + index;
+        }
+
+        // return the value
+        return hashValue;
+
+      },
+
+      // _appendElements
+      // ---------------
+      //      Helps create the table of contents list by appending subheader elements
+
+      _appendSubheaders: function(self, ul) {
+
+        // The current element index
+        var index = $(this).index(self.options.selectors),
+
+          // Finds the previous header DOM element
+          previousHeader = $(self.options.selectors).eq(index - 1),
+
+          currentTagName = +$(this).prop("tagName").charAt(1),
+
+          previousTagName = +previousHeader.prop("tagName").charAt(1),
+
+          lastSubheader;
+
+        // If the current header DOM element is smaller than the previous header DOM element or the first subheader
+        if (currentTagName < previousTagName) {
+
+          // Selects the last unordered list HTML found within the HTML element calling the plugin
+          self.element.find(subheaderClass + "[data-tag=" + currentTagName + "]").last().append(self._nestElements($(this), index));
+
+        }
+
+        // If the current header DOM element is the same type of header(eg. h4) as the previous header DOM element
+        else if (currentTagName === previousTagName) {
+
+          ul.find(itemClass).last().after(self._nestElements($(this), index));
+
+        } else {
+
+          // Selects the last unordered list HTML found within the HTML element calling the plugin
+          ul.find(itemClass).last().
+
+          // Appends an unorderedList HTML element to the dynamic `unorderedList` variable and sets a common class name
+          after($("<ul/>", {
+
+            "class": subheaderClassName,
+
+            "data-tag": currentTagName
+
+          })).next(subheaderClass).
+
+          // Appends a list item HTML element to the last unordered list HTML element found within the HTML element calling the plugin
+          append(self._nestElements($(this), index));
+        }
+
+      },
+
+      // _setEventHandlers
+      // ----------------
+      //      Adds jQuery event handlers to the newly generated table of contents
+      _setEventHandlers: function() {
+
+        // _Local variables_
+
+        // Stores the plugin context in the self variable
+        var self = this,
+
+          // Instantiates a new variable that will be used to hold a specific element's context
+          $self,
+
+          // Instantiates a new variable that will be used to determine the smoothScroll animation time duration
+          duration;
+
+        // Event delegation that looks for any clicks on list item elements inside of the HTML element calling the plugin
+        this.element.on("click.tocify", "li", function(event) {
+
+          if (self.options.history) {
+
+            window.location.hash = $(this).attr("data-unique");
+
+          }
+
+          // Removes highlighting from all of the list item's
+          self.element.find("." + self.focusClass).removeClass(self.focusClass);
+
+          // Highlights the current list item that was clicked
+          $(this).addClass(self.focusClass);
+
+          // If the showAndHide option is true
+          if (self.options.showAndHide) {
+
+            var elem = $('li[data-unique="' + $(this).attr("data-unique") + '"]');
+
+            self._triggerShow(elem);
+
+          }
+
+          self._scrollTo($(this));
+
+        });
+
+        // Mouseenter and Mouseleave event handlers for the list item's within the HTML element calling the plugin
+        this.element.find("li").on({
+
+          // Mouseenter event handler
+          "mouseenter.tocify": function() {
+
+            // Adds a hover CSS class to the current list item
+            $(this).addClass(self.hoverClass);
+
+            // Makes sure the cursor is set to the pointer icon
+            $(this).css("cursor", "pointer");
+
+          },
+
+          // Mouseleave event handler
+          "mouseleave.tocify": function() {
+
+            if (self.options.theme !== "bootstrap") {
+
+              // Removes the hover CSS class from the current list item
+              $(this).removeClass(self.hoverClass);
+
+            }
+
+          }
+        });
+
+        // only attach handler if needed (expensive in IE)
+        if (self.options.extendPage || self.options.highlightOnScroll || self.options.scrollHistory || self.options.showAndHideOnScroll) {
+          // Window scroll event handler
+          $(window).on("scroll.tocify", function() {
+
+            // Once all animations on the page are complete, this callback function will be called
+            $("html, body").promise().done(function() {
+
+              // Local variables
+
+              // Stores how far the user has scrolled
+              var winScrollTop = $(window).scrollTop(),
+
+                // Stores the height of the window
+                winHeight = $(window).height(),
+
+                // Stores the height of the document
+                docHeight = $(document).height(),
+
+                scrollHeight = $("body")[0].scrollHeight,
+
+                // Instantiates a variable that will be used to hold a selected HTML element
+                elem,
+
+                lastElem,
+
+                lastElemOffset,
+
+                currentElem;
+
+              if (self.options.extendPage) {
+
+                // If the user has scrolled to the bottom of the page and the last toc item is not focused
+                if ((self.webkit && winScrollTop >= scrollHeight - winHeight - self.options.extendPageOffset) || (!self.webkit && winHeight + winScrollTop > docHeight - self.options.extendPageOffset)) {
+
+                  if (!$(extendPageClass).length) {
+
+                    lastElem = $('div[data-unique="' + $(itemClass).last().attr("data-unique") + '"]');
+
+                    if (!lastElem.length) return;
+
+                    // Gets the top offset of the page header that is linked to the last toc item
+                    lastElemOffset = lastElem.offset().top;
+
+                    // Appends a div to the bottom of the page and sets the height to the difference of the window scrollTop and the last element's position top offset
+                    $(self.options.context).append($("<div/>", {
+
+                      "class": extendPageClassName,
+
+                      "height": Math.abs(lastElemOffset - winScrollTop) + "px",
+
+                      "data-unique": extendPageClassName
+
+                    }));
+
+                    if (self.extendPageScroll) {
+
+                      currentElem = self.element.find('li.' + self.focusClass);
+
+                      self._scrollTo($('div[data-unique="' + currentElem.attr("data-unique") + '"]'));
+
+                    }
+
+                  }
+
+                }
+
+              }
+
+              // The zero timeout ensures the following code is run after the scroll events
+              setTimeout(function() {
+
+                // _Local variables_
+
+                // Stores the distance to the closest anchor
+                var closestAnchorDistance = null,
+
+                  // Stores the index of the closest anchor
+                  closestAnchorIdx = null,
+
+                  // Keeps a reference to all anchors
+                  anchors = $(self.options.context).find("div[data-unique]"),
+
+                  anchorText;
+
+                // Determines the index of the closest anchor
+                anchors.each(function(idx) {
+                  var distance = Math.abs(($(this).next().length ? $(this).next() : $(this)).offset().top - winScrollTop - self.options.highlightOffset);
+                  if (closestAnchorDistance == null || distance < closestAnchorDistance) {
+                    closestAnchorDistance = distance;
+                    closestAnchorIdx = idx;
+                  } else {
+                    return false;
+                  }
+                });
+
+                anchorText = $(anchors[closestAnchorIdx]).attr("data-unique");
+
+                // Stores the list item HTML element that corresponds to the currently traversed anchor tag
+                elem = $('li[data-unique="' + anchorText + '"]');
+
+                // If the `highlightOnScroll` option is true and a next element is found
+                if (self.options.highlightOnScroll && elem.length) {
+
+                  // Removes highlighting from all of the list item's
+                  self.element.find("." + self.focusClass).removeClass(self.focusClass);
+
+                  // Highlights the corresponding list item
+                  elem.addClass(self.focusClass);
+
+                }
+
+                if (self.options.scrollHistory) {
+
+                  if (window.location.hash !== "#" + anchorText) {
+
+                    window.location.replace("#" + anchorText);
+
+                  }
+                }
+
+                // If the `showAndHideOnScroll` option is true
+                if (self.options.showAndHideOnScroll && self.options.showAndHide) {
+
+                  self._triggerShow(elem, true);
+
+                }
+
+              }, 0);
+
+            });
+
+          });
+        }
+
+      },
+
+      // Show
+      // ----
+      //      Opens the current sub-header
+      show: function(elem, scroll) {
+
+        // Stores the plugin context in the `self` variable
+        var self = this,
+          element = elem;
+
+        // If the sub-header is not already visible
+        if (!elem.is(":visible")) {
+
+          // If the current element does not have any nested subheaders, is not a header, and its parent is not visible
+          if (!elem.find(subheaderClass).length && !elem.parent().is(headerClass) && !elem.parent().is(":visible")) {
+
+            // Sets the current element to all of the subheaders within the current header
+            elem = elem.parents(subheaderClass).add(elem);
+
+          }
+
+          // If the current element does not have any nested subheaders and is not a header
+          else if (!elem.children(subheaderClass).length && !elem.parent().is(headerClass)) {
+
+            // Sets the current element to the closest subheader
+            elem = elem.closest(subheaderClass);
+
+          }
+
+          //Determines what jQuery effect to use
+          switch (self.options.showEffect) {
+
+            //Uses `no effect`
+            case "none":
+
+              elem.show();
+
+              break;
+
+              //Uses the jQuery `show` special effect
+            case "show":
+
+              elem.show(self.options.showEffectSpeed);
+
+              break;
+
+              //Uses the jQuery `slideDown` special effect
+            case "slideDown":
+
+              elem.slideDown(self.options.showEffectSpeed);
+
+              break;
+
+              //Uses the jQuery `fadeIn` special effect
+            case "fadeIn":
+
+              elem.fadeIn(self.options.showEffectSpeed);
+
+              break;
+
+              //If none of the above options were passed, then a `jQueryUI show effect` is expected
+            default:
+
+              elem.show();
+
+              break;
+
+          }
+
+        }
+
+        // If the current subheader parent element is a header
+        if (elem.parent().is(headerClass)) {
+
+          // Hides all non-active sub-headers
+          self.hide($(subheaderClass).not(elem));
+
+        }
+
+        // If the current subheader parent element is not a header
+        else {
+
+          // Hides all non-active sub-headers
+          self.hide($(subheaderClass).not(elem.closest(headerClass).find(subheaderClass).not(elem.siblings())));
+
+        }
+
+        // Maintains chainablity
+        return self;
+
+      },
+
+      // Hide
+      // ----
+      //      Closes the current sub-header
+      hide: function(elem) {
+
+        // Stores the plugin context in the `self` variable
+        var self = this;
+
+        //Determines what jQuery effect to use
+        switch (self.options.hideEffect) {
+
+          // Uses `no effect`
+          case "none":
+
+            elem.hide();
+
+            break;
+
+            // Uses the jQuery `hide` special effect
+          case "hide":
+
+            elem.hide(self.options.hideEffectSpeed);
+
+            break;
+
+            // Uses the jQuery `slideUp` special effect
+          case "slideUp":
+
+            elem.slideUp(self.options.hideEffectSpeed);
+
+            break;
+
+            // Uses the jQuery `fadeOut` special effect
+          case "fadeOut":
+
+            elem.fadeOut(self.options.hideEffectSpeed);
+
+            break;
+
+            // If none of the above options were passed, then a `jqueryUI hide effect` is expected
+          default:
+
+            elem.hide();
+
+            break;
+
+        }
+
+        // Maintains chainablity
+        return self;
+      },
+
+      // _triggerShow
+      // ------------
+      //      Determines what elements get shown on scroll and click
+      _triggerShow: function(elem, scroll) {
+
+        var self = this;
+
+        // If the current element's parent is a header element or the next element is a nested subheader element
+        if (elem.parent().is(headerClass) || elem.next().is(subheaderClass)) {
+
+          // Shows the next sub-header element
+          self.show(elem.next(subheaderClass), scroll);
+
+        }
+
+        // If the current element's parent is a subheader element
+        else if (elem.parent().is(subheaderClass)) {
+
+          // Shows the parent sub-header element
+          self.show(elem.parent(), scroll);
+
+        }
+
+        // Maintains chainability
+        return self;
+
+      },
+
+      // _addCSSClasses
+      // --------------
+      //      Adds CSS classes to the newly generated table of contents HTML
+      _addCSSClasses: function() {
+
+        // If the user wants a jqueryUI theme
+        if (this.options.theme === "jqueryui") {
+
+          this.focusClass = "ui-state-default";
+
+          this.hoverClass = "ui-state-hover";
+
+          //Adds the default styling to the dropdown list
+          this.element.addClass("ui-widget").find(".toc-title").addClass("ui-widget-header").end().find("li").addClass("ui-widget-content");
+
+        }
+
+        // If the user wants a twitterBootstrap theme
+        else if (this.options.theme === "bootstrap") {
+
+          this.element.find(headerClass + "," + subheaderClass).addClass("nav nav-list");
+
+          this.focusClass = "active";
+
+        }
+
+        // If the user wants a twitterBootstrap theme
+        else if (this.options.theme === "bootstrap3") {
+
+          this.element.find(headerClass + "," + subheaderClass).addClass("list-group");
+
+          this.element.find(itemClass).addClass("list-group-item");
+
+          this.focusClass = "active";
+
+        }
+
+        // If a user does not want a prebuilt theme
+        else {
+
+          // Adds more neutral classes (instead of jqueryui)
+
+          this.focusClass = tocFocusClassName;
+
+          this.hoverClass = tocHoverClassName;
+
+        }
+
+        //Maintains chainability
+        return this;
+
+      },
+
+      // setOption
+      // ---------
+      //      Sets a single Tocify option after the plugin is invoked
+      setOption: function() {
+
+        // Calls the jQueryUI Widget Factory setOption method
+        $.Widget.prototype._setOption.apply(this, arguments);
+
+      },
+
+      // setOptions
+      // ----------
+      //      Sets a single or multiple Tocify options after the plugin is invoked
+      setOptions: function() {
+
+        // Calls the jQueryUI Widget Factory setOptions method
+        $.Widget.prototype._setOptions.apply(this, arguments);
+
+      },
+
+      // _scrollTo
+      // ---------
+      //      Scrolls to a specific element
+      _scrollTo: function(elem) {
+
+        var self = this,
+          duration = self.options.smoothScroll || 0,
+          scrollTo = self.options.scrollTo,
+          currentDiv = $('div[data-unique="' + elem.attr("data-unique") + '"]');
+
+        if (!currentDiv.length) {
+
+          return self;
+
+        }
+
+        // Once all animations on the page are complete, this callback function will be called
+        $("html, body").promise().done(function() {
+
+          // Animates the html and body element scrolltops
+          $("html, body").animate({
+
+            // Sets the jQuery `scrollTop` to the top offset of the HTML div tag that matches the current list item's `data-unique` tag
+            "scrollTop": currentDiv.offset().top - ($.isFunction(scrollTo) ? scrollTo.call() : scrollTo) + "px"
+
+          }, {
+
+            // Sets the smoothScroll animation time duration to the smoothScrollSpeed option
+            "duration": duration
+
+          });
+
+        });
+
+        // Maintains chainability
+        return self;
+
+      }
+
+    });
+
+  })); //end of plugin
+</script>
+<script>
+
+/**
+ * jQuery Plugin: Sticky Tabs
+ *
+ * @author Aidan Lister <aidan@php.net>
+ * adapted by Ruben Arslan to activate parent tabs too
+ * http://www.aidanlister.com/2014/03/persisting-the-tab-state-in-bootstrap/
+ */
+(function($) {
+  "use strict";
+  $.fn.rmarkdownStickyTabs = function() {
+    var context = this;
+    // Show the tab corresponding with the hash in the URL, or the first tab
+    var showStuffFromHash = function() {
+      var hash = window.location.hash;
+      var selector = hash ? 'a[href="' + hash + '"]' : 'li.active > a';
+      var $selector = $(selector, context);
+      if($selector.data('toggle') === "tab") {
+        $selector.tab('show');
+        // walk up the ancestors of this element, show any hidden tabs
+        $selector.parents('.section.tabset').each(function(i, elm) {
+          var link = $('a[href="#' + $(elm).attr('id') + '"]');
+          if(link.data('toggle') === "tab") {
+            link.tab("show");
+          }
+        });
+      }
+    };
+
+
+    // Set the correct tab when the page loads
+    showStuffFromHash(context);
+
+    // Set the correct tab when a user uses their back/forward button
+    $(window).on('hashchange', function() {
+      showStuffFromHash(context);
+    });
+
+    // Change the URL when tabs are clicked
+    $('a', context).on('click', function(e) {
+      history.pushState(null, null, this.href);
+      showStuffFromHash(context);
+    });
+
+    return this;
+  };
+}(jQuery));
+
+window.buildTabsets = function(tocID) {
+
+  // build a tabset from a section div with the .tabset class
+  function buildTabset(tabset) {
+
+    // check for fade and pills options
+    var fade = tabset.hasClass("tabset-fade");
+    var pills = tabset.hasClass("tabset-pills");
+    var navClass = pills ? "nav-pills" : "nav-tabs";
+
+    // determine the heading level of the tabset and tabs
+    var match = tabset.attr('class').match(/level(\d) /);
+    if (match === null)
+      return;
+    var tabsetLevel = Number(match[1]);
+    var tabLevel = tabsetLevel + 1;
+
+    // find all subheadings immediately below
+    var tabs = tabset.find("div.section.level" + tabLevel);
+    if (!tabs.length)
+      return;
+
+    // create tablist and tab-content elements
+    var tabList = $('<ul class="nav ' + navClass + '" role="tablist"></ul>');
+    $(tabs[0]).before(tabList);
+    var tabContent = $('<div class="tab-content"></div>');
+    $(tabs[0]).before(tabContent);
+
+    // build the tabset
+    var activeTab = 0;
+    tabs.each(function(i) {
+
+      // get the tab div
+      var tab = $(tabs[i]);
+
+      // get the id then sanitize it for use with bootstrap tabs
+      var id = tab.attr('id');
+
+      // see if this is marked as the active tab
+      if (tab.hasClass('active'))
+        activeTab = i;
+
+      // remove any table of contents entries associated with
+      // this ID (since we'll be removing the heading element)
+      $("div#" + tocID + " li a[href='#" + id + "']").parent().remove();
+
+      // sanitize the id for use with bootstrap tabs
+      id = id.replace(/[.\/?&!#<>]/g, '').replace(/\s/g, '_');
+      tab.attr('id', id);
+
+      // get the heading element within it, grab it's text, then remove it
+      var heading = tab.find('h' + tabLevel + ':first');
+      var headingText = heading.html();
+      heading.remove();
+
+      // build and append the tab list item
+      var a = $('<a role="tab" data-toggle="tab">' + headingText + '</a>');
+      a.attr('href', '#' + id);
+      a.attr('aria-controls', id);
+      var li = $('<li role="presentation"></li>');
+      li.append(a);
+      tabList.append(li);
+
+      // set it's attributes
+      tab.attr('role', 'tabpanel');
+      tab.addClass('tab-pane');
+      tab.addClass('tabbed-pane');
+      if (fade)
+        tab.addClass('fade');
+
+      // move it into the tab content div
+      tab.detach().appendTo(tabContent);
+    });
+
+    // set active tab
+    $(tabList.children('li')[activeTab]).addClass('active');
+    var active = $(tabContent.children('div.section')[activeTab]);
+    active.addClass('active');
+    if (fade)
+      active.addClass('in');
+
+    if (tabset.hasClass("tabset-sticky"))
+      tabset.rmarkdownStickyTabs();
+  }
+
+  // convert section divs with the .tabset class to tabsets
+  var tabsets = $("div.section.tabset");
+  tabsets.each(function(i) {
+    buildTabset($(tabsets[i]));
+  });
+};
+
+</script>
+<style type="text/css">.hljs-literal {
+color: #990073;
+}
+.hljs-number {
+color: #099;
+}
+.hljs-comment {
+color: #998;
+font-style: italic;
+}
+.hljs-keyword {
+color: #900;
+font-weight: bold;
+}
+.hljs-string {
+color: #d14;
+}
+</style>
+<script src="data:application/javascript;base64,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"></script>
+
+<style type="text/css">
+  code{white-space: pre-wrap;}
+  span.smallcaps{font-variant: small-caps;}
+  span.underline{text-decoration: underline;}
+  div.column{display: inline-block; vertical-align: top; width: 50%;}
+  div.hanging-indent{margin-left: 1.5em; text-indent: -1.5em;}
+  ul.task-list{list-style: none;}
+      </style>
+
+<style type="text/css">code{white-space: pre;}</style>
+<style type="text/css">
+  pre:not([class]) {
+    background-color: white;
+  }
+</style>
+<script type="text/javascript">
+if (window.hljs) {
+  hljs.configure({languages: []});
+  hljs.initHighlightingOnLoad();
+  if (document.readyState && document.readyState === "complete") {
+    window.setTimeout(function() { hljs.initHighlighting(); }, 0);
+  }
+}
+</script>
+
+
+
+<style type="text/css">
+h1 {
+  font-size: 34px;
+}
+h1.title {
+  font-size: 38px;
+}
+h2 {
+  font-size: 30px;
+}
+h3 {
+  font-size: 24px;
+}
+h4 {
+  font-size: 18px;
+}
+h5 {
+  font-size: 16px;
+}
+h6 {
+  font-size: 12px;
+}
+.table th:not([align]) {
+  text-align: left;
+}
+</style>
+
+
+
+
+<style type="text/css">
+.main-container {
+  max-width: 940px;
+  margin-left: auto;
+  margin-right: auto;
+}
+code {
+  color: inherit;
+  background-color: rgba(0, 0, 0, 0.04);
+}
+img {
+  max-width:100%;
+}
+.tabbed-pane {
+  padding-top: 12px;
+}
+.html-widget {
+  margin-bottom: 20px;
+}
+button.code-folding-btn:focus {
+  outline: none;
+}
+summary {
+  display: list-item;
+}
+</style>
+
+
+
+<!-- tabsets -->
+
+<style type="text/css">
+.tabset-dropdown > .nav-tabs {
+  display: inline-table;
+  max-height: 500px;
+  min-height: 44px;
+  overflow-y: auto;
+  background: white;
+  border: 1px solid #ddd;
+  border-radius: 4px;
+}
+
+.tabset-dropdown > .nav-tabs > li.active:before {
+  content: "";
+  font-family: 'Glyphicons Halflings';
+  display: inline-block;
+  padding: 10px;
+  border-right: 1px solid #ddd;
+}
+
+.tabset-dropdown > .nav-tabs.nav-tabs-open > li.active:before {
+  content: "";
+  border: none;
+}
+
+.tabset-dropdown > .nav-tabs.nav-tabs-open:before {
+  content: "";
+  font-family: 'Glyphicons Halflings';
+  display: inline-block;
+  padding: 10px;
+  border-right: 1px solid #ddd;
+}
+
+.tabset-dropdown > .nav-tabs > li.active {
+  display: block;
+}
+
+.tabset-dropdown > .nav-tabs > li > a,
+.tabset-dropdown > .nav-tabs > li > a:focus,
+.tabset-dropdown > .nav-tabs > li > a:hover {
+  border: none;
+  display: inline-block;
+  border-radius: 4px;
+  background-color: transparent;
+}
+
+.tabset-dropdown > .nav-tabs.nav-tabs-open > li {
+  display: block;
+  float: none;
+}
+
+.tabset-dropdown > .nav-tabs > li {
+  display: none;
+}
+</style>
+
+<!-- code folding -->
+
+
+
+<style type="text/css">
+
+#TOC {
+  margin: 25px 0px 20px 0px;
+}
+@media (max-width: 768px) {
+#TOC {
+  position: relative;
+  width: 100%;
+}
+}
+
+@media print {
+.toc-content {
+  /* see https://github.com/w3c/csswg-drafts/issues/4434 */
+  float: right;
+}
+}
+
+.toc-content {
+  padding-left: 30px;
+  padding-right: 40px;
+}
+
+div.main-container {
+  max-width: 1200px;
+}
+
+div.tocify {
+  width: 20%;
+  max-width: 260px;
+  max-height: 85%;
+}
+
+@media (min-width: 768px) and (max-width: 991px) {
+  div.tocify {
+    width: 25%;
+  }
+}
+
+@media (max-width: 767px) {
+  div.tocify {
+    width: 100%;
+    max-width: none;
+  }
+}
+
+.tocify ul, .tocify li {
+  line-height: 20px;
+}
+
+.tocify-subheader .tocify-item {
+  font-size: 0.90em;
+}
+
+.tocify .list-group-item {
+  border-radius: 0px;
+}
+
+
+</style>
+
+
+
+</head>
+
+<body>
+
+
+<div class="container-fluid main-container">
+
+
+<!-- setup 3col/9col grid for toc_float and main content  -->
+<div class="row-fluid">
+<div class="col-xs-12 col-sm-4 col-md-3">
+<div id="TOC" class="tocify">
+</div>
+</div>
+
+<div class="toc-content col-xs-12 col-sm-8 col-md-9">
+
+
+
+
+<div class="fluid-row" id="header">
+
+
+
+<h1 class="title toc-ignore">Problem set 3: Worked solutions</h1>
+<h3 class="subtitle">Statistics and statistical programming<br />
+Northwestern University<br />
+MTS 525</h3>
+<h4 class="author">Aaron Shaw</h4>
+<h4 class="date">October 12, 2020</h4>
+
+</div>
+
+
+<div id="programming-challenges" class="section level1">
+<h1>Programming challenges</h1>
+<div id="pc1.-learn-about-the-data." class="section level2">
+<h2>PC1. Learn about the data.</h2>
+<p>The documentation makes it sound like the Illinois data is particularly messy/incomplete. It explains that the data is limited to 2012-2017 (instead of extending back to 2004) “due to format issues and relevance.” The notes point out that data related to searches vary substantially year-to-year, suggesting that the measurement was inconsistent and that caution should be exercised in the analysis of this measure in particular. The <code>search_conducted</code> variable seems to include searches of any kind (vehicles, driver, passengers). I also notice that the codebook does not provide much insight into <em>how</em> <code>subject_race</code> and <code>subject_sex</code> information were collected, measured, and/or recoded. I think it’s best to presume that the records reflect the judgments/notes/documents accessed by the officer(s) and/or police forces involved rather than the perspectives of the subjects of the traffic stops. There are likely some inconsistencies in these measures as well.</p>
+</div>
+<div id="pc2.-import-explore-clean" class="section level2">
+<h2>PC2. Import, explore, clean</h2>
+<p>I’ll use the tidyverse <code>read_csv()</code> function to do this. While I’m at it, I’ll import the <code>lubridate</code> library because I already know this is time series data and there are a bunch of helpful functions in <code>lubridate</code> that I will probably want to use.</p>
+<pre class="r"><code>library(tidyverse)
+library(lubridate)
+data.dir &lt;- &quot;https://communitydata.science/~ads/teaching/2020/stats/data/week_05&quot;
+filename &lt;- &quot;il_statewide_sample-2020_04_01.csv&quot;
+ilstops &lt;- read_csv(url(paste(data.dir, filename, sep = &quot;/&quot;)))</code></pre>
+<pre><code>## Warning: 30129 parsing failures.
+##  row  col expected     actual         file
+## 1153 beat a double east       &lt;connection&gt;
+## 1155 beat a double Rt1/325N   &lt;connection&gt;
+## 1158 beat a double village    &lt;connection&gt;
+## 1160 beat a double PULASKI CO &lt;connection&gt;
+## 1161 beat a double P21        &lt;connection&gt;
+## .... .... ........ .......... ............
+## See problems(...) for more details.</code></pre>
+<pre class="r"><code>ilstops</code></pre>
+<pre><code>## # A tibble: 127,064 x 29
+##    raw_row_number date       time  location  beat subject_age subject_race
+##             &lt;dbl&gt; &lt;date&gt;     &lt;tim&gt;    &lt;dbl&gt; &lt;dbl&gt; &lt;lgl&gt;       &lt;chr&gt;       
+##  1             71 2012-01-01 06:00    60160  1924 NA          white       
+##  2             81 2012-01-01 07:34       NA  1654 NA          asian/pacif…
+##  3            170 2012-01-01 18:35       NA  1324 NA          hispanic    
+##  4            274 2012-01-02 00:18       NA  1633 NA          white       
+##  5            344 2012-01-02 09:08    60608  1232 NA          white       
+##  6            379 2012-01-02 11:00    60621   733 NA          black       
+##  7            410 2012-01-02 13:00       NA  1613 NA          hispanic    
+##  8            570 2012-01-02 21:01    60644  1512 NA          black       
+##  9            729 2012-01-03 09:40    60618  2131 NA          white       
+## 10            789 2012-01-03 13:29    60639  2534 NA          hispanic    
+## # … with 127,054 more rows, and 22 more variables: subject_sex &lt;chr&gt;,
+## #   department_id &lt;dbl&gt;, department_name &lt;chr&gt;, type &lt;chr&gt;, violation &lt;chr&gt;,
+## #   citation_issued &lt;lgl&gt;, warning_issued &lt;lgl&gt;, outcome &lt;chr&gt;,
+## #   contraband_found &lt;lgl&gt;, contraband_drugs &lt;lgl&gt;, contraband_weapons &lt;lgl&gt;,
+## #   search_conducted &lt;lgl&gt;, search_person &lt;lgl&gt;, search_vehicle &lt;lgl&gt;,
+## #   search_basis &lt;chr&gt;, reason_for_stop &lt;chr&gt;, vehicle_make &lt;chr&gt;,
+## #   vehicle_year &lt;dbl&gt;, raw_DriverRace &lt;dbl&gt;, raw_ReasonForStop &lt;dbl&gt;,
+## #   raw_TypeOfMovingViolation &lt;dbl&gt;, raw_ResultOfStop &lt;dbl&gt;</code></pre>
+<div id="recode-and-explore-key-variables" class="section level3">
+<h3>Recode and explore key variables</h3>
+<p>Right off the bat, I see a lot of <code>NA</code> values and some things likely to need recoding. Let’s look a little more closely at the variables we are going to work with.</p>
+<pre class="r"><code>key_variables &lt;- c(&quot;date&quot;, &quot;vehicle_year&quot;, &quot;subject_race&quot;, &quot;subject_sex&quot;, &quot;search_conducted&quot;)
+
+lapply(ilstops[key_variables], class)</code></pre>
+<pre><code>## $date
+## [1] &quot;Date&quot;
+## 
+## $vehicle_year
+## [1] &quot;numeric&quot;
+## 
+## $subject_race
+## [1] &quot;character&quot;
+## 
+## $subject_sex
+## [1] &quot;character&quot;
+## 
+## $search_conducted
+## [1] &quot;logical&quot;</code></pre>
+<p>Indeed, I want to create some factors for the <code>subject_race</code> and <code>subject_sex</code> variables.</p>
+<pre class="r"><code>ilstops$subject_race &lt;- factor(ilstops$subject_race)
+ilstops$subject_sex &lt;- factor(ilstops$subject_sex)</code></pre>
+</div>
+</div>
+<div id="pc3-summarize-key-variables" class="section level2">
+<h2>PC3 Summarize key variables</h2>
+<p>Now, let’s look at summaries of the key variables. Keep in mind that these are all variables calculated over all traffic stops in the dataset.</p>
+<pre class="r"><code>lapply(ilstops[key_variables], summary)</code></pre>
+<pre><code>## $date
+##         Min.      1st Qu.       Median         Mean      3rd Qu.         Max. 
+## &quot;2012-01-01&quot; &quot;2013-06-29&quot; &quot;2015-01-22&quot; &quot;2015-01-09&quot; &quot;2016-07-17&quot; &quot;2017-12-31&quot; 
+## 
+## $vehicle_year
+##    Min. 1st Qu.  Median    Mean 3rd Qu.    Max.    NA&#39;s 
+##    1900    2001    2006    2005    2010    2021     137 
+## 
+## $subject_race
+## asian/pacific islander                  black               hispanic 
+##                   4053                  25627                  16940 
+##                  other                  white                   NA&#39;s 
+##                    335                  80105                      4 
+## 
+## $subject_sex
+## female   male   NA&#39;s 
+##  47698  79364      2 
+## 
+## $search_conducted
+##    Mode   FALSE    TRUE    NA&#39;s 
+## logical  120942    5947     175</code></pre>
+<p>Note that the number of missing (<code>NA</code>) values is quite different depending on the variable.</p>
+<div id="recode-nonsense-vehicle_year-values" class="section level3">
+<h3>Recode nonsense <code>vehicle_year</code> values</h3>
+<p>Review those summaries and you might notice something a little funny about the <code>vehicle_year</code> variable. Doesn’t this data end in 2017? None of the <code>vehicle_year</code> values should be more recent than 2018 (if we’re feeling generous since car manufacturers sometimes start selling “next year’s car” at the end of a given year?). I’m also modestly skeptical that the police pulled over a car from 1900, but at least that’s a plausible value! In the abence of additional information about the provenance of this variable or reasons why values larger than 2018 might be reasonable, I will go ahead and recode anything impossibly new as missing (<code>NA</code>).</p>
+<pre class="r"><code>ilstops$vehicle_year[ilstops$vehicle_year &gt; 2018] &lt;- NA
+
+summary(ilstops$vehicle_year)</code></pre>
+<pre><code>##    Min. 1st Qu.  Median    Mean 3rd Qu.    Max.    NA&#39;s 
+##    1900    2001    2006    2005    2010    2018     139</code></pre>
+<pre class="r"><code>hist(ilstops$vehicle_year)</code></pre>
+<p><img src="data:image/png;base64,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" width="672" /></p>
+</div>
+</div>
+<div id="pc4.-summarize-bivariate-relationships" class="section level2">
+<h2>PC4. Summarize bivariate relationships</h2>
+<p>For bivariate summaries and comparisons, I’ll walk through things variable by variable. In all of these cases except for the <code>date</code> variable, I’m considering the bivariate relationships in aggregate (rather than taking the “over time” character of the data into account). This is good to keep in mind since it might mean that the summaries mask temporal trends.</p>
+<div id="searches-by-vehicle-year" class="section level3">
+<h3>Searches by <code>vehicle year</code></h3>
+<p>I’ll start with a continuous predictor, <code>vehicle_year</code>. Since the outcome variable is a dichotomous category, I can look at this by calculating summary statistics within the two outcomes or create a pair of plots. I love visual summaries, so I’ll start with side-by-side boxplots:</p>
+<pre class="r"><code>ilstops %&gt;%
+  filter(!is.na(vehicle_year) &amp; !is.na(search_conducted)) %&gt;%
+  ggplot(aes(x = search_conducted, y = vehicle_year)) +
+  geom_boxplot()</code></pre>
+<p><img src="data:image/png;base64,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" width="672" /></p>
+<p>It looks like stops of older vehicles are, on average, a little bit more likely to result in a search, but stops involving the very oldest vehicles tend to result in fewer searches.</p>
+<p>With that in mind, I’ll calculate summary statistics across the two groups. One way to do this is with a call to the <code>tapply</code> function nested within a call to the <code>with()</code> command. I don’t think we’ve used <code>with</code> yet, but it can allow me to identify the object I want to use (the first argument to <code>with()</code>) to run some command on (the second argument, in this case the call to <code>tapply()</code>).</p>
+<pre class="r"><code>with(
+  ilstops,
+  tapply(vehicle_year, search_conducted, summary)
+)</code></pre>
+<pre><code>## $`FALSE`
+##    Min. 1st Qu.  Median    Mean 3rd Qu.    Max.    NA&#39;s 
+##    1900    2001    2006    2005    2010    2018     132 
+## 
+## $`TRUE`
+##    Min. 1st Qu.  Median    Mean 3rd Qu.    Max.    NA&#39;s 
+##    1900    1999    2003    2003    2007    2017       7</code></pre>
+<p>Indeed, as I suspected based on the histograms, the center of the distribution for stops resulting in searches is a little lower (older vehicles) than among stops not resulting in searches. I’ll calculate the standard deviations with a separate command:</p>
+<pre class="r"><code>with(ilstops, tapply(vehicle_year, search_conducted, sd, na.rm = TRUE))</code></pre>
+<pre><code>##    FALSE     TRUE 
+## 6.748166 6.655972</code></pre>
+<p>The stops not resulting in searches (<code>FALSE</code>) have a larger/wider spread of vehicle ages than those that do result in searches (<code>TRUE</code>). This difference is likely driven by the larger number of old vehicle outliers inovlved in stops that do not result in searches. This is a nice example of how the visual and numerical summaries might help convey a somewhat subtle feature of my data.</p>
+</div>
+<div id="searches-by-subject_sex" class="section level3">
+<h3>Searches by <code>subject_sex</code></h3>
+<p>Next, we’ll focus on the <code>subject_sex</code> variable. Both the outcome and this predictor variable are categorical measures that take only two (non-missing) values. That means that I can focus on creating contingency tables to report the raw numbers of traffic stops in each of the four possible combinations of the two variables. I’ve provided a couple of methods here:</p>
+<pre class="r"><code>table(ilstops$subject_sex, ilstops$search_conducted)</code></pre>
+<pre><code>##         
+##          FALSE  TRUE
+##   female 46098  1536
+##   male   74842  4411</code></pre>
+<pre class="r"><code>## the xtabs() command produces nicely formatted output that includes both variable names
+xtabs(~ subject_sex + search_conducted, ilstops)</code></pre>
+<pre><code>##            search_conducted
+## subject_sex FALSE  TRUE
+##      female 46098  1536
+##      male   74842  4411</code></pre>
+<p>One way to get the proportions in Base-R is to call the <code>proportions()</code> function on a contingency table.</p>
+<pre class="r"><code>proportions(
+  xtabs(~ subject_sex + search_conducted, ilstops)
+)</code></pre>
+<pre><code>##            search_conducted
+## subject_sex      FALSE       TRUE
+##      female 0.36329963 0.01210526
+##      male   0.58983190 0.03476321</code></pre>
+<p>That’s a lot of significant digits…especially for proportions. I can clean it up a bit by nesting everything within a call to the <code>round()</code> function and restricting the output to just two significant digits.</p>
+<pre class="r"><code>round(
+  proportions(
+    xtabs(~ subject_sex + search_conducted, ilstops)
+  ),
+  digits = 2
+)</code></pre>
+<pre><code>##            search_conducted
+## subject_sex FALSE TRUE
+##      female  0.36 0.01
+##      male    0.59 0.03</code></pre>
+<p>Much better! However, notice that these proportions are all calculated against the <em>total</em> number of non-missing values. I can specify row or column proportions with an additional argument to the <code>proportions</code> function to specify which margins to use. Following R conventions, <code>margin=1</code> calculates proportions row-wise and <code>margin=2</code> calculates them column-wise:</p>
+<pre class="r"><code>## Row-wise proportions
+round(
+  proportions(
+    xtabs(~ subject_sex + search_conducted, ilstops), 1
+  ),
+  digits = 2
+)</code></pre>
+<pre><code>##            search_conducted
+## subject_sex FALSE TRUE
+##      female  0.97 0.03
+##      male    0.94 0.06</code></pre>
+<pre class="r"><code>## Column-wise proportions
+round(
+  proportions(
+    xtabs(~ subject_sex + search_conducted, ilstops), 2
+  ),
+  digits = 2
+)</code></pre>
+<pre><code>##            search_conducted
+## subject_sex FALSE TRUE
+##      female  0.38 0.26
+##      male    0.62 0.74</code></pre>
+<p>These different marginal proportions support distinct statements about the data. Here is an example for each:</p>
+<ul>
+<li><strong>Within the rows (<code>subject_sex</code>):</strong> <em>the proportion of searches in stops of male drivers is twice as large as the proportion in stops of female drivers.</em><br />
+</li>
+<li><strong>Within the columns (<code>search_conducted</code>):</strong> <em>stops that resulted in searches involved male drivers almost <span class="math inline">\(75\%\)</span> of the time.</em></li>
+</ul>
+</div>
+<div id="searches-by-subject_race" class="section level3">
+<h3>Searches by <code>subject_race</code></h3>
+<p>The <code>subject_race</code> variable is also categorical, but takes one of five non-missing values in this data. As a result, the basic structure/approach for my summaries is almost identical to that pursued with <code>subject_sex</code>.</p>
+<p>First, tabular summaries of raw counts and proportions:</p>
+<pre class="r"><code>xtabs(~ subject_race + search_conducted, ilstops)</code></pre>
+<pre><code>##                         search_conducted
+## subject_race             FALSE  TRUE
+##   asian/pacific islander  3981    68
+##   black                  23742  1806
+##   hispanic               15865  1049
+##   other                    320    14
+##   white                  77033  3010</code></pre>
+<p>In this case, I think the marginal proportions are far more interesting than the aggregate proportions. Here are proportions of searches within each category of <code>subject_race</code> (row-wise given the way I’ve structured the call to <code>xtabs</code>):</p>
+<pre class="r"><code>## Proportions of searches among stops within groups
+round(
+  proportions(
+    xtabs(~ subject_race + search_conducted, ilstops), 1
+  ),
+  digits = 2
+)</code></pre>
+<pre><code>##                         search_conducted
+## subject_race             FALSE TRUE
+##   asian/pacific islander  0.98 0.02
+##   black                   0.93 0.07
+##   hispanic                0.94 0.06
+##   other                   0.96 0.04
+##   white                   0.96 0.04</code></pre>
+<p>Notice that the proportion of black and hispanic drivers with stops resulting in searches is higher than within other groups.</p>
+<p>Now, I’ll calculate the proportions of each <code>subject_race</code> category among stops resulting in searches and stops that do not result in searches:</p>
+<pre class="r"><code>## Proportions of groups within those stops (not) resulting in searches
+round(
+  proportions(
+    xtabs(~ subject_race + search_conducted, ilstops), 2
+  ),
+  digits = 2
+)</code></pre>
+<pre><code>##                         search_conducted
+## subject_race             FALSE TRUE
+##   asian/pacific islander  0.03 0.01
+##   black                   0.20 0.30
+##   hispanic                0.13 0.18
+##   other                   0.00 0.00
+##   white                   0.64 0.51</code></pre>
+<p>Here the noteworthy comparisons again arise within the black and hispanic categories. Both groups account for a substantially larger proportion of stops resulting in searches vs. those that do not result in searches.</p>
+<p>For the sake of completeness/comparison, here’s a way to do similar cross-tabulations in single a chunk of tidyverse code. I include conditional proportions of all stops to facilitate comparison with some of the tables I created earlier as well:</p>
+<pre class="r"><code>ilstops %&gt;%
+  group_by(subject_race) %&gt;%
+  filter(!is.na(subject_race)) %&gt;%
+  summarize(
+    n_stops = n(),
+    prop_total_stops = round(n() / nrow(ilstops), digits = 3),
+    across(
+      search_conducted,
+      list(
+        sum = ~ sum(.x, na.rm = TRUE),
+        over_n_stops = ~ round(mean(.x, na.rm = TRUE), digits = 3)
+      )
+    )
+  ) %&gt;%
+  arrange(desc(n_stops))</code></pre>
+<pre><code>## # A tibble: 5 x 5
+##   subject_race    n_stops prop_total_stops search_conducted… search_conducted_o…
+##   &lt;fct&gt;             &lt;int&gt;            &lt;dbl&gt;             &lt;int&gt;               &lt;dbl&gt;
+## 1 white             80105            0.63               3010               0.038
+## 2 black             25627            0.202              1806               0.071
+## 3 hispanic          16940            0.133              1049               0.062
+## 4 asian/pacific …    4053            0.032                68               0.017
+## 5 other               335            0.003                14               0.042</code></pre>
+<p>Notice the use of <code>across()</code> within a call to <code>summarize()</code> provides one way to calculate conditional summariy info. I also use a nested call to <code>arrange()</code> and <code>desc()</code> at the end to sort my results in descending order by one of the columns.</p>
+</div>
+<div id="searches-by-date" class="section level3">
+<h3>Searches by <code>date</code></h3>
+<p>Last, but not least, I can inspect how the number of searches is distributed over time. I can do this a few ways, but once again will start with a visual summary because that is how I prefer to approach things. Remember that <code>search_conducted</code> is a dichotomous categorical measure, so a boxplot is a good starting point.</p>
+<pre class="r"><code>ilstops %&gt;%
+  ggplot(aes(x = search_conducted, y = date)) +
+  geom_boxplot()</code></pre>
+<p><img src="data:image/png;base64,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" width="672" /></p>
+<p>How considerate of ggplot to include the <code>NA</code> category by default! Notice how seemingly all of the missing values come from 2012-2014? There’s some of that missing data the SOPP folks warned us about in their documentation. Let’s inspect things a bit more closely:</p>
+<pre class="r"><code>ilstops %&gt;%
+  group_by(search_conducted) %&gt;%
+  summarize(
+    min = min(date, na.rm = TRUE),
+    max = max(date, na.rm = TRUE)
+  )</code></pre>
+<pre><code>## # A tibble: 3 x 3
+##   search_conducted min        max       
+##   &lt;lgl&gt;            &lt;date&gt;     &lt;date&gt;    
+## 1 FALSE            2012-01-01 2017-12-31
+## 2 TRUE             2012-01-01 2017-12-30
+## 3 NA               2012-01-02 2014-01-10</code></pre>
+<p>All the missing data for <code>search_conducted</code> comes within a two-year range.</p>
+<p>I can use <code>filter</code> to look at the <code>search_conducted</code> variable within that two year range alone:</p>
+<pre class="r"><code>ilstops %&gt;%
+  filter(date &lt; as_date(&quot;2014-01-11&quot;)) %&gt;%
+  group_by(search_conducted) %&gt;%
+  summarize(
+    n = n()
+  )</code></pre>
+<pre><code>## # A tibble: 3 x 2
+##   search_conducted     n
+##   &lt;lgl&gt;            &lt;int&gt;
+## 1 FALSE            40076
+## 2 TRUE              2044
+## 3 NA                 175</code></pre>
+<p><em>Most</em> of the <code>search_conducted</code> data between 2012-2014 isn’t missing, even if that’s the only period within which missing values for <code>search_conducted</code> appear. There’s not much we can do about this, but it’s worth taking into consideration as we go.</p>
+</div>
+</div>
+<div id="pc5-searches-within-subject_race-groups-over-time" class="section level2">
+<h2>PC5 Searches within <code>subject_race</code> groups over time</h2>
+<p>The goal here is to synthesize pieces of the work we did in PC3 and PC4 to analyze subgroups of the stops and the stops resulting in searches over time. I’ll start by reorganizing/tidying the data to support the creation of my time series plots.</p>
+<div id="binning-data-within-time-periods" class="section level3">
+<h3>5.1 Binning data within time periods</h3>
+<p>I want to plot this data as a time series of some numeric measure, but right now my outcome variable is a categorical indicator. How can I plot TRUE/FALSE values over time? One good way is to bin my data into meaningful time-periods (e.g., months or weeks) and count the number of TRUE values (and/or calculate proportions of TRUE values) within each bin.</p>
+<p>Let’s do that using some of the handy features of <code>lubridate</code> for working with date objects. I have demonstrated a few approaches to binning and grouping in the R tutorial materials, but here’s yet another that takes advantage of some <code>lubridate</code> features (specifically the call to <code>format</code> within the <code>mutate</code> step below) quite effectively:</p>
+<pre class="r"><code>## library(lubridate)
+
+ilstops_monthly &lt;- ilstops %&gt;%
+  filter(!is.na(subject_race)) %&gt;%
+  mutate(
+    yearmonth = format(date, &quot;%Y-%m&quot;)
+  ) %&gt;%
+  group_by(yearmonth) %&gt;%
+  summarize(
+    stops = n(),
+    searches = sum(as.numeric(search_conducted), na.rm = T),
+    prop_searched = round(searches / stops, 3)
+  )
+
+ilstops_monthly</code></pre>
+<pre><code>## # A tibble: 72 x 4
+##    yearmonth stops searches prop_searched
+##    &lt;chr&gt;     &lt;int&gt;    &lt;dbl&gt;         &lt;dbl&gt;
+##  1 2012-01    1864       94         0.05 
+##  2 2012-02    1907      106         0.056
+##  3 2012-03    1975       86         0.044
+##  4 2012-04    1725       78         0.045
+##  5 2012-05    1909       70         0.037
+##  6 2012-06    1816       83         0.046
+##  7 2012-07    1584       73         0.046
+##  8 2012-08    1778       70         0.039
+##  9 2012-09    1700       81         0.048
+## 10 2012-10    1674       93         0.056
+## # … with 62 more rows</code></pre>
+<p>Looks great, except do you notice how my new <code>yearmonth</code> variable is no longer recognized as a date or date-time object? I think I can fix that with the handy <code>parse_date_time</code> function from <code>lubridate</code>:</p>
+<pre class="r"><code>ilstops_monthly$date &lt;- parse_date_time(ilstops_monthly$yearmonth, &quot;ym&quot;)
+class(ilstops_monthly$date)</code></pre>
+<pre><code>## [1] &quot;POSIXct&quot; &quot;POSIXt&quot;</code></pre>
+<p>Now I can plot it since <code>ggplot</code> knows what to do with objects in the <code>POSIXct</code> class. I’m also going to add a type of layer we haven’t worked with yet (<code>geom_smooth</code>) which incorporates a smoothed conditional mean to the plot (appropriately enough in this dataset, in the form of a thin blue line).</p>
+<pre class="r"><code>ilstops_monthly %&gt;%
+  ggplot(aes(x = date, y = stops)) +
+  geom_line() +
+  geom_smooth() +
+  labs(title = &quot;Illinois traffic stops&quot;, y = &quot;stops (per month)&quot;)</code></pre>
+<p><img src="data:image/png;base64,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" width="672" /></p>
+<p>Turns out traffic stops are not distributed smoothly across the months of the year (I notice, in particular, the big dips and spikes around the end of each year). It also looks like there is a slight <span class="math inline">\(\cup\)</span>-shaped trend overall, but the shifts do not look very large relative to either the month-to-month variations or the total number of stops per year.</p>
+<p>Here’s a similar plot for searches:</p>
+<pre class="r"><code>ilstops_monthly %&gt;%
+  ggplot(aes(x = date, y = searches)) +
+  geom_line() +
+  geom_smooth() +
+  labs(
+    title = &quot;Illinois traffic stops that result in searches&quot;,
+    y = &quot;searches (per month)&quot;
+  )</code></pre>
+<p><img src="data:image/png;base64,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" width="672" /> Turns out searches are also not distributed smoothly across the months of the year (again, the big dips and spikes around the end of the year). In this case, there seems to be a very slight downward trend on average, but this decrease mostly appears in the first couple of years of the data and does not look very large relative to the month-to-month variations or the total number of searches per year (maybe ten fewer searches per month on average during the latter half of the dataset?).</p>
+<p>Let’s try looking at the proportion of stops that result in searches.</p>
+<pre class="r"><code>ilstops_monthly %&gt;%
+  ggplot(aes(x = date, y = prop_searched)) +
+  geom_line() +
+  geom_smooth() +
+  labs(title = &quot;Proportion of Illinois traffic stops that result in searches&quot;, y = &quot;proportion searched\n(per month)&quot;)</code></pre>
+<p><img src="data:image/png;base64,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" width="672" /> Still quite spiky! Interestingly, we see an inverse-<span class="math inline">\(\cap\)</span> trend in the average here (contrast with the raw counts of traffic stops above), but these fluctuations are, again, very small relative to the underlying variation or the range of measure. Also, notice the narrow y-axis scale sort of hides the fact that almost all the values fall between <span class="math inline">\(4-6\%\)</span>. In other words, all of the variation here is within quite a narrow range.</p>
+</div>
+<div id="plot-stops-within-subject_race-categories" class="section level3">
+<h3>5.2 Plot stops within <code>subject_race</code> categories</h3>
+<p>To do this, I’ll need to go back to the code that created my <code>ilstops_monthly</code> tibble above and incorporate the conditional summary data. To do this, I’ll “explode” the aggregated summaries from the old tibble by inserting an extra variable to <code>group_by()</code> before making my call to <code>summarize</code>. Notice that I go ahead and cleanup the dates again too (since I know to anticipate that issue)</p>
+<pre class="r"><code>ilstops_monthly_grouped &lt;- ilstops %&gt;%
+  filter(!is.na(subject_race)) %&gt;%
+  mutate(
+    yearmonth = format(date, &quot;%Y-%m&quot;)
+  ) %&gt;%
+  group_by(yearmonth, subject_race) %&gt;%
+  summarize(
+    stops = n(),
+    searches = sum(as.numeric(search_conducted), na.rm = T),
+    prop_searched = round(searches / stops, 3)
+  )
+
+ilstops_monthly_grouped$date &lt;- parse_date_time(ilstops_monthly_grouped$yearmonth, &quot;ym&quot;)
+
+ilstops_monthly_grouped</code></pre>
+<pre><code>## # A tibble: 360 x 6
+## # Groups:   yearmonth [72]
+##    yearmonth subject_race       stops searches prop_searched date               
+##    &lt;chr&gt;     &lt;fct&gt;              &lt;int&gt;    &lt;dbl&gt;         &lt;dbl&gt; &lt;dttm&gt;             
+##  1 2012-01   asian/pacific isl…    56        1         0.018 2012-01-01 00:00:00
+##  2 2012-01   black                350       33         0.094 2012-01-01 00:00:00
+##  3 2012-01   hispanic             254       21         0.083 2012-01-01 00:00:00
+##  4 2012-01   other                  7        0         0     2012-01-01 00:00:00
+##  5 2012-01   white               1197       39         0.033 2012-01-01 00:00:00
+##  6 2012-02   asian/pacific isl…    53        0         0     2012-02-01 00:00:00
+##  7 2012-02   black                378       37         0.098 2012-02-01 00:00:00
+##  8 2012-02   hispanic             223       26         0.117 2012-02-01 00:00:00
+##  9 2012-02   other                  4        0         0     2012-02-01 00:00:00
+## 10 2012-02   white               1249       43         0.034 2012-02-01 00:00:00
+## # … with 350 more rows</code></pre>
+<p>Great! Now, I need to add the values for total stops and total searches each month (note that this will be the same for all five groups/rows within any given value of <code>date</code> or <code>yearmonth</code>) so that I can calculate the proportion of total stops and total searches accounted for by each group. There are a bunch of ways we could go about this. Since I’ve made it this far with tidyverse code, I’ll keep going in that direction and use the <code>mutate</code> verb to keep all my existing rows while also creating some new ones. Notice that I have to do this twice because my first call to <code>group_by</code> is only by <code>yearmonth</code> so that I can create the monthly totals, whereas my second call to <code>group_by</code> once again looks wihtin <code>yearmonth</code> and <code>subject_race</code> to calculate the proportions. In order to make sure that nothing weird happens along the way I insert a call to <code>ungroup</code> as well (which does exactly what you might hope following an operation performed on some data that’s already grouped in some way).</p>
+<pre class="r"><code>ilstops_monthly_grouped &lt;- ilstops_monthly_grouped %&gt;%
+  group_by(yearmonth) %&gt;%
+  mutate(
+    total_stops = sum(stops),
+    total_searches = sum(searches)
+  ) %&gt;%
+  ungroup() %&gt;%
+  group_by(yearmonth, subject_race) %&gt;%
+  mutate(
+    prop_total_stops = stops / total_stops,
+    prop_total_searches = searches / total_searches
+  )
+
+ilstops_monthly_grouped</code></pre>
+<pre><code>## # A tibble: 360 x 10
+## # Groups:   yearmonth, subject_race [360]
+##    yearmonth subject_race stops searches prop_searched date               
+##    &lt;chr&gt;     &lt;fct&gt;        &lt;int&gt;    &lt;dbl&gt;         &lt;dbl&gt; &lt;dttm&gt;             
+##  1 2012-01   asian/pacif…    56        1         0.018 2012-01-01 00:00:00
+##  2 2012-01   black          350       33         0.094 2012-01-01 00:00:00
+##  3 2012-01   hispanic       254       21         0.083 2012-01-01 00:00:00
+##  4 2012-01   other            7        0         0     2012-01-01 00:00:00
+##  5 2012-01   white         1197       39         0.033 2012-01-01 00:00:00
+##  6 2012-02   asian/pacif…    53        0         0     2012-02-01 00:00:00
+##  7 2012-02   black          378       37         0.098 2012-02-01 00:00:00
+##  8 2012-02   hispanic       223       26         0.117 2012-02-01 00:00:00
+##  9 2012-02   other            4        0         0     2012-02-01 00:00:00
+## 10 2012-02   white         1249       43         0.034 2012-02-01 00:00:00
+## # … with 350 more rows, and 4 more variables: total_stops &lt;int&gt;,
+## #   total_searches &lt;dbl&gt;, prop_total_stops &lt;dbl&gt;, prop_total_searches &lt;dbl&gt;</code></pre>
+<pre class="r"><code>## I think the html drops a few of our new columns, so I&#39;ll inspect them manually here
+head(ilstops_monthly_grouped[, c(&quot;yearmonth&quot;, &quot;subject_race&quot;, &quot;total_searches&quot;, &quot;prop_total_stops&quot;, &quot;prop_total_searches&quot;)])</code></pre>
+<pre><code>## # A tibble: 6 x 5
+## # Groups:   yearmonth, subject_race [6]
+##   yearmonth subject_race       total_searches prop_total_stops prop_total_searc…
+##   &lt;chr&gt;     &lt;fct&gt;                       &lt;dbl&gt;            &lt;dbl&gt;             &lt;dbl&gt;
+## 1 2012-01   asian/pacific isl…             94          0.0300             0.0106
+## 2 2012-01   black                          94          0.188              0.351 
+## 3 2012-01   hispanic                       94          0.136              0.223 
+## 4 2012-01   other                          94          0.00376            0     
+## 5 2012-01   white                          94          0.642              0.415 
+## 6 2012-02   asian/pacific isl…            106          0.0278             0</code></pre>
+<p>Now my plotting code will look pretty similar to the univariate plots above, just with the addition of another aesthetic…</p>
+<pre class="r"><code>ilstops_monthly_grouped %&gt;%
+  ggplot(aes(x = date, y = stops, color = subject_race)) +
+  geom_line() +
+  geom_smooth() +
+  labs(title = &quot;Illinois traffic stops&quot;, y = &quot;stops (per month)&quot;)</code></pre>
+<p><img src="data:image/png;base64,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" width="672" /></p>
+</div>
+<div id="plot-searches-within-subject_race-categories" class="section level3">
+<h3>5.3 Plot searches within <code>subject_race</code> categories</h3>
+<pre class="r"><code>ilstops_monthly_grouped %&gt;%
+  ggplot(aes(x = date, y = searches, color = subject_race)) +
+  geom_line() +
+  geom_smooth() +
+  labs(title = &quot;Illinois traffic stops resulting in searches&quot;, y = &quot;searches (per month)&quot;)</code></pre>
+<p><img src="data:image/png;base64,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" width="672" /></p>
+</div>
+<div id="plot-proportion-of-searches-within-subject_race-categories" class="section level3">
+<h3>5.4 Plot proportion of searches within <code>subject_race</code> categories</h3>
+<p>The question for this one only asked about proportions of total searches accounted for by each group of <code>subject_race</code>, when several other interesting baselines/comparisons are available. For example, the proportion of total stops across groups as well as stops resulting in searches <em>within</em> each group also strike me as potentially illustrative. I’ll plot each of these proportions just for the sake of completeness. Let’s start with the proportion of total stops plotted across groups.</p>
+<pre class="r"><code>ilstops_monthly_grouped %&gt;%
+  ggplot(aes(x = date, y = prop_total_stops, color = subject_race)) +
+  geom_line() +
+  geom_smooth() +
+  labs(title = &quot;Proportion of total Illinois traffic stops by race/ethnicity&quot;, y = &quot;proportion of total stops\n(per month)\n&quot;)</code></pre>
+<p><img src="data:image/png;base64,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" width="672" /></p>
+<p>Onwards to the proportion of stops resulting in searches calculated <em>within</em> each group of <code>subject_race</code>:</p>
+<pre class="r"><code>ilstops_monthly_grouped %&gt;%
+  ggplot(aes(x = date, y = prop_searched, color = subject_race)) +
+  geom_line() +
+  geom_smooth() +
+  labs(title = &quot;Proportion of Illinois traffic stops that result in searches&quot;, y = &quot;proportion searched\n(calculated per month)&quot;)</code></pre>
+<p><img src="data:image/png;base64,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" width="672" /></p>
+<p>Hmm, that looks pretty messy. The proportions compress everything into a narrow region of the y-axis. Also, the data for some of these groups (especially “other”) fluctuates all over the place, which is a distration since that’s the smallest category.</p>
+<p>Instead of plotting all the proportions on the same figure, it might be easier to see/compare variations if I use facets and don’t include the “other” category (because the y-axis scale is just so different).</p>
+<pre class="r"><code>ilstops_monthly_grouped %&gt;%
+  filter(subject_race != &quot;other&quot;) %&gt;%
+  ggplot(aes(x = date, y = prop_searched, color = subject_race)) +
+  geom_line() +
+  geom_smooth() +
+  facet_grid(rows = vars(subject_race)) +
+  labs(title = &quot;Proportion of Illinois traffic stops that result in searches by race/ethnicity&quot;, y = &quot;proportion searched\n(per month)\n&quot;)</code></pre>
+<p><img src="data:image/png;base64,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" width="672" /></p>
+<p>The title runs a little long and facet label for “asian/pacific islander” gets truncated. If I were preparing this for publication I’d fix those issues, but for now, I’m going to acknowledge them and move on.</p>
+<p>Now, let’s look at the proportion of total searches.</p>
+<pre class="r"><code>ilstops_monthly_grouped %&gt;%
+  ggplot(aes(x = date, y = prop_total_searches, color = subject_race)) +
+  geom_line() +
+  geom_smooth() +
+  labs(title = &quot;Proportion of total Illinois traffic stops resulting in searches by race/ethnicity&quot;, y = &quot;proportion of total searches\n(per month)\n&quot;)</code></pre>
+<p><img src="data:image/png;base64,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" width="672" /></p>
+</div>
+</div>
+<div id="pc6.-calculate-baseline-population-proportions" class="section level2">
+<h2>PC6. Calculate baseline population proportions</h2>
+<p>As recommended, I’ll calculate this using the <a href="https://www.openintro.org/data/index.php?data=county_complete"><code>county_complete</code></a> provided as part of the <code>openintro</code> package to build the relevant variables for Illinois in 2010. You’ll want to review the <a href="https://www.openintro.org/data/index.php?data=county_complete">list of variables</a> to figure out what you need to use here.</p>
+<p>Note that what we’re looking for are <em>statewide-proportions</em> of race/ethnic groups in 2010 within the same categories in the SOPP traffic stop data for Illinois. The <code>county_complete</code> dataset does not provide these values directly, but instead has county-level populations and county-level proportions of slightly different race/ethnic groups over multiple different years across all the counties in the United States. That means that we’ll need to take some steps (like the following) to generate state-level proportion estimates:</p>
+<ul>
+<li>Restrict the dataset to Illinois counties only.<br />
+</li>
+<li>Multiply county-level populations by county-level proportions for salient groups to create county-level population counts within groups.<br />
+</li>
+<li>Sum county-level populations within groups to create state-level group populations.<br />
+</li>
+<li>Divide state-level group populations by total state population (sum of all county populations) to calculate state-level proportions.</li>
+</ul>
+<p>In the code chunk below, I’ll tackle these steps in a very “verbose” and redundant way:</p>
+<pre class="r"><code>library(openintro)
+data(county_complete)
+
+il_county &lt;- county_complete[county_complete$state == &quot;Illinois&quot;, ]
+
+## total state population is easy
+il_pop2010 &lt;- sum(il_county$pop2010)
+
+## Turn IL county-level proportions into counts
+il_county$white2010 &lt;- il_county$pop2010 * il_county$white_2010
+il_county$black2010 &lt;- il_county$pop2010 * il_county$black_2010
+il_county$hispanic2010 &lt;- il_county$pop2010 * il_county$hispanic_2010
+
+## We&#39;ll have to sum these two groups to approximate &quot;Asian / Pacific Islander&quot; from SOPP data
+il_county$asian2010 &lt;- (il_county$pop2010 * il_county$asian_2010)
+il_county$pi2010 &lt;- (il_county$pop2010 * il_county$pac_isl_2010)
+
+## Then go through summing and dividing by the total population
+##
+## prop.white
+sum(il_county$white2010) / il_pop2010</code></pre>
+<pre><code>## [1] 71.53672</code></pre>
+<pre class="r"><code>## prop.black
+sum(il_county$black2010) / il_pop2010</code></pre>
+<pre><code>## [1] 14.5509</code></pre>
+<pre class="r"><code>## prop.hispanic
+sum(il_county$hispanic2010) / il_pop2010</code></pre>
+<pre><code>## [1] 15.82155</code></pre>
+<pre class="r"><code>## prop.api
+(sum(il_county$asian2010, na.rm = TRUE) + sum(il_county$pi2010, na.rm = TRUE)) / il_pop2010</code></pre>
+<pre><code>## [1] 4.576022</code></pre>
+<p>That is all perfectly accurate; however, the code is verbose and redundant because it calculates each (intermediate and group) value with separate lines of code that are very repetitive. There are many more concise ways to go about this, all of which make it faster, less prone to typing mistakes, and easier to extend. How to proceed? Anytime I find myself repeating the same steps of analysis more than once, I consider creating a function and then calling that function within a call to an <code>*apply()</code> function. Here’s what that might look like here:</p>
+<pre class="r"><code>gen_subgroup_prop &lt;- function(group_var, d = il_county) {
+  total_pop2010 &lt;- sum(d[&quot;pop2010&quot;]) # state population total
+  county_ests &lt;- d[&quot;pop2010&quot;] * d[group_var] # county-level counts within groups
+  group_prop &lt;- sum(county_ests, na.rm = TRUE) / total_pop2010 # state-level proportions within groups
+  return(round(group_prop, 4)) # prettify output
+}
+
+## give it a try:
+gen_subgroup_prop(&quot;hispanic_2010&quot;)</code></pre>
+<pre><code>## [1] 15.8215</code></pre>
+<pre class="r"><code>## Now let&#39;s tie everything together
+groups &lt;- c(&quot;white_2010&quot;, &quot;black_2010&quot;, &quot;hispanic_2010&quot;, &quot;asian_2010&quot;, &quot;pac_isl_2010&quot;)
+
+sapply(groups, gen_subgroup_prop)</code></pre>
+<pre><code>##    white_2010    black_2010 hispanic_2010    asian_2010  pac_isl_2010 
+##       71.5367       14.5509       15.8215        4.5717        0.0043</code></pre>
+<p>I need to add those last two values together to reproduce the “asian/pacific islander” category from the SOPP data. Note also that there are slight differences between my results here and my results above. Any differences <strong>should</strong> be due entirely to rounding and, in a scenario where this analysis was being prepared for publication and/or informing a policy decision, I’d ansp;ite;y want to check that and think carefully how many significant digits to include in my reported estimates. The Illinois population is sufficiently large that even a few hundredths of a percent turn into very meaningful numbers of people!</p>
+</div>
+</div>
+<div id="statistical-questions" class="section level1">
+<h1>Statistical questions</h1>
+<p>Note that my “solutions” below reflect possible interpretations, but are not intended to be exhaustive or exclusive of other possible interpretations. We’ll discuss these in our class meeting.</p>
+<div id="sq1.-interpret-analysis-from-pcs3-5" class="section level2">
+<h2>SQ1. Interpret analysis from PCs3-5</h2>
+<p>Some interpretation appeared above alongside specific results. Here is a brief summary of several striking points.</p>
+<ul>
+<li>Overall, traffic stops and searches in Illinois between 2012-2018 were distributed unequally across different racial/ethnic groups.
+<ul>
+<li>The proportion of stops resulting in searches is higher among stops of black and hispanic drivers (<span class="math inline">\(7\%\)</span> and <span class="math inline">\(6\%\)</span> respectively vs. <span class="math inline">\(4\%\)</span> or less for all other groups).<br />
+</li>
+<li>Black and hispanic drivers account for a higher proportion of stops resulting in searches than stops not resulting in searches (<span class="math inline">\(30\%\)</span> and <span class="math inline">\(18\%\)</span> vs. <span class="math inline">\(20\%\)</span> and <span class="math inline">\(13\%\)</span> respectively).</li>
+</ul></li>
+<li>Over time, the tpyical number of traffic stops, stops resulting in searches, and the proportion of stops resulting in searches fluctuate widely, but have fairly stationary central trends.
+<ul>
+<li>The typical number of monthly stops dipped a bit between 2014-2015.<br />
+</li>
+<li>The typical number of monthly stops resulting in searches may have decreased 2012-2015, but held flat 2015-2018.<br />
+</li>
+<li>The typical monthly proportion of stops resulting in searches increased a little 2012-2014, but then declined a little bit 2014-2017.</li>
+</ul></li>
+<li>Comparing across the racial/ethnic groups identified in the data, the <em>typical monthly numbers of stops and searches</em> vary substantially and exhibit distinct trends.
+<ul>
+<li>The typical number of black and hispanic drivers stopped per month has increased.<br />
+</li>
+<li>The typical number of white drivers stopped per month has decreased.<br />
+</li>
+<li>The typical number of asian/pacific islander and “other” category drivers per month has remained nearly flat.<br />
+</li>
+<li>The typical number of stops resulting in searches among black, white, and hispanic drivers per month have tended to be quite close to each other, and sometimes overlap,</li>
+<li>The typical number of stops resulting in searches among white drivers per month has increased slightly.</li>
+<li>The typical number of stops resulting in searches among black and hispanic drivers per month has decreased slightly.</li>
+</ul></li>
+<li>The typical <em>proportion of stops resulting in searches within racial/ethnic groups</em> have shifted somewhat over time.
+<ul>
+<li>The proportion has increased slightly among white and asian/pacific islander drivers.<br />
+</li>
+<li>The proportion has decreased slightly among black and hispanice drivers.</li>
+</ul></li>
+<li>The typical <em>proportion of total stops accounted for by each racial/ethnic group</em> has shifted somewhat over time.
+<ul>
+<li>The proportion has decreased substantially among white drivers.<br />
+</li>
+<li>The proportion has increased among black drivers (substantially) and among hispanic drivers (slightly).</li>
+</ul></li>
+<li>The typical <em>proportion of total searches accounted for by each racial/ethnic group</em> has also shifted over time.
+<ul>
+<li>As of 2012, the proportion accounted for by black and white drivers was nearly identical (close to <span class="math inline">\(40\%\)</span> each)!</li>
+<li>The proprtion has increased substantially among white drivers (up to .<br />
+</li>
+<li>The proportion has decreased substantially among black and hispanic drivers.</li>
+</ul></li>
+</ul>
+</div>
+<div id="sq2.-contextualize-sq1-interpretation-in-relation-to-pc6" class="section level2">
+<h2>SQ2. Contextualize SQ1 interpretation in relation to PC6</h2>
+<p>Several noteworthy comparisons come looking across the different proportions for traffic stops and searches and comparing those against the baseline population proportions accounted for by each racial/ethnic group category. Here are several specific observations:</p>
+<ul>
+<li>The white proportion of the state population is consistently larger than either the proportion of white drivers involved in all traffic stops or the proportion of white drivers involved in all stops resulting in searches.<br />
+</li>
+<li>The hispanic proportion of the state population is about the same as the proportion of hispanic drivers involved in all traffic stops and usually a little smaller than the proportion of hispanic drivers involved in all stops resulting in searches (although this latter difference shrank over the period of data collection).<br />
+</li>
+<li>The black proportion of the state population is consistently and substantially smaller than the proportion of black drivers involved in all traffic stops and consistently and substantially smaller than the proportion of black drivers involved in all stops resulting in searches. This latter difference in particular stands out as it is almost more than double the baseline population proportion.<br />
+</li>
+<li>Overall, these results suggest that race/ethnicity and traffic stops and searches are not indepdendent in a statistical sense. The nature of their relationship is complex and could be due to a number of factors including biased policing practices, socioeconomic inequalities, different levels of behaviors linked to traffic stops within specific sub-groups, regional variations masked by these state-level analyses, political/cultural factors shaping police/driver interactions, and more.</li>
+</ul>
+</div>
+<div id="sq3.-limitations-and-possible-extensions" class="section level2">
+<h2>SQ3. Limitations and possible extensions</h2>
+<p>Again, many possible things worth mentioning here, so I’ll provide a few that stand out to me.</p>
+<ul>
+<li>The generalizability of analysis focused on one state during one 6 year period is limited.</li>
+<li>The data seem very prone to measurement errors of various kinds. In particular, I suspect the race/ethnicity classifications provided by officers are subject to some biases that are hard to identify and might also shift over time/region. The prevalence of missing values during the first two years of the dataset illustrate one aspect of this and may impact estimates of raw counts and proportions.<br />
+</li>
+<li>While the comparisons across racial/ethnic groups and between the traffic stops/searches and baseline population proportions illustrates a number of suggestive patterns, conclusive interpretation or attribution of those patterns to any specific cause or causes is quite difficult in the absence of additional information or assumptions. For one example, see my comments regarding statistical independence and the possible explanations in SQ2 above.</li>
+<li>Extensions of this analysis might seek to investigate how some of the patterns identified in the aggregate sate-level data vary across sub-regions (e.g., counties or police districts) or even in comparison to other states.<br />
+</li>
+<li>Another extension might investigate how specific policy and/or socioeconomic changes may or may not relate to shifting patterns in the data.</li>
+<li>Yet another extension could try to identify some factor that introduces some sudden, uncontrolled (by the drivers or the police) change into the process of traffic stops that could make it possible to identify and estimate the effects of specific explanatory factors. The original SOPP paper is an extraordinary example of this that we can discuss!</li>
+</ul>
+</div>
+</div>
+
+
+
+</div>
+</div>
+
+</div>
+
+<script>
+
+// add bootstrap table styles to pandoc tables
+function bootstrapStylePandocTables() {
+  $('tr.header').parent('thead').parent('table').addClass('table table-condensed');
+}
+$(document).ready(function () {
+  bootstrapStylePandocTables();
+});
+
+
+</script>
+
+<!-- tabsets -->
+
+<script>
+$(document).ready(function () {
+  window.buildTabsets("TOC");
+});
+
+$(document).ready(function () {
+  $('.tabset-dropdown > .nav-tabs > li').click(function () {
+    $(this).parent().toggleClass('nav-tabs-open')
+  });
+});
+</script>
+
+<!-- code folding -->
+
+<script>
+$(document).ready(function ()  {
+
+    // move toc-ignore selectors from section div to header
+    $('div.section.toc-ignore')
+        .removeClass('toc-ignore')
+        .children('h1,h2,h3,h4,h5').addClass('toc-ignore');
+
+    // establish options
+    var options = {
+      selectors: "h1,h2,h3",
+      theme: "bootstrap3",
+      context: '.toc-content',
+      hashGenerator: function (text) {
+        return text.replace(/[.\\/?&!#<>]/g, '').replace(/\s/g, '_');
+      },
+      ignoreSelector: ".toc-ignore",
+      scrollTo: 0
+    };
+    options.showAndHide = true;
+    options.smoothScroll = true;
+
+    // tocify
+    var toc = $("#TOC").tocify(options).data("toc-tocify");
+});
+</script>
+
+<!-- dynamically load mathjax for compatibility with self-contained -->
+<script>
+  (function () {
+    var script = document.createElement("script");
+    script.type = "text/javascript";
+    script.src  = "https://mathjax.rstudio.com/latest/MathJax.js?config=TeX-AMS-MML_HTMLorMML";
+    document.getElementsByTagName("head")[0].appendChild(script);
+  })();
+</script>
+
+</body>
+</html>
diff --git a/psets/pset3-worked_solution.pdf b/psets/pset3-worked_solution.pdf
new file mode 100644 (file)
index 0000000..efa7cab
Binary files /dev/null and b/psets/pset3-worked_solution.pdf differ
diff --git a/psets/pset3-worked_solution.rmd b/psets/pset3-worked_solution.rmd
new file mode 100644 (file)
index 0000000..b7f3ec3
--- /dev/null
@@ -0,0 +1,508 @@
+---
+title: "Problem set 3: Worked solutions"
+subtitle: "Statistics and statistical programming  \nNorthwestern University  \nMTS 525"
+author: "Aaron Shaw"
+date: "October 12, 2020"
+output:
+  html_document:
+    toc: yes
+    toc_depth: 3
+    toc_float:
+      collapsed: true
+      smooth_scroll: true
+    theme: readable
+  pdf_document:
+    toc: yes
+    toc_depth: '3'
+---
+
+```{r setup, include=FALSE}
+knitr::opts_chunk$set(echo = TRUE, message=FALSE, tidy='styler')
+```
+
+# Programming challenges  
+## PC1. Learn about the data.
+
+The documentation makes it sound like the Illinois data is particularly messy/incomplete. It explains that the data is limited to 2012-2017 (instead of extending back to 2004) "due to format issues and relevance." The notes point out that data related to searches vary substantially year-to-year, suggesting that the measurement was inconsistent and that caution should be exercised in the analysis of this measure in particular. The `search_conducted` variable seems to include searches of any kind (vehicles, driver, passengers). I also notice that the codebook does not provide much insight into *how* `subject_race` and `subject_sex` information were collected, measured, and/or recoded. I think it's best to presume that the records reflect the judgments/notes/documents accessed by the officer(s) and/or police forces involved rather than the perspectives of the subjects of the traffic stops. There are likely some inconsistencies in these measures as well.
+
+## PC2. Import, explore, clean
+
+I'll use the tidyverse `read_csv()` function to do this. While I'm at it, I'll import the `lubridate` library because I already know this is time series data and there are a bunch of helpful functions in `lubridate` that I will probably want to use.
+
+```{r import}
+library(tidyverse)
+library(lubridate)
+data.dir <- "https://communitydata.science/~ads/teaching/2020/stats/data/week_05"
+filename <- "il_statewide_sample-2020_04_01.csv"
+ilstops <- read_csv(url(paste(data.dir, filename, sep="/")))
+
+ilstops
+```
+
+### Recode and explore key variables
+
+Right off the bat, I see a lot of `NA` values and some things likely to need recoding. Let's look a little more closely at the variables we are going to work with.
+```{r class check}
+
+key_variables <- c("date", "vehicle_year", "subject_race", "subject_sex", "search_conducted")
+
+lapply(ilstops[key_variables], class)
+```
+Indeed, I want to create some factors for the `subject_race` and `subject_sex` variables. 
+```{r recode factors}
+ilstops$subject_race <- factor(ilstops$subject_race)
+ilstops$subject_sex <- factor(ilstops$subject_sex)
+```
+## PC3 Summarize key variables
+Now, let's look at summaries of the key variables. Keep in mind that these are all variables calculated over all traffic stops in the dataset.  
+```{r summarize}
+lapply(ilstops[key_variables], summary)
+```
+Note that the number of missing (`NA`) values is quite different depending on the variable.
+
+### Recode nonsense `vehicle_year` values
+
+Review those summaries and you might notice something a little funny about the `vehicle_year` variable. Doesn't this data end in 2017? None of the `vehicle_year` values should be more recent than 2018 (if we're feeling generous since car manufacturers sometimes start selling "next year's car" at the end of a given year?). I'm also modestly skeptical that the police pulled over a car from 1900, but at least that's a plausible value! In the abence of additional information about the provenance of this variable or reasons why values larger than 2018 might be reasonable, I will go ahead and recode anything impossibly new as missing (`NA`).
+```{r recode vehicle_year}
+ilstops$vehicle_year[ilstops$vehicle_year > 2018] <- NA
+
+summary(ilstops$vehicle_year)
+hist(ilstops$vehicle_year)
+```
+
+## PC4. Summarize bivariate relationships  
+For bivariate summaries and comparisons, I'll walk through things variable by variable. In all of these cases except for the `date` variable, I'm considering the bivariate relationships in aggregate (rather than taking the "over time" character of the data into account). This is good to keep in mind since it might mean that the summaries mask temporal trends.
+
+### Searches by `vehicle year`  
+I'll start with a continuous predictor, `vehicle_year`. Since the outcome variable is a dichotomous category, I can look at this by calculating summary statistics within the two outcomes or create a pair of plots. I love visual summaries, so I'll start with side-by-side boxplots:
+
+```{r pair boxplot vehicle year x searches}
+ilstops %>%
+  filter(!is.na(vehicle_year) & !is.na(search_conducted)) %>%
+  ggplot(aes(x=search_conducted, y=vehicle_year)) +  geom_boxplot()
+
+```
+
+It looks like stops of older vehicles are, on average, a little bit more likely to result in a search, but stops involving the very oldest vehicles tend to result in fewer searches.
+
+With that in mind, I'll calculate summary statistics across the two groups. One way to do this is with a call to the `tapply` function nested within a call to the `with()` command. I don't think we've used `with` yet, but it can allow me to identify the object I want to use (the first argument to `with()`) to run some command on (the second argument, in this case the call to `tapply()`).
+
+```{r crosstab vehicle year x searches}
+with(ilstops, 
+     tapply(vehicle_year, search_conducted, summary)
+     )
+```
+Indeed, as I suspected based on the histograms, the center of the distribution for stops resulting in searches is a little lower (older vehicles) than among stops not resulting in searches. I'll calculate the standard deviations with a separate command:
+```{r}
+with(ilstops, tapply(vehicle_year, search_conducted, sd, na.rm=TRUE))
+```
+
+The stops not resulting in searches (`FALSE`) have a larger/wider spread of vehicle ages than those that do result in searches (`TRUE`). This difference is likely driven by the larger number of old vehicle outliers inovlved in stops that do not result in searches. This is a nice example of how the visual and numerical summaries might help convey a somewhat subtle feature of my data.
+
+### Searches by `subject_sex`
+
+Next, we'll focus on the `subject_sex` variable. Both the outcome and this predictor variable are categorical measures that take only two (non-missing) values. That means that I can focus on creating contingency tables to report the raw numbers of traffic stops in each of the four possible combinations of the two variables. I've provided a couple of methods here:
+```{r crosstab searches and sex}
+table(ilstops$subject_sex, ilstops$search_conducted)
+
+## the xtabs() command produces nicely formatted output that includes both variable names
+xtabs(~ subject_sex + search_conducted, ilstops)
+
+```
+One way to get the proportions in Base-R is to call the `proportions()` function on a contingency table. 
+```{r props searches}
+proportions(
+  xtabs(~ subject_sex + search_conducted, ilstops)
+  )
+```
+
+That's a lot of significant digits...especially for proportions. I can clean it up a bit by nesting everything within a call to the `round()` function and restricting the output to just two significant digits.
+
+```{r pretty prop searches}
+round(
+  proportions(
+    xtabs(~ subject_sex + search_conducted, ilstops)
+    )
+  , digits=2)
+```
+Much better! However, notice that these proportions are all calculated against the *total* number of non-missing values. I can specify row or column proportions with an additional argument to the `proportions` function to specify which margins to use. Following R conventions, `margin=1` calculates proportions row-wise and `margin=2` calculates them column-wise: 
+
+```{r marginal proportions}
+
+## Row-wise proportions
+round(
+  proportions(
+    xtabs(~ subject_sex + search_conducted, ilstops),1
+    )
+  , digits=2)
+
+## Column-wise proportions
+round(
+  proportions(
+    xtabs(~ subject_sex + search_conducted, ilstops),2
+    )
+  , digits=2)
+```
+These different marginal proportions support distinct statements about the data. Here is an example for each:   
+
+* **Within the rows (`subject_sex`):** *the proportion of searches in stops of male drivers is twice as large as the proportion in stops of female drivers.*  
+* **Within the columns (`search_conducted`):** *stops that resulted in searches involved male drivers almost $75\%$ of the time.*  
+
+### Searches by `subject_race`
+
+The `subject_race` variable is also categorical, but takes one of five non-missing values in this data. As a result, the basic structure/approach for my summaries is almost identical to that pursued with `subject_sex`.
+
+First, tabular summaries of raw counts and proportions:
+```{r}
+xtabs(~ subject_race + search_conducted, ilstops)
+```
+In this case, I think the marginal proportions are far more interesting than the aggregate proportions. Here are proportions of searches within each category of `subject_race` (row-wise given the way I've structured the call to `xtabs`):
+```{r prop searches within group}
+## Proportions of searches among stops within groups
+round(
+  proportions(
+    xtabs(~ subject_race + search_conducted, ilstops), 1)
+  , digits=2)
+```
+Notice that the proportion of black and hispanic drivers with stops resulting in searches is higher than within other groups.
+
+Now, I'll calculate the proportions of each `subject_race` category among stops resulting in searches and stops that do not result in searches:
+```{r prop group within searches}
+## Proportions of groups within those stops (not) resulting in searches
+round(
+  proportions(
+    xtabs(~ subject_race + search_conducted, ilstops), 2)
+  , digits=2)
+```
+Here the noteworthy comparisons again arise within the black and hispanic categories. Both groups account for a substantially larger proportion of stops resulting in searches vs. those that do not result in searches. 
+
+For the sake of completeness/comparison, here's a way to do similar cross-tabulations in single a chunk of tidyverse code. I include conditional proportions of all stops to facilitate comparison with some of the tables I created earlier as well:  
+```{r tidyverse crosstabs}
+ilstops %>%
+  group_by(subject_race) %>%
+  filter(!is.na(subject_race)) %>%
+  summarize(
+    n_stops = n(),
+    prop_total_stops = round(n() / nrow(ilstops), digits=3),
+    across(
+      search_conducted,
+      list(
+        sum = ~ sum(.x, na.rm=TRUE),
+        over_n_stops = ~ round(mean(.x, na.rm=TRUE), digits=3)  
+        )
+      )
+    ) %>%
+      arrange(desc(n_stops))
+```
+
+Notice the use of `across()` within a call to `summarize()` provides one way to calculate conditional summariy info. I also use a nested call to `arrange()` and `desc()` at the end to sort my results in descending order by one of the columns.  
+
+### Searches by `date`
+
+Last, but not least, I can inspect how the number of searches is distributed over time. I can do this a few ways, but once again will start with a visual summary because that is how I prefer to approach things. Remember that `search_conducted` is a dichotomous categorical measure, so a boxplot is a good starting point.
+```{r searches over time}
+ilstops %>%
+  ggplot(aes(x=search_conducted, y=date)) + geom_boxplot()
+  
+```
+
+How considerate of ggplot to include the `NA` category by default! Notice how seemingly all of the missing values come from 2012-2014? There's some of that missing data the SOPP folks warned us about in their documentation. Let's inspect things a bit more closely:
+
+```{r explore missing values in searches}
+ilstops %>%
+  group_by(search_conducted) %>%
+  summarize(
+    min = min(date, na.rm=TRUE),
+    max = max(date, na.rm=TRUE)
+  )
+```
+
+All the missing data for `search_conducted` comes within a two-year range. 
+
+I can use `filter` to look at the `search_conducted` variable within that two year range alone:
+```{r missing search values pre 2014}
+ilstops %>%
+  filter(date < as_date('2014-01-11')) %>%
+  group_by(search_conducted) %>%
+  summarize(
+    n = n()
+  )
+```
+*Most* of the `search_conducted` data between 2012-2014 isn't missing, even if that's the only period within which missing values for `search_conducted` appear. There's not much we can do about this, but it's worth taking into consideration as we go.
+
+
+## PC5 Searches within `subject_race` groups over time
+
+The goal here is to synthesize pieces of the work we did in PC3 and PC4 to analyze subgroups of the stops and the stops resulting in searches over time. I'll start by reorganizing/tidying the data to support the creation of my time series plots.
+
+### 5.1 Binning data within time periods
+
+I want to plot this data as a time series of some numeric measure, but right now my outcome variable is a categorical indicator. How can I plot TRUE/FALSE values over time? One good way is to bin my data into meaningful time-periods (e.g., months or weeks) and count the number of TRUE values (and/or calculate proportions of TRUE values) within each bin. 
+
+
+Let's do that using some of the handy features of `lubridate` for working with date objects. I have demonstrated a few approaches to binning and grouping in the R tutorial materials, but here's yet another that takes advantage of some `lubridate` features (specifically the call to `format` within the `mutate` step below) quite effectively:
+
+```{r bin to months}
+## library(lubridate)
+
+ilstops_monthly <- ilstops %>%
+  filter(!is.na(subject_race)) %>%
+  mutate(
+    yearmonth = format(date, '%Y-%m')
+  ) %>%
+  group_by(yearmonth) %>%
+  summarize(
+    stops = n(),
+    searches = sum(as.numeric(search_conducted), na.rm=T),
+    prop_searched = round(searches / stops,3)
+    )
+
+ilstops_monthly
+```
+Looks great, except do you notice how my new `yearmonth` variable is no longer recognized as a date or date-time object? I think I can fix that with the handy `parse_date_time` function from `lubridate`:
+```{r clean binned dates}
+ilstops_monthly$date <- parse_date_time(ilstops_monthly$yearmonth, "ym")
+class(ilstops_monthly$date)
+```
+
+Now I can plot it since `ggplot` knows what to do with objects in the `POSIXct` class. I'm also going to add a type of layer we haven't worked with yet (`geom_smooth`) which incorporates a smoothed conditional mean to the plot (appropriately enough in this dataset, in the form of a thin blue line).
+
+```{r plot monthly stops}
+ilstops_monthly %>%
+  ggplot(aes(x=date, y=stops)) + geom_line() + geom_smooth() + labs(title="Illinois traffic stops", y="stops (per month)")
+         
+```
+
+Turns out traffic stops are not distributed smoothly across the months of the year (I notice, in particular, the big dips and spikes around the end of each year). It also looks like there is a slight $\cup$-shaped trend overall, but the shifts do not look very large relative to either the month-to-month variations or the total number of stops per year.
+
+Here's a similar plot for searches:
+```{r plot monthly searches}
+ilstops_monthly %>%
+  ggplot(aes(x=date, y=searches)) + geom_line() + geom_smooth() + labs(title="Illinois traffic stops that result in searches",
+                                                                       y="searches (per month)")
+         
+```
+Turns out searches are also not distributed smoothly across the months of the year (again, the big dips and spikes around the end of the year). In this case, there seems to be a very slight downward trend on average, but this decrease mostly appears in the first couple of years of the data and does not look very large relative to the month-to-month variations or the total number of searches per year (maybe ten fewer searches per month on average during the latter half of the dataset?).
+
+Let's try looking at the proportion of stops that result in searches.
+
+```{r plot monthly prop searched}
+ilstops_monthly %>%
+  ggplot(aes(x=date, y=prop_searched)) + geom_line() + geom_smooth() + labs(title="Proportion of Illinois traffic stops that result in searches",  y="proportion searched\n(per month)")
+```
+Still quite spiky! Interestingly, we see an inverse-$\cap$ trend in the average here (contrast with the raw counts of traffic stops above), but these fluctuations are, again, very small relative to the underlying variation or the range of measure. Also, notice the narrow y-axis scale sort of hides the fact that almost all the values fall between $4-6\%$. In other words, all of the variation here is within quite a narrow range.
+
+### 5.2 Plot stops within `subject_race` categories
+
+To do this, I'll need to go back to the code that created my `ilstops_monthly` tibble above and incorporate the conditional summary data. To do this, I'll "explode" the aggregated summaries from the old tibble by inserting an extra variable to `group_by()` before making my call to `summarize`. Notice that I go ahead and cleanup the dates again too (since I know to anticipate that issue)
+
+```{r bin to subgroup months}
+ilstops_monthly_grouped <- ilstops %>%
+  filter(!is.na(subject_race)) %>%
+  mutate(
+    yearmonth = format(date, '%Y-%m')
+  ) %>%
+  group_by(yearmonth, subject_race) %>%
+  summarize( 
+    stops = n(),
+    searches = sum(as.numeric(search_conducted), na.rm=T),
+    prop_searched = round(searches / stops,3)
+    )
+
+ilstops_monthly_grouped$date <- parse_date_time(ilstops_monthly_grouped$yearmonth, "ym")
+
+ilstops_monthly_grouped
+
+```
+Great! Now, I need to add the values for total stops and total searches each month (note that this will be the same for all five groups/rows within any given value of `date` or `yearmonth`) so that I can calculate the proportion of total stops and total searches accounted for by each group. There are a bunch of ways we could go about this. Since I've made it this far with tidyverse code, I'll keep going in that direction and use the `mutate` verb to keep all my existing rows while also creating some new ones. Notice that I have to do this twice because my first call to `group_by` is only by `yearmonth` so that I can create the monthly totals, whereas my second call to `group_by` once again looks wihtin `yearmonth` and `subject_race` to calculate the proportions. In order to make sure that nothing weird happens along the way I insert a call to `ungroup` as well (which does exactly what you might hope following an operation performed on some data that's already grouped in some way).
+
+```{r create total stops, total searches and prop for each within groups}
+ilstops_monthly_grouped <- ilstops_monthly_grouped %>%
+  group_by(yearmonth) %>%
+  mutate(
+    total_stops = sum(stops),
+    total_searches = sum(searches)
+  ) %>%
+  ungroup() %>%
+  group_by(yearmonth, subject_race) %>%
+  mutate(
+    prop_total_stops = stops / total_stops,
+    prop_total_searches = searches / total_searches
+  )
+
+ilstops_monthly_grouped
+
+## I think the html drops a few of our new columns, so I'll inspect them manually here
+head(ilstops_monthly_grouped[,c("yearmonth", "subject_race", "total_searches", "prop_total_stops", "prop_total_searches")])
+```
+
+Now my plotting code will look pretty similar to the univariate plots above, just with the addition of another aesthetic...
+
+```{r plot monthly subgroup stops}
+ilstops_monthly_grouped %>%
+  ggplot(aes(x=date, y=stops, color=subject_race)) + geom_line() + geom_smooth() + labs(title="Illinois traffic stops",  y="stops (per month)")
+         
+```
+
+### 5.3 Plot searches within `subject_race` categories
+```{r plot monthly subgroup searches}
+ilstops_monthly_grouped %>%
+  ggplot(aes(x=date, y=searches, color=subject_race)) + geom_line() + geom_smooth() + labs(title="Illinois traffic stops resulting in searches",  y="searches (per month)")
+
+```
+
+
+### 5.4 Plot proportion of searches within `subject_race` categories
+
+The question for this one only asked about proportions of total searches accounted for by each group of `subject_race`, when several other interesting baselines/comparisons are available. For example, the proportion of total stops across groups as well as stops resulting in searches *within* each group also strike me as potentially illustrative. I'll plot each of these proportions just for the sake of completeness. Let's start with the proportion of total stops plotted across groups.
+```{r plot monthly subgroup prop total stops}
+ilstops_monthly_grouped %>%
+  ggplot(aes(x=date, y=prop_total_stops, color=subject_race)) + geom_line() + geom_smooth() + labs(title="Proportion of total Illinois traffic stops by race/ethnicity",  y="proportion of total stops\n(per month)\n")
+```
+
+Onwards to the proportion of stops resulting in searches calculated *within* each group of `subject_race`:
+
+```{r plot monthly subgroup prop searched}
+ilstops_monthly_grouped %>%
+  ggplot(aes(x=date, y=prop_searched, color=subject_race)) + geom_line() + geom_smooth() + labs(title="Proportion of Illinois traffic stops that result in searches",  y="proportion searched\n(calculated per month)")
+```
+
+Hmm, that looks pretty messy. The proportions compress everything into a narrow region of the y-axis. Also, the data for some of these groups (especially "other") fluctuates all over the place, which is a distration since that's the smallest category. 
+
+Instead of plotting all the proportions on the same figure, it might be easier to see/compare variations if I use facets and don't include the "other" category (because the y-axis scale is just so different).
+
+```{r plot monthly subgroup prop searched facets}
+ilstops_monthly_grouped %>%
+  filter(subject_race != "other") %>%
+  ggplot(aes(x=date, y=prop_searched, color=subject_race)) + geom_line() + geom_smooth() + facet_grid(rows=vars(subject_race)) + labs(title="Proportion of Illinois traffic stops that result in searches by race/ethnicity",  y="proportion searched\n(per month)\n")
+```
+
+The title runs a little long and facet label for "asian/pacific islander" gets truncated. If I were preparing this for publication I'd fix those issues, but for now, I'm going to acknowledge them and move on.
+
+Now, let's look at the proportion of total searches.
+
+```{r plot monthly subgroup prop total searches}
+ilstops_monthly_grouped %>%
+  ggplot(aes(x=date, y=prop_total_searches, color=subject_race)) + geom_line() + geom_smooth() + labs(title="Proportion of total Illinois traffic stops resulting in searches by race/ethnicity",  y="proportion of total searches\n(per month)\n")
+```
+
+## PC6. Calculate baseline population proportions
+
+As recommended, I'll calculate this using the [`county_complete`](https://www.openintro.org/data/index.php?data=county_complete) provided as part of the `openintro` package to build the relevant variables for Illinois in 2010. You'll want to review the [list of variables](https://www.openintro.org/data/index.php?data=county_complete) to figure out what you need to use here. 
+
+Note that what we're looking for are *statewide-proportions* of race/ethnic groups in 2010 within the same categories in the SOPP traffic stop data for Illinois. The `county_complete` dataset does not provide these values directly, but instead has county-level populations and county-level proportions of slightly different race/ethnic groups over multiple different years across all the counties in the United States. That means that we'll need to take some steps (like the following) to generate state-level proportion estimates:  
+
+* Restrict the dataset to Illinois counties only.  
+* Multiply county-level populations by county-level proportions for salient groups to create county-level population counts within groups.  
+* Sum county-level populations within groups to create state-level group populations.  
+* Divide state-level group populations by total state population (sum of all county populations) to calculate state-level proportions.  
+
+In the code chunk below, I'll tackle these steps in a very "verbose" and redundant way:
+```{r verbose county baselines}
+library(openintro)
+data(county_complete)
+
+il_county <- county_complete[county_complete$state == "Illinois",]
+
+## total state population is easy
+il_pop2010 <- sum(il_county$pop2010)
+
+## Turn IL county-level proportions into counts
+il_county$white2010 <- il_county$pop2010*il_county$white_2010
+il_county$black2010 <- il_county$pop2010*il_county$black_2010
+il_county$hispanic2010 <- il_county$pop2010*il_county$hispanic_2010
+
+## We'll have to sum these two groups to approximate "Asian / Pacific Islander" from SOPP data
+il_county$asian2010 <- (il_county$pop2010*il_county$asian_2010)
+il_county$pi2010 <- (il_county$pop2010*il_county$pac_isl_2010)
+
+## Then go through summing and dividing by the total population
+##
+##prop.white
+sum(il_county$white2010) / il_pop2010
+##prop.black 
+sum(il_county$black2010) / il_pop2010
+##prop.hispanic 
+sum(il_county$hispanic2010) / il_pop2010
+##prop.api
+(sum(il_county$asian2010, na.rm=TRUE)+ sum(il_county$pi2010, na.rm=TRUE)) / il_pop2010
+```
+
+That is all perfectly accurate; however, the code is verbose and redundant because it calculates each (intermediate and group) value with separate lines of code that are very repetitive. There are many more concise ways to go about this, all of which make it faster, less prone to typing mistakes, and easier to extend. How to proceed? Anytime I find myself repeating the same steps of analysis more than once, I consider creating a function and then calling that function within a call to an `*apply()` function. Here's what that might look like here:
+```{r county baselines via function}
+gen_subgroup_prop <- function(group_var, d=il_county){
+  total_pop2010 <- sum(d["pop2010"]) # state population total
+  county_ests <- d["pop2010"]*d[group_var] # county-level counts within groups
+  group_prop <- sum(county_ests, na.rm=TRUE) / total_pop2010 # state-level proportions within groups
+  return(round(group_prop, 4)) # prettify output
+}
+
+## give it a try:
+gen_subgroup_prop("hispanic_2010")
+
+## Now let's tie everything together
+groups <- c("white_2010", "black_2010", "hispanic_2010", "asian_2010", "pac_isl_2010")
+
+sapply(groups, gen_subgroup_prop)
+
+```
+
+I need to add those last two values together to reproduce the "asian/pacific islander" category from the SOPP data. Note also that there are slight differences between my results here and my results above. Any differences **should** be due entirely to rounding and, in a scenario where this analysis was being prepared for publication and/or informing a policy decision, I'd ansp;ite;y want to check that and think carefully how many significant digits to include in my reported estimates. The Illinois population is sufficiently large that even a few hundredths of a percent turn into very meaningful numbers of people!
+
+# Statistical questions
+
+Note that my "solutions" below reflect possible interpretations, but are not intended to be exhaustive or exclusive of other possible interpretations. We'll discuss these in our class meeting.
+
+## SQ1. Interpret analysis from PCs3-5  
+
+Some interpretation appeared above alongside specific results. Here is a brief summary of several striking points.  
+
+* Overall, traffic stops and searches in Illinois between 2012-2018 were distributed unequally across different racial/ethnic groups.  
+    + The proportion of stops resulting in searches is higher among stops of black and hispanic drivers ($7\%$ and $6\%$ respectively vs. $4\%$ or less for all other groups).  
+    + Black and hispanic drivers account for a higher proportion of stops resulting in searches than stops not resulting in searches ($30\%$ and $18\%$ vs. $20\%$ and $13\%$ respectively).  
+
+* Over time, the tpyical number of traffic stops, stops resulting in searches, and the proportion of stops resulting in searches fluctuate widely, but have fairly stationary central trends.  
+    + The typical number of monthly stops dipped a bit between 2014-2015.  
+    + The typical number of monthly stops resulting in searches may have decreased 2012-2015, but held flat 2015-2018.  
+    + The typical monthly proportion of stops resulting in searches increased a little 2012-2014, but then declined a little bit 2014-2017.  
+    
+* Comparing across the racial/ethnic groups identified in the data, the *typical monthly numbers of stops and searches* vary substantially and exhibit distinct trends.  
+    + The typical number of black and hispanic drivers stopped per month has increased.  
+    + The typical number of white drivers stopped per month has decreased.  
+    + The typical number of asian/pacific islander and "other" category drivers per month has remained nearly flat.  
+    + The typical number of stops resulting in searches among black, white, and hispanic drivers per month have tended to be quite close to each other, and sometimes overlap,
+    + The typical number of stops resulting in searches among white drivers per month has increased slightly.
+    + The typical number of stops resulting in searches among black and hispanic drivers per month has decreased slightly.
+    
+* The typical *proportion of stops resulting in searches within racial/ethnic groups* have shifted somewhat over time.  
+    + The proportion has increased slightly among white and asian/pacific islander drivers.  
+    + The proportion has decreased slightly among black and hispanice drivers.  
+    
+* The typical *proportion of total stops accounted for by each racial/ethnic group* has shifted somewhat over time.  
+    + The proportion has decreased substantially among white drivers.  
+    + The proportion has increased among black drivers (substantially) and among hispanic drivers (slightly).  
+    
+* The typical *proportion of total searches accounted for by each racial/ethnic group* has also shifted over time.  
+    + As of 2012, the proportion accounted for by black and white drivers was nearly identical (close to $40\%$ each)!
+    + The proprtion has increased substantially among white drivers (up to .  
+    + The proportion has decreased substantially among black and hispanic drivers. 
+
+## SQ2. Contextualize SQ1 interpretation in relation to PC6  
+
+Several noteworthy comparisons come looking across the different proportions for traffic stops and searches and comparing those against the baseline population proportions accounted for by each racial/ethnic group category. Here are several specific observations:  
+
+* The white proportion of the state population is consistently larger than either the proportion of white drivers involved in all traffic stops or the proportion of white drivers involved in all stops resulting in searches.  
+* The hispanic proportion of the state population is about the same as the proportion of hispanic drivers involved in all traffic stops and usually a little smaller than the proportion of hispanic drivers involved in all stops resulting in searches (although this latter difference shrank over the period of data collection).  
+* The black proportion of the state population is consistently and substantially smaller than the proportion of black drivers involved in all traffic stops and consistently and substantially smaller than the proportion of black drivers involved in all stops resulting in searches. This latter difference in particular stands out as it is almost more than double the baseline population proportion.  
+* Overall, these results suggest that race/ethnicity and traffic stops and searches are not indepdendent in a statistical sense. The nature of their relationship is complex and could be due to a number of factors including biased policing practices, socioeconomic inequalities, different levels of behaviors linked to traffic stops within specific sub-groups, regional variations masked by these state-level analyses, political/cultural factors shaping police/driver interactions, and more.  
+
+## SQ3. Limitations and possible extensions  
+
+Again, many possible things worth mentioning here, so I'll provide a few that stand out to me.  
+
+* The generalizability of analysis focused on one state during one 6 year period is limited.
+* The data seem very prone to measurement errors of various kinds. In particular, I suspect the race/ethnicity classifications provided by officers are subject to some biases that are hard to identify and might also shift over time/region. The prevalence of missing values during the first two years of the dataset illustrate one aspect of this and may impact estimates of raw counts and proportions.  
+* While the comparisons across racial/ethnic groups and between the traffic stops/searches and baseline population proportions illustrates a number of suggestive patterns, conclusive interpretation or attribution of those patterns to any specific cause or causes is quite difficult in the absence of additional information or assumptions. For one example, see my comments regarding statistical independence and the possible explanations in SQ2 above. 
+* Extensions of this analysis might seek to investigate how some of the patterns identified in the aggregate sate-level data vary across sub-regions (e.g., counties or police districts) or even in comparison to other states.  
+* Another extension might investigate how specific policy and/or socioeconomic changes may or may not relate to shifting patterns in the data.
+* Yet another extension could try to identify some factor that introduces some sudden, uncontrolled (by the drivers or the police) change into the process of traffic stops that could make it possible to identify and estimate the effects of specific explanatory factors. The original SOPP paper is an extraordinary example of this that we can discuss! 
\ No newline at end of file

Community Data Science Collective || Want to submit a patch?